Sei sulla pagina 1di 99

TOPNOTCH MEDICAL BOARD PREP PATHOLOGY SUPEREXAM

For inquiries visit www.topnotchboardprep.com.ph or email us at topnotchmedicalboardprep@gmail.com


DEAR TOPNOTCH FRIENDS:

PLEASE FOLLOW THESE INSTRUCTIONS:

1. These questions are previous diagnostic, midterm, and finals exams of Topnotch, almost all of them made by Topnotch Board Exam Topnotchers.
2. Answer this Topnotch Superexam seriously 100-items at a time. Cover the Explanations Column. Do not immediately look at the answers from the
answer key. Thats not the correct way of answering sample exams. You need to treat these MCQs as exercises and not as handouts.
3. Time yourself. 1.5 hours per 100-item block.
4. After answering each 100-item block, refer to the Topnotch Answer Key for the correct answers. Please be careful of frameshift mutations when
checking your answers check every 10 items. (the format of the answer key was designed for you to practice against frameshift mutations)
5. The Topnotch Superexams are EXERCISES for the actual med boards. They will not appear verbatim in your future exams. More than knowing whats
the correct answer, its more important for you to:
a. Know why the other choices are wrong
b. Know why the other choices were included in the first place
c. Know the explanation to the correct answer
6. Sharpen your mind by answering the Topnotch Superexams. Most of these questions based on past feedback are more difficult than the actual questions
in the med boards. In these exams made by Board Exam Topnotchers, if youre getting a score of 60/100 , thats already a good score. More than 80/100
is outstanding.

Item QUESTION EXPLANATION AUTHOR TOPNOTCH
# EXAM
1 Which of the following is least likely to facilitate TGF - transforming growth factor. KRISTEL TANHUI DIAGNOSTIC
chemotaxis: (TOP 3 - AUG 2015 EXAM - MARCH
A. LTB4 Questions in the boards can sometimes be phrased as MED BOARDS; 2016
B. IL8 least likely or most likely, so try to accustom TOPNOTCH MD
C. C5a yourself to choosing the best answer in such FROM LA SALLE)
D. TGF circumstances.
E. N-formylmethionine
Source: Robbins and Cotran Pathologic Basis of
Disease 8th ed p. 50
2 The following are anaphylotoxins The anaphylatoxins are components of the KRISTEL TANHUI DIAGNOSTIC
A. C3a complement system which are involved in (TOP 3 - AUG 2015 EXAM - MARCH
B. C4a anaphylaxis. Source: Robbins and Cotran Pathologic MED BOARDS; 2016
C. C5a Basis of Disease 8th ed p. 57 TOPNOTCH MD
D. A and C only FROM LA SALLE)
E. All of the above

3 A 56 year old hypertensive male presented with Infarcts in the brain result to liquefactive necrosis KRISTEL TANHUI DIAGNOSTIC
left sided hemiparesis on waking up. A CT scan while those in all other organs except the brain exhibit (TOP 3 - AUG 2015 EXAM - MARCH
was done which revealed an infarct in the coagulative necrosis. MED BOARDS; 2016
distribution of the MCA. Which of the following TOPNOTCH MD
is the expected gross pathological finding in the Source: Robbins and Cotran Pathologic Basis of FROM LA SALLE)
brain? Disease 8th ed p. 15
A. Coagulative necrosis
B. Caseous necrosis
C. Liquefactive necrosis
D. Fibrinoid necrosis
E. Gangrenous necrosis
4 A 45 year old woman presents with recurrent This is a case of CML. It is associated with 9:22 KRISTEL TANHUI DIAGNOSTIC
infections and on PE was found to have marked translocation and may be treated with imatinib (TOP 3 - AUG 2015 EXAM - MARCH
splenomegaly. Her leukocyte count is increased mesylate. MED BOARDS; 2016
to 300,000. The differential count reveals the Basophilia is a rare finding. It is strongly indicative of TOPNOTCH MD
presence of myeloblasts and promyelocytes, CML. FROM LA SALLE)
with predominance of myelocytes,
metamyelocytes, bands and segmented There are a couple of conditions that cause increased
neutrophils. Basophils are also increased in WBC, either a leukemia or an infection. The increase in
number. The patient is not anemic. Leukocyte WBC count due to an infection is called a leukemoid
alkaline phosphatase is decreased. Which of the reaction. This can be differentiated from CML via the
following describes a major characteristic of leukocyte alkaline phosphatase test. In a leukemoid
this disorder? reaction LAP is increased.
A. 9:22 translocation
B. Expansion of mature B lymphocytes within Source: Topnotch handout on Pathology.
multiple lymph nodes
C. Hypogammaglobulinemia
D. Neoplastic cells exhibiting hair-like
filamentous projections
E. Peak incidence occurs at 65 years

5 A 50 year old man seeks consult due to a This is a case of Mycosis fungoides, which is a T cell KRISTEL TANHUI DIAGNOSTIC
pruritic rash which he has had over the past 8 lymphoma of the skin. Atypical CD4+ T cells with (TOP 3 - AUG 2015 EXAM - MARCH
months. On PE, there were erythematous, cerebriform nuclei are found on biopsy. The disorder MED BOARDS; 2016
eczematoid patches and raised plaques may remain confined to the skin for several years. TOPNOTCH MD
distributed asymmetrically over the chest and When the neoplastic cells invade the skin and become FROM LA SALLE)
abdomen. On biopsy of the lesions, atypical systemic, this is called Sezary syndrome. Sezary
CD4+ T cells with cerebriform nuclei were syndrome is the leukemic form of this cutaneous T cell
found. What is a possible outcome in the course lymphoma and is characterized by the combination of
of this condition? skin lesions and circulating neoplastic cells.
A. Acute leukemia
B. Myelofibrosis Acute leukemia and myelofibrosis are courses in the
C. Sezary Syndrome natural history of the myeloproliferative syndromes.
D. A and B only
E. All of the above Source: Topnotch handout on Pathology

TOPNOTCH MEDICAL BOARD PREP PATHOLOGY SUPEREXAM Page 1 of 99


For inquiries visit www.topnotchboardprep.com.ph or email us at topnotchmedicalboardprep@gmail.com
TOPNOTCH MEDICAL BOARD PREP PATHOLOGY SUPEREXAM
For inquiries visit www.topnotchboardprep.com.ph or email us at topnotchmedicalboardprep@gmail.com
Item QUESTION EXPLANATION AUTHOR TOPNOTCH
# EXAM
6 A 70 year male presented with a 12 hour 0-4hrs: None to Variable waviness of the fiber KRISTEL TANHUI DIAGNOSTIC
history of anginal chest pain. The stat troponin I 1-3 days: neutrophilic infiltrate (TOP 3 - AUG 2015 EXAM - MARCH
and ECG studies confirm the diagnosis of acute 3-7 days: macrophage infiltrate MED BOARDS; 2016
myocardial infarction. Unfortunately, he 1-2 wks: granulation tissue TOPNOTCH MD
succumbed to a fatal arrythmia during his 30 >2 mos: dense collagenous scar FROM LA SALLE)
minute minute stay in the ER before a definitive
intervention could be applied. What are the Source: Robbins and Cotran Pathologic Basis of
expected microscopic findings on LM for his Disease 8th ed p. 550
heart?
A. Dense collagenous scar
B. Variable waviness of the fiber
C. Coagulation necrosis with neutrophilic
infiltrate
D. Beginning disintegration of muscle fibers
with macrophage infiltrate
E. None, its too early for histopathologic
changes to occur.
7 Which of the following findings is an unlikely Trea-bark appearance of the ascending aorta is a KRISTEL TANHUI DIAGNOSTIC
finding in malignant hypertension? characteristic of tertiary syphilis. (TOP 3 - AUG 2015 EXAM - MARCH
A. Multiple punctate hemorrhage on the surface Source: Robbins and Cotran Pathologic Basis of MED BOARDS; 2016
of both kidneys Disease 8th ed p. 950 TOPNOTCH MD
B. Trea-bark appearance of the ascending aorta FROM LA SALLE)
C. Fibrinoid necrosis of arterioles
D. Onion skinning of arterioles
E. None of the above

8 A 65 year old female presents with fever, This is a case of Giant cell arteritis. Along with KRISTEL TANHUI DIAGNOSTIC
headache and diplopia. On palpation, involvement of the branches of the carotid artery, (TOP 3 - AUG 2015 EXAM - MARCH
tenderness is noted along the course of the there is a close association with polymyalgia MED BOARDS; 2016
temporal artery. A biopsy would most likely rheumatica. TOPNOTCH MD
confirm which of the following diagnosis? FROM LA SALLE)
A. Takayasu arteritis Takayasu arteritis is pulseless disease. It is more
B. Retinoblastoma common in women of childbearing age. Initial
C. Giant cell arteritis symptoms are usually nonspecific, including fatigue,
D. Kaposi sarcoma weight loss, and fever. With progression, vascular
E. Katayama disease symptoms appear and dominate the clinical picture,
including reduced blood pressure and weaker pulses
in the upper extremities; ocular disturbances,
including visual defects, retinal hemorrhages, and
total blindness; and neurologic deficits. Involvement
of the more distal aorta may lead to claudication of the
legs; pulmonary artery involvement may cause
pulmonary hypertension. Narrowing of the coronary
ostia may lead to myocardial infarction, and
involvement of the renal arteries leads to systemic
hypertension in roughly half of patients.

Source: Robbins and Cotran Pathologic Basis of
Disease 8th ed p. 512
9 A 50 year old male was recently diagnosed with Tumor necrosis factor or cachectin is secreted by KRISTEL TANHUI DIAGNOSTIC
PTB. On physical examination, he appears activated macrophages and is also responsible for the (TOP 3 - AUG 2015 EXAM - MARCH
emaciated. Which of the following is also called cachexia in cancer patients. MED BOARDS; 2016
cachectin and is responsible for the weight loss TOPNOTCH MD
and wasting noted in the patient? Source: Robbins and Cotran Pathologic Basis of FROM LA SALLE)
A. IL1 Disease 8th ed p. 320
B. IL2
C. PAF
D. TGF-beta
E. TNF

10 Pneumoconiosis refers to nonneoplastic lung Silicosis upper lobes, eggshell calcification, PTB KRISTEL TANHUI DIAGNOSTIC
reaction to inhalation of mineral dusts (TOP 3 - AUG 2015 EXAM - MARCH
encountered in the work place. Which Source: Robbins and Cotran Pathologic Basis of MED BOARDS; 2016
pneumoconiosis is characterized by discrete Disease 8th ed p. 699 TOPNOTCH MD
pale to blackened nodules in the upper zones of FROM LA SALLE)
the lungs. Radiographically it presents as
eggshell calcifications. This disease may be
progressive even if patient is no longer exposed
and it has also been associated with increased
susceptibility to PTB.
A. Coal workers pneumoconiosis
B. Silicosis
C. Asbestosis
D. Byssinosis
E. Sarcoidosis

TOPNOTCH MEDICAL BOARD PREP PATHOLOGY SUPEREXAM Page 2 of 99


For inquiries visit www.topnotchboardprep.com.ph or email us at topnotchmedicalboardprep@gmail.com
TOPNOTCH MEDICAL BOARD PREP PATHOLOGY SUPEREXAM
For inquiries visit www.topnotchboardprep.com.ph or email us at topnotchmedicalboardprep@gmail.com
Item QUESTION EXPLANATION AUTHOR TOPNOTCH
# EXAM
11 A 40 yr old male presents with fever and cough Patient has bacterial pneumonia. Streptococcus is a KRISTEL TANHUI DIAGNOSTIC
of 5 days duration. PE revealed increased popular agent for causing lobar pneumonia. (TOP 3 - AUG 2015 EXAM - MARCH
bronchial breath sounds over the lower Stages of inflammatory response in lobar pneumonia: MED BOARDS; 2016
segment of the right lobe posteriorly. Chest - Congestion: red, heavy, boggy lung TOPNOTCH MD
xray showed lobar consolidation on the right - Red hepatization: massive, confluent exudation with FROM LA SALLE)
lower lobe and culture was positive for neutrophils, red cells and fibrin
pneumonococcus. Which is the prominent - Gray hepatization
inflammatory cells of this exudate? - Resolution
A. Platelets
B. Basophils Source: Robbins and Cotran Pathologic Basis of
C. Eosinophils Disease 8th ed p.712
D. Neutrophils
E. Macrophage

12 19 year old male presents with acute onset When you study for patho and all basic sciences, try to KRISTEL TANHUI DIAGNOSTIC
hematuria, oliguria and periorbital edema make sure you know how the disease will present (TOP 3 - AUG 2015 EXAM - MARCH
which is worse in the morning and gradually clinically cause they like to correlate things. The MED BOARDS; 2016
improves through the day. On PE, blood hardcore micro and biochem or patho knowledge will TOPNOTCH MD
pressure is elevated and urinalysis shows trace only get you halfway cause sometimes they will not FROM LA SALLE)
proteinuria, and many RBCs. ASOT is negative give you the diagnosis.
and DNAse is positive. Which is the expected
finding in light microscopy for this patient? This is a case of PSGN. ASOT may be negative if the
A. Diffuse endocapillary proliferation nephritic strain comes from a skin infection because
B. Subepithelial humps skin lipids bind to streptolysin O. DNAse is the most
C. Normal appearing sensitive test for skin infection with Group A
D. Dense deposits streptococcus. The patient in this case presented with
E. Extracapillary proliferation of crescents the classic nephritic syndrome of hypertension,
hematuria and oliguria.
A and B are both PSGN but B is an electron microscopy
finding.

Source: Robbins and Cotran Pathologic Basis of
Disease 8th ed p.918
13 Patient sought consult for chest pain and Dont just focus on the electron and LM findings and KRISTEL TANHUI DIAGNOSTIC
hemoptysis associated with hematuria, and sacrifice immunofluorescence. SIMILAR TO PREVIOUS (TOP 3 - AUG 2015 EXAM - MARCH
signs and symptoms of uremia. His blood BOARD EXAM CONCEPT/PRINCIPLE. MED BOARDS; 2016
pressure is elevated and he has grade III TOPNOTCH MD
bipedal edema. As the clinician in charge, you A rule of thumb is if the pathophysiology involves FROM LA SALLE)
suspect that he is suffering from an immune complex deposition, it would usually present
autoimmune disease in which antibodies with a granular pattern. If the pathophysiology
against type IV collagen attack the basement involves antibodies against basement membranes, the
membrane of the lungs and kidneys. On pattern is usually linear.
immunofluorescence renal biopsy would most
probably reveal: This is actually a case of Goodpasture syndrome which
A. Granular IgG and C3 in GBM and mesangium involves antibodies against the basement membrane
B. Linear IgG and C3 of the lungs and the kidneys and the findings on
C. Negative immunofluorescent studies show linear IgG and C3.
D. Focal IgM or C3
E. IgA in the mesangium Source: Robbins and Cotran Pathologic Basis of
Disease 8th ed p. 709, 918

14 Patent urachus is a risk factor for which type of Transitional cell carcinoma is associated with KRISTEL TANHUI DIAGNOSTIC
carcinoma of the bladder? smoking. (TOP 3 - AUG 2015 EXAM - MARCH
A. Transitional cell carcinoma Squamous cell carcinoma is associated with chronic MED BOARDS; 2016
B. Squamous cell carcinoma irritation and S. haematobium infection. TOPNOTCH MD
C. Adenocarcinoma FROM LA SALLE)
D. Clear cell carcinoma Source: Robbins and Cotran Pathologic Basis of
E. Sarcoma Disease 8th ed p. 979

15 In which of the following nutritional deficiency Kwashiorkor is a protein deficiency while marasmus a KRISTEL TANHUI DIAGNOSTIC
is hepatic steatosis an expected finding? balanced deficiency of all macronutrients. (TOP 3 - AUG 2015 EXAM - MARCH
A. Vitamin A deficiency MED BOARDS; 2016
B. Vitamin E deficiency Source: Robbins and Cotran Pathologic Basis of TOPNOTCH MD
C. Copper deficiency Disease 8th ed p. 429 FROM LA SALLE)
D. Kwashiorkor
E. Marasmus

16 Which of the following is not a characteristic of Benign ulcers may have mucosal margins which KRISTEL TANHUI DIAGNOSTIC
a benign peptic ulcer? overhang the base slightly but is usually level with the (TOP 3 - AUG 2015 EXAM - MARCH
A. Sharply punched out defect surrounding mucosa. Heaped up margins are more MED BOARDS; 2016
B. Heaped-up margins characteristic of malignant ulcers. TOPNOTCH MD
C. Hemorrhage and fibrin deposition in the FROM LA SALLE)
gastric serosa Source: Robbins and Cotran Pathologic Basis of
D. Perforation Disease 8th ed p. 780
E. Malignant transformation is very rare

TOPNOTCH MEDICAL BOARD PREP PATHOLOGY SUPEREXAM Page 3 of 99


For inquiries visit www.topnotchboardprep.com.ph or email us at topnotchmedicalboardprep@gmail.com
TOPNOTCH MEDICAL BOARD PREP PATHOLOGY SUPEREXAM
For inquiries visit www.topnotchboardprep.com.ph or email us at topnotchmedicalboardprep@gmail.com
Item QUESTION EXPLANATION AUTHOR TOPNOTCH
# EXAM
17 Which of the following is the least likely Napkin ring constrictions are characteristic of left KRISTEL TANHUI DIAGNOSTIC
characteristic of right-sided colonic sided colonic malignancy. Right sided lesions present (TOP 3 - AUG 2015 EXAM - MARCH
malignancy? as polypoid, exophytic masses. Left sided lesions MED BOARDS; 2016
A. Adenocarcinoma present clinically as obstruction while right sided ones TOPNOTCH MD
B. Napkin ring constriction and luminal present as anemia. FROM LA SALLE)
narrowing
C. Desmoplastic response Source: Robbins and Cotran Pathologic Basis of
D. Liver metastasis Disease 8th ed p. 824
E. None of the above
18 Which of the following is the most common Most common cause KRISTEL TANHUI DIAGNOSTIC
cause of chronic pancreatitis? Acute pancreatitis gallstones (TOP 3 - AUG 2015 EXAM - MARCH
A. Gallstones Chronic pancreatitis alcoholism MED BOARDS; 2016
B. Alcoholism TOPNOTCH MD
C. Hypercholesterolemia Source: Robbins and Cotran Pathologic Basis of FROM LA SALLE)
D. Smoking Disease 8th ed p. 896
E. Hereditary predisposition

19 A 22 year old male consults for ptosis and This is a case of myasthenia gravis KRISTEL TANHUI DIAGNOSTIC
diplopia which usually begins to occur in the (TOP 3 - AUG 2015 EXAM - MARCH
late afternoon and improves upon waking up Antibodies to presynaptic Ca channel preventing MED BOARDS; 2016
after a long good sleep. Recently he also noted release of Acetylcholine Lambert Eaton myasthenic TOPNOTCH MD
dysphagia and muscle weakness which also syndrome (paraneoplastic syndrome) FROM LA SALLE)
improves with rest. What is the
pathophysiology of his condition? Source: Robbins and Cotran Pathologic Basis of
A. Antibodies to presynaptic Ca channel Disease 8th ed p. 1275
preventing release of Acetylcholine
B. Antibodies to Acetylcholine receptors
C. Autoimmune peripheral demyelination
D. Inflammation of the spinal cord
E. Patient appears to be faking it. He should be
referred to psych.
20 HSV 1 is the most common cause of viral Accustom yourself to answering question phrased as KRISTEL TANHUI DIAGNOSTIC
encephalitis. Which is the least likely expected most likely or least likely. Examiners may want to (TOP 3 - AUG 2015 EXAM - MARCH
pathological finding? phrase it that way. MED BOARDS; 2016
A. The encephalitis most severely affects the TOPNOTCH MD
frontal lobe. HSV1 encephalitis classically most severely affects the FROM LA SALLE)
B. The infection is necrotizing and often temporal lobe. It most commonly presents as
hemorrhagic in the most severely affected alterations in mood, behavior and memory.
regions
C. Cowdry type A intranuclear viral inclusion
bodies may be found
D. All of the above
E. None of the above
21 A 4-year-old boy presents with recurrent joint SIMILAR TO PREVIOUS BOARD EXAM LESTER BRYAN CO MIDTERM 1
pain involving the knees and hips. He had CONCEPT/PRINCIPLE. Classic hemophilia (factor VIII (TOP 10 - AUG 2015 EXAM - MARCH
always bruised easily, and recently the parents deficiency) is an abnormality of the intrinsic pathway MED BOARDS; 2016
had seen blood in his urine. A presumptive of coagulation proximal to the final common pathway, TOPNOTCH MD
diagnosis of classic hemophilia (hemophilia A) which begins at factor X Xa activation. This defect FROM UST)
is made, and coagulation blood tests are leads to a prolonged APTT. The other laboratory tests
performed. Which of the following is the most listed remain normal, because the bleeding time is a
likely set of findings of coagulation screening measure of platelet plug formation, the PT is a
tests? measure of the extrinsic pathway of coagulation, and
A. Normal bleeding time, platelet count, and the thrombin time is an assay of the conversion of
thrombin time; prolonged PT and APTT fibrinogen to fibrin. The presumptive diagnosis is
B. Normal bleeding time, platelet count, confirmed by specific
thrombin time, and APTT; prolonged PT factor VIII assay.
C. Normal bleeding time, platelet count,
thrombin time, and PT; prolonged APTT
D. Normal platelet count and thrombin time;
prolonged bleeding time, PT, and APTT
E. Prolonged bleeding time, PT, APTT, and
thrombin time; decreased platelet count
22 A 9-year-old girl is diagnosed with acute The most common cause of death that occurs during LESTER BRYAN CO MIDTERM 1
rheumatic fever. Instead of recovering as acute rheumatic fever is cardiac failure secondary to (TOP 10 - AUG 2015 EXAM - MARCH
expected, her condition worsens, and she dies. myocarditis. MED BOARDS; 2016
Which of the following is the most likely cause TOPNOTCH MD
of death? FROM UST)
A. Central nervous system involvement
B. Endocarditis
C. Myocarditis
D. Pericarditis
E. Streptococcal sepsis

TOPNOTCH MEDICAL BOARD PREP PATHOLOGY SUPEREXAM Page 4 of 99


For inquiries visit www.topnotchboardprep.com.ph or email us at topnotchmedicalboardprep@gmail.com
TOPNOTCH MEDICAL BOARD PREP PATHOLOGY SUPEREXAM
For inquiries visit www.topnotchboardprep.com.ph or email us at topnotchmedicalboardprep@gmail.com
Item QUESTION EXPLANATION AUTHOR TOPNOTCH
# EXAM
23 A 25-year-old man presents with hematuria, SIMILAR TO PREVIOUS BOARD EXAM LESTER BRYAN CO MIDTERM 1
periorbital edema, hypertension, and CONCEPT/PRINCIPLE. The clinical description is that (TOP 10 - AUG 2015 EXAM - MARCH
hemoptysis. He has also experienced nausea, of Goodpasture syndrome (antiglomerular basement MED BOARDS; 2016
vomiting, fever, and chills. Serologic testing is membrane disease), caused by antibodies directed TOPNOTCH MD
positive for antiglomerular basement against antigens in the glomerular and pulmonary FROM UST)
membrane antibodies. Which of the following is alveolar basement membranes. Because antigens are
the classic histologic finding in this renal an intrinsic component of the basement membrane,
disease?
labeled antibodies paint the surface of the basement
A. Linear immunofluorescence membrane, resulting in the characteristic linear
B. Lumpy-bumpy immunofluorescence immunofluorescent pattern characteristic of this
C. Spike and dome appearance of the disorder. Lumpy-bumpy immunofluorescence is
glomerular basement membrane very coarse, granular immunofluorescence found in
D. Subendothelial immune complex deposition poststreptococcal immune complex deposit disease.
E. Tram-track appearance of the glomerular The tram-track appearance is seen in
basement membrane on electron microscopy membranoproliferative glomerulonephritis. The
spike and dome appearance is seen in membranous
glomerulonephritis. Subendothelial immune complex
deposition is seen in lupus nephropathy.
24 A 19-year-old young woman who emigrated Inflammation and stenosis of branches of the aortic LESTER BRYAN CO MIDTERM 1
from Taiwan 8 years ago presents with fever, arch is known as Takayasu arteritis, or pulseless (TOP 10 - AUG 2015 EXAM - MARCH
malaise, myalgias, and arthritis and coldness disease. It most commonly occurs in young Asian MED BOARDS; 2016
in her upper extremities. She has a weak radial females. Buerger disease usually affects young Jewish TOPNOTCH MD
pulse bilaterally, and a magnetic resonance males and involves the arteries of the extremities. The FROM UST)
angiogram demonstrates nearly 75% stenosis disease is exacerbated by smoking and can lead to
of the main arteries originating from the aorta. gangrene of the extremities. Kawasaki disease affects
She likely has which of the following the branches of the coronary arteries. Raynaud
rheumatologic conditions? disease is due to vasospasm of small vessels of the
A. Buerger disease fingers and toes, leading to cyanosis and pallor of the
B. Kawasaki disease affected tissues. Temporal arteritis is usually
C. Raynaud disease encountered in older patients and affects the branches
D. Takayasu arteritis of the carotid artery, most commonly the temporal
E. Temporal arteritis artery.
25 A 45-year-old man presents with involuntary This is a case of Huntington disease, which is an LESTER BRYAN CO MIDTERM 1
facial grimaces and movements of the fingers. autosomal dominant, fatal, progressive degeneration (TOP 10 - AUG 2015 EXAM - MARCH
His mother had had similar symptoms and atrophy of the striatum (caudate nucleus and MED BOARDS; 2016
beginning at about the same age. Her disorder putamen). The disorder is characterized by an TOPNOTCH MD
had progressed to dancing movements, increased number of trinucleotide (CAG) repeats in FROM UST)
writhing of the arms and legs, and eventually the HD (huntingtin) gene on the short arm of
coma and death. His maternal grandfather had chromosome 4. Degeneration of the upper and lower
had a similar disorder but at an age older than motor neurons is characteristic of ALS. Dopamine
the mother. Which of the following is most depletion and depigmentation of the substantia nigra
characteristic of this disease? is characteristic of Parkinson disease. Neurofibrillary
A. Degeneration of upper and lower motor tangles and amyloid plaques are found in Alzheimer
neurons disease. Pick bodies can be found in Pick disease,
B. Dopamine depletion and depigmentation of which clinically resembles Alzheimer disease.
the substantia nigra
C. Increased number of trinucleotide repeats in
a gene on chromosome 4
D. Neurofibrillary tangles and amyloid plaques
in the cerebral cortex
E. Pick bodies, characterized by round
intracytoplasmic inclusions consisting of
neurofilaments
26 A 14-year-old girl presents with prolonged von Willebrand disease, a disorder transmitted by LESTER BRYAN CO MIDTERM 1
bleeding from wounds and minor trauma and autosomal modes of inheritance (both dominant and (TOP 10 - AUG 2015 EXAM - MARCH
severe menorrhagia. Family history reveals that recessive) is the most common hereditary bleeding MED BOARDS; 2016
her father also has prolonged bleeding from disorder. There are many variants, all marked by TOPNOTCH MD
wounds and minor trauma, as does her brother. either qualitative or quantitative deficiencies of vWF. FROM UST)
Which of the following is the most likely
mechanism of this patients disorder?
A. Absence of platelet glycoprotein IIb-IIIa
B. Antiplatelet antibodies reacting with platelet
surface glycoproteins
C. Deficiency of factor VIII
D. Deficiency of factor IX
E. Deficiency of vWF
27 A 42-year-old man is seen because of a long Cardiomyopathies are noninflammatory myocardial LESTER BRYAN CO MIDTERM 1
history of slowly developing congestive heart disorders that are not associated with coronary artery (TOP 10 - AUG 2015 EXAM - MARCH
failure. His blood pressure is normal. Coronary obstruction, hypertension, valvular disease, congenital MED BOARDS; 2016
artery angiography reveals no vascular disease. heart disease, or infectious disease. They are most TOPNOTCH MD
No heart murmurs are heard. The white blood often characterized by otherwise unexplained FROM UST)
cell count, differential, and erythrocyte ventricular dysfunction, such as cardiac failure,
sedimentation rate are normal. The most likely ventricular enlargement, or ventricular arrhythmias.
diagnosis is
A. carcinoid heart disease
B. cardiomyopathy
C. coarctation of the aorta
D. constrictive pericarditis
E. myocardial infarction

TOPNOTCH MEDICAL BOARD PREP PATHOLOGY SUPEREXAM Page 5 of 99


For inquiries visit www.topnotchboardprep.com.ph or email us at topnotchmedicalboardprep@gmail.com
TOPNOTCH MEDICAL BOARD PREP PATHOLOGY SUPEREXAM
For inquiries visit www.topnotchboardprep.com.ph or email us at topnotchmedicalboardprep@gmail.com
Item QUESTION EXPLANATION AUTHOR TOPNOTCH
# EXAM
28 A 28-year-old woman complains of fatigue, SIMILAR TO PREVIOUS BOARD EXAM LESTER BRYAN CO MIDTERM 1
dyspnea, and malaise. She also notes that her CONCEPT/PRINCIPLE. Paroxysmal nocturnal (TOP 10 - AUG 2015 EXAM - MARCH
urine has been reddishbrown, particularly with hemoglobinuria results in an acquired MED BOARDS; 2016
the first void of the morning. Subsequent intracorpuscular defect in the ability to synthesize GPI TOPNOTCH MD
studies confirm that she has paroxysmal anchors required for appropriate placement of FROM UST)
nocturnal hemoglobinuria. Which of the complement regulatory proteins on the surface of red
following best describes the defect leading to blood cells. Functional deficiency of such proteins as
this condition? CD55 and CD59 renders the cells sensitive to
A. Anti-intrinsic factor antibodies complement-mediated lysis. Anti-intrinsic factor
B. Deficiency of the intracellular structural antibodies are seen in pernicious anemia. Spectrin is
protein spectrin deficient in hereditary spherocytosis. Ineffective
C. Impaired synthesis of the cell-surface GPI erythropoiesis is seen in megaloblastic anemia due to
anchor folate or vitamin B12 deficiencies. Substitution of
D. Ineffective erythropoiesis valine for glutamic acid in the -globin gene underlies
E. Substitution of a valine for a glutamate the defect in sickle cell anemia.
residue in the -globin gene
29 A 45-year-old man presents with abdominal Berry aneurysms, which occur in 10% to 15% of LESTER BRYAN CO MIDTERM 1
pain and hypertension. On physical patients with adult polycystic kidney disease, are (TOP 10 - AUG 2015 EXAM - MARCH
examination, he is found to have an abdominal small saccular lesions that develop at the site of MED BOARDS; 2016
mass. Further workup confirms the diagnosis of congenital weakness of cerebral arteries, especially TOPNOTCH MD
adult polycystic kidney disease. Which of the those of the circle of Willis. Rupture of these FROM UST)
following vascular complications is associated aneurysms is the most common cause of subarachnoid
with this condition? hemorrhage. Arteriovenous fistulas are often
A. Arteriovenous fistula secondary to trauma. Dissecting aneurysm is
B. Atherosclerotic aneurysm associated with hypertension or with diseases
C. Berry aneurysm affecting the vascular media, most notably Marfan
D. Dissecting aneurysm syndrome. Syphilitic (luetic) aneurysm is associated
E. Luetic aneurysm with tertiary syphilis.
30 A 55-year-old man presents with a large, black- Malignant melanoma arises from melanocytes or LESTER BRYAN CO MIDTERM 1
colored, asymmetric skin lesion with ill-defined nevus cells, is most often associated with excessive (TOP 10 - AUG 2015 EXAM - MARCH
borders on his back. He reports a family history sun exposure, and is most common in fair-skinned MED BOARDS; 2016
of malignant melanoma. Which of the following persons. Of the clinical variants of malignant TOPNOTCH MD
clinical variants of malignant melanoma has the melanoma, nodular melanoma has the worst FROM UST)
poorest prognosis? prognosis. Malignant melanomas have a better
A. Lentigo maligna melanoma prognosis when characterized by a long period of
B. Superficial spreading melanoma radial (superficial) growth, as opposed to early
C. Nodular melanoma vertical growth (as in nodular melanoma).
D. Acral-lentiginous melanoma
E. uveal melanoma
31 When ordering academic attire for a recent A mosaic pattern of bone caused by increases in both LESTER BRYAN CO MIDTERM 1
graduation, a 65-year-old university professor osteoblastic and osteoclastic activity is characteristic (TOP 10 - AUG 2015 EXAM - MARCH
is surprised to find that his hat size has of Paget disease of bone (osteitis deformans). Serum MED BOARDS; 2016
increased. Shortly thereafter, in a routine alkaline phosphatase is markedly increased. Hearing TOPNOTCH MD
checkup, serum alkaline phosphatase activity is loss is common (from narrowing of the auditory FROM UST)
found to be markedly elevated. Serum calcium foramen and compression of the eighth cranial nerve),
and phosphorus are normal. Examination and an increase in hat size due to frontal bossing is
reveals enlargement of the skull with frontal often noted.
bossing and enlarged maxilla, and hearing loss
is evident. Which of the following abnormalities
is associated with the bone disorder suggested
by these findings?
A. Brown tumor of bone
B. Defective calcification of osteoid matrix
C. Mosaic pattern of bone
D. Polyostotic fibrous dysplasia with severe
deformity
E. Subperiosteal hemorrhage and osteoporosis
32 A 3-year-old African-American man with a Sickle cell anemia is the most common hereditary LESTER BRYAN CO MIDTERM 1
history since early childhood of severe anemia anemia in persons of African lineage. Leg ulcers and (TOP 10 - AUG 2015 EXAM - MARCH
requiring many transfusions has nonhealing leg recurring painful crises are characteristic. In sickle cell MED BOARDS; 2016
ulcers and recurrent periods of abdominal and anemia, in contrast to sickle cell trait, sickle cells are TOPNOTCH MD
chest pain. These signs and symptoms are most often seen on the peripheral blood smear. FROM UST)
likely to be associated with which of the
following laboratory abnormalities?
A. Decreased erythropoietin
B. Increased erythrocyte osmotic fragility
C. Schistocytes
D. Sickle cells on peripheral blood smear
E. Teardrop-shaped cells
33 For the past week, a 65-year-old woman has Pseudomembranous colitis is caused by overgrowth of LESTER BRYAN CO MIDTERM 1
been treated for a severe infection with broad- C. difficile. This organism produces exotoxin that (TOP 10 - AUG 2015 EXAM - MARCH
spectrum antibiotics, and she had recovered induces necrosis of the superficial mucosa, leading to MED BOARDS; 2016
well. Over the past day, however, she has pseudomembrane formation. The bacteria itself does TOPNOTCH MD
developed foul-smelling, voluminous, greenish, not invade the mucosa. This condition most often FROM UST)
watery diarrhea, as well as abdominal pain and occurs in patients with a history of broad-spectrum
fever. Which of the following is the mechanism antibiotic use, because elimination of normal
associated with this condition? intestinal flora promotes overgrowth of C. difficile.
A. Aggregation of bacterial colonies on the
lumen, forming pseudomembranes
B. Bacterial release of exotoxin, inducing
necrosis of the mucosa
C. Physical invasion of bacteria into the
superficial mucosa, leading to
pseudomembrane formation
D. Selective killing of C. difficile bacteria by
antibiotics

TOPNOTCH MEDICAL BOARD PREP PATHOLOGY SUPEREXAM Page 6 of 99


For inquiries visit www.topnotchboardprep.com.ph or email us at topnotchmedicalboardprep@gmail.com
TOPNOTCH MEDICAL BOARD PREP PATHOLOGY SUPEREXAM
For inquiries visit www.topnotchboardprep.com.ph or email us at topnotchmedicalboardprep@gmail.com
Item QUESTION EXPLANATION AUTHOR TOPNOTCH
# EXAM
E. Spread of the previous infection to the colon

34 A 50-year-old woman with a 20-year history of SIMILAR TO PREVIOUS BOARD EXAM LESTER BRYAN CO MIDTERM 1
type 2 diabetes mellitus presents with CONCEPT/PRINCIPLE. Diabetic nephropathy (TOP 10 - AUG 2015 EXAM - MARCH
proteinuria, hypoalbuminemia, edema, and manifests clinically as the nephrotic syndrome; MED BOARDS; 2016
hyperlipidemia. She has not monitored her however, this syndrome is compounded by renal TOPNOTCH MD
serum glucose levels over the past several failure and hypertension. Ultrastructural changes FROM UST)
years. What is the classic morphologic finding include a marked increase in the thickness of the
in this condition? glomerular basement membrane and mesangial
A. Crescentic formation in glomeruli on light accumulation of glycosylated basement membranelike
microscopy material. Light microscopy findings include diffuse
B. Intramembranous and epimembranous glomerulosclerosis (a diffuse increase in mesangial
immune complex deposits in the glomerular matrix) and nodular glomerulosclerosis (nodular
basement membrane on electron microscopy accumulations of mesangial matrix).
C. Nodular accumulations of mesangial matrix
on light microscopy
D. Sclerosis within capillary tufts that involves
only some glomeruli and only parts of affected
glomeruli on light microscopy
E. Wire-loop abnormalities from immune
complex deposits and thickening of the
glomerular basement membrane on light
microscopy
35 A 23-year-old woman presents with cervical SIMILAR TO PREVIOUS BOARD EXAM LESTER BRYAN CO MIDTERM 1
and mediastinal lymphadenopathy. Biopsy of a CONCEPT/PRINCIPLE. The diagnosis is Hodgkin (TOP 10 - AUG 2015 EXAM - MARCH
cervical lymph node reveals a nodular lymphoma, nodular sclerosing subtype. This form of MED BOARDS; 2016
appearance with fibrous bands, effacement of Hodgkin lymphoma differs from other forms of TOPNOTCH MD
the lymph node architecture, and numerous classical Hodgkin lymphoma in being the most FROM UST)
lacunar cells. Which of the following is common in young women, having a relatively
characteristic of this disorder? favorable clinical course, and having little association
A. Benign neoplasm with EBV infection. Lacunar cells are considered a
B. Frequent association with EBV infection Reed-Sternberg cell variant, and the diagnosis of NS
C. Most often a complication of human can be based on the finding of fibrous bands and
immunodeficiency virus infection lacunar cells.
D. Peak incidence in early childhood
E. Relatively favorable clinical course
36 A 15-year-old boy presents with a pathologic In young patients, bone malignancies showing LESTER BRYAN CO MIDTERM 1
fracture following a minor injury on the soccer prominent cartilaginous differentiation are almost (TOP 10 - AUG 2015 EXAM - MARCH
field. The area of fracture is surrounded by a assuredly chondroblastic osteosarcomas, rather than MED BOARDS; 2016
large tumor which shows marked chondrosarcomas. Conventional chondrosarcomas TOPNOTCH MD
pleomorphism, high mitotic activity, and occur almost exclusively in older patients. FROM UST)
extensive cartilaginous differentiation on
microscopy. The most likely diagnosis is
A. giant cell tumor
B. osteochondroma
C. chondrosarcoma
D. osteosarcoma
E. Ewing sarcoma
37 A 45-year-old woman presents to her primary Primary biliary cirrhosis is an autoimmune condition LESTER BRYAN CO MIDTERM 1
care physician with jaundice, pruritus, and that typically presents in middle-aged women. The (TOP 10 - AUG 2015 EXAM - MARCH
periocular and intradigital xanthomas. Her itching and hypercholesterolemia are secondary to MED BOARDS; 2016
laboratory results indicate a significantly severe obstructive jaundice. Leptospirosis is a TOPNOTCH MD
increased alkaline phosphatase as well as a condition caused by a treponemal bacterium that FROM UST)
positive test for antimitochondrial antibodies. results in jaundice, renal failure, and hemorrhagic
The most likely cause of her symptoms is phenomena. Macronodular cirrhosis is usually a result
A. leptospirosis of hepatitis B or hepatitis C infection. Primary
B. macronodular cirrhosis sclerosing cholangitis is associated with ulcerative
C. primary biliary cirrhosis colitis and with an increased incidence of
D. primary sclerosing cholangitis cholangiocarcinoma. Secondary biliary cirrhosis is
E. secondary biliary cirrhosis caused by extrahepatic biliary obstruction.
38 The chest radiograph of a 23-year-old medical Cavitation occurs only in secondary tuberculosis. Both LESTER BRYAN CO MIDTERM 1
student reveals a calcified cavitary pulmonary primary and secondary tuberculosis are characterized (TOP 10 - AUG 2015 EXAM - MARCH
lesion. The tuberculin test is positive, but by caseating granulomas, often with Langhans giant MED BOARDS; 2016
sputum smears and cultures are negative for cells, which heal by scarring and calcification. The skin TOPNOTCH MD
Mycobacterium tuberculosis. A presumptive test result for tuberculin sensitivity is positive in both FROM UST)
diagnosis of secondary tuberculosis is made. If forms.
further studies, including a biopsy, were
performed, which of the following
findings would justify the diagnosis of
secondary tuberculosis, as contrasted to
primary tuberculosis?
A. Calcification
B. Caseating granulomas
C. Cavitation
D. Langhans giant cells

TOPNOTCH MEDICAL BOARD PREP PATHOLOGY SUPEREXAM Page 7 of 99


For inquiries visit www.topnotchboardprep.com.ph or email us at topnotchmedicalboardprep@gmail.com
TOPNOTCH MEDICAL BOARD PREP PATHOLOGY SUPEREXAM
For inquiries visit www.topnotchboardprep.com.ph or email us at topnotchmedicalboardprep@gmail.com
Item QUESTION EXPLANATION AUTHOR TOPNOTCH
# EXAM
E. Positive tuberculin test result

39 A 40-year-old woman presents with a painless Approximately 80% to 90% of salivary gland tumors LESTER BRYAN CO MIDTERM 1
mass anterior to her left ear. The mass had originate in the parotid gland and, of these, (TOP 10 - AUG 2015 EXAM - MARCH
been slowly enlarging over the past year. The approximately 70% are pleomorphic adenomas. The MED BOARDS; 2016
mass is firm and nontender. Computed term mixed tumor properly applies to this benign TOPNOTCH MD
tomography and magnetic resonance imaging tumor, which often demonstrates myxoid and FROM UST)
reveal a well-circumscribed, homogeneous cartilage-like elements in addition to stellate or
mass within the left parotid gland. Biopsy fusiform epithelial cells. Complete surgical resection is
reveals anastomosing strands of stellate and difficult because of the tumors proximity to the facial
fusiform epithelial cells embedded in a myxoid nerve, and, thus, recurrence is frequent.
stroma. Which of the following is a
characteristic of the lesion?
A. It is also called papillary cystadenoma
lymphomatosum.
B. It is most often localized to the
submandibular gland.
C. It is the most common malignant salivary
gland tumor.
D. Recurrence often takes place after surgical
resection.
E. Surgical resection should not be performed,
because this condition is usually already
metastatic on diagnosis.
40 A 70-year-old man presents with severe bone The diagnosis is multiple (plasma cell) myeloma, a LESTER BRYAN CO MIDTERM 1
pain and frequent respiratory infections. Serum neoplastic proliferation of malignant plasma cells (TOP 10 - AUG 2015 EXAM - MARCH
protein electrophoresis demonstrates an M (mature B cells, not T cells). Death is often caused by MED BOARDS; 2016
protein spike in the gamma region. renal insufficiency caused by myeloma kidney. The TOPNOTCH MD
Radiographs of the skull, long bones, and spine average age of presentation is approximately 70 years FROM UST)
demonstrate multiple punched-out lesions, of age. IgM myeloma is very uncommon. Both the
and bone marrow aspiration demonstrates neoplastic cells and the serum protein spike are
large numbers of neoplastic plasma cells. Which monoclonal rather than polyclonal, and the
of the following statements is true of this monoclonal spike protein is most frequently an IgG or
disorder? an IgA.
A. Although this patient presents at 70 years of
age, the average age of presentation is 50 years
of age.
B. Renal insufficiency is a common cause of
death.
C. The M spike is most often an IgM.
D. The M spike is most often polyclonal in
nature.
E. This disorder is the most common T-cell
neoplasm.
41 Spongiotic dermatitis is the histologic pattern Contact dermatitis is a type of spongiotic dermatitis GEORGE MICHAEL MIDTERM 2
for which of the following dermatoses? with edema and perivascular lymphocytic infiltrate. SOSUAN (TOP 5 - EXAM - MARCH
A. Discoid lupus erythematosus AUG 2015 MED 2016
B. Contact dermatitis BOARDS; TOPNOTCH
C. Psoriasis MD FROM UST)
D. Erythema nodosum
E. Erythema multiforme

42 Oral lesions may be seen secondary to fungal Special histochemical stains (Grocott-Gomori GEORGE MICHAEL MIDTERM 2
pulmonary infections which spread by methenamine silver (GMS), or periodic acid-Schiff SOSUAN (TOP 5 - EXAM - MARCH
hematogenously or by direct inoculation of (PAS) highlight fungi in sections. AUG 2015 MED 2016
infected sputum. Which of the following maybe BOARDS; TOPNOTCH
used to stain these fungal elements MD FROM UST)
A. Gomori methenamine stain
B. PAS
C. Gram's stain
D. Both A and B
E. AOTA
43 A 72 y/o patient with diabetes complains of Malignant otitis externa is most commonly associated GEORGE MICHAEL MIDTERM 2
severe pain and bloody discharge of the right with Pseudomonas aeruginosa, which is a gram- SOSUAN (TOP 5 - EXAM - MARCH
ear. Which of the following would a Gram stain negative bacillus. AUG 2015 MED 2016
of material from surgical debridement be BOARDS; TOPNOTCH
expected to show? MD FROM UST)
A. Gram-positive cocci
B. Gram-negative cocci
C. Gram-positive bacilli
D. Gram-negative bacilli
E. Septate, branching hyphae

TOPNOTCH MEDICAL BOARD PREP PATHOLOGY SUPEREXAM Page 8 of 99


For inquiries visit www.topnotchboardprep.com.ph or email us at topnotchmedicalboardprep@gmail.com
TOPNOTCH MEDICAL BOARD PREP PATHOLOGY SUPEREXAM
For inquiries visit www.topnotchboardprep.com.ph or email us at topnotchmedicalboardprep@gmail.com
Item QUESTION EXPLANATION AUTHOR TOPNOTCH
# EXAM
44 Which of the following is incorrectly matched? All of them are correct GEORGE MICHAEL MIDTERM 2
A. Bronchopneumonia: Patchy multifocal SOSUAN (TOP 5 - EXAM - MARCH
consolidation AUG 2015 MED 2016
B. Lobar pneumonia: Lobar consolidation BOARDS; TOPNOTCH
C. Atypical pneumonia: Inflammation within MD FROM UST)
alveolar interstitium
D. AOTA
E. NOTA
45 Leading cause of blood transfusion related TRALI is currently the leading cause of transfusion GEORGE MICHAEL MIDTERM 2
mortality mortality. Donor antibodies to recipient leukocyte SOSUAN (TOP 5 - EXAM - MARCH
A. Transfusion related acute lung injury antigens activate complement and encourage AUG 2015 MED 2016
B. Graft versus host reaction granulocytes to aggregate within the pulmonary BOARDS; TOPNOTCH
C. Hemolytic transfusion reaction microvasculature. The diagnosis is based on clinical MD FROM UST)
D. Sepsis findings that are similar to ARDS but arise within 6
E. Anaphylaxis hours after transfusion. TRALI has a 510% mortality
rate, but with interim ventilatory support, most
symptoms resolve within
96 hours.
46 Paradoxical embolus is usually seen among In patients with cardiac septal defects, an embolus GEORGE MICHAEL MIDTERM 2
patients with: may detour into the left heart as a paradoxical SOSUAN (TOP 5 - EXAM - MARCH
A. Tetralogy of Fallot embolus to organs and the brain. AUG 2015 MED 2016
B. Transposition of great vessels BOARDS; TOPNOTCH
C. Cardiac septal defect MD FROM UST)
D. Tricuspid atresia
E. NOTA

47 A 13 y/o F girl with asthma seeks medical Charcot-Leyden crystals are a sign of eosinophilic GEORGE MICHAEL MIDTERM 2
attention at her pediatricians office degranulation, SOSUAN (TOP 5 - EXAM - MARCH
complaining of shortness of breath. What and Curschmann spirals are a sign of excess mucus in AUG 2015 MED 2016
would sputum cytology tests reveal? sputum of asthma patients. BOARDS; TOPNOTCH
A. Fungal hyphae MD FROM UST)
B. Acid-fast bacilli
C. Keratin pearls
D. Charcot-Leyden crystals
E. Atypical squamous epithelial cells
48 This type of gastritis is characterized by auto- Type A - Autoimmune gastritis (10%), autoantibodies GEORGE MICHAEL MIDTERM 2
antibodies to gastric parietal cells and intrinsic are made against parietal cells and intrinsic factor, SOSUAN (TOP 5 - EXAM - MARCH
factor. gland atrophy leads to pernicious anemia; Type B - AUG 2015 MED 2016
A. Type A Helicobacter pylori infection (90%), the most common BOARDS; TOPNOTCH
B. Type B cause of gastritis, Increased risk of peptic ulcers and MD FROM UST)
C. Type AB carcinoma
D. Type C
E. Type O

49 A 32 y/o F has progressive dyspnea, cough, and Polyarteritis nodosa typically does not involve the GEORGE MICHAEL MIDTERM 2
fever. A chest radiograph shows bilateral pulmonary vessels. SOSUAN (TOP 5 - EXAM - MARCH
infiltrates. She also notes dark brown urine. AUG 2015 MED 2016
Which of the following statements is FALSE? BOARDS; TOPNOTCH
A. Low levels of serum complement may be MD FROM UST)
seen
B. Blood cultures should be performed
C. Serum ANCA should be performed
D. The differential diagnosis includes systemic
lupus erythematosus
E. Polyarteritis nodosa is the most likely
diagnosis
50 The leather-bottle appearance of the gastric Linitis plastica refers to a thickened leather-bottle GEORGE MICHAEL MIDTERM 2
adenocarcinoma is characteristic of this appearance of the stomach caused by diffuse SOSUAN (TOP 5 - EXAM - MARCH
histologic type. infiltration of the gastric wall by a poorly AUG 2015 MED 2016
A. Intestinal differentiated signet-ring type of adenocarcinoma. BOARDS; TOPNOTCH
B. Diffuse MD FROM UST)
C. Nodular
D. Esophageal
E. Squamous

51 An elderly woman complains of fatigue, anemia, 10% of colon cancers occur in the rectum. Anal GEORGE MICHAEL MIDTERM 2
and bright red blood in stool. Which of the carcinoma is usually human papillomavirus related SOSUAN (TOP 5 - EXAM - MARCH
following is the MOST LIKELY diagnosis? and occurs in younger patients; endometriosis causing AUG 2015 MED 2016
A. Rectal adenocarcinoma rectal bleeding would not be expected in a BOARDS; TOPNOTCH
B. Anal squamous cell carcinoma postmenopausal patient. MD FROM UST)
C. Endometriosis
D. AOTA
E. NOTA

52 True of HBsAg: HBsAg (surface antigen) provides the first evidence of GEORGE MICHAEL MIDTERM 2
A. First evidence of infection infection and appears in the serum before symptoms. SOSUAN (TOP 5 - EXAM - MARCH
B. Appears in the serum after symptoms AUG 2015 MED 2016
C. Co-incide with the elevation of ALT BOARDS; TOPNOTCH
D. AOTA MD FROM UST)
E. Both A and C

TOPNOTCH MEDICAL BOARD PREP PATHOLOGY SUPEREXAM Page 9 of 99


For inquiries visit www.topnotchboardprep.com.ph or email us at topnotchmedicalboardprep@gmail.com
TOPNOTCH MEDICAL BOARD PREP PATHOLOGY SUPEREXAM
For inquiries visit www.topnotchboardprep.com.ph or email us at topnotchmedicalboardprep@gmail.com
Item QUESTION EXPLANATION AUTHOR TOPNOTCH
# EXAM
53 Hallmark of malignant transformation Anaplasia or lack of differentiation is the hallmark of GEORGE MICHAEL MIDTERM 2
A. Keratin pearls malignant transformation. SOSUAN (TOP 5 - EXAM - MARCH
B. Dedifferentiation AUG 2015 MED 2016
C. Lack of differentiation BOARDS; TOPNOTCH
D. Metastasis MD FROM UST)
E. Skip lesions

54 The leading causative agent of injection drug In hepatitis C, chronic disease occurs in 85% of GEORGE MICHAEL MIDTERM 2
use hepatitis: patients. The leading cause is injection drug use. SOSUAN (TOP 5 - EXAM - MARCH
A. Hepatitis A Cirrhosis will develop in 20% of patients within 20 AUG 2015 MED 2016
B. Hepatitis B years. BOARDS; TOPNOTCH
C. Hepatitis C MD FROM UST)
D. Hepatitis D
E. Both B and C

55 Hepatocellular carcinoma with the best Fibrolamellar variant of HCC may have a better GEORGE MICHAEL MIDTERM 2
prognosis prognosis than conventional HCC. It usually occurs in SOSUAN (TOP 5 - EXAM - MARCH
A. Hepatitis B variant young adults without a history of prior liver disease. It AUG 2015 MED 2016
B. Fibrolamellar variant is characterized by oncocytic-like hepatocytes BOARDS; TOPNOTCH
C. Scirrhous type (abundant intracellular mitochondria) infiltrating MD FROM UST)
D. Mucinous type fibrous stroma.
E. NOTA

56 This pathology is characterized by distinctive The decription stated is of a "tram track," GEORGE MICHAEL MIDTERM 2
combination of mesangial and endothelial characteristic of MPGN. SOSUAN (TOP 5 - EXAM - MARCH
proliferation, along AUG 2015 MED 2016
with thickening and duplication of the capillary BOARDS; TOPNOTCH
basement membrane MD FROM UST)
A. Amyloidosis
B. Diabetic nephropathy
C. Membranoproliferative glomerulonephritis
D. IgA nephropathy
E. Post-streptococcal glomerulonephritis
57 This pathological process is characterized by Malignant hypertension is a rapid disease results in GEORGE MICHAEL MIDTERM 2
acute vascular lesions include fibrinoid necrosis acute vascular lesions and parenchymal injury SOSUAN (TOP 5 - EXAM - MARCH
of renal arteries and referred AUG 2015 MED 2016
arterioles, with onion skin lesions to as malignant nephrosclerosis. BOARDS; TOPNOTCH
A. Renal artery stenosis MD FROM UST)
B. Malignant hypertension
C. Interstitial nephritis
D. Hypertensive nephrosclerosis
E. Vasculitis

58 Crescentic glomerulonephritis is most ANCAs (anti-neutrophil cytoplasmic antibodies) are GEORGE MICHAEL MIDTERM 2
associated with: associated with pauci-immune crescentic SOSUAN (TOP 5 - EXAM - MARCH
A. p-ANCA glomerulonephritis. HIV is associated with focal AUG 2015 MED 2016
B. HIV segmental glomerulosclerosis; cryoglobulin and C3 BOARDS; TOPNOTCH
C. Cytomegalovirus nephritic factor may be associated with different types MD FROM UST)
D. Cryoglobulin of membranoproliferative glomerulonephritis.
E. Nephritic factor Cytomegalovirus infection causes tubulointerstitial
nephritis and is seen in immunocompromised
patients.
59 Pretibial myxedema is characterized by: Pretibial myxedema is secondary to hyaluronic acid GEORGE MICHAEL MIDTERM 2
A. Hyaluronic acid deposition in the deposition and lymphocytic infiltrates within the SOSUAN (TOP 5 - EXAM - MARCH
subcutaneous tissue dermis. AUG 2015 MED 2016
B. Lymphocytic inflitrates in the dermis BOARDS; TOPNOTCH
C. Chondroitin sulfate deposition within the MD FROM UST)
dermis
D. Both A and B
E. AOTA
60 This is a disorder characterized by peripheral Refetoff syndrome is a disorder of peripheral GEORGE MICHAEL MIDTERM 2
resistance to circulating thyroid hormone resistance to circulating thyroid hormone. SOSUAN (TOP 5 - EXAM - MARCH
A. Refetoff syndrome AUG 2015 MED 2016
B. Zellweger syndrome BOARDS; TOPNOTCH
C. Reiter syndrome MD FROM UST)
D. Thyroid dysplasmocytic syndrome
E. NOTA

61 What is the interluekin that is vital for IL-8 is the major chemoatactic factor for neutrophils. JAN CHRISTIAN MIDTERM 3
neutrophil chemotaxis? IL-1 is mostly resposnsible for fever. IL-2 stimutales FELICIANO (TOP 2 - EXAM - MARCH
A. IL-1 grwoth of T cells. IL-6 stimulates acute phase protein AUG 2015 MED 2016
B. IL-2 production and IL-10 modulates the immune response BOARDS; TOPNOTCH
C. IL-6 MD FROM UST)
D. IL-8
E. IL-10

62 What cytokine mediates septic shock and plays SIMILAR TO PREVIOUS BOARD EXAM JAN CHRISTIAN MIDTERM 3
a role in cancer cachexia? CONCEPT/PRINCIPLE/ TNF alpha is believed to be the FELICIANO (TOP 2 - EXAM - MARCH
A. TNF alpha cytokine that mediates septic shock and is implicated AUG 2015 MED 2016
B. IL-1 in cachexia of malignancy. BOARDS; TOPNOTCH
C. Inteferon alpha MD FROM UST)
D. Inferterferon gamma
E. Lipopolysaccharide

TOPNOTCH MEDICAL BOARD PREP PATHOLOGY SUPEREXAM Page 10 of 99


For inquiries visit www.topnotchboardprep.com.ph or email us at topnotchmedicalboardprep@gmail.com
TOPNOTCH MEDICAL BOARD PREP PATHOLOGY SUPEREXAM
For inquiries visit www.topnotchboardprep.com.ph or email us at topnotchmedicalboardprep@gmail.com
Item QUESTION EXPLANATION AUTHOR TOPNOTCH
# EXAM
63 This is considered an irreversible histologic Nuclear pyknosis, karyorrhesis and karyolysis are JAN CHRISTIAN MIDTERM 3
manifestation of cellular injury considered irreversible changes of cell injury, The rest FELICIANO (TOP 2 - EXAM - MARCH
A. Cellular swelling of the choices are reversible. AUG 2015 MED 2016
B. Nuclear chromatin clumping BOARDS; TOPNOTCH
C. Nuclear pyknosis MD FROM UST)
D. Ribosomal detachment
E. Membrane blebbing

64 Which statement refers to dystrophic Dystrophic calcification is ca deposition in abnormal JAN CHRISTIAN MIDTERM 3
calcification rather than metastatic tissues usually due to necrosis. It is seen in TB, FELICIANO (TOP 2 - EXAM - MARCH
calcification? infarcts, thrombus, schistosomiasis, congenital CMV, AUG 2015 MED 2016
A. Diffuse and widespread toxoplasmosis and psamomma bodies. The rest of the BOARDS; TOPNOTCH
B. Ca deposition in normal tissues choices refers to metastatic calcification. MD FROM UST)
C. Patients are usually not normocalcemic
D. Occurs in patients on long term
hemodialysis
E. Associated with thrombus and infarcts
65 Which of the ff mechanisms regarding Diapedesis uses PECAM-1 not VCAM (utilized in tight JAN CHRISTIAN MIDTERM 3
leukocyte extravasatation is incorrectly paired? binding together with ICAM). All the other choices are FELICIANO (TOP 2 - EXAM - MARCH
A. Margination and rolling- Selectins correct AUG 2015 MED 2016
B. Margination and rolling- GlyCAM-1 BOARDS; TOPNOTCH
C. Tight-binding- ICAM-1 MD FROM UST)
D. Diapededis- VCAM-1
E. Migration- IL-8

66 Cigarette smoking is carcinogenic to the ff SIMILAR TO PREVIOUS BOARD EXAM JAN CHRISTIAN MIDTERM 3
organs EXCEPT? CONCEPT/PRINCIPLE. Cigarrete smoking is assocated FELICIANO (TOP 2 - EXAM - MARCH
A. Colon with maligancies in the ff organs: bladder, cervix, AUG 2015 MED 2016
B. Bladder esophagus, kidne, larynx, lung, pancreas. Colon and BOARDS; TOPNOTCH
C. Pancreas breast malignancies usually does not have smoking as MD FROM UST)
D. Kidney a risk factor/
E. Cervix

67 Which of the ff statements regarding neoplasia Benign neoplasms and well-differentiated carcinomas JAN CHRISTIAN MIDTERM 3
is INCORRECT? of endocrine glands frequently secrete hormones FELICIANO (TOP 2 - EXAM - MARCH
A. Tumor is said to be benign when its gross character tic of their origin. Well-differentiated AUG 2015 MED 2016
and microscopic appearances are considered squamous cell carcinomas of the epidermis synthesize BOARDS; TOPNOTCH
relatively innocent keratin, and well-differentiated hepatocellular MD FROM UST)
B. Environmental influences not genetic factors carcinomas elaborate bile. All other statements are
appear to be the dominant risk factors for most correct
cancers.
C. Hamartomas are disorganized but benign
masses composed of cells indigenous to the
involved site.
D. Benign neoplasms and poorly differentiated
carcinomas of endocrine glands frequently
secrete hormones characteristic of their origin
E. Once the tumor cells breach the basement
membrane, the tumor is said to be invasive.
68 Overall, what is the most commonly mutated Point mutations of RAS family genes constitute the JAN CHRISTIAN MIDTERM 3
proto-oncogene seen in approximately 15% to most common type of abnormality involving proto- FELICIANO (TOP 2 - EXAM - MARCH
20% of all human tumors? oncogenes in human tumors. Approximately 15% to AUG 2015 MED 2016
A. RET 20% of all human tumors express mutated RAS BOARDS; TOPNOTCH
B. BCL-2 proteins, but in some types of cancers the frequency of MD FROM UST)
C. RAS RAS mutations is much higher. p53 and Rb are tumor
D. p53 suprresor genes not proto-oncogene
E. Rb

69 Which of these statements regarding True enough, aortas of infants can exhibit fatty JAN CHRISTIAN MIDTERM 3
atherosclerosis is correct? streaks, and such lesions are present in virtually FELICIANO (TOP 2 - EXAM - MARCH
A. It literally means hardening of the arteries" all adolescents, even those without known risk AUG 2015 MED 2016
B. Fatt streaks are present in virtually all factors. Arteriosclerosis literally means hardening of BOARDS; TOPNOTCH
adolescents even without risk factors the arteries. Family history and not age is the most MD FROM UST)
C. Age is the most important independent risk important independent risk factor for atherosclerosis.
factor for atherosclerosis. lipid-filled macrophages are called foam cells. Collagen
D. Because the modified lipoproteins cannot be is the major structural component of the fibrous cap,
completely degraded, chronic ingestion leads to and accounts for its mechanical strength and stability.
the formation of lipid-filled macro- phages
called fatty streaks
E. Fibrinogen is the major structural
component of the fibrous cap, and accounts for
its mechanical strength and stability
70 Pertaining to the previous question, where is In descending order, the most extensively involved JAN CHRISTIAN MIDTERM 3
the most common site of atherosclerosis? vessels are the lower abdominal aorta, the coronary FELICIANO (TOP 2 - EXAM - MARCH
A. Abdominal aorta arteries, the popliteal arteries, the internal carotid AUG 2015 MED 2016
B. Coronorary artery arteries, and the vessels of the circle of Willis. BOARDS; TOPNOTCH
C. Internal carotid artery MD FROM UST)
D. Thoracic aorta
E. Popliteal artery

TOPNOTCH MEDICAL BOARD PREP PATHOLOGY SUPEREXAM Page 11 of 99


For inquiries visit www.topnotchboardprep.com.ph or email us at topnotchmedicalboardprep@gmail.com
TOPNOTCH MEDICAL BOARD PREP PATHOLOGY SUPEREXAM
For inquiries visit www.topnotchboardprep.com.ph or email us at topnotchmedicalboardprep@gmail.com
Item QUESTION EXPLANATION AUTHOR TOPNOTCH
# EXAM
71 A 70 yr old male had severe crushing chest pain SIMILAR TO PREVIOUS BOARD EXAM JAN CHRISTIAN MIDTERM 3
and died 3 hours later before he was brought to CONCEPT/PRINCIPLE. The gross and microscopic FELICIANO (TOP 2 - EXAM - MARCH
the ER. Cause of death is myocardial infarction. appearance of an infarct depends on the duration of AUG 2015 MED 2016
What do you expect to see on light microscopy survival of the patient following the MI. Light BOARDS; TOPNOTCH
on time of death? microcopy findings: Less than 30 mins- None; 30 MD FROM UST)
A. Early coagulation necrosis mins-4 hours- Usually none but variable waviness of
B. Marginal contraction band necrosis fibers at border; 4-12 hours- Early coag necrosis; 12-
C. Myocyte hypereosinophilia 24 hrs- pyknosis; myocyte hypereosniphilia and
D. Waviness of fibers marginal contraction band necrosis
E. Pyknosis of nuclei
72 A 50 yr old patient had routine checkup and the The case points to possible mitral valve prolapse JAN CHRISTIAN MIDTERM 3
only finding is a midsystolic click on wherein the underlying pathology is myxomatous FELICIANO (TOP 2 - EXAM - MARCH
auscultation. You are suspecting a valvular degeneration. The chordae tendineae become AUG 2015 MED 2016
defect. Which of the following pathologic elongated and can rupture to produce acute valvular BOARDS; TOPNOTCH
changes is most likely present in the valve? incompetence. MD FROM UST)
A. Destructive vegetations
B Dystrophic calcification
C Fibrinoid necrosis
D Myxomatous degeneration
E Rheumatic fibrosis
73 A patient came to you complaining of chronic Answer is A. Typo error. Choice should have been JAN CHRISTIAN MIDTERM 3
back pain. Radiagraphic exams shows multple Serum IgA. This is highly suggestive of multitple FELICIANO (TOP 2 - EXAM - MARCH
punched out lytic bone lesions. Which lab myeloma. IgG or IgA M proteins are almaot always AUG 2015 MED 2016
abnomrality is most likely? found in multiple myeloma. Additionaly lab criteria is BOARDS; TOPNOTCH
A. Serum IgM M protein is hypercalcemia, monocloonal urinary light chains MD FROM UST)
B. Hypocalemia and rolouex formation.
C. Increased serum alkaline phpsphatase
D. Marked splenomegaly
E. Polyclonal urinary light chains
74 This pneumoconioses is seen as Eggshell calcification of hilar lymph nodes is JAN CHRISTIAN MIDTERM 3
radiographically as an eggshell calcification of suggestive of silicosis. FELICIANO (TOP 2 - EXAM - MARCH
hilar lymph nodes described as stained glass in AUG 2015 MED 2016
appearance? BOARDS; TOPNOTCH
A. Asbestosis MD FROM UST)
B. Berylloisis
C. Coal worker's pneumoconioses
D. Silicosis
E. Hypersenstivity pneumonitis
75 This statement is correct regarding esophageal Some serotypes of Helicobacter pylori are associated JAN CHRISTIAN MIDTERM 3
cancer. with decreased risk of esophageal adenocarcinoma, FELICIANO (TOP 2 - EXAM - MARCH
A. Adenocarcinoma is more common because they cause gastric atrophy, which in turn AUG 2015 MED 2016
worldwide leads to reduced acid secretion and reflux, and BOARDS; TOPNOTCH
B. Half of squamous cell carcinomas occur in reduced incidence of Barrett esophagus. Squamous CA MD FROM UST)
the lower third of the esophagus is most common and occurs at middle third of
C. Reduced rates of Helicobacter pylori esophagus. AdenoCA produces mucin and is often the
infection may be a factor in the increasing intestinal type morphology. It is 7 times more
incidence of esophageal adenocarcinoma common in males than females.
D. Squamous cell CA most commonly produce
mucin and form glands often with intestinal
type morphology
E. Esophageal adenocarcinoma occurs most
frequently in Caucasians and is equally
distributed among genders
76 On endoscopy of a 14 year old boy complaining Grossly, the polyps are large and pedunculated with a JAN CHRISTIAN MIDTERM 3
of bleeding per rectum, a colonic mass was lobulated contour. Histologic examination FELICIANO (TOP 2 - EXAM - MARCH
seen. Histologic examination demonstrates a demonstrates a characteristic arborizing network of AUG 2015 MED 2016
characteristic arborizing network of connective connective tissue, smooth muscle, lamina propria, and BOARDS; TOPNOTCH
tissue, smooth muscle, lamina propria, and glands lined by normal-appearing intestinal MD FROM UST)
glands lined by normal-appearing intestinal epithelium. The arborization and presence of smooth
epithelium. What is the diagnosis? muscle intermixed with lamina propria are helpful in
A. Familial adenomatous polyposis distinguishing polyps of Peutz-Jeghers syndrome from
B. Peutz-Jeghers syndrome juvenile polyps.
C. Juvenile polyposis
D. Lynch syndrome
E. Colon Adenocarcinoma
77 What is the most important characteristic of Size is the most important characteristic that JAN CHRISTIAN MIDTERM 3
adenomatous polyps that best correlates with correlates with risk of malignancy. Although villous FELICIANO (TOP 2 - EXAM - MARCH
risk of malignancy? adenomas contain foci of invasion more frequently AUG 2015 MED 2016
A. Morphology (tubular or villous) than tubular adenomas, villous architecture alone BOARDS; TOPNOTCH
B. Degree of differentiation does not increase cancer risk when polyp size is MD FROM UST)
C. Size considered.
D. Presence of stalk (pedunculated or sessile)
E. APC proto-oncogene mutation

78 A patient is suspected of having alcholic liver SIMILAR TO PREVIOUS BOARD EXAM JAN CHRISTIAN MIDTERM 3
disease possible alcohol hepatitis. You expect to CONCEPT/PRINCIPLE. Do not rely on buzzwords. FELICIANO (TOP 2 - EXAM - MARCH
see what finding in liver biopsy? Understand the morhpology. Choice a refers to AUG 2015 MED 2016
A. Clumped, amorphous, eosinophilic material Mallory bodies and is characertistic of Alcoholic BOARDS; TOPNOTCH
in ballooned hepatocytes made up of tangled hepatitis. Choice B refrs to autoimmune hepatitis. MD FROM UST)
skeins of intermediate filaments Choice C refers to chronic HepB infection. Choice D
B. Plasma cell predominance in the refers to cholestatis. Choice E refers to Primary
mononuclear inflammatory infiltrates sclerosing cholangitis
C. Ground-glass hepatocytes
D. Extensive feathery degeneration of
periportal hepatocytes
E. Circumferential onion skin fibrosis around
TOPNOTCH MEDICAL BOARD PREP PATHOLOGY SUPEREXAM Page 12 of 99
For inquiries visit www.topnotchboardprep.com.ph or email us at topnotchmedicalboardprep@gmail.com
TOPNOTCH MEDICAL BOARD PREP PATHOLOGY SUPEREXAM
For inquiries visit www.topnotchboardprep.com.ph or email us at topnotchmedicalboardprep@gmail.com
Item QUESTION EXPLANATION AUTHOR TOPNOTCH
# EXAM
an increasingly atrophic duct lumen

79 A child was brought to your clinic complaining SIMILAR TO PREVIOUS BOARD EXAM JAN CHRISTIAN MIDTERM 3
of hematuria. He had sore throat 5 days prior to CONCEPT/PRINCIPLE.The case most likely points to FELICIANO (TOP 2 - EXAM - MARCH
consult. What do you expect to find on electron IgA nephropathy. PSGN usually appears 1 to 4 weeks AUG 2015 MED 2016
microscopy? after a streptococcal infection of the pharynx or skin BOARDS; TOPNOTCH
A. discrete, amorphous, electron-dense deposits (impetigo). Choice A refers to PSGN, Choice B is MD FROM UST)
on the epithelial side of the membrane, often miniman change disease. Choice C refers to MPGN
having the appearance of humps Type 1. Choice E is RPGN
B. uniform and diffuse effacement of foot
processes
C. double contour or tram-track appearance
D. presence of electron-dense deposits
predominantly in the mesangium
E. proliferation of parietal cells forming
crescents
80 What is the microscopic finding in diabetic SIMILAR TO PREVIOUS BOARD EXAM JAN CHRISTIAN MIDTERM 3
glomerulonephropathy? CONCEPT/PRINCIPLE. The most important FELICIANO (TOP 2 - EXAM - MARCH
A. Mesangial expansion glomerular lesions are capillary basement membrane AUG 2015 MED 2016
B. GBM thickening thickening, diffuse mesangial sclerosis, and nodular BOARDS; TOPNOTCH
C. Eosinophilic nodular glomerulosclerosiss glomerulosclerosis aka Kimmelstiel-Wilson disease. MD FROM UST)
D. Renal atherosclerosis Renal atherosclerosis and arteriolosclerosis constitute
E. All of the above part of the macrovascular disease in diabetics.

81 1. A 58 year old male presented with left sided liquefactive necrosis is characterized by digestion of ANDREW TIU (TOP 1 FINAL EXAM -
weakness associated with headache and dead cells resulting in transmoration of tissue into a - AUG 2015 MED MARCH 2016
vomiting 4 hours prior to admission. Patient is liquid viscous mass. for unknown reasons, hypoxic BOARDS; TOPNOTCH
a known hypertensive and currently takes death of cells within CNS often manifests as such. It is MD FROM CIM)
Losartan. What is the most probable pattern of also seen in bacterial and fungal infections.
tissue necrosis seen in this patient? Coagulative necrosis - architecture of dead tissues is
a. gangrenous necrosis preserved for a span of at least some days.
b. coagulative necrosis Gangrenous necrosis - usually applied to a limb that
c. liquefactive necrosis has lost its blood supply and has undergone typically
d. fat necrosis coagulative necrosis involving multiple tissue planes.
e. none of the above Fat necrosis refers to focal areas of fat destruction
from release of activated pancreatic lipases into
pancreas and peritoneal cavity.
(robbins 8th edition p.16)
82 2. A 20 year old male was bitten by a red ant on contraction of endothelial cells resulting in increased ANDREW TIU (TOP 1 FINAL EXAM -
the dorsum of his foot. After 30 minutes, he endothelial spaces is the most common mechanism of - AUG 2015 MED MARCH 2016
noticed his foot was swollen, erythematous, and vascular leakage and is elicited by histamine, BOARDS; TOPNOTCH
itchy. What is the most likely mechanism for the bradykinin, leukotrienes, substance P, etc. It is called MD FROM CIM)
symptoms? the immediate transient response because it occurs
a. increased transcytosis rapidly after exposure to the mediator. (Robbins 8th
b. endothelial injury edition p. 47)
c. retraction of endothelial cells
d. leukocyte mediated vascular injury
e. none of the above
83 3. After 1 day, patient noted increased pruritus, contraction of endothelial cells resulting in increased ANDREW TIU (TOP 1 FINAL EXAM -
swelling, and erythema over the dorsum of his endothelial spaces is the most common mechanism of - AUG 2015 MED MARCH 2016
foot after repeatedly scratching it. What is now vascular leakage and is elicited by histamine, BOARDS; TOPNOTCH
the most likely mechanism for the symptoms? bradykinin, leukotrienes, substance P, etc. It is called MD FROM CIM)
a. increased transcytosis the immediate transient response because it occurs
b. endothelial injury rapidly after exposure to the mediator.
c. retraction of endothelial cells (Robbins 8th edition p. 47)
d. leukocyte mediated vascular injury
e. none of the above
84 4. A 28 year old female, CRB, presents 1 month rim or peripheral staining pattern are most indicative ANDREW TIU (TOP 1 FINAL EXAM -
history of easy fatigability, knee pain, mouth of antibodies to dsDNA. Homogeneous or diffuse - AUG 2015 MED MARCH 2016
sores, and tea colored urine. Past medical usually reflects antibodies to chromatin, histones, and BOARDS; TOPNOTCH
history and family history was unremarkable. occasionally dsDNA. Speckled pattern is least specific MD FROM CIM)
On workup, the physician noted a positive ANA and is the most commonly observed. This includes Sm
and anti - dsDNA. Which of the following antigen, RNP, SS-A, SS-B.
patterns of nuclear fluorescence most likely (RObbins 8th edition p. 214)
suggests the type of antibody present in the
patient?
a. homogeneous
b. rim
c. speckled
d. nucleolar
e. none of the above

TOPNOTCH MEDICAL BOARD PREP PATHOLOGY SUPEREXAM Page 13 of 99


For inquiries visit www.topnotchboardprep.com.ph or email us at topnotchmedicalboardprep@gmail.com
TOPNOTCH MEDICAL BOARD PREP PATHOLOGY SUPEREXAM
For inquiries visit www.topnotchboardprep.com.ph or email us at topnotchmedicalboardprep@gmail.com
Item QUESTION EXPLANATION AUTHOR TOPNOTCH
# EXAM
85 5. Patient CRB was then started on steroids. robbins 8th edition p. 213 ANDREW TIU (TOP 1 FINAL EXAM -
Creatinine was noted to be slightly elevated on - AUG 2015 MED MARCH 2016
follow up. Which of the following is the BOARDS; TOPNOTCH
hallmark of her disease? MD FROM CIM)
a. malar rash
b. glomerulonephritis
c. production of autoantibodies
d. normocytic normochromic anemia
e. none of the above
86 6. A 2 year old female presented with 12 week robbins 8th edition p. 249 ANDREW TIU (TOP 1 FINAL EXAM -
history of recurrent, fever, cough, 15% weight - AUG 2015 MED MARCH 2016
loss, rash, and generalized lymphadenopathy. BOARDS; TOPNOTCH
Patient was started with antibiotics without MD FROM CIM)
relief of symptoms. Biopsy of the lymph node
was done which showed follicles depleted of
cells and the organized network of follicular
dendritic cells is disrupted. What is the most
likely diagnosis?
a. hodgkin's lymphoma
b. AIDS
c. ALL
d. Miliary TB
e. none of the above
87 7. A 56 year old farmer comes to you with benzene - leukemia, hodgkins lymphoma (principal ANDREW TIU (TOP 1 FINAL EXAM -
complaints of a painful chronic skin ulcer. component of light oil); beryllium - lung cancer - AUG 2015 MED MARCH 2016
Biopsy was done which revealed squamous cell (missile fuel and space vehicles); chromium - lung BOARDS; TOPNOTCH
carcinoma. On further workup, patient also has cancer (metal alloys, paints, pigments, preservatives) MD FROM CIM)
pulmonary nodules on the right middle lobe Robbins 8th edition p. 274
and left upper lobe. Which of the following
agents is most likely responsible?
a. benzene
b. beryllium
c. chromium
d. arsenic
e. none of the above
88 8. JCT, 5 year old child presents with fever, neck Mumps encephalitis causes perivenous demyelination ANDREW TIU (TOP 1 FINAL EXAM -
mass, and decreased level of consciousness. and perivascular mononuclear cuffing. Aseptic - AUG 2015 MED MARCH 2016
Past medical history includes pneumonia at 8 meningitis is the most common extrasalivary gland BOARDS; TOPNOTCH
months old and German measles at 2 years old. complication of mumps infection occuring in 10% of MD FROM CIM)
Biopsy of neck mass showed a edematous cases.
interstitium diffusely infiltrated with Robbins 8th edition p. 250
macrophages, lymphocytes, and plasma cells.
Which of the following is the most likely
etiologic agent?
a. SSPE
b. mumps
c. polio
d. EBV
e. none of the above
89 9. A 72 year old male chronic smoker presented Components of cigarette smoke particularly ANDREW TIU (TOP 1 FINAL EXAM -
with abdominal pain and bone pains at night. polycyclic hydrocarbons and nitrosamines are potent - AUG 2015 MED MARCH 2016
Serum calcium was noted to be elevated. Which carcinogens in animals and likely to be directly BOARDS; TOPNOTCH
of the following is the organ specific carcinogen involved in the development of lung cancer in humans. MD FROM CIM)
in tobacco smoke? Robbins 8th edition p. 411-412
a. tar
b. formaldehyde
c. polycyclic aromatic hydrocarbons
d. nicotine
e. none of the above
90 10. A previously healthy neonate presented Robbins 8th edition p.456 ANDREW TIU (TOP 1 FINAL EXAM -
with tachypnea, retractions, and cyanosis. CRP - AUG 2015 MED MARCH 2016
was elevated. What is the most likely diagnosis? BOARDS; TOPNOTCH
a. RDS type I MD FROM CIM)
b. RDS type II
c. ARDS
d. BPD
e. none of the above
91 11. Which of the following refers to a plaque Atherosclerotic plaques have 3 principal components: ANDREW TIU (TOP 1 FINAL EXAM -
witha superficial fibrous cap composed of 1) cells 2) ECM 3)intracellular and extracellular lipid. - AUG 2015 MED MARCH 2016
smooth muscle cells and relaively dense Robbins 8th edition p. 502 BOARDS; TOPNOTCH
collagen with a lipid core? MD FROM CIM)
a. atherosclerotic plaque
b. fatty streak
c. all of the above
d. none of the above
e. none of the above
92 12. A 58 year old retired Chinese teacher came In acute cor pulmonale, there is marked dilation of ANDREW TIU (TOP 1 FINAL EXAM -
in for multiple injuries secondary to fall in the right ventricle without hypertrophy. The rest of the - AUG 2015 MED MARCH 2016
bathroom. X ray was done and showed a choices refer to chronic cor pulmonale. BOARDS; TOPNOTCH
femoral fracture. ORIF was done and 48 hours Robbins 8th edition p. 560 MD FROM CIM)
post -op, patient was noted to be dyspneic,
tachycardic with O2 sats 68%. Despite
resuscitative efforts, patient died. What is the
most likely autopsy findings of the heart?
a. thickened right ventricular wall
b. dilated right ventricular wall
TOPNOTCH MEDICAL BOARD PREP PATHOLOGY SUPEREXAM Page 14 of 99
For inquiries visit www.topnotchboardprep.com.ph or email us at topnotchmedicalboardprep@gmail.com
TOPNOTCH MEDICAL BOARD PREP PATHOLOGY SUPEREXAM
For inquiries visit www.topnotchboardprep.com.ph or email us at topnotchmedicalboardprep@gmail.com
Item QUESTION EXPLANATION AUTHOR TOPNOTCH
# EXAM
c. disappareance of fat in the ventricular wall
d. regurgitation and fibrous thickening of
tricuspid valve
e. none of the above

93 13. An 8 year old male came for complaints of B - RHD, C - NBTE, D - Libmann sacks endocarditis ANDREW TIU (TOP 1 FINAL EXAM -
exertional dyspnea, recurrent high grade fever, robbins 8th edition p. 567 - AUG 2015 MED MARCH 2016
subcutaneous nodules in the pulps of the digits, BOARDS; TOPNOTCH
and tea colored urine. A year ago, patient was MD FROM CIM)
noted to have recurrent sore throat however no
consult was done and no medications were
taken. Which of the following is the hallmark
seen in this case?
a. friable bulky destructive vegetations
b. small warty vegetations along the lines of
closure
c. small bland vegetations along the lines of
closure
d. medium sized vegetations on either sides of
the valve leaflets
e. none of the above
94 14. One week prior to consult, an 18 year old Nodes involved in acute lymphadenitis are enlarged ANDREW TIU (TOP 1 FINAL EXAM -
nursing student came in for complaints of and painful. When pyogenic organisms are the cause, - AUG 2015 MED MARCH 2016
impacted wisdom tooth on the right side of the the centers of the follicles may undergo necrosis. BOARDS; TOPNOTCH
mandible. After 24 hours, a painful neck mass Choices A, B, and D refer to chronic nonspecific MD FROM CIM)
was noted. Which of the following morphology lymphadenitis.
of the neck mass may undergo necrosis and pus (robbins 8th edition p.595)
formation?
a. follicular hyperplasia
b. paracortical hyperplasia
c. acute lymphadenitis
d. reticular hyperplasia
e. none of the above
95 15. What stage of inflammatory response of the It is characterized by vascular engorgement, intra- ANDREW TIU (TOP 1 FINAL EXAM -
lung is described when the lung is heavy, boggy, alveolar fluid with few neutrophils and often the - AUG 2015 MED MARCH 2016
and red? presence of numerous bacteria. In red hepatization, BOARDS; TOPNOTCH
a. congestion the lobe now appears red, firm, and airless with a liver MD FROM CIM)
b. red hepatization - like consistency.
c. gray hepatization (robbins 8th edition p. 713)
d. resolution
e. none of the above
96 16. A 13 year old female presented with A - goodpasture's syndrome; B - MPGN type 1; D - ANDREW TIU (TOP 1 FINAL EXAM -
headache and decreased urine output after a 4 MPGN type 2; E - IgA nephropathy - AUG 2015 MED MARCH 2016
week history of skin infection. Which of the Robbins 8th edition p.921 BOARDS; TOPNOTCH
following is the most likely morphology of the MD FROM CIM)
kidney?
a. linear GBM fluorescence for Ig and
complement
b. discrete subendothelial electron dense
deposits
c. granular deposits of IgG, IgM, and C3 in the
mesangium and along the GBM
d. GBM transformed into an irregular ribbon
like, electron dense structure
e. mesangial deposition of IgA
97 17. An 17 year old female presented with B - uniform diffuse thickening of the glomerular ANDREW TIU (TOP 1 FINAL EXAM -
elevated blood pressure, tea colored urine, and capillary wall - AUG 2015 MED MARCH 2016
pallor. Kidney biopsy was done which revealed C - glomeruli show mesangial widening and BOARDS; TOPNOTCH
glomeruli being large, hypercellular, and having endocapillary proliferation MD FROM CIM)
an accentuated "lobular appearance". Which of D - crescents which are proliferation of parietal cells
the following is the most likely diagnosis? and migration of monocytes and macrophages into the
a. membranoproliferative glomerulonephritis urinary space
b. membranous nephropathy E - collapse of capillary loops, increase in matrix, and
c. IgA nephropathy segmental deposition of plasma proteins aong
d. rapid crescentic glomerulonephritis capillary wall (hyalinosis)
e. focal segmental glomerulosclerosis Robbins 8th edition p. 929
98 18. A 63 year old female, ECT, came in for Robbins 8th edition p. 945 ANDREW TIU (TOP 1 FINAL EXAM -
complaints of bilateral knee pain. ECT has been - AUG 2015 MED MARCH 2016
self medicating Ibuprofen for 5 years. For the BOARDS; TOPNOTCH
past week, ECT has been having recurrent MD FROM CIM)
headaches, vague abdominal pain, and easy
fatigability. Which of the following occurs first
in renal damage in analgesic nephropathy?
a. cortical tubulointerstitial nephritis
b. papillary necrosis
c. ischemic kidney injury
d. toxic kidney injury
e. none of the above

TOPNOTCH MEDICAL BOARD PREP PATHOLOGY SUPEREXAM Page 15 of 99


For inquiries visit www.topnotchboardprep.com.ph or email us at topnotchmedicalboardprep@gmail.com
TOPNOTCH MEDICAL BOARD PREP PATHOLOGY SUPEREXAM
For inquiries visit www.topnotchboardprep.com.ph or email us at topnotchmedicalboardprep@gmail.com
Item QUESTION EXPLANATION AUTHOR TOPNOTCH
# EXAM
99 19. Which of the following nipple discharges robbins 8th edition p. 1068 ANDREW TIU (TOP 1 FINAL EXAM -
would most likely signify malignancy? - AUG 2015 MED MARCH 2016
a. bloody BOARDS; TOPNOTCH
b. serous MD FROM CIM)
c. milky
d. both a and b
e. none of the above

100 20. A 14 year old female presented with a Yes during our exam, it was female and not male but ANDREW TIU (TOP 1 FINAL EXAM -
breast mass. Biopsy was done which showed the description was really referring to gynecomastia. - AUG 2015 MED MARCH 2016
dense collagenous connective tissue and A - delicate cellular, and often myxoid stroma which BOARDS; TOPNOTCH
marked micropapillary epithelial hyperplasia of resembles normal intralobular stroma. B - MD FROM CIM)
the duct lining. Which of the following is the nonproliferative changes which includes cysts,
most likely diagnosis? fibrosis, and adenosis. D - bulbous protrusions due to
a. fibroadenoma presence of nodules of proliferating stroma covered
b. fibrocystic disease by epithelium. E - multiple branching fibrovascular
c. gynecomastia cores each having a connective tissue axis lined by
d. phylloides tumor luminal and myoepithelial cells
e. Papilloma Robbins 8th edition p. 1093
101 Characteristic cytologic feature of medullary Page 73 of Topnotch Handout. Medullary thyroid ANGELA PAULINE P. DIAGNOSTIC
thyroid cancer: carcinoma has polygonal to spindle shaped cells, CALIMAG-LOYOLA EXAM - AUG
A. Orphan annie acellular amyloid deposits, and C-cell hyperplasia. (TOP 8 - FEB 2015 2015
B. Vacuolated nucleus MED BOARDS;
C. Amyloid stroma TOPNOTCH MD
D. Ground glass FROM UST)
E. Pleomorphic giant cells

102 Which of the following is a calcitonin secreting Page 73 of Topnotch Handout. Medully thyroid ANGELA PAULINE P. DIAGNOSTIC
tumor of the thyroid gland? carcinoma is a neuroendocrine neoplasma derived CALIMAG-LOYOLA EXAM - AUG
A. Papillary CA from the parafollicular cells or c-cells which secrete (TOP 8 - FEB 2015 2015
B. Follicular CA calcitonin and is important in diagnosis. MED BOARDS;
C. Medullary CA TOPNOTCH MD
D. Hurthle cell CA FROM UST)
E. Anaplastic CA

103 23y/o female consults because of bipedal Page 125 of Topnotch Handout. The most common ANGELA PAULINE P. DIAGNOSTIC
edema & passing out of tea colored urine. On and severe type of Lupus nephritis is Type IV: Diffuse CALIMAG-LOYOLA EXAM - AUG
PE, her BP is 160/100. she has +2 bipedal proliferative GN which has a wire-loop capillary (TOP 8 - FEB 2015 2015
edema. The urinalysis showed +4 protein, 50- appearance. MED BOARDS;
60RBC/hpf, 0-1 WBC/hpf and occasional red TOPNOTCH MD
cell casts. Serum creatinine is elevated 2mg/dL FROM UST)
from a previously normal level of 0.9 mg/dL 1
week ago. If this is a case of SLE, which of the
following is the most common and severe
histopathologic type?
A. Mesangial GN
B. Focal proliferative GN
C. Membranous GN
D. Difffuse proliferative GN
E. Cresenteric GN
104 A 40 y/o female was found to have a diastolic Page 112 of hand out. Mitral stenosis causes an ANGELA PAULINE P. DIAGNOSTIC
rumble at the apex. The chest x-ray showed increase in LA size due to restriction of blood flow CALIMAG-LOYOLA EXAM - AUG
uplifting of the left main stem bronchus, from the left atrium to the left ventricle as a result of a (TOP 8 - FEB 2015 2015
retrosternal fullness, and dilated main narrowed mitral passage. Murmurs associated with MED BOARDS;
pulmonary artery. What is the most likely MS are opening snap, accentuated S1, and diastolic TOPNOTCH MD
diagnosis? A. rumble. Plain film chest xray findings are FROM UST)
Mitral Stenosis cardiomegaly, double right heart border (enlarged left
B. Mitral Regurgitation atrium and normal right atrium), prominent left atrial
C. Aortic Stenosis appendage and splaying of the subcarinal angle (>120
D. Pulmonic Stenosis degrees).
E. Aortic Regurgitation
105 What is the expected thyroid function test in Page 71 of Topnotch Handout. Primary ANGELA PAULINE P. DIAGNOSTIC
primary hyperthyroidism? hyperthyroidism is the term used when the pathology CALIMAG-LOYOLA EXAM - AUG
A. decreased TSH, increased T4
is within the thyroid gland. Secondary (TOP 8 - FEB 2015 2015
B. decreased TSH, decreased T4
hyperthyroidism is the term used when the thyroid MED BOARDS;
C. increased TSH, increased T4
gland is stimulated by excessive thyroid-stimulating TOPNOTCH MD
D. increased TSH, decreased T4 hormone (TSH) in the circulation.The diagnosis of FROM UST)
E. None of the above hyperthyroidism is confirmed by blood tests that
show a decreased thyroid-stimulating hormone (TSH)
level and elevated T4 and T3 levels. A low TSH level
typically indicates that the pituitary gland is being
inhibited or "instructed" by the brain to cut back on
stimulating the thyroid gland, having sensed increased
levels of T4 and/or T3 in the blood.
106 What can generally differentiate between Page 115of Topnotch Handout. Asthma and COPD are ANGELA PAULINE P. DIAGNOSTIC
asthma and COPD in spirometry? both obstructive lung diseases. On spirometry FEV1, CALIMAG-LOYOLA EXAM - AUG
a. FEV1 FVC and FEV1/FVC are decreased in both how ever (TOP 8 - FEB 2015 2015
b. reversibility asthma is a reversible condition. MED BOARDS;
c. PEF TOPNOTCH MD
d. TLC FROM UST)
E. FVC

TOPNOTCH MEDICAL BOARD PREP PATHOLOGY SUPEREXAM Page 16 of 99


For inquiries visit www.topnotchboardprep.com.ph or email us at topnotchmedicalboardprep@gmail.com
TOPNOTCH MEDICAL BOARD PREP PATHOLOGY SUPEREXAM
For inquiries visit www.topnotchboardprep.com.ph or email us at topnotchmedicalboardprep@gmail.com
Item QUESTION EXPLANATION AUTHOR TOPNOTCH
# EXAM
107 Ransons criteria is used in acute panceatitis Page 101 of Topnotch Handout. Ranson's criteria is a ANGELA PAULINE P. DIAGNOSTIC
for: criteria for prognostication on admission and for the CALIMAG-LOYOLA EXAM - AUG
A. Diagnosis
first 48 hours. (TOP 8 - FEB 2015 2015
B. Monitor abscess formation MED BOARDS;
C. Predict possible etiology TOPNOTCH MD
D. Decide surgical intervention FROM UST)
E. Prognostication

108 Which collagen type is commonly found in the Page 8 of Topnotch Handout. Epidermolysis bullosa is ANGELA PAULINE P. DIAGNOSTIC
dermoepidermal junction and is usually caused by genetic defects (or mutations) within the CALIMAG-LOYOLA EXAM - AUG
defective in Epidermolysis bullosa: A. human COL7A1 gene encoding the protein type VII (TOP 8 - FEB 2015 2015
Type I collagen (collagen VII). MED BOARDS;
B. Type 3 TOPNOTCH MD
C. Type 5 FROM UST)
D. Type 7
E. Type 9
109 In a patient with pulmonary embolism the Page 11 of Topnotch Handout. The most common ECG ANGELA PAULINE P. DIAGNOSTIC
most common ECG manifestation that should manifestation is a sinus tachycardia. Right ventricular CALIMAG-LOYOLA EXAM - AUG
be expected is? A. strain pattern or S1Q3T3 is found in only 6% of (TOP 8 - FEB 2015 2015
Incomplete right bundle branch block patients. MED BOARDS;
B. Sinus tachycardia TOPNOTCH MD
C. Right ventricular strain pattern FROM UST)
D. S1Q3T3
E. Junctional rhythm
110 A patient diagnosed with drug-induced lupus Page 20 of Topnotch Handout. A patient with drug ANGELA PAULINE P. DIAGNOSTIC
would most likely be positive for which induced lupus will be positive to anti-histone. A- CALIMAG-LOYOLA EXAM - AUG
autoantibody? A. Sjorgren syndrome, B-CREST syndrome, D-SLE, E- (TOP 8 - FEB 2015 2015
Anti-SS-A Primary biliary cirrhosis MED BOARDS;
B. Anti-centromere TOPNOTCH MD
C. Anti-histone FROM UST)
D. Anti-dsDNA
E. Anti-mitochondrial
111 A 59 y/o male, received a blood group identical Page 23 of Topnotch Handout. Hyperacute rejection ANGELA PAULINE P. DIAGNOSTIC
living unrelated kidney graft. During surgery no occurs within minutes to hours due to preformed CALIMAG-LOYOLA EXAM - AUG
abnormalities occurred. Four hours after the antibodies. It is a Type II hypersensitibity reaction. (TOP 8 - FEB 2015 2015
transplantation, it was noted that diuresis Morphological features: thrombotic occlusion of MED BOARDS;
suddenly decreased. Upon repeat laparotomy capillaries and fibrinoid necrosis occurs in arterial TOPNOTCH MD
the transplanted kidney showed signs of walls. FROM UST)
hyperacute rejection and had to be removed.
Which pathological examination findings are
consistent with hyperacute rejection?
A. Thrombosis of capillaries and fibrinoid
necrosis in arterial walls
B. Presence of interstitial mononuclear cell
infiltration and edema
C. Necrotizing vasculitis withendothelial cell
necrosis
D. Interstitial fibrosis and tubular atrophy
with loss of renal parenchyma
E. Neutrophilic infiltration, deposition of Ig,
complement and fibrin
112 A 32-day-old female infant was admitted due to Page 25 of Topnotch Handout. This is a case of ANGELA PAULINE P. DIAGNOSTIC
delayed umbilical cord detachment and Leukocyte adhesion deficiency syndrome it is an CALIMAG-LOYOLA EXAM - AUG
omphalitis. After admission, CBC revealed autosomal recessive disease involving a mutation in (TOP 8 - FEB 2015 2015
severe leukocytosis, and there was poor clinical integrins hence defective adhesion LFA-1 proteins on MED BOARDS;
response to several kinds of antibiotics. She is the surface of phagocytes. It is commonly manifested TOPNOTCH MD
diagnosed to have aan immunodefiency disease bu severe pyogenic infections in infancy and delayed FROM UST)
specifically a phagocyte disorder. What is the cord separation.
most probable pathophysiologic mechanism of
this disease?
A. Mutation in WASP gene for actin filament
assembly
B. Mutation in DNA repair enzymes
C. Lack of NADPH oxidase activity
D. Failure of phagolysosomal fusion
E. Defective LFA-1 proteins
113 Which electrolyte abnormality is not usually Page 29 of Topnotch Handout. Tumor lysis syndrome ANGELA PAULINE P. DIAGNOSTIC
found in tumor lysis syndrome?
is an oncologic emergency that is caused by massive CALIMAG-LOYOLA EXAM - AUG
A. Hypocalcemia tumor cell lysis with the release of large amounts of (TOP 8 - FEB 2015 2015
B. Hypercalcemia potassium, phosphate, and nucleic acids into the MED BOARDS;
C. Hyperphosphatemia systemic circulation. TOPNOTCH MD
D. Hyperuricemia FROM UST)
E. Hyperkalemia

114 Early chronic myeloid leukemia (CML) and Page 30 of Topnotch Handout. CML must be ANGELA PAULINE P. DIAGNOSTIC
leukemoid reaction (LR) sometimes show differentiated from leukemoid reactions in which CALIMAG-LOYOLA EXAM - AUG
similar histological pictures. To differentiate there is a marked increase in myeloid elements (TOP 8 - FEB 2015 2015
between a leukemoid reaction and CML the secondary to infection, chronic inflammation and MED BOARDS;
following should be requested:
other causes. Both present with extreme leukocytosis TOPNOTCH MD
A. Neutrophil alkaline phosphatase however CML has low NAP and CRP. Cytogenetic FROM UST)
B. C Reactive Protein testing will determine the presence of the BCR-ABL
C. Cytogenetic testing gene.
D. A and B only
E. All of the above

TOPNOTCH MEDICAL BOARD PREP PATHOLOGY SUPEREXAM Page 17 of 99


For inquiries visit www.topnotchboardprep.com.ph or email us at topnotchmedicalboardprep@gmail.com
TOPNOTCH MEDICAL BOARD PREP PATHOLOGY SUPEREXAM
For inquiries visit www.topnotchboardprep.com.ph or email us at topnotchmedicalboardprep@gmail.com
Item QUESTION EXPLANATION AUTHOR TOPNOTCH
# EXAM
115 This type of Hodgkin's lymphoma which has an Page 32 of Topnotch Handout. Refer to the table on ANGELA PAULINE P. DIAGNOSTIC
intermediate prognosis, has a highly significant types of hodgkin's lymphoma. Mixed cellularity CALIMAG-LOYOLA EXAM - AUG
association with EBV infection and numerous involved lymph nodes are diffusely effaced by a (TOP 8 - FEB 2015 2015
R-S cells in a mixed inflammatory background heterogenous cellular infiltrate. Plentiful RS cells MED BOARDS;
that obliterates the normal architecture:
admixed with lymphocytes. It has an intermediate TOPNOTCH MD
A. Lymphocyte depleted prognosis and EBV-infected in 70% of cases. FROM UST)
B. Lymphocyte predominant
C. Lymphocyte rich
D. Mixed cellularity
E. Nodular sclerosis
116 Cold agglutinin disease is a form of Page 38 of Topnotch Handout. IgM antibodies ANGELA PAULINE P. DIAGNOSTIC
autoimmune hemolytic anemia caused by cold- generally cause cold agglutinin disease. M-Malamig! CALIMAG-LOYOLA EXAM - AUG
reacting autoantibodies. Autoantibodies bind to (TOP 8 - FEB 2015 2015
the erythrocyte membrane leading to MED BOARDS;
premature erythrocyte destruction. Which TOPNOTCH MD
antibody is commonly involved? FROM UST)
A. IgM
B. IgG
C. IgA
D. IgD
E. IgE
117 Incomplete excision of a dentigerous cyst may Page 42 of Topnotch Handout. Dentigerous cyst ANGELA PAULINE P. DIAGNOSTIC
result in a neoplastic transformation, originates around the crown of an unerupted tooth, CALIMAG-LOYOLA EXAM - AUG
specifically into a/an: A. often associated with an impacted third molar. (TOP 8 - FEB 2015 2015
Odontoma Complete excision is curative, however incomplete MED BOARDS;
B. Ameloblastoma excision may result to recurrence or neoplastic TOPNOTCH MD
C. Basal cell carcinoma transformation into an ameloblastoma or a squamous FROM UST)
D. Cholesteatoma cell carcinoma.
E. Paraganglioma
118 A primary melanoma located in which part of SIMILAR TO PREVIOUS BOARD EXAM ANGELA PAULINE P. DIAGNOSTIC
the body has the worst prognosis?
CONCEPT/PRINCIPLE. Page 44 of Topnotch Handout. CALIMAG-LOYOLA EXAM - AUG
A. Sole Anatomic location of the primary melanoma is an (TOP 8 - FEB 2015 2015
B. Palm important independent predictor of SLN status and MED BOARDS;
C. Scalp prognosis. Patients with primary melanomas of the TOPNOTCH MD
D. Chest head/neck and trunk have a worse prognosis than FROM UST)
E. Back primary melanomas of other anatomic locations.

119 Penile carcinoma in situ has a strong SIMILAR TO PREVIOUS BOARD EXAM ANGELA PAULINE P. DIAGNOSTIC
association with:
CONCEPT/PRINCIPLE. Page 52 of Topnotch Handout. CALIMAG-LOYOLA EXAM - AUG
A. HPV 6 Penile Carcinoma in situ is strongly associated with (TOP 8 - FEB 2015 2015
B. HPV 11 HPV 16 infection. MED BOARDS;
C. HPV 16 TOPNOTCH MD
D. HPV 18 FROM UST)
E. HPV 31

120 Psammoma bodies are frequently encountered Page 59 of Topnotch Handout. PSaMMoma bodies are ANGELA PAULINE P. DIAGNOSTIC
in the following conditions except?
encountered in A-D. CALIMAG-LOYOLA EXAM - AUG
A. Papillary thyroid CA
(TOP 8 - FEB 2015 2015
B. Serous cystadeno CA MED BOARDS;
C. Meningioma TOPNOTCH MD
D. Mesothelioma FROM UST)
E. Medullary thyroid CA

121 Which of the following is an effect of PAF? Platelet activating factor is a phospholipid-dervided LYNN DARYL MIDTERM 1
A. Vasodilation mediator that has multiple inflammatory effects. It FELICIANO EXAM - AUG
B. Bronchoconstriction cause platelet aggregation, vasoconstriction, VILLAMATER, MD 2015
C. Decreased vascular permeability bronchoconstriction, increased venular permeability, (TOP 5 - FEB 2015
D. Decreased platelet aggregation increased leukocyte adhesion to endothelium and MED BOARDS;
E. Decreased leukocyte adhesion chemotaxis. (Robbin's) PAF causes TOPNOTCH MD
bronchoconstriction and also vasodilation in low FROM EAC)
doses. SIMILAR TO PREVIOUS BOARD EXAM
CONCEPT.
122 Cell-derived mediators of inflammation, in Options B and C are properties of plasma-derived LYNN DARYL MIDTERM 1
contrast with plasma-derived mediators mediators of inflammation. Option D is incorrect. FELICIANO EXAM - AUG
A. Are normally sequestered in granule and Both types of mediators can act on one or few target VILLAMATER, MD 2015
can be rapidly secreted by granule exocytosis in cells. SIMILAR TO PREVIOUS BOARD EXAM CONCEPT. (TOP 5 - FEB 2015
response to stimulus. MED BOARDS;
B. Are produced mainly in the liver. TOPNOTCH MD
C. Are inactive precursors that must be FROM EAC)
activated, usually by a series of proteolytic
cleavages to acquire their biologic properties.
D. Act only in one or few target cell types.
E. None of the above.
123 The following presents with granulomatous Histoplasma mimics tuberculosis. All except E would LYNN DARYL MIDTERM 1
inflammation EXCEPT: show granulomatous type of inflammation in the FELICIANO EXAM - AUG
A. Histoplasmosis affected organs. SIMILAR TO PREVIOUS BOARD EXAM VILLAMATER, MD 2015
B. Tuberculosis CONCEPT. (TOP 5 - FEB 2015
C. Sarcoidosis MED BOARDS;
D. Schistomiasis TOPNOTCH MD
E. Molluscum contagiosum FROM EAC)

TOPNOTCH MEDICAL BOARD PREP PATHOLOGY SUPEREXAM Page 18 of 99


For inquiries visit www.topnotchboardprep.com.ph or email us at topnotchmedicalboardprep@gmail.com
TOPNOTCH MEDICAL BOARD PREP PATHOLOGY SUPEREXAM
For inquiries visit www.topnotchboardprep.com.ph or email us at topnotchmedicalboardprep@gmail.com
Item QUESTION EXPLANATION AUTHOR TOPNOTCH
# EXAM
124 What is the mechanism of edema in nephrotic Increased permeability to plasma proteins resulting LYNN DARYL MIDTERM 1
syndrome? from either structural or physicochemical alteration FELICIANO EXAM - AUG
A. Glomerular injury allows protein to escape from the plasma into the VILLAMATER, MD 2015
B. Increased capillary permeability to proteins urinary space. Massive proteinuria depletes serum (TOP 5 - FEB 2015
C. Decreased protein absorption albumin resulting in hypoalbuminemia, and thus MED BOARDS;
D. Tubulointerstitial disorder decreased colloid osmotic pressure of the blood with TOPNOTCH MD
E. Increased hydrostatic pressure subsequent accumulation of fluid in the interstitial FROM EAC)
tissues. Sodium and water retention due to
compensatory secretion of aldosterone and
stimulation of sympathetic system also contributes
and aggravates the edema. SIMILAR TO PREVIOUS
BOARD EXAM CONCEPT.
125 A 24-year old male patient presented with This is a morphologic description of Tay-Sachs LYNN DARYL MIDTERM 1
cherry-red spots in the macula. Morphology of Disease, a deficiency of hexosaminidase. SIMILAR TO FELICIANO EXAM - AUG
the brain shows neurons ballooned with PREVIOUS BOARD EXAM CONCEPT. VILLAMATER, MD 2015
cytoplasmic vacuoles. What enzyme is deficient (TOP 5 - FEB 2015
in this condition? MED BOARDS;
A. Alpha 1,4-glucosidase TOPNOTCH MD
B. Sphingomyelinase FROM EAC)
C. Arylsulfatase
D. Iduronidase
E. Hexosaminidase
126 A 48-year old male patient who underwent Hyperacute rejection occurs few minutes to few hours LYNN DARYL MIDTERM 1
kidney transplant suddenly developed bloody after transplant. Option A describes Acute cellular FELICIANO EXAM - AUG
urine few hours after the procedure. rejection. Options C and D are mophologic findings VILLAMATER, MD 2015
Morphologic changes in this pattern of rejection found in acute humoral rejection. Option E describes (TOP 5 - FEB 2015
will reveal: the morphology of kidney in chronic rejection. MED BOARDS;
A. Extensive interstitial mononuclear cell TOPNOTCH MD
infiltration and edema FROM EAC)
B. Thrombotic occlusion of capillaries and
fibrinoid necrosis
C. Necrotizing vasculitis with endothelial cell
necrosis
D. Neutrophilic infiltration and deposition of
immunoglobulin, complement, and fibrin
E. Interstitial fibrosis and tubular atrophy
with loss of renal parenchyma.
127 What is the hallmark of tissue repair? SIMILAR TO PREVIOUS BOARD EXAM CONCEPT. LYNN DARYL MIDTERM 1
A. Vasoconstriction FELICIANO EXAM - AUG
B. Blood clot formation VILLAMATER, MD 2015
C. Granulation tissue (TOP 5 - FEB 2015
D. Tissue remodeling MED BOARDS;
E. Wound contraction TOPNOTCH MD
FROM EAC)

128 The first step in phagocytosis is: Steps in phagocytosis: 1. Recognition and attachment; LYNN DARYL MIDTERM 1
A. Formation of phagocytic vacuole 2. Engulfment and formation of phagocytic vacuole, 3. FELICIANO EXAM - AUG
B. Degradation of ingested material Killing and degradation of ingested material. SIMILAR VILLAMATER, MD 2015
C. Recognition of particle TO PREVIOUS BOARD EXAM CONCEPT. (TOP 5 - FEB 2015
D. Attachment MED BOARDS;
E. Engulfment TOPNOTCH MD
FROM EAC)

129 An 18-year old male patient presents with easy The diagnosis of AML is based on the presence of at LYNN DARYL MIDTERM 1
fatigability, fever, and cutaneous bleeding. least 20% myeloid blasts in the bone marrow. Robbins FELICIANO EXAM - AUG
Bone marrow biopsy showed 40% myeloblast. 9th ed., p. 613 . The most common manifestation of VILLAMATER, MD 2015
What is the most likely diagnosis? AML include fever, easy fatigability and bleeding. (TOP 5 - FEB 2015
A. ALL SIMILAR TO PREVIOUS BOARD EXAM CONCEPT. MED BOARDS;
B. AML TOPNOTCH MD
C. CML FROM EAC)
D. Burkitt's lymphoma
E. Adult T cell lymphoma
130 Morphologic finding/s in alcoholic hepatitis The rest are features of viral hepatitis. SIMILAR TO LYNN DARYL MIDTERM 1
include: PREVIOUS BOARD EXAM CONCEPT. FELICIANO EXAM - AUG
A. Councilman bodies VILLAMATER, MD 2015
B. Hepatocyte swelling and necrosis (TOP 5 - FEB 2015
C. Lymphoid aggregates within portal tracts MED BOARDS;
D. Hepatocyte apoptosis TOPNOTCH MD
E. All of the above FROM EAC)

131 The most common cause of sudden cardiac SIMILAR TO PREVIOUS BOARD EXAM CONCEPT. LYNN DARYL MIDTERM 1
death in Myocardial infaction is FELICIANO EXAM - AUG
A. Congestive heart failure VILLAMATER, MD 2015
B. Ventricular fibrillation (TOP 5 - FEB 2015
C. Pulmonary edema MED BOARDS;
D. Acute pericarditis TOPNOTCH MD
E. Ventricular rupture FROM EAC)

132 Juxta-articular osteopenia is characteristic of SIMILAR TO PREVIOUS BOARD EXAM CONCEPT. LYNN DARYL MIDTERM 1
A. Systemic lupus erythematosus FELICIANO EXAM - AUG
B. Rheumatoid arthritis VILLAMATER, MD 2015
C. Osteoarthritis (TOP 5 - FEB 2015
D. Ankylosing spondylitis MED BOARDS;
E. Gouty arthritis TOPNOTCH MD
FROM EAC)

TOPNOTCH MEDICAL BOARD PREP PATHOLOGY SUPEREXAM Page 19 of 99


For inquiries visit www.topnotchboardprep.com.ph or email us at topnotchmedicalboardprep@gmail.com
TOPNOTCH MEDICAL BOARD PREP PATHOLOGY SUPEREXAM
For inquiries visit www.topnotchboardprep.com.ph or email us at topnotchmedicalboardprep@gmail.com
Item QUESTION EXPLANATION AUTHOR TOPNOTCH
# EXAM
133 True of female hemophilia carrier Hemophilia is an X-linked recessive disorders wherein LYNN DARYL MIDTERM 1
A. Both X chromosomes are defective heterozygous female or female carriers does not FELICIANO EXAM - AUG
B. Decrease Factor VIII express full phenotypic change because of paired VILLAMATER, MD 2015
C. 25% of her offspring are affected normal allele, with random inactivation of one of the X (TOP 5 - FEB 2015
D. One of the X chromosome shows chromosome leading to variability. Only 1 of the X MED BOARDS;
abnormality chromosome is abnormal. Decrease in Factor VIII is a TOPNOTCH MD
E. Transmits disease to half her sons and half manifestation of affected male offspring. Option C is FROM EAC)
her daughters characteristic of AR while Option E is a property of X-
Linked Dominant disorders.
134 An 8-year old male presented with sunburn- This is a case of Staphylococcal scalded-skin syndrome LYNN DARYL MIDTERM 1
like rash that spread over the entire body and or Ritter disease caused by S. aureus. It is FELICIANO EXAM - AUG
evolves into fragile bullae. Desquamation of distinguished from toxic epidermal necrolysis or VILLAMATER, MD 2015
epidermis follows at the level of granulosa Lyells disease which is secondary to drug (TOP 5 - FEB 2015
layer. This is most likely caused by: hypersensitivity and causes desquamation at the level MED BOARDS;
A. Staphylococcus aureus of epidermal-dermal junction. TOPNOTCH MD
B. Streptococcus pyogenes FROM EAC)
C. Viridans streptococcus
D. Drug hypersensitivity
E. Pseudomonas aeruginosa
135 A 62-year old female presented with fever and Pseudomonas causes necrotizing pneumonia, LYNN DARYL MIDTERM 1
cough for the past few days. She also had vasculitis accompanied by thrombosis and FELICIANO EXAM - AUG
necrotizing oval lesion on her extremities. The hemorrhage. It proliferates widely, penetrating VILLAMATER, MD 2015
most likely etiology is: deeply into the veins and spreads hematogenously. (TOP 5 - FEB 2015
A. Staphylococcus aureus Ecthyma gangrenosum, well-demarcated necrotic and MED BOARDS;
B. Streptococcus pyogenes hemorrhagic oval skin lesion, often appear. TOPNOTCH MD
C. Clostridium perfringens FROM EAC)
D. Bacillus anthracis
E. Pseudomonas aeruginosa
136 The most likely renal pathology in multiple Renal insufficiency in multiple myeloma can be due to LYNN DARYL MIDTERM 1
myeloma is: Bence Jones proteins which are directly toxic to FELICIANO EXAM - AUG
A. Tubulo-interstitial nephritis epithelial cell, and accumulation of light chains which VILLAMATER, MD 2015
B. Membranoproliferative glomerulonephritis results to tubulo-interstitial nephritis. Other causes (TOP 5 - FEB 2015
C. Acute glomerulonephritis include hypercalcemia nad hyperuricemia. MED BOARDS;
D. Focal segmental glomerulosclerosis TOPNOTCH MD
E. Tubular necrosis FROM EAC)

137 A 45-year old male, smoker, presented with Chronic cough, copious sputum in a smoker points to LYNN DARYL MIDTERM 1
cough, copious sputum, and progressive chronic bronchitis as the diagnosis. Option A - FELICIANO EXAM - AUG
dyspnea on exertion for the last two years. On emphysema. Option C - Bronchiectasis. Option D - VILLAMATER, MD 2015
physical examination, he has wheezes all over Asthma; Option E - Idiopathic interstitial fibrosis. (TOP 5 - FEB 2015
his lung fields, and cyanotic lips. Morphologic SIMILAR TO PREVIOUS BOARD EXAM CONCEPT. MED BOARDS;
findings of the lungs in this condition will most TOPNOTCH MD
likely show a/an: FROM EAC)
A. Abnormally large alveoli separated by thin
septa and deformed respiratory bronchioles.
B. Enlargement of the mucus-secreting glands
of the bronchi and hyperemia and edema of the
mucus membranes.
C. Dilatation of bronchi and bronchioles
caused by destruction of the muscle and elastic
tissue.
D. Thickening of the basement membrane,
edema and inflammatory infiltrate in the
bronchial walls.
E. Cobblestone pleural surfaces and patchy
interstitial fibrosis varying in intensity.
138 The most common malignancy of the stomach: SIMILAR TO PREVIOUS BOARD EXAM CONCEPT. LYNN DARYL MIDTERM 1
A. Carcinoid FELICIANO EXAM - AUG
B. Lymphoma VILLAMATER, MD 2015
C. Adenocarcinoma (TOP 5 - FEB 2015
D. Squamous cell carcinoma MED BOARDS;
E. Gastrointestinal stromal tumor TOPNOTCH MD
FROM EAC)

139 A 57-year old, obese male was brought to the This is a case of acute myocardial infarction. LYNN DARYL MIDTERM 1
hospital because of sudden onset of chest pain, Myocardial necrosis begins at approximately 30 FELICIANO EXAM - AUG
characterized as squeezing, accompanied by minutes after coronary occlusion. SIMILAR TO VILLAMATER, MD 2015
diaphoresis and nausea. Neutrophils were PREVIOUS BOARD EXAM CONCEPT. (TOP 5 - FEB 2015
noted to be elevated. What is the MED BOARDS;
pathophysiologic mechanism of his condition? TOPNOTCH MD
A. Tissue necrosis FROM EAC)
B. Alveolar edema
C. Infection
D. Inflammation of airways
E. B and C
140 Thrombosis of what vessel will most most Right coronary artery (30-40%) infarct involves the LYNN DARYL MIDTERM 1
likely involve the posterior portion of the inferior/posterior wall of LV, posterior portion of FELICIANO EXAM - AUG
ventricular septum? ventricular septum, inferior/posterior RV free wall in VILLAMATER, MD 2015
A. Left anterior descending coronary artery some cases; LADA involves the anterior wall of LV (TOP 5 - FEB 2015
B. Left circumflex coronary artery near apex, anterior portion of ventricular septum and MED BOARDS;
C. Marginal branch of left circumflex artery apex circumferentially; Left circumflex artery involves TOPNOTCH MD
D. Right coronary artery the lateral wall of left ventricle except the apex. FROM EAC)
E. Left main coronary artery (Robbin's)

TOPNOTCH MEDICAL BOARD PREP PATHOLOGY SUPEREXAM Page 20 of 99


For inquiries visit www.topnotchboardprep.com.ph or email us at topnotchmedicalboardprep@gmail.com
TOPNOTCH MEDICAL BOARD PREP PATHOLOGY SUPEREXAM
For inquiries visit www.topnotchboardprep.com.ph or email us at topnotchmedicalboardprep@gmail.com
Item QUESTION EXPLANATION AUTHOR TOPNOTCH
# EXAM
141 A 30 year old male admitted for dengue fever Increased capillary permeability is the EDWARD HARRY MIDTERM 2
on the 7th day of illness develops pleural pathophysiologic mechanism of pleural effusion in VALLAJERA, MD EXAM - AUG
effusion, what is the explanation for the pleural dengue fever more than decreased platelets (TOP 8 - FEB 2015 2015
effusion? MED BOARDS;
A. Decreased plasma oncotic pressure TOPNOTCH MD
B. Increased capillary permeability FROM PERPETUAL
C. Increased capillary hydrostatic pressure BINAN)
D. Decreased platelets
E. None of the above
142 JS, a 70 year old male who was a smoker of 40 The major (primary bronchus) is the most common EDWARD HARRY MIDTERM 2
pack years was diagnosed with squamous cell site of development of squamous cell carcinoma of the VALLAJERA, MD EXAM - AUG
carcinoma of the lung, which among the lung (TOP 8 - FEB 2015 2015
following is the most common site of origin of MED BOARDS;
this lesion? TOPNOTCH MD
A. Trachea FROM PERPETUAL
B. Secondary bronchus BINAN)
C. Primary bronchus
D. Terminal bronchioles
E. None of the above
143 RLDL, a 40 year old female was a diagnosed Diffuse proliferative GN is the most common as well as EDWARD HARRY MIDTERM 2
case of SLE, she had a renal biopsy done and the most severe form of glomerulonephritis VALLAJERA, MD EXAM - AUG
results were brought to you, you know that the (TOP 8 - FEB 2015 2015
most common type of renal lesion of lupus has: MED BOARDS;
A. Mesangial lupus glomerulonephritis TOPNOTCH MD
B. Diffuse proliferative glomerulonephritis FROM PERPETUAL
C. Membranous glomerulopathy BINAN)
D. Focal proliferative glomerulopathy
E. None of the above

144 What is the cytokine that stimulates collagen TGF-B stimulates collagen synthesis, IL-1 stimulates EDWARD HARRY MIDTERM 2
synthesis fever as well as TNF, VEGF causes angiogenesis, PDGF VALLAJERA, MD EXAM - AUG
A. TNF causes collagenase secretion. (TOP 8 - FEB 2015 2015
B. IL-1 MED BOARDS;
C. VEGF TOPNOTCH MD
D. TGF-B FROM PERPETUAL
E. PDGF BINAN)

145 MIV, a 48 year old female underwent Pap Koilocytic atypia is a characteristic finding in the pap EDWARD HARRY MIDTERM 2
smear, which of the following cytologic findings smear which suggests HPV infection VALLAJERA, MD EXAM - AUG
suggest the presence of HPV infection (TOP 8 - FEB 2015 2015
A. Atypical ductal cells MED BOARDS;
B. Diffuse thickening of the basement TOPNOTCH MD
membrane FROM PERPETUAL
C. Koilocytic atypia BINAN)
D. A and B
E. None of the above
146 RF, a 30 year old male underwent lymph node Starry sky pattern is seen in Burkitt's lymphoma EDWARD HARRY MIDTERM 2
biopsy, the result showed a starry sky pattern, which is associated with EBV infection VALLAJERA, MD EXAM - AUG
you know that his malignancy is associated (TOP 8 - FEB 2015 2015
with? MED BOARDS;
A. Hepatitis B infection TOPNOTCH MD
B. Cytomegalovirus FROM PERPETUAL
C. Herpes zoster BINAN)
D. Epstein Barr virus
E. Infection with viruses belonging to filoviridae
family
147 Which among the following is important in the HIV invades the CD4 cells which are also responsible EDWARD HARRY MIDTERM 2
pathology of HIV infection? for the cell mediated immunity VALLAJERA, MD EXAM - AUG
A. CD8 (TOP 8 - FEB 2015 2015
B. CD4 MED BOARDS;
C. Macrophages TOPNOTCH MD
D. Dendritic cells FROM PERPETUAL
E. Neutrophils BINAN)

148 A patient was diagnosed with melanoma, one of The number of skin dendritic cells is not a prognostic EDWARD HARRY MIDTERM 2
the following is a prognostic factor for factor for melanoma VALLAJERA, MD EXAM - AUG
melanoma except (TOP 8 - FEB 2015 2015
A. tumor depth MED BOARDS;
B. presence of tumor infiltrating lymphocytes TOPNOTCH MD
C. number of dendritic cells FROM PERPETUAL
D. location of the tumor BINAN)
E. gender

149 BB, a 70 year old chronic alcoholic presents to Mallory bodies are eosinophilic cytoplasmic clumps in EDWARD HARRY MIDTERM 2
you with changes in sensorium, you requested a hepatocytes signifying liver injury, councilman bodies VALLAJERA, MD EXAM - AUG
liver biopsy, which of the following would you are eosinophilic globules seen in acute hepatitis, Negri (TOP 8 - FEB 2015 2015
expect to see: bodies are associated with rabies while psamomma MED BOARDS;
A. Councilman bodies bodies are associated with meningioma, prolactinoma, TOPNOTCH MD
B. Mallory bodies ovarian serous cystadenoma and papillary thyroid CA. FROM PERPETUAL
C. Negri bodies BINAN)
D. Psamomma bodies
E. None of the above

TOPNOTCH MEDICAL BOARD PREP PATHOLOGY SUPEREXAM Page 21 of 99


For inquiries visit www.topnotchboardprep.com.ph or email us at topnotchmedicalboardprep@gmail.com
TOPNOTCH MEDICAL BOARD PREP PATHOLOGY SUPEREXAM
For inquiries visit www.topnotchboardprep.com.ph or email us at topnotchmedicalboardprep@gmail.com
Item QUESTION EXPLANATION AUTHOR TOPNOTCH
# EXAM
150 MBDM, a 29 year old male underwent a tissue AML presents with >20% of myeloblasts in the bone EDWARD HARRY MIDTERM 2
section that showed 40% myeloid cells, what is marrow. VALLAJERA, MD EXAM - AUG
the diagnosis? (TOP 8 - FEB 2015 2015
A. CLL MED BOARDS;
B. AML TOPNOTCH MD
C. ALL FROM PERPETUAL
D. Mantle cell lymphoma BINAN)
E. Non-Hodgkins lymphoma

151 Most common bone involved in osteoporosis The vertebra are the most commonly affected as it is a EDWARD HARRY MIDTERM 2
A. Pelvis weight bearing bone. VALLAJERA, MD EXAM - AUG
B. Tibia (TOP 8 - FEB 2015 2015
C. Femur MED BOARDS;
D. Vertebra TOPNOTCH MD
E. Ribcage FROM PERPETUAL
BINAN)

152 What is the most common cause of nephritic EDWARD HARRY MIDTERM 2
syndrome in children? VALLAJERA, MD EXAM - AUG
A. Hepatitis B (TOP 8 - FEB 2015 2015
B. Intake of toxic substances MED BOARDS;
C. Antecedent GABHS infection TOPNOTCH MD
D. Viral infections FROM PERPETUAL
E. None of the above BINAN)

153 Which of the following lesions is more prone to Lobular carcinoma in situ is a fertile ground for breast EDWARD HARRY MIDTERM 2
develop breast CA cancer, proliferation without atypia and proliferation VALLAJERA, MD EXAM - AUG
A. Proliferative atypia with atypia is hyperplasia, fibroadenoma is a benign (TOP 8 - FEB 2015 2015
B. Proliferation without atypia condition. MED BOARDS;
C. Carcinoma in situ TOPNOTCH MD
D. Fibroadenoma FROM PERPETUAL
E. None of the above BINAN)

154 Which among the following is involved in Degree of differentiation or tissue grading is not EDWARD HARRY MIDTERM 2
staging a particular malignancy except needed in the staging in most malignancies VALLAJERA, MD EXAM - AUG
A. Presence of metastasis (TOP 8 - FEB 2015 2015
B. Lymph node involvement MED BOARDS;
C. Degree of differentiation TOPNOTCH MD
D. Size of the mass FROM PERPETUAL
E. None of the above BINAN)

155 KB, a 32 year old female fond of using beauty Type IV or delayed hypersensitivity develops usually EDWARD HARRY MIDTERM 2
products tried a new product on her face, 5 after 48 hours on application of offending agent on the VALLAJERA, MD EXAM - AUG
days later, she noticed redness on the areas of skin of the patient. (TOP 8 - FEB 2015 2015
application of the new product, what is the MED BOARDS;
most likely explanation: TOPNOTCH MD
A. IgE mediated immune reaction FROM PERPETUAL
B. Cytotoxin mediated BINAN)
C. Immunoglobulin-antigen complex
D. Cell mediated hypersensitivity
E. None of the above
156 What is the mechanism of DIC in All of the above are the mechanism of DIC EDWARD HARRY MIDTERM 2
meningococcemia? VALLAJERA, MD EXAM - AUG
A. Massive intravascular coagulation occuring (TOP 8 - FEB 2015 2015
everywhere in the circulation MED BOARDS;
B. Endothelial dysfunction TOPNOTCH MD
C. Depletion of clotting factors FROM PERPETUAL
D. All of the above BINAN)
E. None of the above
157 A 2 week old infant had been undergoing The most likely cause is septic shock due to systemic EDWARD HARRY MIDTERM 2
treatment for sepsis when the patient went into vasodilation. VALLAJERA, MD EXAM - AUG
hypotension and died, what is the most likely (TOP 8 - FEB 2015 2015
explanation for the infant's death? MED BOARDS;
A. DIC TOPNOTCH MD
B. Sepsis FROM PERPETUAL
C. Thrombocytopenia BINAN)
D. B and C
E. None of the above
158 The classic anaphylactic reaction is due to: SIMILAR TO PREVIOUS BOARD EXAM EDWARD HARRY MIDTERM 2
A. Th1 CONCEPT/PRINCIPLE. Th2 subset of helper T cells are VALLAJERA, MD EXAM - AUG
B. Th2 overactive in patients who develop the classic (TOP 8 - FEB 2015 2015
C. NK cell anaphylactic reaction MED BOARDS;
D. Macrophage TOPNOTCH MD
E. None of the above FROM PERPETUAL
BINAN)

159 Among the following adaptations of the body to Metaplasia is defined as the change from one adult cell EDWARD HARRY MIDTERM 2
injury, which of the following is a fertile ground type into another and is a fertile ground for VALLAJERA, MD EXAM - AUG
for neoplasia? malignancies, examples of malignancies originating (TOP 8 - FEB 2015 2015
A. Hyperplasia from metaplastic change include Barett's esophagus MED BOARDS;
B. Atrophy leading to esophageal adenoCA, cervical CA from TOPNOTCH MD
C. Hypertrophy squamous metaplasia of cervical columnar cells, SQCA FROM PERPETUAL
D. Metaplasia of the lung due to squamous metaplasia of respiratory BINAN)
E. None of the above epithelium.

TOPNOTCH MEDICAL BOARD PREP PATHOLOGY SUPEREXAM Page 22 of 99


For inquiries visit www.topnotchboardprep.com.ph or email us at topnotchmedicalboardprep@gmail.com
TOPNOTCH MEDICAL BOARD PREP PATHOLOGY SUPEREXAM
For inquiries visit www.topnotchboardprep.com.ph or email us at topnotchmedicalboardprep@gmail.com
Item QUESTION EXPLANATION AUTHOR TOPNOTCH
# EXAM
160 What is the most common type of gastric Gastric adenoCA is the most common type of gastric EDWARD HARRY MIDTERM 2
malignancy? malignancy VALLAJERA, MD EXAM - AUG
A. Lymphoma (TOP 8 - FEB 2015 2015
B. Gastric adenocarcinoma MED BOARDS;
C. Carcinoid tumor TOPNOTCH MD
D. Gastric adenoma FROM PERPETUAL
E. Pleomorphic adenoma BINAN)

161 This can be a pathologic or physiologic process hypertrophy is increase in size of cells. Atrophy is HAROLD JAY S. MIDTERM 3
which is described as the increase in the decrease in number and size of the cell. Metaplasia is BAYTEC, MD (TOP 10 EXAM - AUG
number of cells in an organ. replacement of one differentiated type to another. - FEB 2015 MED 2015
A. Hypertrophy Anaplasia happens when cells lose their morphologic BOARDS; TOPNOTCH
B. Hyperplasia characteristic of a mature cells. MD FROM FEU)
C. Atrophy
D. Metaplasia
E. Anaplasia

162 In Rheumatic Heart Disease, at which Aschoff bodies can be found in all layers of the heart in HAROLD JAY S. MIDTERM 3
layer/layers of the heart can you find Aschoff RF and RHD. SIMILAR TO PREVIOUS BOARD EXAM BAYTEC, MD (TOP 10 EXAM - AUG
bodies? CONCEPT/PRINCIPLE - FEB 2015 MED 2015
A. pericardium BOARDS; TOPNOTCH
B. myocardium MD FROM FEU)
C. endocardium
D. B and C
E. All layers
163 Valvular vegetations characrerized as small, IE are large irregular masses on the valve cusps that HAROLD JAY S. MIDTERM 3
warty vegetations along the lines of closure of can extend onto the chordae. NBTE/marantic BAYTEC, MD (TOP 10 EXAM - AUG
the valve leaflets are usually seen in what endocarditis are small bland vegetations usually - FEB 2015 MED 2015
disease? attached at the line of closure. LSE are small or BOARDS; TOPNOTCH
A. RHD medium sized vegetation on either or both sides of the MD FROM FEU)
B. Infective endocarditis valve leaflets
C. Non bacterial thrombotic endocarditis
D. Libman sacks endocarditis
E. marantic endocarditis
164 In Libman-Sacks Disease, which of the following In Robbins, Mitral and Tricuspid. In Medscape, Mitral HAROLD JAY S. MIDTERM 3
valve is usually affected? and Aortic. BAYTEC, MD (TOP 10 EXAM - AUG
A. Tricuspid - FEB 2015 MED 2015
B. Pulmonary BOARDS; TOPNOTCH
C. Mitral MD FROM FEU)
D. Aortic
E. All valves are equally involve

165 Which of the following caspase is an example of caspases 8, 9, 10 are initiators while caspases 6 and 3 HAROLD JAY S. MIDTERM 3
executioner in apoptosis? are executioners BAYTEC, MD (TOP 10 EXAM - AUG
A. 8 - FEB 2015 MED 2015
B. 9 BOARDS; TOPNOTCH
C. 10 MD FROM FEU)
D. 6
E. 5

166 Wire-loop capillaries are seen in which type of SIMILAR TO PREVIOUS BOARD EXAM HAROLD JAY S. MIDTERM 3
kidney disease? CONCEPT/PRINCIPLE BAYTEC, MD (TOP 10 EXAM - AUG
A. Lupus nephritis - FEB 2015 MED 2015
B. RPGN BOARDS; TOPNOTCH
C. Diabetic nephropathy MD FROM FEU)
D. Polycystic kidney disease
E. Acute tubular necrosis

167 What is the most common and the most severe type IV or the diffuse proliferative glomerulonephritis HAROLD JAY S. MIDTERM 3
type of Lupus nephropathy? is the most common and most severe type. BAYTEC, MD (TOP 10 EXAM - AUG
A. Type I - FEB 2015 MED 2015
B. Type II BOARDS; TOPNOTCH
C. Type III MD FROM FEU)
D. Type IV
E. Type V

168 Minamata disease is an environmental concern Mercury poisoning is the one associated with HAROLD JAY S. MIDTERM 3
because it causes cerebral palsy, deafness, Minamata disease. BAYTEC, MD (TOP 10 EXAM - AUG
blindness, mental retardation and other major - FEB 2015 MED 2015
CNS defects in children in utero. Which of the BOARDS; TOPNOTCH
following elements is the cause of the disease? MD FROM FEU)
A. arsenic
B. lead
C. mercury
D. cadnium
E. chromium
169 All of the following cancers are strongly cancers of the lung, larynx, esophagus, pancreas, HAROLD JAY S. MIDTERM 3
associated with smoking except: bladder, oral cavity are strongly associated with BAYTEC, MD (TOP 10 EXAM - AUG
A. Oral cavity smoking. - FEB 2015 MED 2015
B. pancreas BOARDS; TOPNOTCH
C. esophagus MD FROM FEU)
D. bladder
E. Breast

TOPNOTCH MEDICAL BOARD PREP PATHOLOGY SUPEREXAM Page 23 of 99


For inquiries visit www.topnotchboardprep.com.ph or email us at topnotchmedicalboardprep@gmail.com
TOPNOTCH MEDICAL BOARD PREP PATHOLOGY SUPEREXAM
For inquiries visit www.topnotchboardprep.com.ph or email us at topnotchmedicalboardprep@gmail.com
Item QUESTION EXPLANATION AUTHOR TOPNOTCH
# EXAM
170 Among the types of Hodgkin's lymphoma, this lymphocyte depleted has the highest association with HAROLD JAY S. MIDTERM 3
has the highest association with EBV and also EBV and also has the poorest prognosis.Nodular BAYTEC, MD (TOP 10 EXAM - AUG
has the poorest prognosis. sclerosis is the most common and has excellent - FEB 2015 MED 2015
A. Lymphocyte-depleted prognosis BOARDS; TOPNOTCH
B. Lymphocyte rich MD FROM FEU)
C. Lymphocyte predominant
D. Nodular sclerosis
E. Mixed cellularity

171 Chromosome translocations are important in 9:22 is associated with CML. 14:18 is associated with HAROLD JAY S. MIDTERM 3
diagnosing and prognosticating certain types of Follicular lymphoma. 11:14 is associated with mantle BAYTEC, MD (TOP 10 EXAM - AUG
cancers. Which of the following may be seen in cell lymphoma. 15:17 is AML M3 - FEB 2015 MED 2015
Burkitt's lymphoma? BOARDS; TOPNOTCH
A. t9:22 MD FROM FEU)
B. T8:14
C. T14:18
D. T11:14
E. T15:17
172 Which of the following types of vasculitis is p-ANCA is associated with microscopic polyangitis HAROLD JAY S. MIDTERM 3
highly associated with c-ANCA or the antibodies and Churgstraus while c-ANCA is for Wegener BAYTEC, MD (TOP 10 EXAM - AUG
against proteinase-3? Granulomatosis - FEB 2015 MED 2015
A. PAN BOARDS; TOPNOTCH
B. Takayasu arteritis MD FROM FEU)
C. Wegener Granulomatosis
D. Microscopic polyangitis
E. Churg-Strauss syndrome

173 A patient came in for second opinion about his 30% of patients with PAN are HBsAg positive. HAROLD JAY S. MIDTERM 3
newly diagnosed polyarteritis nodosa. All of the pathology topnotch handout page 106. PAN does not BAYTEC, MD (TOP 10 EXAM - AUG
following statements are true about this disease affect the lungs, affects mainly young adults, usually - FEB 2015 MED 2015
EXCEPT involves medium sized arteries, very responsive to BOARDS; TOPNOTCH
A. Does NOT affect lungs steroid therapy and cyclophosphamide, and 30% of MD FROM FEU)
B. Affects mainly young adults patients are HbsAg positive.
C. Usually involves medium sized arteries
D. NOT associated with hepatitis B
E. very responsive to steroid therapy and
cyclophospahmide
174 Which among the following arteries is HAROLD JAY S. MIDTERM 3
commonly affected in Kawasaki disease? BAYTEC, MD (TOP 10 EXAM - AUG
A. Abdominal Aorta - FEB 2015 MED 2015
B. Thoracic aorta BOARDS; TOPNOTCH
C. Arch of the aorta MD FROM FEU)
D. Pulmonary artery
E. Coronary arteries

175 Which among the following medications is/are Steroids is contraindicated because it promote HAROLD JAY S. MIDTERM 3
essential in treating a patient with Kawasaki rupture of coronary vessels and aggration of the BAYTEC, MD (TOP 10 EXAM - AUG
disease? disease. - FEB 2015 MED 2015
A. ASA BOARDS; TOPNOTCH
B. IVIg MD FROM FEU)
C. Steroids
D. A and B
E. All of the above

176 This is a primary malignant small round-cell HAROLD JAY S. MIDTERM 3


tumor of the bone and soft tissue which usually BAYTEC, MD (TOP 10 EXAM - AUG
involve the mutation of a gene in chromosome - FEB 2015 MED 2015
22. Homer-Wright rosettes can be seen BOARDS; TOPNOTCH
histologically and onion-skin appearance can be MD FROM FEU)
seen radiographically on patients with this
disease.
A. Ewing sarcoma
B. Osteosarcoma
C. Chondrosarcoma
D. Giant Cell Tumor
E. Fibrous Dysplasia
177 Reiter syndrome is an autoimmune disease these three are the triad of Reiter syndrome HAROLD JAY S. MIDTERM 3
which is usually triggered by infections caused BAYTEC, MD (TOP 10 EXAM - AUG
by Shigella, Salmonella, Yersinia, and - FEB 2015 MED 2015
Chlamydia. Which among the following is/are BOARDS; TOPNOTCH
component of the disease? MD FROM FEU)
A. urethritis
B. arthritis
C. uveitis
D. A and B
E. All of the above
178 In infective endocarditis, what is the most HAROLD JAY S. MIDTERM 3
organism involved in patients with native but BAYTEC, MD (TOP 10 EXAM - AUG
previously damaged heart valves? - FEB 2015 MED 2015
A. Strep viridans BOARDS; TOPNOTCH
B. Staph epidermidis MD FROM FEU)
C. Strep bovis
D. Staph aureus
E. Enterococcus

TOPNOTCH MEDICAL BOARD PREP PATHOLOGY SUPEREXAM Page 24 of 99


For inquiries visit www.topnotchboardprep.com.ph or email us at topnotchmedicalboardprep@gmail.com
TOPNOTCH MEDICAL BOARD PREP PATHOLOGY SUPEREXAM
For inquiries visit www.topnotchboardprep.com.ph or email us at topnotchmedicalboardprep@gmail.com
Item QUESTION EXPLANATION AUTHOR TOPNOTCH
# EXAM
179 Which of the following conditions will cause decrease in plasma proteins will decrease oncotic HAROLD JAY S. MIDTERM 3
edema? pressure within the vessel which can cause edema BAYTEC, MD (TOP 10 EXAM - AUG
A. Increase oncotic pressure in the vessels - FEB 2015 MED 2015
B. Decrease plasma volume in blood vessels BOARDS; TOPNOTCH
C. Decrease oncotic pressure outside the MD FROM FEU)
vessels
D. Decrease in plasma proteins
E. None of the above
180 Which of the following diseases is an X-linked Alport syndrome and vitamin D resistant rickets are HAROLD JAY S. MIDTERM 3
dominant disorder? the only most commonly associated with X linked BAYTEC, MD (TOP 10 EXAM - AUG
A. G6PD deficiency dominant genetic transmission. - FEB 2015 MED 2015
B. Hemophilia BOARDS; TOPNOTCH
C. Ehlers Danlos syndrome MD FROM FEU)
D. Marfan syndrome
E. Alport syndrome

181 What is the most common tumor in the SIMILAR TO PREVIOUS BOARD EXAM JEAN PAOLO M. FINAL EXAM -
stomach? CONCEPT/PRINCIPLE. DELFINO, MD (TOP AUG 2015
A. adenocarcinoma 10 - FEB 2015 MED
B. GIST BOARDS; TOPNOTCH
C. carcinoid MD FROM FATIMA)
D. adenoma
E. lymphoma

182 True of Luetic aneurysm? All are true regarding Syphilitic/Luetic aneurysm JEAN PAOLO M. FINAL EXAM -
A. Inflammation begins in the tunica DELFINO, MD (TOP AUG 2015
adventitia 10 - FEB 2015 MED
B. With characteristic tree-barking BOARDS; TOPNOTCH
appearance MD FROM FATIMA)
C. Involvement of aorta favors development of
superimosed atheromatosisof the aortic root
D. All of the above
E. A and C
183 Microscopically, the earliest change of systemic JEAN PAOLO M. FINAL EXAM -
Hypertensive Heart Disease is? DELFINO, MD (TOP AUG 2015
A. Increase in transverse diameter of 10 - FEB 2015 MED
myocytes BOARDS; TOPNOTCH
B. Irregular cellular enlargement MD FROM FATIMA)
C. Irregular nuclear enlargement
D. Interstitial fibrosis
E. All of the above
184 In acute Rheumatic Fever, inflammation and SIMILAR TO PREVIOUS BOARD EXAM JEAN PAOLO M. FINAL EXAM -
Aschoff bodies are commonly found in which CONCEPT/PRINCIPLE.. During acute RF, diffuse DELFINO, MD (TOP AUG 2015
layer of the heart? inflammation and Aschoff bodies may be found in any 10 - FEB 2015 MED
A. pericardium of the 3 layers of the heart- hence the lesion is called a BOARDS; TOPNOTCH
B. myocardium PANCARDITIS. MD FROM FATIMA)
C. endocardium
D. All of the above
E. A and B

185 What is the valve most commonly affected in SIMILAR TO PREVIOUS BOARD EXAM JEAN PAOLO M. FINAL EXAM -
Libman-Sacks endocarditis? CONCEPT/PRINCIPLE.. In SLE, mitral and tricuspid DELFINO, MD (TOP AUG 2015
A. Mitral valvulitis with small, sterile vegetations, called 10 - FEB 2015 MED
B. Tricuspid Libman-Sacks endocarditis is ocassionally BOARDS; TOPNOTCH
C. Aortic encountered. according to harrison, MD FROM FATIMA)
D. Pulmonic "The characteristic endocardial lesions of SLE are
E. A and B verrucous valvular abnormalities known as Libman-
Sacks endocarditis . They most often are located on
the left-sided cardiac valves, particularly on the
ventricular surface of the posterior mitral leaflet, and
are made up almost entirely of fibrin. " So yes, the
correct answer is mitral valve.
186 What is the most severe form of -thalassemia Hydrops fetalis is the most severe form of - JEAN PAOLO M. FINAL EXAM -
which is caused by deletion of all four -globin thalassemia. it is caused by deletion of all four - DELFINO, MD (TOP AUG 2015
genes? globin genes. In the fetus, excess -globin chains form 10 - FEB 2015 MED
A. Thalassemia trait tetramers (hemoglobin Barts) that have such a high BOARDS; TOPNOTCH
B. Silent carrier state affinity for oxygen that they deliver little to tissues. MD FROM FATIMA)
C. Hydrops fetalis
D. Hemoglobin H disease
E. A and D

187 A 58 year old male presented with easy CLL and SLL differ only in the degree of peripheral JEAN PAOLO M. FINAL EXAM -
fatigability, anorexia and weight loss. On PE, blood lymphocytosis. Most affected patients have DELFINO, MD (TOP AUG 2015
there were noted lymphadenopathies and sufficient lymphocytosis to fulfill the diagnostic 10 - FEB 2015 MED
hepatosplenomegaly. CBC revealed requirement for CLL (absolute lymphocyte count BOARDS; TOPNOTCH
leukocytosis, with absolute lymphocyte count >4000 per mm3). In this condition, lymph nodes are MD FROM FATIMA)
3000 per mm3. There were numerous small, diffusely effaced by an infiltrate of predominantly
round lymphocytes with scant cytoplasm which small lymphocytes 6 to 12 m in diameter with round
are frequently disrupted in the process of to slightly irregular nuclei, condensed chromatin, and
making smears. What is the diagnosis? scant cytoplasm. Some of these cells are usually
A. ALL disrupted in the process of making smears, producing
B. SLL so-called smudge cells.
C. CLL
D. CML
E. Multiple Myeloma

TOPNOTCH MEDICAL BOARD PREP PATHOLOGY SUPEREXAM Page 25 of 99


For inquiries visit www.topnotchboardprep.com.ph or email us at topnotchmedicalboardprep@gmail.com
TOPNOTCH MEDICAL BOARD PREP PATHOLOGY SUPEREXAM
For inquiries visit www.topnotchboardprep.com.ph or email us at topnotchmedicalboardprep@gmail.com
Item QUESTION EXPLANATION AUTHOR TOPNOTCH
# EXAM
188 What is the most common lesion of the salivary Mucocele is the most common lesion of the salivary JEAN PAOLO M. FINAL EXAM -
gland? glands and it results from either blockage or rupture DELFINO, MD (TOP AUG 2015
A. Sialolithiasis of a salivary gland duct, with consequent leakage of 10 - FEB 2015 MED
B. Pelomorphic adenoma saliva into the surrounding connective tissue stroma. BOARDS; TOPNOTCH
C. Warthin tumor MD FROM FATIMA)
D. Mucocele
E. Sialadenitis

189 Acute hepatitis is characterized by what With acute hepatitis, hepatocyte injury takes the form JEAN PAOLO M. FINAL EXAM -
morphologic feature? of diffuse swelling (ballooning degeneration;), so the DELFINO, MD (TOP AUG 2015
A. Swollen hepatocytes with irregulaly cytoplasm looks empty and contains only scattered 10 - FEB 2015 MED
clumped cytoplasmic organelles and large clear eosinophilic remnants of cytoplasmic organelles. BOARDS; TOPNOTCH
spaces Feathery degeneration is retention of biliary material MD FROM FATIMA)
B. Presence of feathery degeneration causing foamy appearance of hepatocytes seen in
C. Deposition of fibrous tissue in the portal cholestatic liver injury. C and D are charcteristics of
tracts and periportal septa chronic hepatitis.
D. Bridging inflammation and necrosis
E. All of the above
190 True statement about Crigler-Najjar Syndrome Crigler-Najjar Syndrome type I is an autosomal JEAN PAOLO M. FINAL EXAM -
type I recessive condition wherein there is ABSENT UGT1A1 DELFINO, MD (TOP AUG 2015
A. Liver morphology is normal activity causing indirect hyperbilirubinemia. Liver 10 - FEB 2015 MED
B. There is decreased UGT1A1 enzyme pathology is normal and it is fatal in the neonatal BOARDS; TOPNOTCH
activity period. MRP2 mutation is seen in Dubin-Johnson MD FROM FATIMA)
C. It is generally mild with occasional Syndrome
kernicterus
D. There is mutation in MRP2
E. Hyperbilirubinemia is of direct type
191 Hepatitis B carrier state is most commonly In endemic regions such as Africa and Southeast Asia, JEAN PAOLO M. FINAL EXAM -
acquired via what mode of transmission? spread of Hepatitis B from an infected mother to a DELFINO, MD (TOP AUG 2015
A. Heterosexual transmission neonate during birth (vertical transmission) is 10 - FEB 2015 MED
B. Needle-stick injuries common. These neonatal infections often lead to a BOARDS; TOPNOTCH
C. Vertical transmission carrier state for life. MD FROM FATIMA)
D. Blood transfusion
E. Homosexual transmission

192 Variant of Renal Cell Carcinoma which is made SIMILAR TO PREVIOUS BOARD EXAM JEAN PAOLO M. FINAL EXAM -
up of pale eosinophilic cells, often with a CONCEPT/PRINCIPLE.. Chromophobe renal carcinoma DELFINO, MD (TOP AUG 2015
perinuclear halo, arranged in solid sheets with is made up of pale eosinophilic cells, often with a 10 - FEB 2015 MED
a concentration of the largest cells around perinuclear halo, arranged in solid sheets with a BOARDS; TOPNOTCH
blood vessels? concentration of the largest cells around blood MD FROM FATIMA)
A. Clear cell CA vessels.
B. Papillary CA
C. Chromophobe CA
D. Collecting Duct CA
E. Urothelial CA
193 This agent predisposes susceptible populations The uses of cadmium include yellow pigments and JEAN PAOLO M. FINAL EXAM -
to develop prostate cancer? phosphors; found in solders; used in batteries and as DELFINO, MD (TOP AUG 2015
A. Nickel alloy and in metal platings and coatings. It is 10 - FEB 2015 MED
B. Chromium associated with prostate cancer. Nickel is associated BOARDS; TOPNOTCH
C. Cadmium with nose, lung cancer; Benzene- Leukemia, Hodgkin MD FROM FATIMA)
D. Vinyl chloride lymphoma; Chromium- lungs; Vinyl chloride-
E. Benzene Angiosarcoma, liver.

194 Metastasis unequivocally marks a tumor as All cancers metastasize except glioma and basal cell JEAN PAOLO M. FINAL EXAM -
malignant. What cancer does not metastasize? carcinoma. Lipoma does not metastasize. But the DELFINO, MD (TOP AUG 2015
A. hepatoma question asks what "cancer" does not metastasize. 10 - FEB 2015 MED
B. seminoma Lipoma is benign so it should be ruled out from the BOARDS; TOPNOTCH
C. lipoma choices. Glioma is the correct answer MD FROM FATIMA)
D. glioma
E. meningioma

195 True of chronic bronchitis except Although the numbers of goblet cells increase slightly, JEAN PAOLO M. FINAL EXAM -
A. Grossly, there may be hyperemia, swelling the major change is in the size of the mucous gland DELFINO, MD (TOP AUG 2015
and edema of the mucus membranes (hyperplasia). This increase can be assessed by the 10 - FEB 2015 MED
B. The ratio of the number of mucus glands to ratio of the thickness of the mucous gland layer to the BOARDS; TOPNOTCH
the thickness of the wall is increased thickness of the wall between the epithelium and the MD FROM FATIMA)
C. There is excessive mucinous to cartilage (Reid index).
mucopurulent secretions layering the epithelial
surfaces
D. The major change is in the size of the
mucus glands
E. All are true
196 20 year old male presented with history of Diagnosis for this case is Churg-Strauss Syndrome. It JEAN PAOLO M. FINAL EXAM -
allergic rhinitis, asthma and recurrent sinusitis. is a small-vessel necrotizing vasculitis classically DELFINO, MD (TOP AUG 2015
An autoimmune etiology is suspected. What is associated with asthma, allergic rhinitis, lung 10 - FEB 2015 MED
the most likely autoantibody involved in this infiltrates, peripheral hypereosinophilia, and BOARDS; TOPNOTCH
condition? extravascular necrotizing granulomas. The MD FROM FATIMA)
A. Anti-myeloperoxidase Ab autoantibody implicated in this condition is the anti-
B. Anti-proteinase 3 Ab myeloperoxidase antibody and p-ANCA. B-Wegener's;
C. Anti-endomysial Ab C- Celiac Disease; D- primary biliary cirrhosis; E-
D. Anti-mitochondrial Ab Crohn's disease.
E. Anti-saccharomyces Ab

TOPNOTCH MEDICAL BOARD PREP PATHOLOGY SUPEREXAM Page 26 of 99


For inquiries visit www.topnotchboardprep.com.ph or email us at topnotchmedicalboardprep@gmail.com
TOPNOTCH MEDICAL BOARD PREP PATHOLOGY SUPEREXAM
For inquiries visit www.topnotchboardprep.com.ph or email us at topnotchmedicalboardprep@gmail.com
Item QUESTION EXPLANATION AUTHOR TOPNOTCH
# EXAM
197 What is the most common site of ectopic Aberrantly situated, or ectopic, pancreatic tissue is JEAN PAOLO M. FINAL EXAM -
pancreas? found in about 2% of careful routine postmortem DELFINO, MD (TOP AUG 2015
A. duodenum examinations. The favored sites for ectopia are the 10 - FEB 2015 MED
B. jejunum stomach and duodenum, followed by the jejunum, BOARDS; TOPNOTCH
C. ileum Meckel diverticula, and ileum. MD FROM FATIMA)
D. Meckel's diverticulum
E. spleen

198 8 year old male patient abruptly develops Diagnosis is PSGN. The electron microscopy finding in JEAN PAOLO M. FINAL EXAM -
malaise, fever, nausea, oliguria, and hematuria this condition is subepithelial humps. A- minimal DELFINO, MD (TOP AUG 2015
2 weeks after recovery from a sore throat. The change disease; B- MPGN; C- IgA nephropathy; D- 10 - FEB 2015 MED
patient has red cell casts in the urine, mild Membranous GN BOARDS; TOPNOTCH
proteinuria, periorbital edema, and MD FROM FATIMA)
hypertension. What is the expected electron
microscopy finding in this case?
A. Loss of foot processes
B. Subendothelial deposits
C. Mesangial and paramesangial dense
deposits
D. Subepithelial deposits
E. Subepithelial humps
199 In the heart, grossly apparent bands of The terms steatosis and fatty change describe JEAN PAOLO M. FINAL EXAM -
yellowed myocardium alternating with bands abnormal accumulations of triglycerides within DELFINO, MD (TOP AUG 2015
of darker, red-brown, uninvolved myocardium parenchymal cells. This is most often seen in the liver 10 - FEB 2015 MED
is known as tigered effect. This is due to and heart. In the heart, fat deposits create grossly BOARDS; TOPNOTCH
accumulation of what material in the cardiac apparent bands of yellowed myocardium alternating MD FROM FATIMA)
cells? with bands of darker, red-brown, uninvolved
A. calcium myocardium (tigered effect)
B. glycogen
C. triglyceride
D. cholesterol
E. lipofuscin
200 What is the immunologically mediated Acute Rheumatic Fever is a Type 2 hypersensitivity JEAN PAOLO M. FINAL EXAM -
pathologic lesion seen in Acute Rheumatic reaction. It is an antibody mediated reaction causing DELFINO, MD (TOP AUG 2015
Fever? phagocytosis and lysis of cells; inflammation; in some 10 - FEB 2015 MED
A. Vascular dilation, edema, smooth muscle diseases, functional derangements without cell or BOARDS; TOPNOTCH
contraction tissue injury. A- Type 1; B- type 3; D- type 4 MD FROM FATIMA)
B. Necrotizing vasculitis
C. Phagocytosis and cell lysis
D. Perivascular cellular infiltrates
E. B and C
201 The gene involved in Williams syndrome is: Williams syndrome: supravalvular aortic stenosis, GRACE ARVIOLA, MD DIAGNOSTIC
A. Fibrillin hypercalcemia, cognitive abnormalities, and hallmark (TOP 3 - AUG 2014 EXAM - FEB
B. Collagen facial anomalies. MED BOARDS; 2015
C. Elastin TOPNOTCH MD)
D. Reticulin
E. Chondroitin

202 Anti-centromere antibodies are present in: CREST syndrome: calcinosis, Raynaud's phenomenon, GRACE ARVIOLA, MD DIAGNOSTIC
A. Sjogren syndrome esophageal dysmotility, sclerodactyly, and (TOP 3 - AUG 2014 EXAM - FEB
B. SLE telangiectasia. MED BOARDS; 2015
C. Wegener's granulomatosis TOPNOTCH MD)
D. CREST syndrome
E. Churg-Strauss syndrome

203 The presence of Heinz bodies and bite cells in a Heinz bodies: RBCs with denatured hemoglobin. Bite GRACE ARVIOLA, MD DIAGNOSTIC
patient having hemolytic anemia strongly cells results when splenic macrophages pluck out (TOP 3 - AUG 2014 EXAM - FEB
suggests: these inclusions. MED BOARDS; 2015
A. Sickle cell anemia TOPNOTCH MD)
B. G6PD deficiency
C. Alpha thalassemia
D. Multiple myeloma
E. Hereditary spherocytosis

204 Nocturnal pain relieved by aspirin intake is The pain is probably caused by excessive GRACE ARVIOLA, MD DIAGNOSTIC
characteristic of: prostaglandin E2 (PGE2) production by the (TOP 3 - AUG 2014 EXAM - FEB
A. Osteomyelitis proliferating osteoblasts. MED BOARDS; 2015
B. Osteoid osteoma TOPNOTCH MD)
C. Chondroma
D. Osteosarcoma
E. Ossifying fibroma

205 What is the most important factor in the Location in the jejunum or ileum carries the worst GRACE ARVIOLA, MD DIAGNOSTIC
prognosis of GI carcinoid? prognosis. (TOP 3 - AUG 2014 EXAM - FEB
A. Size MED BOARDS; 2015
B. Nuclear grade TOPNOTCH MD)
C. Location
D. Metastasis
E. Presence of other tumors

TOPNOTCH MEDICAL BOARD PREP PATHOLOGY SUPEREXAM Page 27 of 99


For inquiries visit www.topnotchboardprep.com.ph or email us at topnotchmedicalboardprep@gmail.com
TOPNOTCH MEDICAL BOARD PREP PATHOLOGY SUPEREXAM
For inquiries visit www.topnotchboardprep.com.ph or email us at topnotchmedicalboardprep@gmail.com
Item QUESTION EXPLANATION AUTHOR TOPNOTCH
# EXAM
206 Which condition is most likely to give rise to In hereditary tyrosinemia, almost 40% of patients GRACE ARVIOLA, MD DIAGNOSTIC
hepatocellular carcinoma? develop the tumor despite adequate dietary control. (TOP 3 - AUG 2014 EXAM - FEB
A. Hereditary tyrosinemia However, this condition is extremely rare. MED BOARDS; 2015
B. Chronic Hepatitis B TOPNOTCH MD)
C. Chronic Hepatitis C
D. Chronic Alcoholism
E. Autoimmune hepatitis

207 Mucocutaneous lymph node syndrome AKA Kawasaki disease because it presents with GRACE ARVIOLA, MD DIAGNOSTIC
preferentially affects the: conjunctival and oral erythema and erosion, edema of (TOP 3 - AUG 2014 EXAM - FEB
A. Temporal artery the hands and feet, erythema of the palms and soles, a MED BOARDS; 2015
B. Aortic arch desquamative rash, and cervical lymph node TOPNOTCH MD)
C. Renal arteries involvement. Approximately 20% of untreated
D. Coronary arteries patients develop cardiovascular sequela involving the
E. Tibial and radial arteries coronary arteries.

208 A person with hypothyroidism and Pendred syndrome is caused by a mutation in the GRACE ARVIOLA, MD DIAGNOSTIC
sensorineural deafness most likely has an SLC26A4 gene whose product, pendrin, is an anion (TOP 3 - AUG 2014 EXAM - FEB
abnormality of the: transporter expressed on the apical surface of MED BOARDS; 2015
A. H-P-O axis thyrocytes and in the inner ear. TOPNOTCH MD)
B. Thyroglobulin
C. Anion transporter
D. Iodide channel
E. Peripheral deiodinases

209 What is the most common clinical Primary hyperparathyroidism is the most common GRACE ARVIOLA, MD DIAGNOSTIC
manifestation among patients with Sipple manifestation in MEN 1 (Wermer syndrome). (TOP 3 - AUG 2014 EXAM - FEB
Syndrome? MED BOARDS; 2015
A. Pheochromocytoma TOPNOTCH MD)
B. Medullary carcinoma of the thyroid
C. Parathyroid adenoma
D. Neuroganglioma
E. Pituitary adenoma
210 Coffin lid appearance is seen in calculi Struvite stones are composed of magnesium GRACE ARVIOLA, MD DIAGNOSTIC
composed of: ammonium phosphate. (TOP 3 - AUG 2014 EXAM - FEB
A. Calcium oxalate MED BOARDS; 2015
B. Cystine TOPNOTCH MD)
C. Magnesium ammonium phosphate
D. Uric acid
E. Calcium carbonate

211 Which are considered vascular phenomena in Osler nodes and Roth spots are immunologic GRACE ARVIOLA, MD DIAGNOSTIC
infective endocarditis? phenomena. Osler nodes are subcutaneous nodules in (TOP 3 - AUG 2014 EXAM - FEB
A. Janeway lesions and Osler nodes the pulp of the digits. Roth spots are retinal MED BOARDS; 2015
B. Splinter hemorrhages and Roth spots hemorrhages in the eyes. Splinter hemorrhages are TOPNOTCH MD)
C. Janeway lesions and Roth spots micro-thromoboemboli. Janeway lesions are
D. Splinter hemorrhages and Janeway lesions erythematous or hemorrhagic nontender lesions on
E. Osler nodes and Roth spots the palms or soles.

212 Which heavy metal plays a role in the treatment AML with the t(15,17) (promyelocytic leukemia) is GRACE ARVIOLA, MD DIAGNOSTIC
of relapsing acute promyelocytic leukemia? treated with pharmacologic doses of ATRA (all-trans (TOP 3 - AUG 2014 EXAM - FEB
A. Arsenic retinoic acid) combined with conventional MED BOARDS; 2015
B. Lead chemotherapy, or more recently, with arsenic salts, TOPNOTCH MD)
C. Copper which appear to cause PML-RARa to be degraded.
D. Mercury
E. Zinc

213 Which characteristic of a tumor will make it Poorly vascularized, poorly oxygenated, and a central GRACE ARVIOLA, MD DIAGNOSTIC
less radiosensitive? location makes a tumor less radiosensitive. (TOP 3 - AUG 2014 EXAM - FEB
A. Peripheral location MED BOARDS; 2015
B. Highly oxygenated TOPNOTCH MD)
C. Poorly vascularized
D. A and B
E. B and C

214 Subacute combined degeneration of the spinal The combined degenration of both ascending and GRACE ARVIOLA, MD DIAGNOSTIC
cord is seen in deficiency of which vitamin? descending tracts of the spinal cord is characteristic of (TOP 3 - AUG 2014 EXAM - FEB
A. B1 vitamin B12 deficiency. MED BOARDS; 2015
B. B2 TOPNOTCH MD)
C. B3
D. B5
E. B12

215 A 38 year old male patient presents with a Anti-Saccharomyces antibodies are present in Crohn's GRACE ARVIOLA, MD DIAGNOSTIC
longstanding history of abdominal pain and disease. (TOP 3 - AUG 2014 EXAM - FEB
intermittent diarrhea. Imaging studies showed MED BOARDS; 2015
cobblestone appearance of the colon with TOPNOTCH MD)
ulcers that were sporadically located. Biopsy
revealed non-caseating granulomas. What
antibodies might this patient potentially have?
A. p-ANCA
B. c-ANCA
C. Anti-Saccharomyces antibodies
D. Antibodies to gliadin
E. Anti-mitochondrial antibodies

TOPNOTCH MEDICAL BOARD PREP PATHOLOGY SUPEREXAM Page 28 of 99


For inquiries visit www.topnotchboardprep.com.ph or email us at topnotchmedicalboardprep@gmail.com
TOPNOTCH MEDICAL BOARD PREP PATHOLOGY SUPEREXAM
For inquiries visit www.topnotchboardprep.com.ph or email us at topnotchmedicalboardprep@gmail.com
Item QUESTION EXPLANATION AUTHOR TOPNOTCH
# EXAM
216 What fusion gene is carried in the Philadelphia CML is distinguished from other myeloproliferative GRACE ARVIOLA, MD DIAGNOSTIC
chromosome of CML? disorders by the presence of a chimeric BCR-ABL gene (TOP 3 - AUG 2014 EXAM - FEB
A. C-myc and N-myc derived from portions of the BCR gene on MED BOARDS; 2015
B. BCR-ABL chromosome 22 and the ABL gene on chromosome 9. TOPNOTCH MD)
C. BRCA1 and BRCA2
D. NOD2
E. JAK/STAT

217 A 42 year old female presents with a 5 year The histologic hallmark is the presence of dysohesive GRACE ARVIOLA, MD DIAGNOSTIC
history of palpable breast mass. Biopsy showed infiltrating tumor cells, often arranged in a single file (TOP 3 - AUG 2014 EXAM - FEB
signet ring cells arranged in an Indian file pattern or in loose clusters or sheets. Signet ring MED BOARDS; 2015
pattern. The mass is most likely: cellscontaining an intracytoplasmic mucin droplet are TOPNOTCH MD)
A. Invasive ductal carcinoma common.
B. Invasive lobular carcinoma
C. Paget's disease of the breast
D. Carcinoma in situ
E. Medullary carcinoma of the breast
218 The chest x-ray of a cyanotic infant revealed an This is transposition of the great arteries. Choice A is GRACE ARVIOLA, MD DIAGNOSTIC
egg-shaped heart. Which statement is correct? TAPVC, choice B is TOF, choice D is endocardial (TOP 3 - AUG 2014 EXAM - FEB
A. The pulmonary veins empty into the right cushion defect, and choice E is coarctation of the aorta. MED BOARDS; 2015
atrium. TOPNOTCH MD)
B. The underlying pathology is obstruction of
the right ventricular outflow tract.
C. This condition is common among infants of
diabetic mothers.
D. The infant probably has Down's syndrome.
E. The aorta is constricted at a site just distal
to the ligamentum arteriosum.
219 A patient with retroperitoneal fibrosis is also at Retroperitoneal fibrosis is associated with Reidel GRACE ARVIOLA, MD DIAGNOSTIC
risk of having: thyroiditis, PSC, and right-sided varicocele. (TOP 3 - AUG 2014 EXAM - FEB
A. Left-sided varicocele MED BOARDS; 2015
B. Hashimoto's thyroiditis TOPNOTCH MD)
C. Crohn's disease
D. Primary sclerosing cholangitis
E. Whipple's disease

220 This subtype of Hodgkin's lymphoma is In LP, the Reed-Sternberg cells have a distinctive B- GRACE ARVIOLA, MD DIAGNOSTIC
considered non-classical. cell immunophenotype that differs from that of the (TOP 3 - AUG 2014 EXAM - FEB
A. Nodular sclerosis classical types. MED BOARDS; 2015
B. Lymphocyte-predominant TOPNOTCH MD)
C. Lymphocyte-rich
D. Lymphocyte-depleted
E. Mixed cellularity

221 Which of the following is true regarding dense dense deposit disease (MPGN type II) - is a primary LEAN ANGELO MIDTERM
deposit disease? type MPGN associated with activation of the SILVERIO, MD (TOP 4 EXAM 1 - FEB
A. Characterized by immune complex deposits alternative pathway. This is based on the diminished - AUG 2014 MED 2015
in the glomerular basement membrance and serum levels of factor B and properdin along with BOARDS; TOPNOTCH
activiation of classical pathway normal C1 an C4 levels. Ultrastructurally, type II MD), MD
B. Highly responsive to treatment with MPGN is characterized by deposition of dense material
immunosuppresive drugs such as pulse along the GBM proper. Natural history of patients with
cyclophosphamide combined with steroids this disease showed refractory to combined pulse
C. Diminished serum levels of factor B and cyclophosphamide and steroids. there is also high
properdin incidence of recurrence among transplant patient
D. Dense materials are primarily deposited on compared to type I MPGN . Robbins 8th ed pp 928-
the podocyte-GBM junction 929
E. Less recurrence among allograft recipients
compared to Type I MPGN
222 54 y/o male patient went for an OPD consult A 54 y/o patient complaining of nocturia and LEAN ANGELO MIDTERM
secondary to a chief complaint of urinary frequency along with constellation of clinical findings SILVERIO, MD (TOP 4 EXAM 1 - FEB
frequency and nocturia. PMHx: CABG-2 years such as on diabetic and on insulin therapy, - AUG 2014 MED 2015
ago. FM: (+) DM (+) Htn both parents. he is on hypertensive, (+) Family history, obese, bipedal BOARDS; TOPNOTCH
insulin therapy since he was 45 y/o; other edema without any overt clinical evidence of infection MD), MD
medications include the ff: losartan, metoprolol, is most likely suffering from DM nephropathy. renal
ACEI. pertinent physical examination showed morphologic changes include the following.
the following data: BP 140/80, PR 98 bpm, T Widespread thickening of the capillary basement
37.1C; BMI: 34 kg/m2, soft nontender membrane. diffuse increase of mesangial matrix
abdomen, no suprapubic tenderness, no CVA secondary to GBM thickening with minimal mesangial
tenderness, (+) bipedal edema. which of the cell proliferation. Presence of PAS positive nodules (
following is a correct morphologic Kimmelsteil Wilson nodules) along the periphery of
characteristics behind his most plausible cause the glomerulus. diffuse hyaline arteriolosclerosis
of urinary complaint? affecting both afferent and efferent arterioles.
A. presence of focal thickening of the Robbins 8th ed pp 1140-1141
glomerular capillary basement membrane
B. presence of mesangial proliferation
secondary to hyperplasia of the mesangial cells
C. PAS negative nodular lesions noted on the
periphery of the glomerulus.
D. presence of hyaline arteriolosclerosis
affecting both afferent and efferent arterioles
E. All of the above

TOPNOTCH MEDICAL BOARD PREP PATHOLOGY SUPEREXAM Page 29 of 99


For inquiries visit www.topnotchboardprep.com.ph or email us at topnotchmedicalboardprep@gmail.com
TOPNOTCH MEDICAL BOARD PREP PATHOLOGY SUPEREXAM
For inquiries visit www.topnotchboardprep.com.ph or email us at topnotchmedicalboardprep@gmail.com
Item QUESTION EXPLANATION AUTHOR TOPNOTCH
# EXAM
223 A 45 y/o male patient brought to ER due to the rise of the serum creatinine and presence of LEAN ANGELO MIDTERM
massive hematochezia, he is a known case of oliguria after an hypotensive episode is suggestive of SILVERIO, MD (TOP 4 EXAM 1 - FEB
liver cirrhosis secondary to Chronic Hep B ischemic type AKI. Morphologic changes include - AUG 2014 MED 2015
infection. Clinical findings are of the following: patchy necrosis primarily along the the straight BOARDS; TOPNOTCH
lethargic, BP 60 mmHg palpatory, PR 145 bpm, portion of the PCT and ascending limb of henle, MD), MD
pale palpebral conjunctiva, icteric sclera, cold eosinophilic cast composed of tamm horsfall protein
clammy extremities. he was successfully are noted along the ascending limb and the distal
stabilized after undergoing endoscopic band tubules. In contrast, toxin mediated AKI is manifested
ligation along with blood transfusion and by diffuse damage along the whole length of the PCT.
intravenous hydration. during his hospital stay, based on lab findings and having oliguria, patient is
there was a noted increase in the serum still on maintenance phase. It is the recovery phase
creatinine from 1.1 -2.1 mg/dl accompanied by that is associated with increase urinary volume,
oliguria. which of the following is true hypokalemia and susceptibility to infection. Robbins
regarding the cause of the oliguria? 8th ed pp 937-938
A. the most affected nephron segment is
proximal segment of the proximal convoluted
tubule
B. he is at the stage wherein there is increased
risk for generalized infection and electrolyte
abnormalities like hypokalemia
C. cell swelling, vacuolization and
tubulorrhexis primarily along the descending
limb of henle
D. Eosinophilic hyaline cast noted along the
ascending limb of henle and Distal tubules
E. all of the above
224 A 66 y.o male went for consult secondary to CLL is the most common leukemia of adults in the LEAN ANGELO MIDTERM
recent onset of weight loss accompanied by western countries. It is distinguished to SLL only by SILVERIO, MD (TOP 4 EXAM 1 - FEB
anorexia and easy fatigability. PE findings absolute lymphocytic count of >4000/mm3. - AUG 2014 MED 2015
showed normotensive, tachycardic, pale Morphologic characteristics include diffuse BOARDS; TOPNOTCH
palpebral conjunctiva, enlarged lymph nodes effacement of the LN along with loose aggregrates of MD), MD
on bilateral cervical, axillary, inguinal regions. atypical larger lymphocytes which collectively called
Traube space is obliterated and liver edge is proliferation centers. unlike most leukemias, it is rare
5cm from right subcostal margin. CBC showed for CLL to undergo chromosomal translocation.
normocytic anemia, thrombocytopenia and furthermore, CLL also has an unknown mechanism
lymphocytosis. A diagnosis of chronic that disrupts the normal immune function resulting to
lymphocytic leukemia is entertain, which of the a decrease in antibody production. CLL is also prone
following is consistent regarding the diagnosis? to undergo Richter transformation wherein there is
A. there is diffuse effacement of lymph nodes transformation of the primary cancer to DLBCL.
by predominantly small lymphocytes along
with loose aggregrates of larger activated
lymphocytes.
B. disruption of normal immune function
accompanied with hypergammaglobulinemia
C. prolymphocytic transformation to diffuse B
cell lymphoma is rare
D. There is a high incidence of chromosomal
translocation
E. all of the above.
225 Which of the following factors is associated ALL has one of the greatest prognosis since it is highly LEAN ANGELO MIDTERM
with worse prognosis in Acute Lymphoblastic responsive to chemotherapy, however there are some SILVERIO, MD (TOP 4 EXAM 1 - FEB
Leukemia? parameters associated with worse prognosis. These - AUG 2014 MED 2015
A. Presence of philadelphia chromosome are the following: age under 2 at presentation, BOARDS; TOPNOTCH
B. Peripheral blood blast count of 80,000 presentation at adulthood or adolescence, blast count MD), MD
C. Presentation of symptoms at 5 years of age of more than 100,000; presence of phidelphia
D. presence of chromosomal translocation t chromosome. On the otherhand, favorable prognostic
(12,21) factors include the ff: an age 2-10 years old, a low
E. hyperploidy white cell count, hyperploidy, trisomy of chromosome
4,7,and 10, presence of t(12,21) Robbins 8th ed pp
603
226 A 72 y/o male went for consult secondary to 3 expect clinical scenarios in your board exam. An LEAN ANGELO MIDTERM
days of fever accompanied by productive elderly patient presenting with recurrent pneumonia, SILVERIO, MD (TOP 4 EXAM 1 - FEB
cough. History revealed that he had multiple low back pain and polyuria supported with lab - AUG 2014 MED 2015
episodes of pneumonia within the last 3 years evidence of pancytopenia and radiographic finding of BOARDS; TOPNOTCH
.aside from the primary complaint, he also had compression fractures of lesions on vertebral column MD), MD
polyuria, continuous low back pain even at rest is consistent with multiple myeloma. features of MM
and constipation. PE are normal except for the stems from the effect of plasmacytic growth on axial
ff: pale conjunctiva, bibasilar crackles, skeleton, production of excessive immunoglobulins
hypotonic bowel sounds, paravertebral and alteration of humoral immunity. Because of the
tenderness, smooth, non nodular slightly factors produced by plasma cells, it causes the
enlarged prostate, weak bilateral LE strength activation of osteoclast leading to bone resorption and
2/5. Radiographic examination showed attendant hypercalcemia. Definitive diagnosis is made
pneumonia of bilateral lower lobes, with only by doing bone marrow examination. Patient
incidental finding of thoracic compression usually suffers renal insufficiency secondary to bence
fractures of t5-t6 t8-t9 level and multiple jones proteinuria. Infection is the most common cause
lucencies along the thoracic vertebral bodies. of death among this patient secondary to abnormal
CBC showed normocytic normochromic immunoglobulin production and decreased number of
anemia, leukopenia, thrombocytopenia. UA white blood cells. MGUS and not MM is the most
revealed massive protenuria. which of the common plasma cell dyscrasia
following is consistent with the most possible
primary diagnosis
A. patient is suffering from hypocalcemia and
hypoglobulinemia
B. it is the most common plasma cell dyscrasia
C. definitive diagnosis can be established only
through clinical and radiographic findings

TOPNOTCH MEDICAL BOARD PREP PATHOLOGY SUPEREXAM Page 30 of 99


For inquiries visit www.topnotchboardprep.com.ph or email us at topnotchmedicalboardprep@gmail.com
TOPNOTCH MEDICAL BOARD PREP PATHOLOGY SUPEREXAM
For inquiries visit www.topnotchboardprep.com.ph or email us at topnotchmedicalboardprep@gmail.com
Item QUESTION EXPLANATION AUTHOR TOPNOTCH
# EXAM
D. Infection is the most common cause of
death
E. all of the above

227 Which of the following is not an immune Multiple sclerosis is an example of type IV LEAN ANGELO MIDTERM
complex mediated disease? hypersensitivity reaction. Other type III or immune SILVERIO, MD (TOP 4 EXAM 1 - FEB
A. Systemic Lupus Erythematosus complex mediated reaction aside from the choices are - AUG 2014 MED 2015
B. Multiple sclerosis serum sickness, arthus reaction and reactive bacterial BOARDS; TOPNOTCH
C. Polyarteritis nodosa arthritis. MD), MD
D. Poststreptococcal glomerulonephritis
E. None of the above

228 What is the most serious complication of the most serious complication of chronic tuberculous LEAN ANGELO MIDTERM
Tuberculous meningitis? meningitis is arachnoid fibrosis leading to SILVERIO, MD (TOP 4 EXAM 1 - FEB
A. Tuberculoma formation hydrocephalus and obliterative endarteritis leading to - AUG 2014 MED 2015
B. Fibrinous basal exudates leading to cranial brain infarction. This is SIMILAR TO PREVIOUS BOARDS; TOPNOTCH
nerve palsies BOARD EXAM CONCEPT/PRINCIPLE. MD), MD
C. Choroid plexus involvement leading to
diffuse meningoencephalitis
D. Obliterative endarteritis
E. none of the above
229 A 42 y/o male presents with right abdominal Von hippel lindau disease is a type of familial tumor LEAN ANGELO MIDTERM
mass associated with gross hematuria. He also syndrome characterized by renal cell carcinoma, SILVERIO, MD (TOP 4 EXAM 1 - FEB
had episodes of diaphoresis and refractory pheochromocytoma and cerebellar - AUG 2014 MED 2015
hypertension. Imaging showed renal mass and hemangioblastoma. Sipple syndrome aka MEN type BOARDS; TOPNOTCH
adrenal medullary tumor. What is your primary IIA is a triad of parathyroid hyperplasia, medullary MD), MD
diagnosis? carcinoma, pheochromocytoma. Tuberous sclerosis is
A. Sipple syndrome associated with renal angiomyolipoma, retinal
B. Tuberous sclerosis hamartoma,cortical tubers, cardiac rhabdomyoma,
C. Von hippel lindau syndrome shagreen patches and ash leaf patch. Gorlin
D. Li Fraumeni syndrome syndrome.Li Fraumeni syndrome is caused by p53
E. None of the above mutation and associated with sarcomas, breast cancer,
adrenal cortical tumors, leukemia and gliomas.
230 What is the most striking histologic finding in SIMILAR TO PREVIOUS BOARD EXAM LEAN ANGELO MIDTERM
Desquamative Interstitial Pneumonia CONCEPT/PRINCIPLE last aug 2014. taken verbatim SILVERIO, MD (TOP 4 EXAM 1 - FEB
secondary to smoking? Robbins 8th ed pp 704 - AUG 2014 MED 2015
A. Thickened alveolar septa due to sparse BOARDS; TOPNOTCH
inflammatory lymphocytic infiltrate MD), MD
B. Mild interstitial fibrosis
C. Necrotic type II pneumocytes
D. accumulation of a large number of
macrophages with brown pigment cytoplasmic
bodies.
E. none of the above
231 An 18 yo female patient presents with chronic Menetrier disease is characterized by diffuse LEAN ANGELO MIDTERM
epigastric pain associated with weight loss, hyperplasia of the foveolar epithelium of the body and SILVERIO, MD (TOP 4 EXAM 1 - FEB
diarrhea, and peripheral edema. Upon fundus of the stomach. It is secondary to proliferation - AUG 2014 MED 2015
endoscopy, menetrier disease was given as a of mucous neck cells and not the gastric connective BOARDS; TOPNOTCH
diagnosis. Which of the following is true tissue. The hyperplasia is secondary to the excessive MD), MD
regarding the diagnosis? secretion of TGF alpha. although it is a self limiting
A. there is an irregular enlargement of the condition, it has an increased risk for the development
gastric rugal folds secondary to hyperplasia of of gastric adenocarcinoma.
gastric connective tissue
B. associated with excessive secretion of
platelet derived growth factor
C. it is a progressive unremmittng condition
however no risk for gastric adenocarcinoma
D. characterized by hyperplasia of foveolar
mucous neck cells
E. all of the above
232 Which of the following pathogenic organism Rotavirus is the most common cause of severe LEAN ANGELO MIDTERM
can cause diarrhea by selectively destroying childhood diarrhea. It affects only the apical and mid SILVERIO, MD (TOP 4 EXAM 1 - FEB
apical mature enterocytes of the small villous enterocytes leading to loss of absorptive - AUG 2014 MED 2015
intestine? capacity of the small intestine. All the other virus BOARDS; TOPNOTCH
A. Norovirus shows nonspecific affectation of different population MD), MD
B. adenovirus of enterocytes. Robbins 8th ed pp 804-805.
C. Rotavirus
D. Norwalk virus
E. All of the above

TOPNOTCH MEDICAL BOARD PREP PATHOLOGY SUPEREXAM Page 31 of 99


For inquiries visit www.topnotchboardprep.com.ph or email us at topnotchmedicalboardprep@gmail.com
TOPNOTCH MEDICAL BOARD PREP PATHOLOGY SUPEREXAM
For inquiries visit www.topnotchboardprep.com.ph or email us at topnotchmedicalboardprep@gmail.com
Item QUESTION EXPLANATION AUTHOR TOPNOTCH
# EXAM
233 A 61 y/o male presents with hesitancy, dysuria, Benign Prostatic hyperplasia is characterized by LEAN ANGELO MIDTERM
and nocturia. BPH is entertained. Which of the hyperplasia of the stromal and epithelial cells along SILVERIO, MD (TOP 4 EXAM 1 - FEB
following is true regarding the diagnosis? ther prostatic periurethral zone. The hallmark is the - AUG 2014 MED 2015
A. This is secondary to hypertrophy of the presence of nodularity. The compression of the BOARDS; TOPNOTCH
stromal and epithelial cells adjacent normal prostate by the nodules creates a MD), MD
B. the microscopic hallmark is nodularity pseudocapsular plane. the pathogenesis behind the
C. presence of true capsule creates a plane hyperplasia is not secondary to increase mitosis but
between the normal tissue and prostatic rather impaired cell death.
nodules
D. the cause of the nodularity is secondary to
increase mitosis of the epithelial cells
E. all of the above
234 what is the most serious consequence of Alkaptonuria is an autosomal recessive disorder LEAN ANGELO MIDTERM
alkaptonuria? secondary to lack of homogentisic oxidase which SILVERIO, MD (TOP 4 EXAM 1 - FEB
A. Renal failure converts homogentisic acid to methylacetoacetic acid. - AUG 2014 MED 2015
B. Hypertrophic cardiomyopathy It causes black discoloration of the urine if it allows to BOARDS; TOPNOTCH
C. Arthropathy stand on ambient air. It accumulates throughout the MD), MD
D. Liver failure body causing blue to black pigmentation of soft tissue
E. None of the above such as ears, nose and face. however, it also causes
pigmentation of the articular cartilages causing its loss
of resiliency and fibrillation. there is no association
b/w alkaptonuria and cardiomyopathy,liver failure
and renal failure.
235 Which of the following describes the resolution B- congestion, C- red hepatization, D- gray LEAN ANGELO MIDTERM
stage of lobar pneumonia hepatization. Robbins 8th ed pp 712-713 SILVERIO, MD (TOP 4 EXAM 1 - FEB
A. Progressive enzymatic digestion of alveolar - AUG 2014 MED 2015
exudates producing granular semifluid debris BOARDS; TOPNOTCH
ingested by macrophage MD), MD
B. Vascular dilatation with intraalveolar fluid
and presence of numerous bacteria
C. massive confluent exudation with
neutrophils, rbc, and alveolar fibrin
D. progressive disintegration of RBC and
presence of fibrinosuppurative exudates
E. none of the above
236 A 28 y/o female veterinarian presents with 3 the most likely impression regarding this case is LEAN ANGELO MIDTERM
week history dry cough accompanied by low histoplasmosis. TB is set aside because of the absence SILVERIO, MD (TOP 4 EXAM 1 - FEB
grade fever and night sweats. CXR revealed of positive sputum smear along with negative PPD. - AUG 2014 MED 2015
cavitary lesion noted on bilateral lung apices. Histoplasmosis clinical and morphological BOARDS; TOPNOTCH
AFB showed negative for 3 specimen. PPD is presentation greatly mimicks that of TB. H. MD), MD
also negative. Which of the following is true capsulatum is internalized by macrophage after
regarding the most plausible diagnosis in this opsonization. there is also presence of caseation
case? necrosis. fulminant disemminated histoplasmosis can
A. the organism is internalized primarily by occur in immunocompromised state. This patient is
NK cells and atypical lymphocytes after also veterinarian which most likely expose to bird
opsonization with antibody droppings.
B. there is absence of caseation necrosis
C. produces concentric calcification of the
lesion ( tree bark appearance) when drug
control is achieved
D. rarely become disseminated even in
immunocompromised state.
E. all are correct.
237 A 59 y/o female presents with rapidly enlarging based on the rapidity of symptoms, the most likely LEAN ANGELO MIDTERM
anterior neck mass accompanied with diagnosis is anaplastic carcinoma. B- Medullary SILVERIO, MD (TOP 4 EXAM 1 - FEB
proggressive of dyspnea, dysphagia and carcinoma. C- papillary carcinoma. D- follicular - AUG 2014 MED 2015
hoarseness of 1 month duration. What would carcinoma BOARDS; TOPNOTCH
be the most consistent pathologic finding if MD), MD
biopsy is done to the mass?
A. large pleomorphic multinucleated giant
cells with fusiform cells
B. spindle cells with amyloid deposits on
adjacent stroma
C. finely dispersed chromatin giving a
optically clear or empty appearance of the
nucleus
D. uniform cells forming small follicles
containing colloid substance
E. None of the above.
238 A 45 y/o male presents with a rapidly growing based on the characteristics, this is most likely a case LEAN ANGELO MIDTERM
nodular skin lesion on the forehead with noted of basal cell carcinoma. A- squamous cell carcinoma, c- SILVERIO, MD (TOP 4 EXAM 1 - FEB
telangiectasia on top of the lesion. If biopsy was seborrheic keratoses, d- trichoepithelioma, e- lichen - AUG 2014 MED 2015
done, which of the following will be consistent planus BOARDS; TOPNOTCH
with the most plausible diagnosis? MD), MD
A. Highly anaplastic with necrosis and
presence of polygonal cells with numerous
areas of keratinization
B. basaloid cells with hyperchromatic nuclei
with palisading alignment in the periphery of
the tumor
C. hyperkeratosis with horn cyst
D. basaloid cell with with hair like
differentiation
E. presence of civatte bodies

TOPNOTCH MEDICAL BOARD PREP PATHOLOGY SUPEREXAM Page 32 of 99


For inquiries visit www.topnotchboardprep.com.ph or email us at topnotchmedicalboardprep@gmail.com
TOPNOTCH MEDICAL BOARD PREP PATHOLOGY SUPEREXAM
For inquiries visit www.topnotchboardprep.com.ph or email us at topnotchmedicalboardprep@gmail.com
Item QUESTION EXPLANATION AUTHOR TOPNOTCH
# EXAM
239 A 49 y/o female presented to the ER secondary Pagets disease or Osteitis deformans is a rare skeletal LEAN ANGELO MIDTERM
to right tibial fracture after a fall from a disease characterized by haphazard mosiaic pattern of SILVERIO, MD (TOP 4 EXAM 1 - FEB
standing height, Xray revealed middle lamellar bone formation. Clinically it affects the axial - AUG 2014 MED 2015
transverse fracture of the right tibia with noyed skeleton leading to compression of multiple spinal and BOARDS; TOPNOTCH
enlarged, sclerotic irregular thickening of both cranial nerve exits. Bony overgrowth of skull base also MD), MD
cortical and cancellous bone. further physical causes different cranial nerve palsies. they also
examination noted weakness of bilateral hip prominence of facial bones presenting as leontiasis
flexors and knee flexors, sensory deficit on L3- ossea.
S1 dermatome bilateral. patient presents with
prominent zygoma and supraorbital ridge with
frontal bossing. she also has lateral rectus palsy
on (R), (L) peripheral facial palsy. which of the
following is the most likely diagnosis of the
patient?
A. Pagets disease
B. early onset idiopathic osteoporosis
C. osteomalacia
D. multiple enchondromatosis
E. Osteopetrosis
240 What is the most common malignancy of the basal cell carcinoma is the most common malignancy LEAN ANGELO MIDTERM
eyelid? of the eyelid with predilection on the lower lid and SILVERIO, MD (TOP 4 EXAM 1 - FEB
A. Squamous cell carcinoma medial canthus. This is followed by sebaceous - AUG 2014 MED 2015
B. Basal cell carcinoma carcinoma and squamous cell carcinoma. Robbins 8th BOARDS; TOPNOTCH
C. Sebaceous carcinoma ed p 1348 MD), MD
D. lymphangioma
E. Hemangioma

241 In pathologic examination of a brain of a person SIMILAR TO PREVIOUS BOARD EXAM KEVIN BRYAN LO, MIDTERM 2
who died from rabies which of the following CONCEPT/PRINCIPLE, in our exam we had to choose MD (TOP 7 - AUG EXAM - FEB
areas of the brain where the pathognomonic between cerebellum or hippocampus, both answers 2014 MED BOARDS; 2015
negri bodies will be found? are correct and should be either cerebellum or TOPNOTCH MD)
A. cerebrum hippocampus
B. cerebellum
C. hippocampus
D. thalamus
E. B and C
242 a 65 year old male patient presents with a SIMILAR TO PREVIOUS BOARD EXAM KEVIN BRYAN LO, MIDTERM 2
chronic history of decreasing urinary stream, CONCEPT/PRINCIPLE MD (TOP 7 - AUG EXAM - FEB
dribbling of urine and difficulty in voding, What 2014 MED BOARDS; 2015
is the pathophysiologic process involved in a TOPNOTCH MD)
person diagnosed to have BPH?
A. hypertrophy
B. hyperplasia
C. neoplasia
D. metaplasia
E. none of the above
243 What percentage of patients with hepatitis B SIMILAR TO PREVIOUS BOARD EXAM KEVIN BRYAN LO, MIDTERM 2
eventually proceed to have hepatocellular CONCEPT/PRINCIPLE, almost exact choices, page 96 MD (TOP 7 - AUG EXAM - FEB
carcinoma? of topnotch handout in the diagram, I did not get this 2014 MED BOARDS; 2015
A. 6-15% as well during our exam :) TOPNOTCH MD)
B. 12-20%
C. >30%
D. <5%
E. 2-8%

244 A 60 year old patient with longstanding type 2 SIMILAR TO PREVIOUS BOARD EXAM KEVIN BRYAN LO, MIDTERM 2
DM recently underwent renal transplantation CONCEPT/PRINCIPLE, A is hyperacute, B is acute, C is MD (TOP 7 - AUG EXAM - FEB
of 1 kidney for end stage renal disease chronic 2014 MED BOARDS; 2015
secondary to diabetic nephropathy, after a TOPNOTCH MD)
period of 4 weeks after transplantation, he
developed sudden weight gain, bilateral pedal
edema, pain or tenderness near the right flank
area and fever, histologic examination would
most likely reveal:
A. thrombotic occlusion of capillaries and
fibrinoid necrosis
B. extensive interstitial mononuclear cell
infiltration and edema
C. vascular changes, interstitial fibrosis and
tubular atrophy with loss of renal parenchyma
D. A and B
E. all of the above
245 40 year old male patient presented with SIMILAR TO PREVIOUS BOARD EXAM KEVIN BRYAN LO, MIDTERM 2
repeated episodes of bloody stools. Workup CONCEPT/PRINCIPLE, no granulomas, no strictures, MD (TOP 7 - AUG EXAM - FEB
and evaluation revealed multiple superficial limited to the colon, submucosal inflammation, no skip 2014 MED BOARDS; 2015
broad based ulcers, with diffuse inflammation lesions TOPNOTCH MD)
limited to the colon, biopsy revealed
inflammation limited only to the submucosal
area, which of the following is the most likely
condition?
A. Crohn's disease
B. bacillary dysentery
C. amoebic colitis
D. ulcerative colitis
E. none of the above

TOPNOTCH MEDICAL BOARD PREP PATHOLOGY SUPEREXAM Page 33 of 99


For inquiries visit www.topnotchboardprep.com.ph or email us at topnotchmedicalboardprep@gmail.com
TOPNOTCH MEDICAL BOARD PREP PATHOLOGY SUPEREXAM
For inquiries visit www.topnotchboardprep.com.ph or email us at topnotchmedicalboardprep@gmail.com
Item QUESTION EXPLANATION AUTHOR TOPNOTCH
# EXAM
246 50 year old male chronic smoker presents with SIMILAR TO PREVIOUS BOARD EXAM KEVIN BRYAN LO, MIDTERM 2
chronic cough dyspnea and sputum production. CONCEPT/PRINCIPLE, panacinar is with alpha 1 MD (TOP 7 - AUG EXAM - FEB
He also has bouts of easy fatigability. Patient antitrypsin deficiency 2014 MED BOARDS; 2015
also presents with increased anteroposterior TOPNOTCH MD)
diameter of the chest wall. Patient most likely
has this type of emphysema
A. panacinar
B. centrilobular
C. paraseptal
D. alveolar
E. none of the above
247 A 56 year old male presenting with recent onset SIMILAR TO PREVIOUS BOARD EXAM KEVIN BRYAN LO, MIDTERM 2
of bipedal edema, BP was 120/80 Heart rate of CONCEPT/PRINCIPLE, most questions were cases, MD (TOP 7 - AUG EXAM - FEB
90. Does not recall any history of recent wordy, 2 step cases. A lot of renal pathology. 2014 MED BOARDS; 2015
infections but has been said to have multiple Subendothelial, granular pattern nephrotic syndrome TOPNOTCH MD)
sexual partners as well. Over the past few is membranoproliferative type
weeks, he noticed his urine to be foamy and
bubbly, and would have puffy eyelids especially
on waking in the morning. Further examination
revealed urine protein of +3, RBCs 0-2 WBC 0-
1, pus cells (+1), no casts and crystals, urine
specific gravity of 1.015, urine pH of 5.5. A renal
biopsy was eventually done revealing large
hypercellular glomeruli, thickened glomerular
basement membrane and increased mesangial
matrix. Immunofluorescence revealed granular
pattern of C3, IgG and C1q, electron microscopy
revealed subendothelial deposits, which of the
following is the most likely finding?
A. focal segmental glomerulosclerosis
B. membranous glomerulonephritis
C. rapidly progressive glomerulonephritis
D. membranoproliferative glomerulonephritis
E. minimal change disease
248 What is the most common etiologic factor in SIMILAR TO PREVIOUS BOARD EXAM KEVIN BRYAN LO, MIDTERM 2
acute myeloid leukemia? CONCEPT/PRINCIPLE, vague question, a lot of genetic MD (TOP 7 - AUG EXAM - FEB
A. radiation exposure mutations like down syndrome are commonly 2014 MED BOARDS; 2015
B. benzene associated with acute leukemias, all the others are also TOPNOTCH MD)
C. genetic mutation common causes of leukemias but less common.
D. carcinogen exposure
E. previous treatment for leukemia

249 What is the pathophysiologic mechanism in SIMILAR TO PREVIOUS BOARD EXAM KEVIN BRYAN LO, MIDTERM 2
which patients with protein energy CONCEPT/PRINCIPLE, malnutrition with low protein MD (TOP 7 - AUG EXAM - FEB
malnutrition sometimes have fatty liver levels leads to low synthesis of necessary proteins like 2014 MED BOARDS; 2015
especially with protein malnutrition? globulins and lipoprotein transporters like LDL HDL TOPNOTCH MD)
A. increased mobilization of fat stores from VLDL that transport cholesterol and trigylcerides thus
adipose tissue fats are continuously stored in the liver and not
B. increased glucose stored as fat distributed properly to the tissues
C. increased endogenous synthesis of
triglycerides
D. impaired transport of fats
E. all of the above
250 A 50 year old male with long standing history of SIMILAR TO PREVIOUS BOARD EXAM KEVIN BRYAN LO, MIDTERM 2
chronic alcoholism presents with jaundice, CONCEPT/PRINCIPLE, eosinophilic clumps are MD (TOP 7 - AUG EXAM - FEB
palmar erythema, distended abdomen, spider mallory bodies with both macro micronodular 2014 MED BOARDS; 2015
nevi near in the upper anterior chest wall. What cirrhosis in the setting of long standing alcoholism and TOPNOTCH MD)
is the most likely histologic finding present in hallmarks of cirrhosis, D is chronic hepatitis,C is acute
the liver? hepatitis B is cholestatic change
A. councilman bodies
B. fine foamy appearance with feathery
degeneration and accumulation of bile pigment
plugs in dilated canaliculi
C. apoptosis bridging necrosis, ballooning
degeneration, interface hepatitis, ground glass
hepatocyes
D. hepatocyte apoptosis bridging necrosis and
fibrosis, deposition of fibrous tissue,
E. hepatocyte swelling necrosis, eosinophilic
clumps in hepatocytes, both micro and
macronodular cirrhosis
251 What pathological entity is characterized by SIMILAR TO PREVIOUS BOARD EXAM KEVIN BRYAN LO, MIDTERM 2
widespread small vessel vasculitis, interstitial CONCEPT/PRINCIPLE, diffuse skin fibrosis and organ MD (TOP 7 - AUG EXAM - FEB
and perivascular fibrosis of the skin and changes including esophageal dysmotility are due to 2014 MED BOARDS; 2015
multiple organs, progressive atrophy and small vessel vasculitis and fibrosis TOPNOTCH MD)
collagenous replacement of alimentary canal
especially the esophagus, clawlike deformities
and limited range of motion of joints and
fingers, raynauds phenomenon, fibrinoid
necrosis of renal arterioles
A. wegeners granulomatosis
B. systemic lupus erythematosus
C. systemic sclerosis
D. churg strauss
E. polyarteritis nodosa

TOPNOTCH MEDICAL BOARD PREP PATHOLOGY SUPEREXAM Page 34 of 99


For inquiries visit www.topnotchboardprep.com.ph or email us at topnotchmedicalboardprep@gmail.com
TOPNOTCH MEDICAL BOARD PREP PATHOLOGY SUPEREXAM
For inquiries visit www.topnotchboardprep.com.ph or email us at topnotchmedicalboardprep@gmail.com
Item QUESTION EXPLANATION AUTHOR TOPNOTCH
# EXAM
252 A 30 year old male patient with 9 months SIMILAR TO PREVIOUS BOARD EXAM KEVIN BRYAN LO, MIDTERM 2
history of chronic cough with sputum CONCEPT/PRINCIPLE - obliterative endarteritis cause MD (TOP 7 - AUG EXAM - FEB
production accompanied by low grade the multiple microinfarcts from CNS TB 2014 MED BOARDS; 2015
afternoon fevers, night sweats, weight loss TOPNOTCH MD)
suddenly presents with fever, altered
sensorium and nuchal rigidity. Cranial CT scan
shows basal enhancement and multiple
infarcts, what is the pathophysiologic
mechanism of these infarcts?
A. thrombotic occlusion of the vessels causes
multiple infarcts
B. embolic seeding from extrapulmonary
tuberculotic sites
C. vasospasm induced by hemorrhage in the
CNS
D. obliterative endarteritis produced by CNS
infection
E. accelerated atherosclerotic changes
253 A 16 year old female patient presents with 2 B symptoms, mediastinal mass, contiguous growth KEVIN BRYAN LO, MIDTERM 2
months history of easy fatigability, body most likely hodgkin's lymphoma MD (TOP 7 - AUG EXAM - FEB
malaise, afternoon fevers, night sweats and 2014 MED BOARDS; 2015
weight loss. There was noted occasional cough TOPNOTCH MD)
with multiple palpable cervical lymph nodes
noted which prompted an initial assessment of
pulmonary tuberculosis, however, a chest xray
revealed an incidental mediastinal mass with
no apical lesions noted. What is the most likely
consideration?
A. primary bronchogenic carcinoma
B. hodgkin's lymphoma
C. non hodgkin's lymphoma
D. pulmonary tuberculoma
E. metastatic pulmonary carcinoma
254 50 year old postmenopausal female patient SIMILAR TO PREVIOUS BOARD EXAM KEVIN BRYAN LO, MIDTERM 2
presenting with a history of vertebral CONCEPT/PRINCIPLE, punched out lesions and MD (TOP 7 - AUG EXAM - FEB
compression fractures and hip fracture was multiple pathological fractures 2014 MED BOARDS; 2015
worked up due to repeated complains of leg TOPNOTCH MD)
and extremity pains. She was noted to have 4
episodes of flu over the past 2 months. Work
ups revealed mild anemia, slightly elevated
serum calcium and multiple lytic bone lesions
were noted over the ribs and some punched out
defects were seen in the skull xrays, which of
the following conditions does this patient most
likely have
A. osteoporosis
B. multiple myeloma
C. waldenstrom's macroglobulinemia
D. myelodysplastic syndrome
E. osteomalacia
255 35 year old female presents with an SIMILAR TO PREVIOUS BOARD EXAM KEVIN BRYAN LO, MIDTERM 2
erythematous rash on the cheeks aggravated by CONCEPT/PRINCIPLE, most specific and indicates a MD (TOP 7 - AUG EXAM - FEB
exposure to sunlight, she has been having a probable renal involvement of disease 2014 MED BOARDS; 2015
history of repeated recurrent joint pains and TOPNOTCH MD)
swelling over the ankles and wrists, she has
some oral apthous ulcers accompanied by
headaches and easy fatigability. She came in for
consult due to a recent onset of bipedal edema
accompanied by tea colored urine. Initial work
ups were requested which revealed urine
protein of +2, urine RBC of 5-10, urine WBC of
0-1, pus cells of +1, rbc casts +1, no crystals.
ANA titers were also elevated. What is the
autoantibody specific to this disease which also
associated with renal pathological involvement
A. anti-smith
B. ANA
C. anti-dsDNA
D. anti-histone
E. Anti-Ro
256 In relation to the question above, if a kidney SIMILAR TO PREVIOUS BOARD EXAM KEVIN BRYAN LO, MIDTERM 2
biopsy is done, what is the most CONCEPT/PRINCIPLE, a lot of multiple stem questions MD (TOP 7 - AUG EXAM - FEB
pathognomonic expected finding? 2014 MED BOARDS; 2015
A. nonspecific mesangial proliferation TOPNOTCH MD)
B. tram track appearance of the basement
membrane
C. spike and dome appearance
D. chicken wire loop appearance
E. focal segmental glomerulosclerosis

TOPNOTCH MEDICAL BOARD PREP PATHOLOGY SUPEREXAM Page 35 of 99


For inquiries visit www.topnotchboardprep.com.ph or email us at topnotchmedicalboardprep@gmail.com
TOPNOTCH MEDICAL BOARD PREP PATHOLOGY SUPEREXAM
For inquiries visit www.topnotchboardprep.com.ph or email us at topnotchmedicalboardprep@gmail.com
Item QUESTION EXPLANATION AUTHOR TOPNOTCH
# EXAM
257 In relation to question above, this disease SIMILAR TO PREVIOUS BOARD EXAM KEVIN BRYAN LO, MIDTERM 2
manifestation belongs to what type of immune CONCEPT/PRINCIPLE, immune complex type 3, ABO MD (TOP 7 - AUG EXAM - FEB
hypersensitivity? incompatibility also came out as type 2 2014 MED BOARDS; 2015
A. type 1 hypersensitivity TOPNOTCH MD)
B. type 2
C. type 3
D. type 4
E. non of the above

258 Which of the following cytokines secreted by T SIMILAR TO PREVIOUS BOARD EXAM KEVIN BRYAN LO, MIDTERM 2
helper lymphocytes leads to direct strong CONCEPT/PRINCIPLE from boards, IL1 and IL6 are MD (TOP 7 - AUG EXAM - FEB
activation of macrophages? pyrogens and initiate inflammation and fever, TNF 2014 MED BOARDS; 2015
A. IL1 alpha also promotes inflammation and cachexia, TGF TOPNOTCH MD)
B. TNF alpha beta inhibits inflammation and promotes cell repair
C. TNF beta
D. IFN gamma
E. IL-6

259 20 year old patient developed paraparesis. 2 SIMILAR TO PREVIOUS BOARD EXAM KEVIN BRYAN LO, MIDTERM 2
weeks after developing a diarrheal episode, CONCEPT/PRINCIPLE, GBS albuminocytologic MD (TOP 7 - AUG EXAM - FEB
patient started developing gradual ascending dissociation 2014 MED BOARDS; 2015
paralysis. No other sensory and CN deficits TOPNOTCH MD)
were seen, lumbar puncture was done, which of
the following is an expected finding?
A. oligoclonal bands IgG, CSF pleocytosis
B. increased CSF protein, WBCs 0-1
C. increased CSF protein, neutrophils 5000
D. increased CSF protein, lymphocytes 500
E. normal CSF protein, normal glucose WBCs 0-
1
260 30 year old female patient presenting with SIMILAR TO PREVIOUS BOARD EXAM KEVIN BRYAN LO, MIDTERM 2
nonspecific pneumonitis, cough, intermittent CONCEPT/PRINCIPLE, asthmatic manifestations, MD (TOP 7 - AUG EXAM - FEB
bouts of bronchial asthma with long standing pulmonary, skin, other involved organs but spares the 2014 MED BOARDS; 2015
history of allergic rhinitis. Presents with renal vessels TOPNOTCH MD)
nodular skin lesions, biopsy of skin lesions
reveal capillary vasculitis with granulomas and
eosinophilic necrosis. There was no noted renal
involvement, the most likely pathological
condition in this case is?
A. polyarteritis nodosa
B. churg strauss
C. wegeners granulomatosis
D. microscopic polyangitis
E. giant cell arteritis
261 All of the following are ultrastructural changes Karyorrhexis is nuclear fragmentation which is RAYMUND MARTIN MIDTERM 3
of reversible cell injury except: irreversible LI, MD (TOP 1 - AUG EXAM - FEB
A. Dilation of the endoplasmic reticulum 2014 MED BOARDS; 2015
B. Cell membrane blebbing TOPNOTCH MD)
C. Karyorrhexis
D. Detachment of polysomes
E. None of the above

262 A patient was diagnosed with Paget's disease Although the use of glucocorticoids may predispose to RAYMUND MARTIN MIDTERM 3
and has been treated with bisphophonates. osteoporotic fracture. The description indicates LI, MD (TOP 1 - AUG EXAM - FEB
Currently, he presents at your clinic with left Codman's triangle and is indicative of osteosarcoma. 2014 MED BOARDS; 2015
hip pain that is not relieved by over the counter Paget's disease is a predisposing factor to TOPNOTCH MD)
pain relievers? He has history of taking low osteosarcoma
dose steroids for his asthma. Imaging reveals
lytic and sclerotic changes with periosteal
elevation and reactive new bone formation.
What is the most likely diagnosis?
A. Osteopetrosis
B. Glucocorticoid-induced osteoporosis
C. Osteoarthritis
D. Osteosarcoma
E. Rheumatoid arthritis
263 Which type of small vessel vasculitis is Churg-Strauss spares the kidney and is characterized RAYMUND MARTIN MIDTERM 3
characterized by eosinophil-rich by association with asthma and eosinophilia LI, MD (TOP 1 - AUG EXAM - FEB
granulomatous inflammation involving the 2014 MED BOARDS; 2015
respiratory tract and is associated with asthma TOPNOTCH MD)
and blood eosinophilia with sparing of the
kidneys?
A. Polyarteritis nodosa
B. Wegener granulomatosis
C. Takayasu arteritis
D. Leukocytoclastic vasculitis
E. Churg-Strauss syndrome
264 Which if the following cytokines has an IL-12 - Th1; IL-4 - Th2; IL10 -suppression of RAYMUND MARTIN MIDTERM 3
important role in IFN- production in Th1- inflammation LI, MD (TOP 1 - AUG EXAM - FEB
mediated chronic inflammation? 2014 MED BOARDS; 2015
A. IL-12 TOPNOTCH MD)
B. IL-10
C. IL-4
D. IL-2
E. TNF-

TOPNOTCH MEDICAL BOARD PREP PATHOLOGY SUPEREXAM Page 36 of 99


For inquiries visit www.topnotchboardprep.com.ph or email us at topnotchmedicalboardprep@gmail.com
TOPNOTCH MEDICAL BOARD PREP PATHOLOGY SUPEREXAM
For inquiries visit www.topnotchboardprep.com.ph or email us at topnotchmedicalboardprep@gmail.com
Item QUESTION EXPLANATION AUTHOR TOPNOTCH
# EXAM
265 A patient with Duchenne muscular dystrophy is Allelic heterogeneity refers to different mutations at RAYMUND MARTIN MIDTERM 3
a muscular disorder involving the dystrophin the same locus. Variable expressivity is the same LI, MD (TOP 1 - AUG EXAM - FEB
gene that usually leads to a male patient being mutation but with different phenotypic 2014 MED BOARDS; 2015
wheelchair-bound during his teenage years. In characteristics. Anticipation occurs in trinucleotide TOPNOTCH MD)
contrast, Becker muscular dystrophy is a milder repeats and is characterized by an ealier and more
but similar form of disease wherein afflicted severe onset of disease in suceeding generations.
patients can usually still walk at the same age Incomplete penetrance means some patients with the
period. This is an example of: disease causing mutation may not present signs or
A. Pleiotropy symptoms; usually in autosominal dominant.
B. Variable expressivity Pleiotropy occurs in mitochondrial disorders.
C. Anticipation
D. Incomplete penetrance
E. Allelic heterogeneity
266 A 34 year old male smoker presents to your Thromboangiitis obliterans or Buerger disease usually RAYMUND MARTIN MIDTERM 3
clinic with leg pain. History is unremarkable in male smokers less than 35 years old. LI, MD (TOP 1 - AUG EXAM - FEB
except for a 15 year history of smoking 2014 MED BOARDS; 2015
consuming 1 pack per day. Pain worsens with TOPNOTCH MD)
exercise and is also not totally relieved by rest.
Biopsy of the affected leg reveals segmental
vasculitis of small to medium vessels
accompanied with luminal thrombosis and
microabscesses extending into contiguous veins
and nerve. What is the most likely diagnosis?
A. Thromboangiitis obliterans
B. Polyarteritis nodosa
C. Peripheral arterial atherosclerosis
D. Microscopic polyangiitis
E. Infectious vasculitis
267 A 32 year old male patient claims that he has Thymoma - associated with myasthenia gravis; Small RAYMUND MARTIN MIDTERM 3
difficulty with prolonged chewing of food and cell Ca - associated with Lambert-Eaton (improves LI, MD (TOP 1 - AUG EXAM - FEB
that he experiences diplopia when watching TV with prolonged stimulation) 2014 MED BOARDS; 2015
for long periods of time. What is neoplasm most TOPNOTCH MD)
associated with this condition?
A. Small cell lung Ca
B. Thymoma
C. Hodgkin's lymphoma
D. Renal Cell Ca
E. Breast Ca
268 A 10 year old child presents to your clinic with A is type 1 hypersensitivity; B is type II, C is type IV, D RAYMUND MARTIN MIDTERM 3
complaints of dark urine, periorbital edema, is type III LI, MD (TOP 1 - AUG EXAM - FEB
malaise, and body weakness. You elicit a history 2014 MED BOARDS; 2015
of throat infection 3 weeks prior. You expect TOPNOTCH MD)
which of the following pathological lesion for
this type of hypersensitivity reaction:
A. Vascular dilation edema, smooth muscle
contraction
B. Phagocytosis and lysis of cells
C. Perivascular cellular infiltrates, edema, and
granuloma formation
D. Inflammation with associated necrotizing
vasculitis
E. None of these
269 An young athlete dies suddenly and Hypertrophic cardiomyopathy is most commonly RAYMUND MARTIN MIDTERM 3
unexpectedly. Autopsy reveals massive caused by a mutation in myosin heavy chain LI, MD (TOP 1 - AUG EXAM - FEB
myocardial hypertrophy with disproportionate 2014 MED BOARDS; 2015
thickening of the ventricular septum. What is TOPNOTCH MD)
the most likely pathologic etiology of this
condition?
A. Previous viral myocardial infection
B. Atherosclerotic blockage of coronary arteries
C. Radiation-induced myocardial fibrosis
D. Mutation of gene encoding beta-myosin
heavy chain
E. None of the above
270 You are assigned to assist a transplant surgeon A and D is acute rejection; C is hyperacute rejection RAYMUND MARTIN MIDTERM 3
who will be performing a kidney transplant. In LI, MD (TOP 1 - AUG EXAM - FEB
order to prepare yourself for possible questions 2014 MED BOARDS; 2015
that you may be asked during the surgery, you TOPNOTCH MD)
recall your pathology knowledge.You know that
chronic rejection will be characterized by:
A. Extensive interstitial mononuclear cell
infiltration and edema
B. Dominated by vascular changes, interstitial
fibrosis
C. Occurs due to preformed antibodies is the
circulation
D. Takes the form of necrotizing vasculitis with
endothelial cell necrosis
E. A and B

TOPNOTCH MEDICAL BOARD PREP PATHOLOGY SUPEREXAM Page 37 of 99


For inquiries visit www.topnotchboardprep.com.ph or email us at topnotchmedicalboardprep@gmail.com
TOPNOTCH MEDICAL BOARD PREP PATHOLOGY SUPEREXAM
For inquiries visit www.topnotchboardprep.com.ph or email us at topnotchmedicalboardprep@gmail.com
Item QUESTION EXPLANATION AUTHOR TOPNOTCH
# EXAM
271 Which of following characteristics indicate A, B, and C is indicative of non-neoplastic proliferation RAYMUND MARTIN MIDTERM 3
neoplastic/malignant lymph node LI, MD (TOP 1 - AUG EXAM - FEB
proliferation? 2014 MED BOARDS; 2015
A. Marked variation in the shape and size of the TOPNOTCH MD)
follicles
B. Presence of phagocytic macrophages and
recognizable light and dark zones
C. Preservation of the lymph node architecture
D. Marked by monoclonal proliferation
E. All of these
272 A patient presents to your clinic with chronic Crohn's disease is most common in the ileum though it RAYMUND MARTIN MIDTERM 3
diarrhea and perianal abscess occasionally may occur anywhere in the GI tract LI, MD (TOP 1 - AUG EXAM - FEB
associated with low-grade fever. She had a 2014 MED BOARDS; 2015
prior history of intestinal fistula formation TOPNOTCH MD)
which was managed conservatively. Physical
examination reveals mouth ulcers with no
other significant findings. What is the most
commonly affected site in patients with this
condition?
A. Ileum
B. Sigmoid colon
C. Jejunum
D. Stomach
E. Duodenum
273 A mother brings to you her 1 year old child due This refers to Bruton's which is an X-linked RAYMUND MARTIN MIDTERM 3
to recurrent respiratory bacterial infections immunodeficiency syndrome LI, MD (TOP 1 - AUG EXAM - FEB
and otitis media. You suspect an 2014 MED BOARDS; 2015
immunodeficiency syndrome due to failure of TOPNOTCH MD)
preB cells to undergo Ig gene rearrangement
and to develop into mature B cells. The
mutation responsible is encoded in which
chromosome?
A. Chromosome 17
B. Chromosome 7
C. Chromosome 16
D. Chromosome 22
E. Chromosome X
274 A female patient presenting with right breast Charateristic description of invasive lobular Ca RAYMUND MARTIN MIDTERM 3
mass but with no other associated symptoms LI, MD (TOP 1 - AUG EXAM - FEB
undergoes biopsy which revealed presence of 2014 MED BOARDS; 2015
dyscohesive infiltrating tumor cells, often TOPNOTCH MD)
arranged in single file or in loose clusters or
sheets with associated signet-ring cells
containing an intracytoplasmic mucin droplet.
What is the diagnosis?
A. Invasive ductal carcinoma
B. Ductal carcinoma in situ
C. Paget's disease
D. Invasive lobular carcinoma
E. Medullary carcinoma
275 A patient complains of polydipsia and polyuria. ADH levels are adequate but kidney does not respond RAYMUND MARTIN MIDTERM 3
Laboratory tests reveal a dilute urine and in nephogenic type LI, MD (TOP 1 - AUG EXAM - FEB
hypernatremia. If this condition is a nephogenic 2014 MED BOARDS; 2015
type of diabetes insipidus, what will be TOPNOTCH MD)
expected during water deprivation test?
A. Decreased ADH levels
B. Elevated ADH levels
C. Elevated urine osmolality
D. Correction of hypernatremia
E. Low serum osmolality
276 An infant born term to G1P1 mother dies. Prior Characteristic description of cytomegalovirus in utero RAYMUND MARTIN MIDTERM 3
to death, the infant was noted to have infection. LI, MD (TOP 1 - AUG EXAM - FEB
intrauterine growth retardation, jaundice, 2014 MED BOARDS; 2015
hepatosplenomegaly, anemia, and bleeding due TOPNOTCH MD)
to thrombocytopenia. Periventricular
calcification is observed on imaging and
histology reveals prominent intranuclear
basophilic inclusions surrounded by a clear
halo. In utero infection is suspected. Which is
the most likely etiologic agent?
A. Toxoplasma
B. Herpes virus
C. Cytomegalovirus
D. Rubella
E. Treponema pallidum

TOPNOTCH MEDICAL BOARD PREP PATHOLOGY SUPEREXAM Page 38 of 99


For inquiries visit www.topnotchboardprep.com.ph or email us at topnotchmedicalboardprep@gmail.com
TOPNOTCH MEDICAL BOARD PREP PATHOLOGY SUPEREXAM
For inquiries visit www.topnotchboardprep.com.ph or email us at topnotchmedicalboardprep@gmail.com
Item QUESTION EXPLANATION AUTHOR TOPNOTCH
# EXAM
277 A patient presents with chronic cough CD4 lymphocytes and IFN- gamma are responsible for RAYMUND MARTIN MIDTERM 3
associated with night sweats, weight loss, and granulomatous reaction (type IV) LI, MD (TOP 1 - AUG EXAM - FEB
body malaise. Sputum microscopy reveals acid 2014 MED BOARDS; 2015
fast bacilli and X-ray reveals right upper lobe TOPNOTCH MD)
density? Granulomatous reaction which is
typical of this condition is brought about by
which of the following:
A. CD8 lymphocytes, Humoral immune system
B. CD4 lymphocytes, IFN
C. CD4 lymphocytes, IL-4
D. CD8 lymophocytes, NK cells
E. B cells and macrophages
278 A 16 year old African American man presents to Autosplenectomy occurs in early childhood in sickle RAYMUND MARTIN MIDTERM 3
the emergency room severe chest pain, fever, cell anemia LI, MD (TOP 1 - AUG EXAM - FEB
tachypnea, and cough. Imaging reveals lung 2014 MED BOARDS; 2015
infiltrates and ABG shows hypoxemia. Further TOPNOTCH MD)
probing reveals that patient had previous
episodes of painful crises and anemia.
Peripheral smear reveals target cells and
characteristically shaped erythrocytes. An Hb
electrophoresis is ordered but patient
deteriorates and dies. Autopsy examination of
the spleen will most likely demonstrate:
A. Red pulp expansion
B. Extramedullary hematopoiesis
C. Small fibrotic atrophy
D. Congestive changes and increase in volume
E. A and B
279 Baretts esophagus is what type of abnormal Intestinal metaplasia - replacement with simple RAYMUND MARTIN MIDTERM 3
cell/tissue change? columnar with goblet cells that predispose to LI, MD (TOP 1 - AUG EXAM - FEB
A. Dysplasia adenocarcinoma 2014 MED BOARDS; 2015
B. Metaplasia TOPNOTCH MD)
C. Anaplasia
D. Neoplastic change
E. Necrosis

280 A 30 year old woman presents to your clinic Most common form of kidney disease in SLE is diffuse RAYMUND MARTIN MIDTERM 3
complaining of a butterfly rash over the face, proliferative (type IV). B is focal proliferative (III). C is LI, MD (TOP 1 - AUG EXAM - FEB
fever, joint pains, and photosensitivity. membranous (V). D and E are mesangial (I and II) 2014 MED BOARDS; 2015
Prelimionary laboratory tests reveal positive TOPNOTCH MD)
ANA and thrombocytopenia. The pathologic
picture of the most common form of renal
disease associated with this condition is:
A. >50% of glomeruli exhibiting crescent
formation, fibrinoid necrosis, proliferation of
endothelial and mesangial cells
B. <50% of glomeruli exhibiting crescent
formation, fibrinoid necrosis, proliferation of
endothelial and mesangial cells
C. characterized by diffuse thickening of the
capillary walls
D. slight mesangial cell proliferation and
immune complex deposition
E. moderate mesangial cell proliferation and
immune complex deposition
281 A 65 year old woman with a heavy smoking Dx: Chronic Bronchitis. Hallmark is marked ERIC ROYD FINAL EXAM -
history presents with chronic productive cough hyperplasia of bronchial submucosal glands and TALAVERA, MD (TOP FEB 2015
that has been present for 3 consecutive months bronchial smooth muscle hypertrophy (with 1 - AUG 2014 MED
over the past 2 years. On PE, she is noted to be LYMPHOCYTIC) infiltrates.Choice A pertains to BOARDS; TOPNOTCH
overweight with a bluish tinge to her skin. Asthma, Choice B is bronchiectasis, Choice C is MD)
Auscultation revealed rhonchi on both lung emphysema, Choice E is ARDS
fields. Which of the following is the most likely
histologic finding in this patient's airways?
A. Bronchial smooth muscle hypertrophy with
proliferation of eosinophils
B. Permanent bronchial dilation, which is
filled with mucus and neutrophils
C. Dilation of air spaces with destruction of
alveolar walls
D. Hyperplasia of mucus secreting submucosal
glands
E. Diffuse alveolar damage with leakage of
protein rich fluid into the alveolar spaces
282 A newborn infant who presented with cyanosis The clinical consequence primarily depends on the ERIC ROYD FINAL EXAM -
was diagnosed to have Tetralogy of Fallot severity of the subpulmonary stenosis as this TALAVERA, MD (TOP FEB 2015
(TOF), which of the following features of TOF determines the direction of blood flow. If the stenosis 1 - AUG 2014 MED
would primarily determine the severity of the is mild, the abnormality resembles an isolated VSD BOARDS; TOPNOTCH
disease process? and the shunt may be left to right without cyanosis MD)
A. Ventricular Septal Defect (so called pink tetralogy). As the obstruction
B. Right Ventricular Hypertrophy increases, the greater is the resitance to the RV
C. Subpulmonary stenosis outflow, producing right to left shunting and cyanosis
D. Over riding of the aorta
E. None of the above

TOPNOTCH MEDICAL BOARD PREP PATHOLOGY SUPEREXAM Page 39 of 99


For inquiries visit www.topnotchboardprep.com.ph or email us at topnotchmedicalboardprep@gmail.com
TOPNOTCH MEDICAL BOARD PREP PATHOLOGY SUPEREXAM
For inquiries visit www.topnotchboardprep.com.ph or email us at topnotchmedicalboardprep@gmail.com
Item QUESTION EXPLANATION AUTHOR TOPNOTCH
# EXAM
283 Which of the following is true regarding cancer There is some correlation between tumor burden and ERIC ROYD FINAL EXAM -
cachexia? severity of cachexia. The BMR is increased depsite TALAVERA, MD (TOP FEB 2015
A. There is no correlation between tumor reduced food intake. There is currently no satisfactory 1 - AUG 2014 MED
burden and the severity of cachexia treatment to cancer cachexia other than removal of BOARDS; TOPNOTCH
B. There is a redued basal metabolic rate the underlying cause, the tumor. MD)
accompied by reduced food intake
C. The weight loss seen results equally from
loss of fat and lean muscle
D. Satisfactory treatment can be achieved by
giving total parenteral nutrition alone
E. All of the above
284 A 72 year old male presented with progressive Dx: (most likely) Pancreatic CA. Weight loss, anorexia ERIC ROYD FINAL EXAM -
weight loss, anorexia and abdominal pain and malaise are signs of advanced disease. Majority TALAVERA, MD (TOP FEB 2015
radiating to the back. On PE, the gallbladder (60%) arise in the pancreatic head. Only 15% arise in 1 - AUG 2014 MED
was noted to be enlarged. An abdominal CT the body and 5% in the tail. Obstructive jaundice is BOARDS; TOPNOTCH
scan was done which demonstrated a mass in associated with most cases of carcinoma involving the MD)
the pancreatic head. Which of the following pancreatic head. CA 19-9 are useful in following
statements is true regarding the probable patient response but are too non specific and lack the
disease of this patient? sensitivity for screening.
A. Weight loss, anorexia and generalized
malaise are early signs of the disease
B. Abdominal pain is usually the first
symptom
C. The most common location is in the body of
the pancreas
D. Jaundice is more prominent when there is
involvement of the tail of the pancreas
E. CA 19-9 is a specific and sensitive marker
and can be used to screen for this disease
285 Which of the following tumors of the CNS Meningiomas are predominantly benign tumors of ERIC ROYD FINAL EXAM -
express progesterone receptors which may adult usually attached to the dura, that arise from the TALAVERA, MD (TOP FEB 2015
result in rapid growth during pregnancy? meningthelial cell of the arachnoid. Meningiomas are 1 - AUG 2014 MED
A. Meningioma usually slow growing lesions that present with vague BOARDS; TOPNOTCH
B. Oligodendroglioma non localizing sign , however due to presence of MD)
C. Astrocytoma progesterone receptors there can be rapid growth
D. Ependymoma during pregnancy
E. Medulloblastoma

286 A 78 year old female presented with severe Dx: Multiple Myeloma. Bone marrow aspiration would ERIC ROYD FINAL EXAM -
bone pain, pallor and frequent respiratory tract yield an increase number of plasma cells (>30%) TALAVERA, MD (TOP FEB 2015
infections. Laboratory studies showed 1 - AUG 2014 MED
hypercalcemia and an elevated creatinine. BOARDS; TOPNOTCH
Radiographs of the skull, long bones and spine MD)
demonstrated multiple punch out lesions.
Which of the following statement is FALSE
regarding this disorder?
A. Bone marrow aspiration would
demonstrate an increased number of T cells
B. Neurologic manifestations such as lethargy
and confusion are also present due to the
hypercalcemia
C. Cellular immunity is relatively unaffected
D. Perfomance of protein electrophoresis
would yield an abnormal spike (M spike) which
is most often an IgG
E. Renal failure and infections are major
causes of death from this disease
287 A 13 year old boy was brought to the ER due to the clinical findings are consistent with primary ERIC ROYD FINAL EXAM -
progressive weakness, easy fatigability and adrenocortical insufficiency or addison disease. 70% TALAVERA, MD (TOP FEB 2015
weight loss for the past 4 months. In addition, of cases are autoimmune but recently the most 1 - AUG 2014 MED
he has recently presented with nausea, frequent cause was TB. Hyperpigmentation is due to BOARDS; TOPNOTCH
vomiting and abdominal discomfort. On PE his compensatory hypothalamic production of MD)
blood pressure was noted to be markedly proopiomelanocortin, the precursor peptide of both
decreased; in addition there was note of corticotropin and MSH.
increased pigmentation over the creases of his
skin. What is the most likey diagnosis of this
case?
A. Secondary hyperaldosteronism
B. Cushing syndrome
C. Osteitis fibrosa cystica
D. Pheochromocytoma
E. Addison disease
288 Which of the following interleukins has a IL with predominantly anti inflammatory functions: ERIC ROYD FINAL EXAM -
predominant anti-inflammatory functions? IL-4, IL-3, IL-9, IL-10, IL-11, IL-13 ans IL-19 TALAVERA, MD (TOP FEB 2015
A. IL-1 1 - AUG 2014 MED
B. IL-2 BOARDS; TOPNOTCH
C. IL-5 MD)
D. IL-10
E. IL-8

TOPNOTCH MEDICAL BOARD PREP PATHOLOGY SUPEREXAM Page 40 of 99


For inquiries visit www.topnotchboardprep.com.ph or email us at topnotchmedicalboardprep@gmail.com
TOPNOTCH MEDICAL BOARD PREP PATHOLOGY SUPEREXAM
For inquiries visit www.topnotchboardprep.com.ph or email us at topnotchmedicalboardprep@gmail.com
Item QUESTION EXPLANATION AUTHOR TOPNOTCH
# EXAM
289 A 26 year old male presented with hematuria, Dx: Goodpasture syndrome. Anti-GBM antibodies are ERIC ROYD FINAL EXAM -
periorbital edema, hypertension and directed against both glomerular and alveolar TALAVERA, MD (TOP FEB 2015
hemoptysis. Serological testing was positive for basement membranes which would account for the 1 - AUG 2014 MED
anti-glomerular basement membrane bodies. nephritic syndrome an hemoptysis seen. Choice A is BOARDS; TOPNOTCH
Which of the following is the classic histologic seen in post streptococcal glomerulonephritis. Choice MD)
finding of this disease? C is seen in membranous GN. Choice D is seen in lupus
A. Lumpy bumpy immunofluorescence nephropathy and MPGN while Choice E is seen in
B. Linear immunofluorescence MPGN
C. Spike and dome appearance of the
glomerular basement membrane
D. Subenothelial immune complex deposition
E. Tram track appearance of the glomerular
basement membrane on electron microscopy
290 Which of the following statements is true SCID affects both cell mediated and humoral response. ERIC ROYD FINAL EXAM -
regarding Severe Combined Immunodeficiency It more commonly affects boys than girls. The most TALAVERA, MD (TOP FEB 2015
(SCID)? common form is inherited in a X-linked manner and 1 - AUG 2014 MED
A. It is a syndrome with a characteristic defect involves a common mutation in the common-gamma BOARDS; TOPNOTCH
involving only the humoral response chain of cytokine receptors. The remaininf cases are MD)
B. Majority are inherited in an autosomal inherited as AR, for which the most common cause is
recessive manner ADA deficiency. BMT is the mainstay of treatment and
C. It is the first human disease in which gene it is the first human disease in which gene therapy has
therapy has been successful been successful
D. It more commonly affects girls than boys
E. Deficiency of adenosine deaminase enzyme
(ADA) is seen in the X-linked variant of SCID
291 A 58 year old male, non-smoker, is referred for Dx: Polycythemia vera. The disorder is characterized ERIC ROYD FINAL EXAM -
evaluation of marked erythrocytosis and by prominent erythrocytosis, moderate granulocytosis TALAVERA, MD (TOP FEB 2015
splenomegaly. A CBC showed a marked and thrombocytosis. Because of the hyperviscoity and 1 - AUG 2014 MED
elevation of the RBC, WBC and platelet count. sludging of blood, there is frequet association with BOARDS; TOPNOTCH
Blood uric acid level was also elevated. Oxygen thrombosis or hemorrhagic phenomenon. Marked MD)
saturation was normal at 98%. Which of the splenomegaly and a decreased EPO are other classic
following is characteristic of this disorder ? characteristics. Cushing syndrome and hypoxia are
A. Frequent association with thrombosis or associated with secondary polycythemia which
hemorrhagic phenomenon characterized by an increased EPO. About 3% of
B. Secondary to increased EPO production patients terminate in Acute leukemia not CML
C. It is most often secondary to hypoxia
D. Usually terminates into chronic
myelogenous leukemia
E. A manifestation of Cushing syndrome
292 Which of the following features of liver Hyperestrogenism produces local vasodilation in the ERIC ROYD FINAL EXAM -
cirrhosis is not associated with impaired skin which could account for palmar erythema and TALAVERA, MD (TOP FEB 2015
estrogen metabolism and consequent spider angiomata. It also causes gynecomastia and 1 - AUG 2014 MED
hyperestrogenism ? testicular atrophy among males. Hemorrhoids are BOARDS; TOPNOTCH
A. Palmar erythema primarily due to the formation of portosystemic MD)
B. Internal hemorrhoids shunts.
C. Gynecomastia
D. Testicular atrophy
E. Spider angiomata
293 A 68 year old woman fell and sustained a pelvic Fat embolism syndrome occurs 2-3 days after severe ERIC ROYD FINAL EXAM -
fracture. Due to financial constraints she opted fracture injury and includes progressive CNS TALAVERA, MD (TOP FEB 2015
to just take pain medications for the moment. dysfunction and severe respiratory insufficiency. 1 - AUG 2014 MED
After a couple of days she developed a rapidly Thrombocytopenia is common and petechial BOARDS; TOPNOTCH
progresive respiratory failure which eventually hemorrhage can result from obstruction of the MD)
led to her death. On autopsy, there was note of microvasculature by embolic fat droplets. Respiratory
numerous petechiae over the conjunctiva and insufficiency may be due to injury to pulmonary
chest wall. What could have probably caused microvessels with leakage of fluid into the alveoli
the death of this woman? resulting in ARDS.
A. Saddle embolus
B. Acute tubular necrosis
C. Epidural hematoma
D. Bladder rupture
E. Fat embolization
294 Which of the the following features would point The inflammation in UC is limited to the ERIC ROYD FINAL EXAM -
more to Crohn's disease rather than Ulcerative mucosa/submucosal area. Skip lesions are common in TALAVERA, MD (TOP FEB 2015
colitis ? crohn's disease. 1 - AUG 2014 MED
A. Diffuse colonic involvement BOARDS; TOPNOTCH
B. Marked pseudopolyps MD)
C. Transmural inflammation
D. Toxic megacolon
E. Absence of non caseating granuloma

295 An 18 year old male presented with swelling of Osteoclastoma or giant cell tumor involve both the ERIC ROYD FINAL EXAM -
the left knee, an X-ray was done which showed epihyses and metaphyses. The majority arise in the TALAVERA, MD (TOP FEB 2015
lytic lesions over the said area. A biopsy was knee. The typical location causes arthritis like 1 - AUG 2014 MED
done which showed an abundance of symptoms. Biopsy of the tumor will reveal an BOARDS; TOPNOTCH
multinucleated giant cells with background of abundance of multinucleated giant cells with MD)
mononuclear stromal cells. What is the most background of mononuclear stromal cells
likely diagnosis for this case?
A. Osteoclastoma
B. Osteosarcoma
C. Ewing's tumor
D. Chondrosarcoma
E. Osteochondroma

TOPNOTCH MEDICAL BOARD PREP PATHOLOGY SUPEREXAM Page 41 of 99


For inquiries visit www.topnotchboardprep.com.ph or email us at topnotchmedicalboardprep@gmail.com
TOPNOTCH MEDICAL BOARD PREP PATHOLOGY SUPEREXAM
For inquiries visit www.topnotchboardprep.com.ph or email us at topnotchmedicalboardprep@gmail.com
Item QUESTION EXPLANATION AUTHOR TOPNOTCH
# EXAM
296 A 33 year old woman sought consult for post Risk factors for Cervical CA: all of the aforementioned ERIC ROYD FINAL EXAM -
coital bleeding. Which of the following risk choices + male partner with multiple sexual partners, TALAVERA, MD (TOP FEB 2015
factors would point to cervical carcinoma as the persistent infection with HPV 16 or 18, 1 - AUG 2014 MED
underlying cause? Immunosuppression, certain HLA subtypes, use of oral BOARDS; TOPNOTCH
A. Multiple sexual partners contraceptives MD)
B. Young age at first intercourse
C. High parity
D. Smoking
E. All of the above
297 A 56 year old male presented with a 2 day Dx: acute prostatitis. Biopsy of a man with prostatitis ERIC ROYD FINAL EXAM -
history of fever, chills and dysuria. On PE, the is contraindicated as it may lead to sepsis TALAVERA, MD (TOP FEB 2015
prostate was noted to be exquisitely tender and 1 - AUG 2014 MED
boggy. Which of the following statements is BOARDS; TOPNOTCH
FALSE regarding this condition? MD)
A. The diagnosis should be established by
biopsy which would show focal areas of
necrosis with diffuse edema
B. The causative agens are similar to those
that cause UTI
C. The organisms are usually implanted by
intraprostatic reflux of urine from the posterior
urethra.
D. Organisms can seed the prostate by
lymphohematogenous routes from a distant
foci of infection
E. None of the above
298 Which of the following features is more The presence of increased storage of iron in marrow ERIC ROYD FINAL EXAM -
suggestive of anemia of chronic disease rather macrophages, a high serum ferritin level and a TALAVERA, MD (TOP FEB 2015
than iron deficiency anemia? reduced total iron binding capacity readily rules out 1 - AUG 2014 MED
A. Low serum ferritin level IDA as the cause BOARDS; TOPNOTCH
B. Low MCV MD)
C. Low TIBC
D. High serum iron
E. None of the above

299 A 62 year old male, known hypertensive, Dx: Aortic dissection. The dissection is usually ERIC ROYD FINAL EXAM -
presented to the ER due to an acute onset of initiated by a tear of the intimal layer of the blood TALAVERA, MD (TOP FEB 2015
chest pain. He described the chest pain as vessel. Most serious complications occur with 1 - AUG 2014 MED
having a "tearing" quality and radiating to the dissection that involves the aorta from the aortic valve BOARDS; TOPNOTCH
back. 12L ECG showed no findings of ischemia to the aortic arch. More than 90% of cases of aortic MD)
or infarction Which of the following statements dissection are men aged 40-60 with antecedent
is true regarding the most likely condition ? hypertension. A recognizable medial damage seems to
A. It is usally initiated by a tear over the be neither a pre requiste for dissection nor a
adventitia of the blood vessel guarantee that dissection is imminent.
B. The most serious complications occur when
the involvement is distal to the subclavian
artery
C. More than 90% of patients presenting with
this condition have an underlying connective
tissue disorder
D. A damage to the tunica media is a pre
requiste for the the disease condition to occur
E. Majoirty of cases are found within 10 cm of
the the aortic valve
300 A 45 year old woman presented with areas of Dx: Acanthosis nigricans. Acanthosis nigricans is a ERIC ROYD FINAL EXAM -
velvety hyperpigmentation involving the axilla brown to black, poorly defined, velvety TALAVERA, MD (TOP FEB 2015
and posterior surface of the neck. The said hyperpigmentation of the skin. It is usually found in 1 - AUG 2014 MED
areas initially started as smaller macules but body folds, such as the posterior and lateral folds of BOARDS; TOPNOTCH
have now progressed to form palpable plaques. the neck, the armpits, groin, navel, forehead, and other MD)
Which of the following is an important areas. It typically occurs in individuals younger than
association of this skin lesion? age 40, and is associated with obesity or
A. Viral infection endocrinopathies, and is also indicative of visceral
B. Visceral malignancy malignancy such as CA of the lungs, breast, stomach or
C. Asthma uterus
D. Seizure disorders
E. None of the above
301 Which of the following is true about the intrinsic or mitochondrial pathway is the major LEAN ANGELO BACK-UP
intrinsic pathway of apoptosis ? mechanism of apoptosis in the mammalian cells. The SILVERIO, MD (TOP 4 MIDTERM
A. The upregulation of Bcl-2 regulates the role of Bcl 2 ( in the presence of a growth signal) is to - AUG 2014 MED EXAM - FEB
mitochondrial permeability preventing the regulate the permeability of the mitochondrial BOARDS; TOPNOTCH 2015
leakage of cytochrome aa3 membranes limiting the leakage of cytochrome c.once MD), MD
B. Activation of Bax and Bak forms oligomers stress or damage was done to the cell, there is
that create a hole in the mitochondrial upregulation of Bax and Bak. this in turn inhibits the
membrane Bcl2 and forms oligomers creating holes to the
C. caspase 3 is the critical initiating caspase mitochondria leading to release of cytochrome c.
for apoptosis cytosolic cytochrome c binds to Apaf 1 forming
D. upregulation of Smac/DIABLO activates the apoptosome to activate caspase 9. the critical initiator
physiologic inhibitors of apoptosis of caspase activation. Other factors such as
E. all of the above Smac/DIABLO inhibits the inhibitors of apoptosis
including caspase 3. Robbins 8th ed p 29.

TOPNOTCH MEDICAL BOARD PREP PATHOLOGY SUPEREXAM Page 42 of 99


For inquiries visit www.topnotchboardprep.com.ph or email us at topnotchmedicalboardprep@gmail.com
TOPNOTCH MEDICAL BOARD PREP PATHOLOGY SUPEREXAM
For inquiries visit www.topnotchboardprep.com.ph or email us at topnotchmedicalboardprep@gmail.com
Item QUESTION EXPLANATION AUTHOR TOPNOTCH
# EXAM
302 various experimental models states that the sirtuins have histone deacetylase activity and are LEAN ANGELO BACK-UP
most important way of promoting cellular thought to promote the expression of several genes SILVERIO, MD (TOP 4 MIDTERM
longevity is through caloric restriction. Which whose products increase longevity. These products - AUG 2014 MED EXAM - FEB
of the family of proteins is responsible for such include proteins that increase metabolic activity, BOARDS; TOPNOTCH 2015
effect? reduce apoptosis, stimulate protein folding, and MD), MD
A. Insulin Growth factor inhibit the harmful effects of oxygen free radicals.
B. telomerase Robbins 8th ed p 41.
C. sirtuins
D. transforming growth factor beta
E. none of the above
303 Which of the followingl nuclear ultrastructural when a cell is exposed to transient heat, it undergoes LEAN ANGELO BACK-UP
changes can be seen in the affected cell, when cellular swelling. These are the ultrastructural SILVERIO, MD (TOP 4 MIDTERM
the latter is exposed to transient heat ? changes of reversible cell injury: Plasma membrane - - AUG 2014 MED EXAM - FEB
A. disaggregation blebbing, blunting and loss of microvilli, mitochondria BOARDS; TOPNOTCH 2015
B. Appearance of amorphous densities - swelling, appearance of small amorphous densities, MD), MD
C. Myelin figures ER- dilation, polysome detachment, myelin figures.
D. Blebbing nuclear 0 disaggregation of granular and fibirllar
E. All of the above elements.Robbins 8th ed p 14.

304 the following proteins are leukocyte molecules P selectin is expressed in the endothelium during LEAN ANGELO BACK-UP
responsible for endothelial adhesion except? leukocyte adhesion. Other endothelial molecutes are SILVERIO, MD (TOP 4 MIDTERM
A. integrin E -selectin, Glycam 1, ICAM 1, VCAM 1. Robbins 8th ed - AUG 2014 MED EXAM - FEB
B. Sialyl Lewis X modified proteins p 49. BOARDS; TOPNOTCH 2015
C. L selectin MD), MD
D. P selectin
E. None of the above

305 what type of substance accumulate in Niemann cholesterol and cholesterol esters accumulate in the LEAN ANGELO BACK-UP
pick disease Type C? following conditions: atherosclerosis, xanthomas, SILVERIO, MD (TOP 4 MIDTERM
A. protein cholesterolosis of gallbladder, and Niemann pick - AUG 2014 MED EXAM - FEB
B. TAG disease type C. page 34-35. BOARDS; TOPNOTCH 2015
C. cholesterol MD), MD
D. calcium
E. Glycogen

306 A 65 y/o male went for consult secondary, this is a case of diffuse Large B cell lymphoma. Its LEAN ANGELO BACK-UP
fever, chronic malaise, and weight loss. Upon characteristics are anaplastic relatively large cells SILVERIO, MD (TOP 4 MIDTERM
physical examination, multiple with diffuse pattern of growth. Its cytogenetic, gene - AUG 2014 MED EXAM - FEB
lymphadenophathies were noted on the expression, profiling and immunohistochemical is BOARDS; TOPNOTCH 2015
oropharyngeal area. hepatosplenomegaly is heterogenous. However, 30% presents with BCL6 MD), MD
also noted. Biopsy of the lymph node showed dysregulation. A- follicular lymphoma. C- ( starry sky
generalized effacement of sinusoid structure by pattern) Burkitts lymphoma. D- mantle cell
sheets of large cells 4-5x the size of a normal lymphoma. Robbins 8th ed - 606-608
lymphocyte. which of the following is true
about the diagnosis ?
A. t(14,18) translocation is the hallmark for
this condition
B. BCL6 dysregulation is common in around
30% of the cases
C. it exhibits a high mitotic index with
numerous apoptotic cells interspersed with
macrophages
D. it is correlated with cyclin D1
overexpression
E. none of the above
307 what is the most striking histologic finding in (SIMILAR TO PREVIOUS BOARD EXAM LEAN ANGELO BACK-UP
desquamative interstitial pneumonia ( CONCEPT/PRINCIPLE) Robbins 8th ed p 704 SILVERIO, MD (TOP 4 MIDTERM
smoking related interstitial disease)? - AUG 2014 MED EXAM - FEB
A. Multiple macrophages containing dusty BOARDS; TOPNOTCH 2015
brown cytoplasmic pigments MD), MD
B. Thickening of alveolar septa
C. presence of lamellar bodies in the
macrophage
D. presence of plump cuboidal pneumocytes
along the septa
E. massive interstitial fibrosis
308 A 48 y/o male complains of intermittent low (SIMILAR TO PREVIOUS BOARD EXAM LEAN ANGELO BACK-UP
grade fever, fatigue and weight loss of CONCEPT/PRINCIPLE). This is a case of AML. Robbins SILVERIO, MD (TOP 4 MIDTERM
approximately 1 month duration. Physical 8th ed page 621-624 - AUG 2014 MED EXAM - FEB
examination showed BP 100/60, PR 105 bpm, BOARDS; TOPNOTCH 2015
PP 25 cpm, pale palpebral conjunctiva, palatal MD), MD
and cutaneous petechia.liver edge was palpated
4 cm below the right subcostal margin.blood
smear showed large pleomorphic cells with
multiple nucleoli and cytoplasmic needle like
azurophilic granules. which of the following is
true about his condition ?
A. t(8,21) balanced translocation imparts a
favorable prognosis
B. aberrant tyrosine kinase activation is a
universal feature of the disease
C. diagnosis requires at least 20% blast in the
bone marrow
D. all of the above
E. none of the above

TOPNOTCH MEDICAL BOARD PREP PATHOLOGY SUPEREXAM Page 43 of 99


For inquiries visit www.topnotchboardprep.com.ph or email us at topnotchmedicalboardprep@gmail.com
TOPNOTCH MEDICAL BOARD PREP PATHOLOGY SUPEREXAM
For inquiries visit www.topnotchboardprep.com.ph or email us at topnotchmedicalboardprep@gmail.com
Item QUESTION EXPLANATION AUTHOR TOPNOTCH
# EXAM
309 a 78 y/o male complains of sever chronic low this is a case of multiple myeloma. Serum M protein is LEAN ANGELO BACK-UP
back pain accompanied by weakness and almost always greater than 3gm/dl. ( asymptomatic SILVERIO, MD (TOP 4 MIDTERM
lethargy. PE reveals pale palpebral conjunctiva, patients with < 3gm/dl M protein and without skeletal - AUG 2014 MED EXAM - FEB
bilateral basal crackles and hepatomegaly. lesion is called MGUS). IgG rearrangement is universal BOARDS; TOPNOTCH 2015
Lumbar Xray showed multiple osteolytic lesion in MM. myeloma cells produce factors that activated MD), MD
scattered around the vertebral bodies. which of RANKL which in turn stimulates osteoclastic activity.
the following is consistent about his condition? (SIMILAR TO PREVIOUS BOARD EXAM
A. serum M protein level is 2.5 g/dl CONCEPT/PRINCIPLE)
B. rearrangements involving the Ig heavy chain
is rare
C. proliferation and survival of neoplastic cells
are dependent on cytokines (IL6)
D. RANKL upregulation causes apoptosis of
osteoblast
E. all of the above.
310 A 45 y/o obese female complains of the difference between reflux and eosinophilic LEAN ANGELO BACK-UP
retrosternal pain aggravated after a high fat esophagitis is that the latter is characterized by SILVERIO, MD (TOP 4 MIDTERM
meal. She also noted frequent regurgitation of abundance of intraepithelial eosinophils. Robbins 8th - AUG 2014 MED EXAM - FEB
sour tasting gastric contents especially at the ed p770. ( SIMILAR TO PREVIOUS BOARD EXAM BOARDS; TOPNOTCH 2015
middle of her sleep.pastmedical history shows CONCEPT/PRINCIPLE) MD), MD
that she is asthmatic, hypertensive and diabetic.
EGD was done and biopsy was performed in the
lower esophageal region showing mild number
of intraepithelial eosinophils and basal zone
hyperplasia. what is the diagnosis?
A. chemical esophagitis
B. reflux esophagitis
C. eosinophilic esophagitis
D. hiatal hernia
E. none of the above
311 which of the following is a consistent finding in Systemic sclerosis is associated with anti Scl70 while LEAN ANGELO BACK-UP
scleroderma except? CREST or limited scleroderma is correlated to SILVERIO, MD (TOP 4 MIDTERM
A. Anticentromere antibody is correlated to anticentromere antibody. Intimal proliferation of - AUG 2014 MED EXAM - FEB
CREST syndrome . vascular tissue with progressive fibrosis is BOARDS; TOPNOTCH 2015
B. Intimal vascular proliferation is the most characteristic of this disease. joint destruction is not MD), MD
consistent finding common in systemic sclerosis ( distinguishing feature
C. Renal abnormalities resembles that of a against RA). Robbins 8th ed p 224
malignant hypertension
D. synovial inflammation and joint destruction
is similar to that of RA.
312 which of the following is true about the children diagnosed at <18 months of age generally has LEAN ANGELO BACK-UP
neuroblastoma ? a good prognosis regardless of the stage. N myc SILVERIO, MD (TOP 4 MIDTERM
A. Children younger than 18months have a amplication is related to Neuroblastoma not c myc. - AUG 2014 MED EXAM - FEB
worst prognosis regardless of stage 90% of neuroblastoma secretes catecholamines BOARDS; TOPNOTCH 2015
B. C myc amplification is the most important regardless of location. Robbins 8th ed p 475-476 MD), MD
genetic abnormality used in risk stratification
C. only neuroblastoma cells that are located in
the adrenal medulla produces catecholamines
D. presence of Homer wright pseuodorosettes
E. all of the above
313 What is the most characteristic lesion of HIV HIV is highly associated with a variant of FSGS known LEAN ANGELO BACK-UP
associated nephropathy ? as collapsing glomerulopathy. It is characterized by SILVERIO, MD (TOP 4 MIDTERM
A. Capillary wall hyalinosis retraction or collapse of the entire glomerulus and - AUG 2014 MED EXAM - FEB
B. Duplication of the basement membrane hypertrophy and proliferation of visceral epithelial BOARDS; TOPNOTCH 2015
C. Retraction of the entire glomerulus cells. Robbins 8th ed p 926 MD), MD
D. Diffuse mesangial proliferation
E. Fibrin crescents

314 What is the most serious complication of the most serious complication of chronic tuberculous LEAN ANGELO BACK-UP
Tuberculous meningitis? meningitis is arachnoid fibrosis leading to SILVERIO, MD (TOP 4 MIDTERM
A. Tuberculoma formation hydrocephalus and obliterative endarteritis leading to - AUG 2014 MED EXAM - FEB
B. Fibrinous basal exudates leading to cranial brain infarction. This is a SIMILAR TO PREVIOUS BOARDS; TOPNOTCH 2015
nerve palsies BOARD EXAM CONCEPT/PRINCIPLE. MD), MD
C. Choroid plexus involvement leading to
diffuse meningoencephalitis
D. Obliterative endarteritis
E. none of the above
315 which of the following morphologic changes 30% of polyarteritis nodosa is associated with LEAN ANGELO BACK-UP
consistent with hepatitis B associated Hepatitis B virus infection. PAN is a segmental SILVERIO, MD (TOP 4 MIDTERM
vasculitis? transmural necrotizing inflammation of small and - AUG 2014 MED EXAM - FEB
A. Pulmonary circulation is commonly medium sized arteries typically involving the renal BOARDS; TOPNOTCH 2015
affected and visceral vessels sparing the pulmonary MD), MD
B. Focal transmural necrotizing lesions of all circulation. all stages of activity coexist in different
stages of activity vessels. this is unlike microscopic polyangitis wherein
C. Associated with peripheral all lesions are at the same stage of activity and
hypereosinophilia pulmonary circulation is commonly affected.
D. granulomatous inflammation with elastic hypereosinophilia is common in churge strauss
lamina fragmentation syndrome while elastic lamina fragmentation is
E. all of the above characteristic of giant cell arteritis. Robbins 8th ed
513-515

TOPNOTCH MEDICAL BOARD PREP PATHOLOGY SUPEREXAM Page 44 of 99


For inquiries visit www.topnotchboardprep.com.ph or email us at topnotchmedicalboardprep@gmail.com
TOPNOTCH MEDICAL BOARD PREP PATHOLOGY SUPEREXAM
For inquiries visit www.topnotchboardprep.com.ph or email us at topnotchmedicalboardprep@gmail.com
Item QUESTION EXPLANATION AUTHOR TOPNOTCH
# EXAM
316 A 63 y/o male went for consult secondary to this is a case of lichen planus. It is a form of interface LEAN ANGELO BACK-UP
skin lesion noted on his forehead. PE showed dermatitis with characteristic necrolytic basal layer at SILVERIO, MD (TOP 4 MIDTERM
multiple pruritic, violaceous polygonal papule the tip of dermal papilla ( civatte bodies). B-mycoses - AUG 2014 MED EXAM - FEB
with lacelike pattern. Which of the morphologic fungoides, C- psoriasis, D- pemphigus vulgaris BOARDS; TOPNOTCH 2015
changes is consistent with the diagnosis? Robbins 8th ed p 1192 MD), MD
A. civatte bodies at the basal epidermis
B. pautrier microabscesses
C. thinned out stratum granulosum
D. acantholytic blisters
E. all of the above
317 A 25 y/o female complains of a constant left Robbins 8th ed page 1224. osteoid osteoma are by LEAN ANGELO BACK-UP
midthigh pain most severe during at sleep. She definition less than 2cm in greatest dimension SILVERIO, MD (TOP 4 MIDTERM
claims that taking effectively relieved of her occuring more on the appendicular skeleton. It usually - AUG 2014 MED EXAM - FEB
symptoms. Femoral xray done showing 2cm affects teenagers with 2:1 ratio predilection for BOARDS; TOPNOTCH 2015
subperiosteal lesion located at the midshaft women. 50% arise in the femur. Most common MD), MD
with dense sclerotic background. which of the symptom of this condition is severe nocturnal pain
following is the most likely diagnosis? effectively relieved by aspirin. the pain is secondary to
A. osteoblastoma the production of prostaglandin E2 by the
B. osteoma proliferating osteoblast. morphology will show a
C. osteoid osteoma tremendous amount of reactive bone formation
D. osteochondroma around the tumor ( nidus).
E. none of the above
318 what is the hallmark feature in diagnosing diagnosis of parathyroid carcinoma based on cytologic LEAN ANGELO BACK-UP
parathyroid carcinoma? detail is unreliable, and invasion of surrounding SILVERIO, MD (TOP 4 MIDTERM
A. Nuclear atypia tissues and metastasis are the only reliable criteria. - AUG 2014 MED EXAM - FEB
B. Follicular formation of oxyphil cells (SIMILAR TO PREVIOUS BOARD EXAM BOARDS; TOPNOTCH 2015
C. Local invasion CONCEPT/PRINCIPLE) Robbins 8th pp 1127-1128 MD), MD
D. Mitotic index
E. Capsular invasion

319 what major fibril protein responsible for A - primary amyloidosis, C- systemic senile LEAN ANGELO BACK-UP
secondary amyloidosis on a chronic kidney amyloidosis, D- medullary carcinoma E-alzheimers SILVERIO, MD (TOP 4 MIDTERM
disease patient on prolonged hemodialysis ? disease. Robbins 8th ed p 252 - AUG 2014 MED EXAM - FEB
A. Amyloid light chain BOARDS; TOPNOTCH 2015
B. Beta microglobulin MD), MD
C. transthyretin
D. calcitonin
E. Amyloid precursor protein

320 A 32 y/o male was brought to ER secondary to SIMILAR TO PREVIOUS BOARD EXAM LEAN ANGELO BACK-UP
personality changes. He appears to be CONCEPT/PRINCIPLE. Arboviral encephalitis is SILVERIO, MD (TOP 4 MIDTERM
combative, confused and agitated. 30mins at characterized with neuronophagia and microgial - AUG 2014 MED EXAM - FEB
the ER, patient experienced status epilepticus nodules. Robbins 8th ed p 1302. BOARDS; TOPNOTCH 2015
which despite all efforts he eventually perished. MD), MD
Biopsy of the brain revealed single cell necrosis
with neuronophagia. what is the primary
diagnosis?
A. Herpes simplex encephalitis
B. CMV encephalitis
C. Arboviral encephalitis
D. HIV encephalitis
E. HSV encephalitis
321 True of cancer cachexia: Weight loss in cancer cachexia results equally from a DEBBIE ROSE BACK-UP
A. Weight loss results more from loss of muscle loss of muscle and of fat. It is NOT caused by the TANENGSY, MD (TOP MIDTERM
than of fat. nutritional demands of the tumor. BMR is increased in 5 - AUG 2014 MED EXAM - FEB
B. It is caused by nutritional demands of the patients with cancer. It is suspected that TNF BOARDS; TOPNOTCH 2015
tumor. produced by macrophages mediates cachexia. Robbins MD)
C. It has no satisfactory treatment other than & Cotran Pathologic Basis of Disease 8th edition, p.320
removal of the underlying cause.
D. Basal metabolic rate is decreased in patients
with cancer.
E. It is suspected that Il-4 produced by
macrophages mediates cachexia.
322 A paraneoplastic syndrome characterized by Trosseau phenomenon is a migratory DEBBIE ROSE BACK-UP
gray-black patches of verrucous thrombophlebitis associated with cancer of pancreas TANENGSY, MD (TOP MIDTERM
hyperkeratosis: or lungs. Robbins & Cotran Pathologic Basis of Disease 5 - AUG 2014 MED EXAM - FEB
A. carcinoid syndrome 8th edition, p.321-322 BOARDS; TOPNOTCH 2015
B. trousseau phenomenon MD)
C. dermatomyositis
D. acanthosis nigricans
E. hypertrophic osteoarthropathy

323 Causes of non-immune fetal hydrops, except: Robbins & Cotran Pathologic Basis of Disease 8th DEBBIE ROSE BACK-UP
A. high output heart failure edition, p.461 TANENGSY, MD (TOP MIDTERM
B. turner syndrome 5 - AUG 2014 MED EXAM - FEB
C. parvovirus B19 infection BOARDS; TOPNOTCH 2015
D. maternal Rh isoimmunization MD)
E. CMV infection

TOPNOTCH MEDICAL BOARD PREP PATHOLOGY SUPEREXAM Page 45 of 99


For inquiries visit www.topnotchboardprep.com.ph or email us at topnotchmedicalboardprep@gmail.com
TOPNOTCH MEDICAL BOARD PREP PATHOLOGY SUPEREXAM
For inquiries visit www.topnotchboardprep.com.ph or email us at topnotchmedicalboardprep@gmail.com
Item QUESTION EXPLANATION AUTHOR TOPNOTCH
# EXAM
324 This condition is characterized by a sharply Churg-Strauss syndrome - granulomas with DEBBIE ROSE BACK-UP
segmental acute and chronic vasculitis of geographic patterns of central necrosis and vasculitis. TANENGSY, MD (TOP MIDTERM
medium sized and small arteries, Takayasu arteritis - histologic appearance is 5 - AUG 2014 MED EXAM - FEB
predominantly of the extremities: indistinguishable from temporal arteritis. Polyarteritis BOARDS; TOPNOTCH 2015
A. thromboangiitis obliterans nodosa - segmental transmural necrotizing MD)
B. Churg-Strauss syndrome inflammation of small and medium arteries. Temporal
C. Takayasu arteritis arteritis - nodular intimal thickening reducing luminal
D. polyarteritis nodosa diameter, granulomatous inflammation leading to
E. temporal arteritis elastic lamina fragmentation. Robbins & Cotran
Pathologic Basis of Disease 8th edition, p.512-514
325 The most common primary tumor of the heart: The most frequent primary tumor of the heart in DEBBIE ROSE BACK-UP
A. lipoma children, on the other hand, is rhabdomyosarcoma. TANENGSY, MD (TOP MIDTERM
B. myxoma Robbins & Cotran Pathologic Basis of Disease 8th 5 - AUG 2014 MED EXAM - FEB
C. rhabdomyoma edition, p.584 BOARDS; TOPNOTCH 2015
D. sarcoma MD)
E. papillary fibroelastoma

326 The most specific morphologic finding in Spherocytosis is distinctive but not pathognomonic of DEBBIE ROSE BACK-UP
hereditary spherocytosis: Hereditary Spherocytosis. Robbins & Cotran TANENGSY, MD (TOP MIDTERM
A. reticulocytosis Pathologic Basis of Disease 8th edition, p.643 5 - AUG 2014 MED EXAM - FEB
B. hemosiderosis BOARDS; TOPNOTCH 2015
C. spherocytosis MD)
D. extramedullary hematopoiesis
E. cholelithiasis

327 The most common hereditary disease von Willebrand factor deficiency, on the other hand, is DEBBIE ROSE BACK-UP
associated with life threatening bleeding: the most common inherited bleeding disorder of TANENGSY, MD (TOP MIDTERM
A. von Willebrand factor deficiency humans. Robbins & Cotran Pathologic Basis of Disease 5 - AUG 2014 MED EXAM - FEB
B. hemophilia A 8th edition, p.672 BOARDS; TOPNOTCH 2015
C. hemophilia B MD)
D. Bernard Soulier syndrome
E. Glanzmann Thrombasthenia

328 The major condition/s associated with Airway remodeling is a histologic finding in bronchial DEBBIE ROSE BACK-UP
bronchiectasis: asthma. Robbins & Cotran Pathologic Basis of Disease TANENGSY, MD (TOP MIDTERM
A. obstruction 8th edition, p.692 5 - AUG 2014 MED EXAM - FEB
B. infection BOARDS; TOPNOTCH 2015
C. airway remodeling MD)
D. A & B
E. all of the above

329 True of typhoid fever, except: The oval ulcers in typhoid fever are oriented along the DEBBIE ROSE BACK-UP
A. Humans are its sole reservoir. axis of the ileum. Robbins & Cotran Pathologic Basis of TANENGSY, MD (TOP MIDTERM
B. Gallbladder colonization is associated with Disease 8th edition, p.801 5 - AUG 2014 MED EXAM - FEB
the chornic carrier state. BOARDS; TOPNOTCH 2015
C. Morphologically oval ulcers perpendicular to MD)
the axis of the ileum are seen.
D. Liver, bone marrow, lymph nodes may show
typhoid nodules.
E. Patients with sickle cell disease are
susceptible to Salmonella osteomyelitis.
330 A patient presents with diarrhea, weight loss, PAS(+) macrophages are also seen in Intestinal TB, DEBBIE ROSE BACK-UP
malabsorption, along with arthritis and fever. but it is also usually AFB(+). Robbins & Cotran TANENGSY, MD (TOP MIDTERM
Biopsy of the small intestine reveals Pathologic Basis of Disease 8th edition, p.803 5 - AUG 2014 MED EXAM - FEB
accumulation of distended, foamy macrophages BOARDS; TOPNOTCH 2015
containing PAS(+), diastase resistant granules. MD)
Rod-shaped bacilli can be identified by electron
microscopy. What is the most likely diagnosis?
A. Whipple disease
B. intestinal TB
C. lactase deficiency
D. pseudomembranous colitis
331 The disease that is most likely to give rise to Robbins & Cotran Pathologic Basis of Disease 8th DEBBIE ROSE BACK-UP
hepatocellular carcinoma: edition, p.878 TANENGSY, MD (TOP MIDTERM
A. hepatitis C 5 - AUG 2014 MED EXAM - FEB
B. hepatitis B BOARDS; TOPNOTCH 2015
C. non-alcoholic steatohepatitis MD)
D. hereditary tyrosinemia
E. alpha1-antitrypsin deficiency

332 The most common congenital anomaly of the Pancreas divisum is the failure of fusion of fetal duct DEBBIE ROSE BACK-UP
pancreas: systems of dorsal and ventral pancreatic primordia. TANENGSY, MD (TOP MIDTERM
A. agenesis Robbins & Cotran Pathologic Basis of Disease 8th 5 - AUG 2014 MED EXAM - FEB
B. annular pancreas edition, p.892 BOARDS; TOPNOTCH 2015
C. ectopic pancreas MD)
D. pancreas divisum
333 Most pancreatic cancers arise from: Head - origin of 60% of pancreatic cancer; body - 15%; DEBBIE ROSE BACK-UP
A. pancreatic head tail - 5%. Robbins & Cotran Pathologic Basis of Disease TANENGSY, MD (TOP MIDTERM
B. pancreatic body 8th edition, p.902 5 - AUG 2014 MED EXAM - FEB
C. pancreatic tail BOARDS; TOPNOTCH 2015
D. diffusely involving the entire gland MD)

TOPNOTCH MEDICAL BOARD PREP PATHOLOGY SUPEREXAM Page 46 of 99


For inquiries visit www.topnotchboardprep.com.ph or email us at topnotchmedicalboardprep@gmail.com
TOPNOTCH MEDICAL BOARD PREP PATHOLOGY SUPEREXAM
For inquiries visit www.topnotchboardprep.com.ph or email us at topnotchmedicalboardprep@gmail.com
Item QUESTION EXPLANATION AUTHOR TOPNOTCH
# EXAM
334 Foot process effacement associated with In Minimal change disease, the prinicipal lesion is in DEBBIE ROSE BACK-UP
normal glomeruli by light microscopy makes the visceral epithelial cells which show a uniform and TANENGSY, MD (TOP MIDTERM
this diagnosis: diffuse effacement of foot processes. Robbins & Cotran 5 - AUG 2014 MED EXAM - FEB
A. membranous nephropathy Pathologic Basis of Disease 8th edition, p.925 BOARDS; TOPNOTCH 2015
B. minimal change disease MD)
C. PSGN
D. MPGN
E. Berger disease

335 A 27-year-old female with hypertension was Choice A is seen in males, diabetics, advanced age. DEBBIE ROSE BACK-UP
found to have renal artery stenosis. What is the Choice B in malignant HPN (fibronoid necrosis TANENGSY, MD (TOP MIDTERM
expected morphologic finding on examination? described). Choice C (the answer) refers to 5 - AUG 2014 MED EXAM - FEB
A. a concentrically placed atheromatous plaque fibromuscular dysplasia, ocuring in females, in their BOARDS; TOPNOTCH 2015
with superimposed thrombosis 3rd-4th decades. Choice D describes onion skinning MD)
B. eosinophilic granular change in blood vessel seen in malignant HPN. Robbins & Cotran Pathologic
wall staining (+) for fibrin Basis of Disease 8th edition, p.951
C. fibromuscular thickening involving the media
of artery
D. intimal thickening caused by proliferation of
elongated, concentrically arranged smooth
muscle cells
336 Consistent with benign ulcers, except: SIMILAR TO PREVIOUS BOARD EXAM DEBBIE ROSE BACK-UP
A. oval, sharply punched out defect CONCEPT/PRINCIPLE. Robbins & Cotran Pathologic TANENGSY, MD (TOP MIDTERM
B. mucosal margin with slight overhang from Basis of Disease 8th edition, p.780-781 5 - AUG 2014 MED EXAM - FEB
base BOARDS; TOPNOTCH 2015
C. hemorrhage and fibrin deposition on gastric MD)
serosa
D. heaped up margins
E. thin layer of fibrinoid deposit at base
underlaid by predominantly neutrophilic
inflammatory infiltrate
337 True of ulcerative colitis, except: The earliest lesion in Crohn's disease is an aphthous DEBBIE ROSE BACK-UP
A. normal serosal surface of colon ulcer. Robbins & Cotran Pathologic Basis of Disease TANENGSY, MD (TOP MIDTERM
B. toxic megacolon may complicate 8th edition, p.811-812 5 - AUG 2014 MED EXAM - FEB
C. earliest lesion is an aphthous ulcer BOARDS; TOPNOTCH 2015
D. backwash ileitis MD)
E. absence of granulomas

338 A 21-year-old male presenting with recurrent Robbins & Cotran Pathologic Basis of Disease 8th DEBBIE ROSE BACK-UP
colicky abdominal pain from GI obstruction and edition, p.818 TANENGSY, MD (TOP MIDTERM
transient intussesceptions. On physical 5 - AUG 2014 MED EXAM - FEB
examination, dark blue to brown maculesa are BOARDS; TOPNOTCH 2015
noted around his mouth, eyes, nostrils, buccal MD)
mucosa. What is the most likely diagnosis?
A. Cronkhite Canada syndrome
B. Cowden syndrome
C. Bannayan-Ruvalbaca-Riley syndrome
D. Peutz-Jeghers syndrome
339 The main regulatory factor for iron absorption? Robbins & Cotran Pathologic Basis of Disease 8th DEBBIE ROSE BACK-UP
A. ferritin edition, p.862 TANENGSY, MD (TOP MIDTERM
B. hepcidin 5 - AUG 2014 MED EXAM - FEB
C. transferrin BOARDS; TOPNOTCH 2015
D. serum Fe MD)
E. TIBC

340 What refers to de novo formation of blood Angiogenesis/neovascularization is new vessel DEBBIE ROSE BACK-UP
vessels during embryogenesis? formation in the mature organism. Arteriogenesis TANENGSY, MD (TOP MIDTERM
A. angiogenesis involved remodeling of existing arteries in response to 5 - AUG 2014 MED EXAM - FEB
B. neovascularization chronic changes in pressure and flow. Robbins & BOARDS; TOPNOTCH 2015
C. arteriogenesis Cotran Pathologic Basis of Disease 8th edition, p.489 MD)
D. vasculogenesis
341 A 56 year old hypertensive and diabetic male JESSICA MAE BACK-UP
complained of chest pains and was brought to SANCHEZ, MD (TOP 4 MIDTERM
the emergency room. ECG showed ST segment - AUG 2014 MED EXAM - FEB
elevation and lab work-up showed elevated BOARDS; TOPNOTCH 2015
troponin and CK-MB. What type of necrosis is MD)
expected in the cardiac muscle?
A. Coagulation
B. Liquefactive
C. Gangrenous
D. Enzymatic fat
342 A 50 year old male alcoholic presents with liver This patient presents with signs of chronic liver JESSICA MAE BACK-UP
failure. He has spider angiomata and testicular disease. The pathologic hallmark of chronic liver SANCHEZ, MD (TOP 4 MIDTERM
atrophy. A liver biopsy would reveal which of disease is the presence of fibrosis. - AUG 2014 MED EXAM - FEB
the following? BOARDS; TOPNOTCH 2015
A. Ballooning degeneration of hepatocytes MD)
B. Disorganized liver cells with Councilman
bodies
C. Regenerating hepatic nodules surrounded by
extensive fibrous tissue
D. Severe congestion with centrilobular atrophy

TOPNOTCH MEDICAL BOARD PREP PATHOLOGY SUPEREXAM Page 47 of 99


For inquiries visit www.topnotchboardprep.com.ph or email us at topnotchmedicalboardprep@gmail.com
TOPNOTCH MEDICAL BOARD PREP PATHOLOGY SUPEREXAM
For inquiries visit www.topnotchboardprep.com.ph or email us at topnotchmedicalboardprep@gmail.com
Item QUESTION EXPLANATION AUTHOR TOPNOTCH
# EXAM
343 An 18 year old girl sustained a gaping wound in JESSICA MAE BACK-UP
the thigh which was not sutured. The healed SANCHEZ, MD (TOP 4 MIDTERM
wound showed an elevated scar which did not - AUG 2014 MED EXAM - FEB
go beyond the original margin of the wound. BOARDS; TOPNOTCH 2015
Which of the following abnormalities in repair MD)
occurred in her case?
A. Fibromatosis
B. Deficient wound contraction
C. Failure of collagen maturation
D. Excessive granulation tissue formation
344 A 59 year old woman had loss of consciousness This patient had an ischemic stroke. For unknown JESSICA MAE BACK-UP
that persisted for over an hour. When she reasons, hypoxic death of cells within the CNS often SANCHEZ, MD (TOP 4 MIDTERM
became arousable, she cannot speak nor move manifests as liquefactive necrosis. - AUG 2014 MED EXAM - FEB
her right arm or leg. A cerebral angiogram BOARDS; TOPNOTCH 2015
revealed an occlusion to her left middle Reference: Robbins, Pathologic Basis of Disease, 8th MD)
cerebral artery. Months later, a CT scan shows a ed. p. 15
large 5 cm cystic area in her left parietal lobe
cortex. This CT finding is most likely the
consequence of resolution from which of the
following cellular events?
A. Apoptosis
B. Atrophy
C. Coagulation necrosis
D. Liquefactive necrosis
345 A 15 year old girl had episodes of sneezing with Allergic reactions and atopic diseases, such as allergic JESSICA MAE BACK-UP
watery eyes and runny nose for the past 2 rhinitis, are examples of type I hypersensitivity SANCHEZ, MD (TOP 4 MIDTERM
weeks. On physical examination, she has red, reactions which are mediated by substances released - AUG 2014 MED EXAM - FEB
swollen nasal mucosa. She has had similar from mast cells, such as histamine. BOARDS; TOPNOTCH 2015
episodes in the summer, when the amount of MD)
pollen in the air is high. Her symptoms are most
likely to be mediated by the release of which of
the following chemical mediators?
A. Complement C3b
B. Histamine
C. Platelet activating factor
D. Immunoglobulin G
346 Classic polyarteritis nodosa spares the blood Reference: Topnotch Pathology Handouts JESSICA MAE BACK-UP
vessels in which of the following organs? SANCHEZ, MD (TOP 4 MIDTERM
A. Spleen - AUG 2014 MED EXAM - FEB
B. Kidneys BOARDS; TOPNOTCH 2015
C. Heart MD)
D. Lungs
347 A 12 year old girl was brought to a Diagnosis is Rheumatic Fever. Based on the Jones JESSICA MAE BACK-UP
paediatrician because of joint pains. This was Criteria, fever and joint pains are only minor criteria SANCHEZ, MD (TOP 4 MIDTERM
accompanied by moderate grade fever. On PE, (migratory polyarthritis is a major criterion), and - AUG 2014 MED EXAM - FEB
the paediatrician noted diastolic murmurs and evidence of prior Strep infection, though a required BOARDS; TOPNOTCH 2015
friction rub. The patient had a history of criterion, is not considered as one of the five major MD)
pharyngitis 10 weeks prior to consultation. The criteria.
major criterion seen in this patient that would
help in your diagnosis is: A. Carditis
B. Fever
C. Joint pains
D. Previous history of pharyngitis
348 A 30 year old pedicab driver complains of PE findings suggest consolidation. Choice B is JESSICA MAE BACK-UP
productive cough, fever, and dyspnea. He was consistent with pathologic findings of consolidation. SANCHEZ, MD (TOP 4 MIDTERM
diagnosed with Bacterial Pneumonia. On PE, - AUG 2014 MED EXAM - FEB
dullness on percussion over the left lung field BOARDS; TOPNOTCH 2015
was elicited. What microscopic finding can MD)
explain these signs and symptoms?
A. Interstitial edema and interstitial capillary
vessel congestion
B. Intraalveolar leukocyte accumulation with
red cell exudation
C. Minimal fluid accumulation in pleural
cavities with fibrin deposition
D. Pulmonary cavitary formation with
intracavitary necrotic debris accumulation
349 A 60 year old patient with an 80 pack year The most likely diagnosis in this case is Squamous cell JESSICA MAE BACK-UP
smoking history was seen at the ER because of carcinoma. Keratin pearls are seen in this type of SANCHEZ, MD (TOP 4 MIDTERM
productive cough, weight loss, and an episode cancer. - AUG 2014 MED EXAM - FEB
of hemoptysis. A chest X-ray showed a 6 cm BOARDS; TOPNOTCH 2015
mass on the middle lobe. Bronchoscopy MD)
revealed an endophytic mass almost
obstructing the segmental bronchus. Which
pathology is associated with the most likely
diagnosis for this case?
A. Keratin pearl formation
B. Mucin lakes
C. Psammoma body formation
D. Giant cell formation

TOPNOTCH MEDICAL BOARD PREP PATHOLOGY SUPEREXAM Page 48 of 99


For inquiries visit www.topnotchboardprep.com.ph or email us at topnotchmedicalboardprep@gmail.com
TOPNOTCH MEDICAL BOARD PREP PATHOLOGY SUPEREXAM
For inquiries visit www.topnotchboardprep.com.ph or email us at topnotchmedicalboardprep@gmail.com
Item QUESTION EXPLANATION AUTHOR TOPNOTCH
# EXAM
350 Which of the following is an essential Reference: Robbins, Pathologic Basis of Disease, 8th JESSICA MAE BACK-UP
component in the definition of Barrett ed. p. 770 SANCHEZ, MD (TOP 4 MIDTERM
esophagus? - AUG 2014 MED EXAM - FEB
A. Columnar epithelium BOARDS; TOPNOTCH 2015
B. Intestinal type epithelium with goblet cells MD)
C. Fundic type gastric epithelium
D. Cardiac type gastric epithelium
351 A 5 year old boy with stunted growth has JESSICA MAE BACK-UP
worsening headaches for 2 months. A cranial SANCHEZ, MD (TOP 4 MIDTERM
CT scan reveals a 1.5 cm mass expanding the - AUG 2014 MED EXAM - FEB
sella turcica. The mass is cystic with scattered BOARDS; TOPNOTCH 2015
calcification. Which of the following is the most MD)
likely diagnosis?
A. Neuroblastoma
B. Craniopharyngioma
C. Pituitary adenoma
D. Mature teratoma
352 In benign hypertension, the arterioles of the Onion skinning and fibrinoid necrosis are associated JESSICA MAE BACK-UP
kidney would show: with malignant hypertension. SANCHEZ, MD (TOP 4 MIDTERM
A. Arteriolitis - AUG 2014 MED EXAM - FEB
B. Onion-skinning BOARDS; TOPNOTCH 2015
C. Fibrinoid necrosis MD)
D. Hyaline thickening
353 A 5 year old boy is brought to the paediatrician The most common cause of nephrotic syndrome in JESSICA MAE BACK-UP
because of periorbital and bipedal edema. children is Minimal Change Disease. A renal biopsy SANCHEZ, MD (TOP 4 MIDTERM
Urinalysis showed the following: sugar - would show no changes in light microscopy, but may - AUG 2014 MED EXAM - FEB
negative; protein - 4+; RBC - 0-1/hpf; show effacement of podocytes on electron BOARDS; TOPNOTCH 2015
leucocytes - 0-1/lpf. Blood chemistry shows: microscopy. MD)
albumin - 2g/dL; cholesterol - 1.8 mmol/L.
Light microscopy of a kidney biopsy of this
patient would most likely show:
A. Normocellular glomeruli
B. Glomeruli with endocapillary
hypercellularity
C. Glomeruli with extracapillary
hypercellularity
D. Hypocellular glomeruli
354 A 20 year old female was diagnosed with SLE 5 Wire loop lesions seen in lupus nephritis reflect active JESSICA MAE BACK-UP
years ago. A renal biopsy is performed because disease. SANCHEZ, MD (TOP 4 MIDTERM
she presents now with nephrotic and nephritic - AUG 2014 MED EXAM - FEB
syndromes. Light microscopy of the biopsy Reference: Robbins, Pathologic Basis of Disease, 8th BOARDS; TOPNOTCH 2015
would most likely show: ed. p. 219 MD)
A. Spike and dome appearance
B. Tram-track basement membrane
C. Wire-loop lesions
D. Normocellular glomeruli
355 A 10 year old boy consulted a physician Reference: Robbins, Pathologic Basis of Disease, 8th JESSICA MAE BACK-UP
because of tea coloured urine and puffiness of ed. p. 919 SANCHEZ, MD (TOP 4 MIDTERM
the eyelids, especially noted in the morning. - AUG 2014 MED EXAM - FEB
Blood pressure is 140/90. These symptoms BOARDS; TOPNOTCH 2015
were noted 3 weeks after he had fever and sore MD)
throat. Urinalysis showed: protein - 4+; sugar -
negative; leucocytes - 0-1/hpf; RBC - 10-
20/hpf; red cell cast - 1-2/lpf; granular cast - 0-
1/lpf; waxy cast - 0-1/lpf. Electron microscopy
of this patients renal biopsy would most likely
show:
A. Electron dense deposits with fingerprint
appearance
B. Subepithelial humps
C. No electron dense deposits
D. Subendothelial humps and spikes
356 A 5 year old boy was admitted for high grade CSF findings are characteristic of Tuberculous JESSICA MAE BACK-UP
fever with nuchal rigidity on PE. CSF meningitis. Viral meningitis would present with SANCHEZ, MD (TOP 4 MIDTERM
examination revealed moderate pleocytosis moderately elevated protein, and usually normal - AUG 2014 MED EXAM - FEB
with lymphocyte predominance, with increased glucose levels. Bacterial meningitis presents with BOARDS; TOPNOTCH 2015
protein and decreased glucose. What is the neutrophil predominance. MD)
most likely diagnosis?
A. Acute bacterial meningitis
B. Viral meningitis
C. Tuberculous meningitis
D. Subarachnoid hemorrhage
357 A 25 year old woman with recent onset of a Reference: Robbins, Pathologic Basis of Disease, 8th JESSICA MAE BACK-UP
major depressive disorder ingests an entire ed. p. 416 SANCHEZ, MD (TOP 4 MIDTERM
bottle of acetaminophen. She becomes - AUG 2014 MED EXAM - FEB
progressively obtunded over the next 8 hours. BOARDS; TOPNOTCH 2015
Which of the following microscopic findings is MD)
most likely to be present in her liver 3 days
following ingestion?
A. Normal histology
B. Extensive necrosis
C. Severe steatosis
D. Bridging fibrosis

TOPNOTCH MEDICAL BOARD PREP PATHOLOGY SUPEREXAM Page 49 of 99


For inquiries visit www.topnotchboardprep.com.ph or email us at topnotchmedicalboardprep@gmail.com
TOPNOTCH MEDICAL BOARD PREP PATHOLOGY SUPEREXAM
For inquiries visit www.topnotchboardprep.com.ph or email us at topnotchmedicalboardprep@gmail.com
Item QUESTION EXPLANATION AUTHOR TOPNOTCH
# EXAM
358 A 9 year old girl has a firm well-circumscribed Diagnosis is Thyroglossal duct cyst. JESSICA MAE BACK-UP
midline nodule on her neck that moves SANCHEZ, MD (TOP 4 MIDTERM
upwards with protrusion of the tongue. FNAB - AUG 2014 MED EXAM - FEB
shows mucus and benign epithelial cells. Which BOARDS; TOPNOTCH 2015
is a correct statement about her condition? MD)
A. The serum TSH is probably increased
B. This is due to cystic dilatation of an
embryologic remnant
C. This is a very common condition especially
among Asians
D. Papillary carcinoma is never associated with
it
359 A 30 year old motorcycle driver develops JESSICA MAE BACK-UP
polyuria and polydipsia following a motor SANCHEZ, MD (TOP 4 MIDTERM
vehicular accident. His condition is most - AUG 2014 MED EXAM - FEB
probably the result of which of the following? BOARDS; TOPNOTCH 2015
A. Deficiency of vasopressin MD)
B. Excess of growth hormone
C. Deficiency of insulin
D. Excess of PTH
360 Which of the following is usually produced by Acidophilic cells: GH, prolactin JESSICA MAE BACK-UP
an acidophilic pituitary adenoma? Basophilic cells: FSH, LH, ACTH, TSH SANCHEZ, MD (TOP 4 MIDTERM
A. TSH - AUG 2014 MED EXAM - FEB
B. FSH/LH BOARDS; TOPNOTCH 2015
C. Growth Hormone MD)
D. ACTH
361 Which of the following is/are the hallmark/s of There are 8 hallmarks of cancer: Selfsufficiency in MAIRRE JAMES BACK-UP
cancer? growth signals, Insensitivity to growthinhibitory GADDI, MD (TOP 4 - MIDTERM
A. Evasion of apoptosis signals, Altered cellular metabolism, Evasion of AUG 2013 MED EXAM - FEB
B. Metastasis apoptosis, Limitless replicative potential BOARDS; TOPNOTCH 2015
C. Invasion (immortality), Sustained angiogenesis, Ability to MD)
D. Sustained angiogenesis invade and metastasize and Ability to evade the host
E. All of the above immune response. Robbins 9th pg 282-283

362 As the pathologist tasked to autopsy a 10/M There is widespread neuronal degeneration and an MAIRRE JAMES BACK-UP
who recently died due to rabies, you know that inflammatory reaction that is most severe in the GADDI, MD (TOP 4 - MIDTERM
Negri bodies can be found in the? brainstem. The basal ganglia, spinal cord, and dorsal AUG 2013 MED EXAM - FEB
A. Cerebral cortex root ganglia may also be involved. Negri bodies are BOARDS; TOPNOTCH 2015
B. Cerebellum cytoplasmic, round to oval, eosinophilic inclusions MD)
C. Pons found in the pyramidal neurons of the hippocampus
D. Medulla and Purkinje cells of the cerebellum. Robbins 9th pg
E. None of the above 1277

363 Which of the following is TRUE regarding Anti-apoptotic proteins include BCL-2 BCL-xL and MAIRRE JAMES BACK-UP
apoptosis? MCL1 while pro-apoptotic proteins include BAX and GADDI, MD (TOP 4 - MIDTERM
A. The intrinsic pathway is initiated by the BAK. Sensors of cellular stress and damage which are AUG 2013 MED EXAM - FEB
release of cytochrome c into the cytoplasm the regulators between the two groups include BAD BOARDS; TOPNOTCH 2015
leading to activation of caspase 9 BIM BID Puma and Noxa Robbins 9th pg 53-56 MD)
B. Apoptosis is inhibited by anti-apoptotic
proteins such as BAX and BAK
C. Triggering of the death receptor pathway
leads to the activation of caspase 8 and 10
D. A and C
E. All of the above
364 A 60/F who has recurrent fever, fatigue, weight Sarcoidosis is a multisystem disease of unknown MAIRRE JAMES BACK-UP
loss, cough and night sweats came to you for etiology; the diagnostic histopathologic feature is the GADDI, MD (TOP 4 - MIDTERM
consult. You ordered chest xray which showed presence of noncaseating granulomas in various AUG 2013 MED EXAM - FEB
enlarged hilar lymph nodes and multiple 1-2 tissues. Clinically, It may be discovered unexpectedly BOARDS; TOPNOTCH 2015
cm non-cavitating lesions. Biopsy of the lesions on routine chest films as bilateral hilar adenopathy. It MD)
was done showing non-caseating granulomata may presents with shortness of breath, cough, chest
with fibrosis. The most probable diagnosis is: pain, hemoptysis and constitutional signs and
A. Lung carcinoma symptoms (fever, fatigue, weight loss, anorexia, night
B. Pulmonary tuberculosis sweats). Robbins 9th pg 693-694
C. Mycobacterium avium-intracellulare
infection
D. Sarcoidosis
E. Silicosis
365 A 60/M presents with seizures and left sided The histologic appearance of glioblastoma is similar to MAIRRE JAMES BACK-UP
weakness. MRI was done which showed a large anaplastic astrocytoma with the additional features of GADDI, MD (TOP 4 - MIDTERM
heterogenous mass on the right cerebral necrosis and vascular/endothelial cell AUG 2013 MED EXAM - FEB
hemisphere. The patient was then referred to proliferation.Tumor cells collect along the edges of the BOARDS; TOPNOTCH 2015
Dr. Karl F who then proceeded to remove the necrotic regions, producing a histologic pattern MD)
mass as best as he could. As the pathologist, you referred to as pseudo-palisading. Robbins 9th pg
examined sections of the mass and you noted 1308
multiple foci of necrosis, hemorrhage and
pseudo-palisading tumor cells. What is your
diagnosis?
A. Oligodendroglioma
B. Glioblastoma multiforme
C. Anaplastic astrocytoma
D. Ependymoma
E. Pilocytic astrocytoma

TOPNOTCH MEDICAL BOARD PREP PATHOLOGY SUPEREXAM Page 50 of 99


For inquiries visit www.topnotchboardprep.com.ph or email us at topnotchmedicalboardprep@gmail.com
TOPNOTCH MEDICAL BOARD PREP PATHOLOGY SUPEREXAM
For inquiries visit www.topnotchboardprep.com.ph or email us at topnotchmedicalboardprep@gmail.com
Item QUESTION EXPLANATION AUTHOR TOPNOTCH
# EXAM
366 A previously well 60/M presents with sudden Angiodysplasia, a lesion with malformed submucosal MAIRRE JAMES BACK-UP
painless massive LGIB and he was then and mucosal blood vessels, occurs most often in the GADDI, MD (TOP 4 - MIDTERM
immediately brought to a local hospital. In the cecum or right colon and usually presents after the AUG 2013 MED EXAM - FEB
hospital, the bleeding was still massive and sixth decade of life.The lesions are characterized by BOARDS; TOPNOTCH 2015
persistent. Sigmoid colonoscopy was done ectatic nests of tortuous veins, venules, and capillaries. MD)
which failed to reveal the site of bleeding. The The vascular channels may be separated from the
only finding was that there were multiple intestinal lumen by only the vascular wall and limited
outpouchings in the sigmoid colon. Due to the injury may therefore result in significant bleeding.
difficulty in localizing and in controlling the Robbins 9th pg 780-781; Symptoms of diverticular
bleeding, total colectomy was done. As the disease include intermittent cramping, continuous
pathologist evaluating the specimen, you found lower abdominal discomfort, constipation, distention,
multiple outpouchings in the sigmoid and or a sensation of never being able to completely empty
tortuous and ectatic submucosal and mucosal the rectum. Occasionally there may be minimal
vessels in the cecum. What is your diagnosis? chronic or intermittent blood loss, and, rarely, massive
A. Diverticulosis hemorrhage. pg 804
B. Diverticulitis
C. Angiodysplasia
D. Ulcerative colitis
E. Mesenteric ischemia
367 A 45/M was diagnosed to have HNPCC. What is In HNPCC DNA mismatch repair deficiency causes MAIRRE JAMES BACK-UP
the underlying molecular pathology which led mutations to accumulate in microsatellite repeats, a GADDI, MD (TOP 4 - MIDTERM
to the development of the patient's condition? condition referred to as microsatellite instability . AUG 2013 MED EXAM - FEB
A. DNA mismatch repair deficiency Robbins 9th pg 812. Choice B-D refers to the BOARDS; TOPNOTCH 2015
B. APC gene mutation at 5q21 adenoma-carcinoma sequence with the last step being MD)
C. K-RAS gene mutation at 12p12 TP53 17q13 mutation pg 811. Loss-of-function
D. Loss of heterozygosity at 18q21 mutations in the gene STK11 are present in
E. Loss of function of the gene STK11 approximately half of individuals with familial Peutz-
Jeghers syndrome pg 806
368 Which of the following is/are small round blue Other examples include rhabdomyosarcoma, MAIRRE JAMES BACK-UP
cell tumors? medulloblastoma, small cell lung carcinoma, small-cell GADDI, MD (TOP 4 - MIDTERM
A. Neuroblastoma lymphom AUG 2013 MED EXAM - FEB
B. Wilm's Tumor BOARDS; TOPNOTCH 2015
C. Ewing sarcoma MD)
D. A and C
E. All of the above

369 A 35-year-old woman had a firm nodule Leiomyosarcomas are uncommon malignant MAIRRE JAMES BACK-UP
palpable on the dome of the uterus six years neoplasms that are thought to arise from myometrial GADDI, MD (TOP 4 - MIDTERM
ago on routine examination. The nodule has or endometrial stromal precursor cells, rather than AUG 2013 MED EXAM - FEB
slowly increased in size and is now about twice from degenration of leiomyomas. Leiomyomas are BOARDS; TOPNOTCH 2015
the size it was when first discovered. She is sharply circumscribed with low mitotic index in MD)
asymptomatic. She opted to have total contrast to leiomyosarcomas Robbins 9th pg 1020
hysterectomy done. As the pathologist, you
found that the uterus was grossly distorted and
asymmetric. The mass measures 14 x 10 x 12
cm located at the posterior midcorpus. Sections
of the mass showed a whorled white surface
with multiple large areas of necrosis and ill-
defined borders. Which of the following is most
likely?
A. Adenocarcinoma
B. Leiomyosarcoma
C. Adenomyosis
D. Leiomyoma
E. Metastasis
370 Which of the following is/are responsible for Macrophages produce IL-1, 6, 8,12, and TNF alpha; all MAIRRE JAMES BACK-UP
the production IL-1? T cells produce IL-2, 3; TH1 cells produce IFN gamma; GADDI, MD (TOP 4 - MIDTERM
A. Macrophage TH2 cells produce IL-4,5,10 Robbins 9th pg 198 AUG 2013 MED EXAM - FEB
B. TH1 cell BOARDS; TOPNOTCH 2015
C. TH2 cell MD)
D. B cells
E. B and C

371 A 35/M presents with the classic triad of This classic triad or Charcot's neurologic triad is MAIRRE JAMES BACK-UP
scanning speech, intention tremor and associated with multiple sclerosis. MS is a type IV GADDI, MD (TOP 4 - MIDTERM
nystagmus. What type of hypersensitivity hypersensitivity reaction Robbins 9th pg 209 AUG 2013 MED EXAM - FEB
reaction is responsible for the patient's BOARDS; TOPNOTCH 2015
disorder? MD)
A. Type I
B. Type II
C. Type III
D. Type IV
E. The illness described is not caused by a
hypersensitivity reaction
372 A 34/F with lupus successfully gave birth to a Anti-Ro/SS-A and Anti-La/SS-B are associated with MAIRRE JAMES BACK-UP
live baby girl. On PE, the neonate was congenital heart block and neonatal lupus Robbins 9th GADDI, MD (TOP 4 - MIDTERM
bradycardic but otherwise asymptomatic with pg 219 AUG 2013 MED EXAM - FEB
no signs of distress. On ECG, there was BOARDS; TOPNOTCH 2015
prolongation of the PR interval followed by a MD)
dropped beat. What marker/s is/are associated
with this condition?
A. Anti-Ro
B. Anti-CCP
C. Anti-dsDNA
D. Anti-Sm
E. C and D

TOPNOTCH MEDICAL BOARD PREP PATHOLOGY SUPEREXAM Page 51 of 99


For inquiries visit www.topnotchboardprep.com.ph or email us at topnotchmedicalboardprep@gmail.com
TOPNOTCH MEDICAL BOARD PREP PATHOLOGY SUPEREXAM
For inquiries visit www.topnotchboardprep.com.ph or email us at topnotchmedicalboardprep@gmail.com
Item QUESTION EXPLANATION AUTHOR TOPNOTCH
# EXAM
373 C3 convertase splits C3 into two distinct All three pathways of complement activation lead to MAIRRE JAMES BACK-UP
fragments, C3a and C3b. C3 convertase is the formation of an active enzyme called the C3 GADDI, MD (TOP 4 - MIDTERM
formed through which pathway/s? conver- tase, which splits C3 into two functionally AUG 2013 MED EXAM - FEB
A. Classical distinct frag- ments, C3a and C3b. Robbins 9th pg 88 BOARDS; TOPNOTCH 2015
B. Alternative MD)
C. Lectin
D. A and B
E. All of the above

374 A 10/M, diagnosed to have mental retardation, CTG - myotonic dystrophy; GAA - Friedreich ataxia; MAIRRE JAMES BACK-UP
underwent karyotypic analysis. The was no CAG - Huntington disease Robbins 9th pg 168-169 GADDI, MD (TOP 4 - MIDTERM
aneuploidy present but a discontinuity of AUG 2013 MED EXAM - FEB
staining was seen in the long arm of the X BOARDS; TOPNOTCH 2015
chromosome. Expansion of what type of MD)
trinucleotide repeat sequence can be expected
from the patient?
A. CTG
B. CGG
C. GAA
D. CAG
E. CCC
375 What disorder is due to a single gene mutation The mutation results in a glutamine to arginine MAIRRE JAMES BACK-UP
causing resistance to cleavage and inactivation substitution at amino acid residue 506 that renders GADDI, MD (TOP 4 - MIDTERM
by protein C resulting in a hypercoaguable factor V resistant to cleavage and inactivation by AUG 2013 MED EXAM - FEB
state? protein C. Robbins 9th pg 123-124 BOARDS; TOPNOTCH 2015
A. Factor V Leiden MD)
B. Sickle cell anemia
C. Protein C deficiency
D. Protein S deficiency
E. Antithrombin III deficiency
376 A 34/F with recurrent headaches was Acute hemorrhage into an adenoma is associated with MAIRRE JAMES BACK-UP
diagnosed to have a suprasellar tumor 2 clinical evidence of rapid enlargement of the lesion, a GADDI, MD (TOP 4 - MIDTERM
months ago. While walking she suddenly situation appropriately termed pituitary apoplexy. AUG 2013 MED EXAM - FEB
complained of blindness and was subsequently Robbins 9th pg 1075 BOARDS; TOPNOTCH 2015
rushed to the hospital. In the ER she was MD)
vomiting and complained of a severe headache.
What is your diagnosis?
A. Pituitary microadenoma
B. Craniopharyngioma
C. Pituitary apoplexy
D. Pituitary macroadenoma
E. Prolactinoma
377 A 18/M presents with periorbital edema, B - FSGS; C - MCD; D - Alport syndrome Robbins 9th MAIRRE JAMES BACK-UP
proteinuria and hematuria. You suspect MPGN pg 917, 919, 920, 924 GADDI, MD (TOP 4 - MIDTERM
type 1 for this case. If you were to do kidney AUG 2013 MED EXAM - FEB
biopsy, you would expect to find? BOARDS; TOPNOTCH 2015
A. Large and hypercellular glomeruli with MD)
thickened and duplicated GBM
B. Sclerotic segments with collapse of capillary
loops, increase in matrix and segmental
deposition of plasma proteins along the
capillary wall
C. Uniform and diffuse effacement of foot
processes in the visceral epithelial cells
D. GBM shows irregular foci of thickening
alternating with thinning and splitting and
lamination of the lamina densa
E. None of the above
378 Which of the following best describes the Robbins 9th pg 793 Choice A describes C. difficile pg MAIRRE JAMES BACK-UP
pathology of a Rotavirus infection? 791 Choice B describes Shigella pg 788 Choice D GADDI, MD (TOP 4 - MIDTERM
A. Pseudomembrane formation made up of an describes Salmonella pg 789 AUG 2013 MED EXAM - FEB
adherent layer of inflammatory cells BOARDS; TOPNOTCH 2015
B. The mucosa of the left colon is hemorrhagic MD)
and ulcerated, and pseudomembranes may be
present
C. Selective infection and destruction of mature
enterocytes in the small intestine with the villus
surface repopulated by immature secretory
cells
D. Enlargement of Peyer patches in the terminal
ileum with oval ulcers that may perforate
E. None of the above
379 The characteristic histologic finding/s of The other characteristic histologic findings of asthma, MAIRRE JAMES BACK-UP
asthma include/s? collectively called airway remodeling, include GADDI, MD (TOP 4 - MIDTERM
A. Thickening of the airway wall thickening of airway wall, subbasement membrane AUG 2013 MED EXAM - FEB
B. Hypertrophy of the bronchial wall muscle fibrosis (due to deposition of type I and III collagen), BOARDS; TOPNOTCH 2015
C. Increase in the size of the submucosal glands increased vascularity, increase in the size of the MD)
and number of airway goblet cells submucosal glands and number of airway goblet cells
D. A and B and hypertrophy and/or hyperplasia of the bronchial
E. All of the above wall muscle Robbins 9th pg 682

TOPNOTCH MEDICAL BOARD PREP PATHOLOGY SUPEREXAM Page 52 of 99


For inquiries visit www.topnotchboardprep.com.ph or email us at topnotchmedicalboardprep@gmail.com
TOPNOTCH MEDICAL BOARD PREP PATHOLOGY SUPEREXAM
For inquiries visit www.topnotchboardprep.com.ph or email us at topnotchmedicalboardprep@gmail.com
Item QUESTION EXPLANATION AUTHOR TOPNOTCH
# EXAM
380 Choristomas are: Choice A refers to hamartomas Robbins 9th pg 473 MAIRRE JAMES BACK-UP
A. Excessive focal overgrowths of cells and Choice B - choriocarcinoma Choice C - teratoma GADDI, MD (TOP 4 - MIDTERM
tissues native to the organ where it occurs AUG 2013 MED EXAM - FEB
B. Malignant germ cell tumors composed of BOARDS; TOPNOTCH 2015
syncitiotrophoblasts and cytotrophoblasts MD)
C. Helter-skelter collection of differentiated
cells or organoid structures (neural tissue,
islands of cartilage, etc) embedded in a myxoid
stroma
D. Collection of normal cells or tissues in
abnormal locations
E. None of the above
381 What is the most common site of metastasis of SCOTT RILEY ONG, BACK-UP
lung cancers? MD (TOP 5 - AUG MIDTERM
A. Bone 2014 MED BOARDS; EXAM - FEB
B. Liver TOPNOTCH MD) 2015
C. Adrenal
D. Brain
E. Kidney

382 The pathognomonic inclusion bodies in nerve SIMILAR TO PREVIOUS BOARD EXAM SCOTT RILEY ONG, BACK-UP
cells infected by the rabies virus are most CONCEPT/PRINCIPLE. The inclusion bodies refer to MD (TOP 5 - AUG MIDTERM
commonly found in which part of the CNS? Negri bodies, which are most commonly seen in the 2014 MED BOARDS; EXAM - FEB
A. Cerebral cortex hippocampus. Cerebellum is the 2nd most common TOPNOTCH MD) 2015
B. Hippocampus site.
C. Basal ganglia
D. Cerebellum
E. Medulla oblongata

383 In which of the following conditions is fibrinous Caseous and adhesive pericarditis are the forms SCOTT RILEY ONG, BACK-UP
pericarditis least likely to occur? associated with TB. MD (TOP 5 - AUG MIDTERM
A. Dressler syndrome 2014 MED BOARDS; EXAM - FEB
B. Tuberculosis TOPNOTCH MD) 2015
C. Systemic lupus erythematosus
D. Rheumatic heart disease
E. None of the above

384 Which of the following laboratory findings will SCOTT RILEY ONG, BACK-UP
you expect in patients with hemophilia A? MD (TOP 5 - AUG MIDTERM
A. Prolonged prothrombin time 2014 MED BOARDS; EXAM - FEB
B. Prolonhged partial thromboplastin time TOPNOTCH MD) 2015
C. Decreased platelet count
D. Prolonged bleeding time
E. Both A and D

385 ABO blood type incompatibility is an example SCOTT RILEY ONG, BACK-UP
of which hypersensitivity reaction? MD (TOP 5 - AUG MIDTERM
A. Type I hypersensitivity reaction 2014 MED BOARDS; EXAM - FEB
B. Type II hypersensitivity reaction TOPNOTCH MD) 2015
C. Type III hypersensitivity reaction
D. Type IV hypersensitivity reaction
E. None of the above

386 The following laboratory findings are SIMILAR TO PREVIOUS BOARD EXAM SCOTT RILEY ONG, BACK-UP
consistent with iron-deficiency anemia except: CONCEPT/PRINCIPLE. MD (TOP 5 - AUG MIDTERM
A. Hypochromic RBCs 2014 MED BOARDS; EXAM - FEB
B. Decreased MCV TOPNOTCH MD) 2015
C. Increased RDW
D. Anisocytosis
E. Decreased MCHC

387 Which of the following is not a small-vessel Polyarteritis nodosa is classically a medium-vessel SCOTT RILEY ONG, BACK-UP
vasculitides? disease. MD (TOP 5 - AUG MIDTERM
A. Polyarteritis nodosa 2014 MED BOARDS; EXAM - FEB
B. Henoch-Schonlein purpura TOPNOTCH MD) 2015
C. Churg-Strauss syndrome
D. Wegener granulomatosis
E. None of the above

388 A renal biopsy that shows hypercellular SCOTT RILEY ONG, BACK-UP
glomeruli on light microscopy, "starry sky" MD (TOP 5 - AUG MIDTERM
pattern of immunofluorescence and 2014 MED BOARDS; EXAM - FEB
subepithelial immune complex humps on TOPNOTCH MD) 2015
electron microscopy is most consistent with
which of the following diagnosis?
A. Rapidly progressive glomerulonephritis
B. Acute poststreptococcal glomerulonephritis
C. IgA nephropathy
D. Membranoproliferative glomerulonephritis
E. Lupus nephritis
389 Which of the following describes the most A: type I. B: type II. C: type III. D: type IV. E: type V SCOTT RILEY ONG, BACK-UP
common type of choledochal cyst? (Caroli disease) MD (TOP 5 - AUG MIDTERM
A. Fusiform dilatation of the common bile duct 2014 MED BOARDS; EXAM - FEB
B. Diverticulum arising from the common bile TOPNOTCH MD) 2015
duct and attached to it by a narrow stalk
C. Focal dilatation of the intraduodenal portion
of the common bile duct
D. Multiple saccular dilatations of the intra- and
TOPNOTCH MEDICAL BOARD PREP PATHOLOGY SUPEREXAM Page 53 of 99
For inquiries visit www.topnotchboardprep.com.ph or email us at topnotchmedicalboardprep@gmail.com
TOPNOTCH MEDICAL BOARD PREP PATHOLOGY SUPEREXAM
For inquiries visit www.topnotchboardprep.com.ph or email us at topnotchmedicalboardprep@gmail.com
Item QUESTION EXPLANATION AUTHOR TOPNOTCH
# EXAM
extra-hepatic bile ducts
E. Saccular dilatations of the intrahepatic bile
ducts without biliary obstruction

390 What is the most common clinical presentation SCOTT RILEY ONG, BACK-UP
of multiple sclerosis? MD (TOP 5 - AUG MIDTERM
A. Nystagmus 2014 MED BOARDS; EXAM - FEB
B. Motor weakness TOPNOTCH MD) 2015
C. Optic neuritis
D. Intention tremor
E. Dementia

391 Which of the following features would make The other choices are more characteristic of Crohn SCOTT RILEY ONG, BACK-UP
you favor a diagnosis of ulcerative colitis over disease. MD (TOP 5 - AUG MIDTERM
Crohn disease? 2014 MED BOARDS; EXAM - FEB
A. Pseudopolyp formation TOPNOTCH MD) 2015
B. Fistula formation
C. Non-caseating granuloma
D. Paneth cell metaplasia
E. Aphthous ulcers

392 Which of the following features would make SIMILAR TO PREVIOUS BOARD EXAM SCOTT RILEY ONG, BACK-UP
you suspect that an ulcer is malignant? CONCEPT/PRINCIPLE. Option B is a classic feature of a MD (TOP 5 - AUG MIDTERM
A. Edematous ulcer collar with overhanging malignant ulcer. In barium studies of the upper GI 2014 MED BOARDS; EXAM - FEB
mucosal edges tract, this will present as the "Carmen meniscus sign". TOPNOTCH MD) 2015
B. Flat-based ulcer with heaped up edges The other options describe a benign ulcer.
C. Radiating folds extending into the crater of
the ulcer
D. Depth of the ulcer is greater than its width
E. Smooth ulcer mound with tapering edges
393 Which of the following viral hepatitis infection Hepatitis C is associated with 50% chronicity but not SCOTT RILEY ONG, BACK-UP
is least associated with fulminant hepatitis? with fulminant hepatitis. Hepatitis D can lead to MD (TOP 5 - AUG MIDTERM
A. Hepatitis B fulminant hepatitis in the setting of co-infection with 2014 MED BOARDS; EXAM - FEB
B. Hepatitis C Hep B. Hepatitis E is associated with fulminant TOPNOTCH MD) 2015
C. Hepatitis D hepatitis among pregnant women.
D. Hepatitis E
E. None of the above

394 Among patients with chronic viral hepatitis SIMILAR TO PREVIOUS BOARD EXAM SCOTT RILEY ONG, BACK-UP
infection who develop liver cirrhosis, what CONCEPT/PRINCIPLE. MD (TOP 5 - AUG MIDTERM
percentage will continue to progress into 2014 MED BOARDS; EXAM - FEB
hepatocellular carcinoma? TOPNOTCH MD) 2015
A. <2%
B. 5%
C. 6-15%
D. 12-20%
E. 50%
395 Which of the following is the most important SIMILAR TO PREVIOUS BOARD EXAM SCOTT RILEY ONG, BACK-UP
prognosticating factor in cases of breast CONCEPT/PRINCIPLE MD (TOP 5 - AUG MIDTERM
carcinoma? 2014 MED BOARDS; EXAM - FEB
A. HER-2/neu status TOPNOTCH MD) 2015
B. Family history
C. Tumor histology
D. Age
E. Axillary node status

396 Which of the following is the most common SCOTT RILEY ONG, BACK-UP
benign neoplasm of the liver? MD (TOP 5 - AUG MIDTERM
A. Hepatic adenoma 2014 MED BOARDS; EXAM - FEB
B. Cavernous hemangioma TOPNOTCH MD) 2015
C. Focal nodular hyperplasia
D. Lipoma
E. Biliary cystadenoma

397 The presence of a hyperpigmented, ulcerating SIMILAR TO PREVIOUS BOARD EXAM SCOTT RILEY ONG, BACK-UP
plaque with irregular borders and asymmetric CONCEPT/PRINCIPLE. Acral lentiginous melanoma, MD (TOP 5 - AUG MIDTERM
shape in which of the following body parts which occurs on the palms and soles and beneath the 2014 MED BOARDS; EXAM - FEB
should make you worry most? nails, has the worst prognosis among the different TOPNOTCH MD) 2015
A. Face types of melanoma.
B. Nape
C. Chest
D. Palms
E. Back

TOPNOTCH MEDICAL BOARD PREP PATHOLOGY SUPEREXAM Page 54 of 99


For inquiries visit www.topnotchboardprep.com.ph or email us at topnotchmedicalboardprep@gmail.com
TOPNOTCH MEDICAL BOARD PREP PATHOLOGY SUPEREXAM
For inquiries visit www.topnotchboardprep.com.ph or email us at topnotchmedicalboardprep@gmail.com
Item QUESTION EXPLANATION AUTHOR TOPNOTCH
# EXAM
398 A 32-year old female presented with symptoms SIMILAR TO PREVIOUS BOARD EXAM SCOTT RILEY ONG, BACK-UP
of fatigue, weight gain, cold intolerance, CONCEPT/PRINCIPLE. Lymphocytic thyroiditis also MD (TOP 5 - AUG MIDTERM
constipation and thinning of her hair. On present with painless enlargement of the thyroid 2014 MED BOARDS; EXAM - FEB
physical examination, you noted a firm, mildly gland, but it does not exhibit Hurthle cells on biopsy. TOPNOTCH MD) 2015
enlarged but painless thyroid gland. Biopsy Subacute thyroiditis presents with painful thryoid
revealed diffuse inflammatory infiltrates, enlargement.
atrophy of the thyroid follicles and
characteristic Hurthle cells. What is your most
likely diagnosis?
A. Hashimoto thyroiditis
B. Lymphocytic thyroiditis
C. Subacute thyroiditis
D. Graves disease
E. Reidel thyroiditis
399 A history of bladder exstrophy is a risk factor Schistosoma infection --> Squamous cell CA. Smoking - SCOTT RILEY ONG, BACK-UP
for the development of which type of bladder -> Transitional cell CA MD (TOP 5 - AUG MIDTERM
cancer? 2014 MED BOARDS; EXAM - FEB
A. Transitional cell carcinoma TOPNOTCH MD) 2015
B. Squamous cell carcinoma
C. Adenocarcinoma
D. Clear cell carcinoma
E. Any of the above

400 Which of the following conditions is SCOTT RILEY ONG, BACK-UP


characterized by findings of giant rugal MD (TOP 5 - AUG MIDTERM
hypertrophy, thickened gastric mucosa, 2014 MED BOARDS; EXAM - FEB
excessive mucus production, hypoproteinemia TOPNOTCH MD) 2015
and hypochlorydia?
A. Non-tropical sprue
B. Eosinophilic gastritis
C. Crohn gastritis
D. Menetrier disease
E. Zollinger-Ellison syndrome
401 A patient with chronic back pain came to you Chronic NSAID use presents with Papillary necrosis on JOSE CARLO DIAGNOSTIC
due to oliguria and had been abusing NSAIDs Renal biopsy. SIMILAR TO PREVIOUS BOARD EXAM MASANGKAY III, MD EXAM - AUG
chronically for the past years, if renal biopsy CONCEPT/PRINCIPLE (TOP 8 - FEB 2014 2014
will be made, which of the following would be MED BOARDS;
seen? TOPNOTCH MD)
A. Acute Tubular Necrosis
B. Papillary Necrosis
C. Effaced foot processes
D. Coagulative Necrosis of the nephrons
E. Nodular Sclerosis
402 A 3 year-old patient was brought to you by his This is a case of Wilm's Tumor, it is the most common JOSE CARLO DIAGNOSTIC
mother due to an abdominal mass which she renal tumor in childhood, it does not cross the midline MASANGKAY III, MD EXAM - AUG
palpated while bathing the child with a urine and mostly involves bilateral kidneys, biopsy will (TOP 8 - FEB 2014 2014
described as "iced-tea-like", upon your present with Triphasic combinations of blastema;, MED BOARDS;
thorough PE you noticed that the mass does not stromal and epithelial cell combinations. TOPNOTCH MD)
cross the midline and involves a synchronous
tumor. Your primary consideration would most
likely reveal which of the following biopsy
findings?
A. Varying amounts of immature
neuroepithelium, cartilage, bone, muslce and
others
B. Flexner-Wintersteiner Rosettes
C. Homer-Wright Pseudorosettes
D. Blastemal, stromal and epithelial cell
combinations
E. glomerulus-like structures composed of a
central blood vessel enveloped by germ cells
403 An elderly patient with a pulmonary mass and Squamous Cell CA will present with paraneoplastic JOSE CARLO DIAGNOSTIC
hypercalcemia underwent right pulmonectomy, hypercalcemia and is usually central in location. MASANGKAY III, MD EXAM - AUG
upon biopsy the pathologist noted the mass to SIMILAR TO PREVIOUS BOARD EXAM (TOP 8 - FEB 2014 2014
have grown from the alveolar duct and alveolar CONCEPT/PRINCIPLE MED BOARDS;
sacs, what would be the primary consideration? TOPNOTCH MD)
A. Small Cell Lung CA
B. Large Cell Lung CA
C. Squamous Cell Lung CA
D. Lung Adenocarcinoma
E. Metastatic Lung CA
404 A patient with Small Cell Lung cancer Lung cancers may metastasize using lymphatic or JOSE CARLO DIAGNOSTIC
underwent PET scan, a metastasis was noted, hematogenous routes to the Adrenals (50%), Liver MASANGKAY III, MD EXAM - AUG
which of the following sites would be the most (30-50%), Brain (20%), Bone (20%) (TOP 8 - FEB 2014 2014
common site of metastasis? MED BOARDS;
A. Bone (Ribs) TOPNOTCH MD)
B. Liver
C. Brain
D. Adrenals
E. Heart

TOPNOTCH MEDICAL BOARD PREP PATHOLOGY SUPEREXAM Page 55 of 99


For inquiries visit www.topnotchboardprep.com.ph or email us at topnotchmedicalboardprep@gmail.com
TOPNOTCH MEDICAL BOARD PREP PATHOLOGY SUPEREXAM
For inquiries visit www.topnotchboardprep.com.ph or email us at topnotchmedicalboardprep@gmail.com
Item QUESTION EXPLANATION AUTHOR TOPNOTCH
# EXAM
405 A 45-year old male patient with recurrent SIMILAR TO PREVIOUS BOARD EXAM JOSE CARLO DIAGNOSTIC
respiratory infections came to your office with CONCEPT/PRINCIPLE MASANGKAY III, MD EXAM - AUG
a peripheral smear result with noted Sea-blue (TOP 8 - FEB 2014 2014
histiocytes by the pathologist, what would be MED BOARDS;
your primary consideration? TOPNOTCH MD)
A. AML
B. ALL
C. CLL
D. CML
E. Hairy Cell Leukemia
406 A walk-in patient with multiple purpuric rashes vWD will present with a prolonged bleeding time due JOSE CARLO DIAGNOSTIC
came to your clinic asking for your to inadequate adhesion of platelets to the severed MASANGKAY III, MD EXAM - AUG
interpretation of his self-requested lab results: epithelium and a prolonged PTT due to the lack of (TOP 8 - FEB 2014 2014
Platelet Count 245,000/L and INR of 1.00 Factor 8 from vWF. MED BOARDS;
Bleeding Time was 15 mins, PTT was TOPNOTCH MD)
prolonged. What would be your impression?
A. TTP
B. Bernard-Soulier Syndrome
C. Glanzmann's Thrombasthenia
D. von Willebrand Disease
E. Hemophilia
407 A 78 year old male patient with a parotid mass Papillary Cystadenoma Lymphomatosum or Warthin's JOSE CARLO DIAGNOSTIC
and a chronic smoking history underwent tumor is the second most common tumor of the MASANGKAY III, MD EXAM - AUG
parotidectomy with findings of follicular parotid gland, occurs virtually in the parotid gland (TOP 8 - FEB 2014 2014
germinal centers and cystic spaces on biopsy, only and is classically seen in elderly patients with a MED BOARDS;
but because of your thirst for knowledge you chronic smoking history. TOPNOTCH MD)
read about it and you knew that this tumor is
virtually restricted to the parotid gland only
and is the second most common salivary
neoplasm. What is this tumor?
A. Pleomprphic Adenoma
B. Adenoid Cystic Adenoma
C. Papillary Cystadenoma Lymphomatosum
D. Mucoepidermoid adenoma
E. Parotid Metastasis from a Lung CA
408 Seborrheic Keratoses in a patient with Gastric JOSE CARLO DIAGNOSTIC
Adenocarcinoma is termed as what sign? MASANGKAY III, MD EXAM - AUG
A. Breslow's Sign (TOP 8 - FEB 2014 2014
B. Lesser-Trelat Sign MED BOARDS;
C. Charcot-Marie-Tooth Sign TOPNOTCH MD)
D. Ormond Sign
E. Zellballen Sign

409 A 34 year old man diagnosed as having type II The reason for this is that with decreased levels of JOSE CARLO DIAGNOSTIC
diabetes mellitus. Laboratory evaluation of his insulin with diabetes mellitus there is increased MASANGKAY III, MD EXAM - AUG
serum also finds hypertriglyceridemia, which is mobilization of free fatty acids from adipose tissue (TOP 8 - FEB 2014 2014
due to his diabetes. The most common type of (increased lipolysis). This increases delivery of free MED BOARDS;
secondary hyperlipidemia associated fatty acids to the liver, which increases production and TOPNOTCH MD)
with diabetes mellitus is characterized by secretion of VLDL by the liver.
elevated serum levels of which one of the
following substances?

A. Chylomicrons
B. High-density lipoproteins
C. Intermediate-density lipoproteins
D. Low-density lipoproteins
E. Very-low-density lipoproteins
410 Which of the following cytokines is secreted by IL1 and TNF-alpha are secreted by Macrophages, IL2 JOSE CARLO DIAGNOSTIC
macrophages and functions as a major Tcells, IL3 BM stem cells, IL4 stimulates IgE, IL5 MASANGKAY III, MD EXAM - AUG
mediator of acute inflammation by stimulating stimulates IgA (TOP 8 - FEB 2014 2014
acute phase reactions with increasing vascular MED BOARDS;
permeability and stimulating fibroblasts? TOPNOTCH MD)

A. Interleukin-1
B. Interleukin-2
C. Interleukin-3
D. Interleukin-4
E. Interleukin-5
411 You encountered a 35-year old patient with This is a classic case of Wegener's Granulomatosis, a JOSE CARLO DIAGNOSTIC
necrotizing granulomas along the respiratory small vessel vasculitis, positive for c-ANCA. MASANGKAY III, MD EXAM - AUG
tract and the lungs. He eventually died of (TOP 8 - FEB 2014 2014
crescentic glomerulonephritis. Which of the MED BOARDS;
following markers would turn out positive in TOPNOTCH MD)
this patient?
A. c-ANCA
B. p-ANCA
C. Anti-centromere
D. ANA
E. Anti-Smith

TOPNOTCH MEDICAL BOARD PREP PATHOLOGY SUPEREXAM Page 56 of 99


For inquiries visit www.topnotchboardprep.com.ph or email us at topnotchmedicalboardprep@gmail.com
TOPNOTCH MEDICAL BOARD PREP PATHOLOGY SUPEREXAM
For inquiries visit www.topnotchboardprep.com.ph or email us at topnotchmedicalboardprep@gmail.com
Item QUESTION EXPLANATION AUTHOR TOPNOTCH
# EXAM
412 Trinucleotide repeat Mutations are Fragile X Syndrome is the 2nd most common cause of JOSE CARLO DIAGNOSTIC
amplifications of a sequence of three mental retardation next to Down's Syndrom, classic MASANGKAY III, MD EXAM - AUG
nucleotides, they also are associated with an finding is macroorchidism (TOP 8 - FEB 2014 2014
Anticipation-type of inheritance, An example is MED BOARDS;
the second most common cause of Mental TOPNOTCH MD)
retardation with a prominent PE finding of
Macroorchidism. What Nucleotide repeat will
be seen in this patient?
A. CAG
B. GAA
C. CTG
D. CGG
E. GCA
413 A 1 year old patient was brought to you by his This is a case of Down Syndrome/ Trisomy 21, 95% of JOSE CARLO DIAGNOSTIC
mother due to noticeable developmental delay, cases are due to nondisjunction of chromosomes on MASANGKAY III, MD EXAM - AUG
you noticed that the patient has specific facies cell division causing 3 chromosome 21. (TOP 8 - FEB 2014 2014
like flat facial profile, epicanthal folds, MED BOARDS;
macroglossia and a prominent Simian Crease. TOPNOTCH MD)
What is the most common chromosomal
structural abnormality seen in this type of
congenital disorder.
A. Nondisjunction
B. Mosaicism
C. Deletion
D. Translocation
E. Transversion
414 A patient came to your office with a chief Contact Dermatitis is a Type IV/Delayed JOSE CARLO DIAGNOSTIC
complaint of erythema and intense pruritus on Hypersensitivity Reaction MASANGKAY III, MD EXAM - AUG
both hands after wearing Latex gloves. What (TOP 8 - FEB 2014 2014
type of Hypersensitivity reaction is this patient MED BOARDS;
having? TOPNOTCH MD)
A. Anaphylactic
B. Cytotoxic
C. Immune Complex
D. Delayed
E. None of the Above
415 An AIDS patient was diagnosed to have JOSE CARLO DIAGNOSTIC
Pneumocystis jiroveci Pneumonia, What is MASANGKAY III, MD EXAM - AUG
probably the patient's CD4 count? (TOP 8 - FEB 2014 2014
A. <500 MED BOARDS;
B. <300 TOPNOTCH MD)
C. <200
D. <100
E. <50

416 What is the most common type of congenital Endocardial Cushion defect and ASD primum type are JOSE CARLO DIAGNOSTIC
heart defect seen in a patient with Down the most common CHD in Down Syndrome patients. MASANGKAY III, MD EXAM - AUG
Syndrome? (TOP 8 - FEB 2014 2014
A. Total Anomalous Pulmonary Venous MED BOARDS;
Return TOPNOTCH MD)
B. Endocardial Cushion Defect
C. Transposition of Great Arteries
D. Atrial Septal Defect, Secundum Type
E. None of the above
417 A 50 year old patient died of Myocardial the histologic description is that of a Coagulative type JOSE CARLO DIAGNOSTIC
Infarction, upon Autopsy/Histologic studies of of necrosis, In the cardiac muscles, this is usually MASANGKAY III, MD EXAM - AUG
his cardiac muscles there was noted evident as early as 4 hours (4-12 hours) after the (TOP 8 - FEB 2014 2014
preservation of architecture of dead tissues, onset of MI. MED BOARDS;
noted as Ghost cells. If the MI occurred at 1:00 TOPNOTCH MD)
PM what is the earliest possible time of the
autopsy?
A. 1:30 PM
B. 2:00 PM
C. 3:30 PM
D. 5:00 PM
E. 6:30 PM
418 Dilated Cardiomyopathy maybe due to the Friedrich's Ataxia may cause a restrictive type of JOSE CARLO DIAGNOSTIC
following, EXCEPT: cardiomyopathy MASANGKAY III, MD EXAM - AUG
A. Pregnancy (TOP 8 - FEB 2014 2014
B. Friedrich's Ataxia MED BOARDS;
C. Chaga's Disease TOPNOTCH MD)
D. Doxorubicin
E. Coxsackie B Myocarditis

419 An autopsy of a patient who died of Libman- LSE is seen in SLE. B:NBTE, C:IE, D:RHD JOSE CARLO DIAGNOSTIC
Sacks Endocarditis will have lesions on the MASANGKAY III, MD EXAM - AUG
heart grossly described as which of the (TOP 8 - FEB 2014 2014
following vegetation descriptions: MED BOARDS;
A. Small or medium sized vegetations on both TOPNOTCH MD)
sides of the valve leaflets
B. Small bland vegetations, usually attached at
the line of closure
C. Large irregular masses on the valve cusps
that extend into the chordae
D. small warty vegetations along the lines of
closure of the valve leaflets

TOPNOTCH MEDICAL BOARD PREP PATHOLOGY SUPEREXAM Page 57 of 99


For inquiries visit www.topnotchboardprep.com.ph or email us at topnotchmedicalboardprep@gmail.com
TOPNOTCH MEDICAL BOARD PREP PATHOLOGY SUPEREXAM
For inquiries visit www.topnotchboardprep.com.ph or email us at topnotchmedicalboardprep@gmail.com
Item QUESTION EXPLANATION AUTHOR TOPNOTCH
# EXAM
E. Large, multiple, regular masses on the valve
cusps that extend into the chordae

420 A patient working for an Aircraft Industry was SIMILAR TO PREVIOUS BOARD EXAM JOSE CARLO DIAGNOSTIC
diagnosed to have Lung Carcinoma, what CONCEPT/PRINCIPLE MASANGKAY III, MD EXAM - AUG
substance may be responsible for this patient's (TOP 8 - FEB 2014 2014
morbidity? MED BOARDS;
A. Silicon TOPNOTCH MD)
B. Cadmium
C. Arsenic
D. Vinyl Chloride
E. Beryllium
421 A 52-year old female came for consult due to on Here we have the typical findings in leukemia - WEBSTER ALINDOG, MIDTERM 1
and off fever, easy bruising and easy recurrent infection/fever (defective immune cells), MD (TOP 3 - FEB EXAM - AUG
fatigability. Significant in P.E. were pallor, pallor and other signs of anemia, and 2014 MED BOARDS; 2014
wasting and presence of hepatosplenomegaly. bruising/bleeding (thrombocytopenia). Sea-blue TOPNOTCH MD)
CBC showed WBC count of >100,000/mm3 and histiocytes (clincher) are found in CML and in other
thrombocytopenia; sea-blue histiocytes were myelodysplastic syndromes (MDS). These cells have
also noted in her bone marrow. The most likely wrinkled, green blue cytoplasm hence the name.
diagnosis for this patient is:
A. Acute promyelocytic leukemia
B. Chronic myelogenous leukemia
C. Histiocytosis X
D. Non-Hodgkin lymphoma
422 Which segment of the respiratory tree is In centriacinar/centrilobular emphysema, the central WEBSTER ALINDOG, MIDTERM 1
primarily affected in centriacinar emphysema or prximal parts of the acini are affected, whereas the MD (TOP 3 - FEB EXAM - AUG
(smoking-related)? distal parts are spared. These are more common and 2014 MED BOARDS; 2014
A. Terminal bronchioles usually more severe in the upper lobes, particularly in TOPNOTCH MD)
B. Respiratory bronchioles the apical segments. In contrast, panacinar
C. Alveolar ducts emphysema involves segments from the respiratory
D. Alveolar sacs bronchiole to the terminal blind alveoli. It tends to
occur more commonly in the lower zones and most
severe at the bases.
423 Achalasia is characterized by: WEBSTER ALINDOG, MIDTERM 1
A. Increased resting tone of LES MD (TOP 3 - FEB EXAM - AUG
B. Lack of esophageal peristalsis 2014 MED BOARDS; 2014
C. Incomplete LES relaxation during TOPNOTCH MD)
deglutition
D. All of the above

424 What is the most frequent type of WEBSTER ALINDOG, MIDTERM 1


hyperfunctioning pituitary adenoma? MD (TOP 3 - FEB EXAM - AUG
A. Gonadotroph adenoma 2014 MED BOARDS; 2014
B. Somatotroph adenoma TOPNOTCH MD)
C. Lactotroph adenoma
D. Corticotroph adenoma

425 A 49-year old hypertensive male is noted to be This is a case of renal artery stenosis and occlusion by WEBSTER ALINDOG, MIDTERM 1
refractory to most oral medications save for an atheromatous plaque is the most common cause MD (TOP 3 - FEB EXAM - AUG
ACE inhibitors. Upon examination, a bruit is (70%, Robbins). This pathology is most common in 2014 MED BOARDS; 2014
heard on auscultation of his kidneys. Elevated elderly males. Although fibromuscular dysplasia can TOPNOTCH MD)
plasma renin was also determined. If we are to also cause renal artery stenosis, we find it less
biopsy the kidneys of this patient we are most appropriate in this case because it is frequent in
likely to identify: younger females. Again, the rule is to choose the best
A. Occlusion of renal artery by an answer. =)
atheromatous plaque
B. Fibromuscular dysplasia of renal artery
C. Adventitial hyperplasia of renal artery
D. Endotheliolysis and vasculogenesis
426 Nodular hyperplasia of the prostate, a common And hence the obstructive symptoms of BPH. Prostatic WEBSTER ALINDOG, MIDTERM 1
disorder of men above 50, arises most tumors, on the other hand, tend to originate from the MD (TOP 3 - FEB EXAM - AUG
frequently and almost exclusively in which part periphery. 2014 MED BOARDS; 2014
of the gland? TOPNOTCH MD)
A. Peripheral zone
B. Posterior aspect
C. Transition zone
D. Any portion of the prostate

427 This is a rare clinical syndrome with major WEBSTER ALINDOG, MIDTERM 1
features of adenomatous colonic polyposis and MD (TOP 3 - FEB EXAM - AUG
CNS tumors, including medulloblastomas and 2014 MED BOARDS; 2014
gliobastomas: TOPNOTCH MD)
A. Gardner syndrome
B. HNPCC syndrome
C. Crohn syndrome
D. Turcot syndrome

TOPNOTCH MEDICAL BOARD PREP PATHOLOGY SUPEREXAM Page 58 of 99


For inquiries visit www.topnotchboardprep.com.ph or email us at topnotchmedicalboardprep@gmail.com
TOPNOTCH MEDICAL BOARD PREP PATHOLOGY SUPEREXAM
For inquiries visit www.topnotchboardprep.com.ph or email us at topnotchmedicalboardprep@gmail.com
Item QUESTION EXPLANATION AUTHOR TOPNOTCH
# EXAM
428 A 55-year old chronic alcoholic male exhibiting This is a case of liver cirrhosis. It has 3 defining WEBSTER ALINDOG, MIDTERM 1
ascites, splenomegaly, periumbilical caput characteristics: 1. bridging fibrous septae; 2. MD (TOP 3 - FEB EXAM - AUG
medusae and skin spider angiomata was parenchymal nodules; and 3. disruption of the 2014 MED BOARDS; 2014
admitted due to anorexia and severe architecture of the entire liver. The vascular TOPNOTCH MD)
malnutrition. Hepatic pathology was reorganization and disruption of the normal structure
immediately suspected and a liver biopsy was leads to portal hypertension (intrahepatic). Finally, it
eventually done over the course. Which of the is said that once cirrhosis has developed, reversal is
following may not be true about the patient's already rare; correction of malnutrition may improve
condition? the patient's clinical presentation but will not cure
A. The biopsy will most likely show bridging definitively the liver problem.
fibrous septae and parenchymal nodules in the
liver.
B. There is reversibility upon correction of the
malnutritive state.
C. It is the predominant cause of intrahepatic
portal hypertension.
D. None of these.
E. All of these.
429 What is the most common clinically significant WEBSTER ALINDOG, MIDTERM 1
congenital anomaly of the pancreas? MD (TOP 3 - FEB EXAM - AUG
A. Annular pancreas 2014 MED BOARDS; 2014
B. Pancreatic agenesis TOPNOTCH MD)
C. Pancreas divisum
D. Ectopic pancreas

430 What consitute the membrane attack complex WEBSTER ALINDOG, MIDTERM 1
of the complement system? MD (TOP 3 - FEB EXAM - AUG
A. C1q,r,s 2014 MED BOARDS; 2014
B. C1q, C3b TOPNOTCH MD)
C. C3b, C4a, C5b
D. C5b, C6-9

431 All of the following are immune-mediated Master the 4 hypersensitivity class and their WEBSTER ALINDOG, MIDTERM 1
hypersensitivity (type III) except for: respective examples. (high yield) MD (TOP 3 - FEB EXAM - AUG
A. Type I DM 2014 MED BOARDS; 2014
B. SLE TOPNOTCH MD)
C. PSGN
D. Serum sickness

432 A 42-year old female came in for consult for This is a case of follicular thyroid carcinoma. It has WEBSTER ALINDOG, MIDTERM 1
presence of a slowly enlarging non-tender little tendency to invade the lymphatics, but vascular MD (TOP 3 - FEB EXAM - AUG
nodule in her anterior neck. Surgical removal invasion is common, with spread to bone, lungs, liver 2014 MED BOARDS; 2014
was done and studies revealed a thyroid tumor and elsewhere. Buzz words: papillary thyroid CA - TOPNOTCH MD)
with light-tan appearance containing small foci orphan Annie eye, psammoma; follicular thyroid CA -
of hemorrhage; (+) fairly uniform cells forming vascular invasion, Hurthle cell; medullary thyroid CA -
small follicles containing colloid with some amyloid deposits, C-cell hyperplasia.
Hurthle cells but no psammoma bodies. Should
it progresses, one can assume that this would
most likely metastasize thru:
A. Direct seeding
B. Lymphatic spread
C. Hematogenous dissemination
D. Metastasis not possible
433 What is the most accepted theory about the WEBSTER ALINDOG, MIDTERM 1
pathogenesis of vitiligo? MD (TOP 3 - FEB EXAM - AUG
A. Post-infectious 2014 MED BOARDS; 2014
B. Poor nutrition TOPNOTCH MD)
C. Autoimmunity
D. Chemical toxicity

434 Which of the following statements is most WEBSTER ALINDOG, MIDTERM 1


consistent with brain abscess? MD (TOP 3 - FEB EXAM - AUG
A. It is a discrete, non-encapsulated lesion 2014 MED BOARDS; 2014
with central liquefactive necrosis. TOPNOTCH MD)
B. It usually originates from adjacent
structures to the brain via the lymphatics.
C. CSF is under increased pressure, with
elevated WBC and protein levels but normal
sugar.
D. The most favored site is the cerebellum.
E. All of the above.
435 What clotting factor is lacking in Hemophilia A? Hemophilia A - factor VIII; Hemophilia B - factor WEBSTER ALINDOG, MIDTERM 1
A. Factor VII IX/Christmas factor; Hemophilia C - factor XI. All will MD (TOP 3 - FEB EXAM - AUG
B. Factor VIII have increased/prolonged PTT. 2014 MED BOARDS; 2014
C. Factor IX TOPNOTCH MD)
D. Factor XIII
E

TOPNOTCH MEDICAL BOARD PREP PATHOLOGY SUPEREXAM Page 59 of 99


For inquiries visit www.topnotchboardprep.com.ph or email us at topnotchmedicalboardprep@gmail.com
TOPNOTCH MEDICAL BOARD PREP PATHOLOGY SUPEREXAM
For inquiries visit www.topnotchboardprep.com.ph or email us at topnotchmedicalboardprep@gmail.com
Item QUESTION EXPLANATION AUTHOR TOPNOTCH
# EXAM
436 A potent vasodilator released during NO = nitric oxide/EDRF; N2O = nitrous oxide/laughing WEBSTER ALINDOG, MIDTERM 1
inflammation which reduces platelet gas, your inhalational anesthetic. MD (TOP 3 - FEB EXAM - AUG
aggregation and adhesion, inhibits several 2014 MED BOARDS; 2014
features of mast cell-induced inflammation and TOPNOTCH MD)
serves as an endogenous regulator of leukocyte
recruitment. It is released from endothelial
cells, hence also known as endothelial-derived
relaxing factor:
A. Leukotrienes
B. Bradykinin
C. Nitric oxide
D. Nitrous oxide
E. Prostaglandin
437 Tay-Sachs disease is a lysosomal storage Disease: enzyme deficient: major accumulating WEBSTER ALINDOG, MIDTERM 1
disease predominated with affectation of metabolite --- Tay-Sachs: hexosaminidase alpha: GM2 MD (TOP 3 - FEB EXAM - AUG
neurons in the brain and the retina. The ganglioside --- Gaucher: glucocerebrosidase: 2014 MED BOARDS; 2014
enzyme deficient is hexosaminidase alpha glucocerebroside --- Neimann-Pick: TOPNOTCH MD)
subunit and the major accumulating metabolite sphingomyelinase: sphingomyelin --- Hurler: alpha-L-
is: iduronidase: dermatan sulfate, heparan sulfate ---
A. Glycogen Hunter: L-iduronosulfate sulfatase: dermatan sulfate,
B. Glucocerebroside heparan sulfate
C. Dermatan sulfate
D. GM2 ganglioside
438 The carcinogenic agent that is mainly found in WEBSTER ALINDOG, MIDTERM 1
missile fuel and space vehicles: MD (TOP 3 - FEB EXAM - AUG
A. Beryllium 2014 MED BOARDS; 2014
B. Radon TOPNOTCH MD)
C. Chromium
D. Benzene

439 This virus causes a benign, self-limited WEBSTER ALINDOG, MIDTERM 1


lymphoproliferative disorder with absolute MD (TOP 3 - FEB EXAM - AUG
lymphocytosis (atypical lymphocytes); 2014 MED BOARDS; 2014
associated with hairy leukoplakia and a number TOPNOTCH MD)
of neoplasms including lymphomas and
nasopharyngeal carcinoma:
A. CMV
B. EBV
C. Adenovirus
D. HTLV-1
440 This refers to the ratio of thickness of the WEBSTER ALINDOG, MIDTERM 1
mucous gland layer in the airways to that of the MD (TOP 3 - FEB EXAM - AUG
wall between the epithelial lining and the 2014 MED BOARDS; 2014
cartilage. It is utilized to assess chronic TOPNOTCH MD)
bronchitis:
A. Reid index
B. De Ritis ratio
C. Gleason score
D. Air passage ratio

441 Hemolysis and vasoocclusive crisis are common Sickle-cell anaemia is caused by a point mutation in JULIET KRISTINE MIDTERM 2
in this form of anemia caused by mutation of the -globin chain of haemoglobin, causing the EVANGELISTA, MD EXAM - AUG
glutamic acid to valine at amino acid 6 of beta hydrophilic amino acid glutamic acid to be replaced (TOP 9 - FEB 2014 2014
chain with the hydrophobic amino acid valine at the sixth MED BOARDS;
A. Hemoglobin C position. TOPNOTCH MD)
B. Thalassemia
C. Hereditary spherocytosis
D. Sickle cell anemia
E. G6PD Deficiency
442 A 58 year-old male, hypertensive was rushed to A subarachnoid hemorrhage is a bleeding into the JULIET KRISTINE MIDTERM 2
the Emergency Room due to loss of subarachnoid space, the area between the arachnoid EVANGELISTA, MD EXAM - AUG
consciousness. His son recalled that his father membrane and the pia mater surrounding the brain. (TOP 9 - FEB 2014 2014
suddenly complained of very severe headache This may occur spontaneously, usually from a MED BOARDS;
after their breakfast followed by loss of ruptured cerebral aneurysm, or may result from head TOPNOTCH MD)
consciousness. What is the most probable injury. Symtpoms of SAH include a severe headache
diagnosis? with a rapid onset (thunderclap headache), vomiting,
A. Intraparenchymal Hemorrhage confusion or a lowered level of consciousness, and
B. Epidural Hemorrhage sometimes seizures.
C. Subarachnoid Hemorrhage
D. Subdural Hemorrhage
E. Interventricular Hemorrhage
443 Which of the following malignant neoplasms is Paraneoplastic syndromes are common in lung cancer, JULIET KRISTINE MIDTERM 2
correctly paired with the appropriate and may be the first manifestation of the disease or its EVANGELISTA, MD EXAM - AUG
paraneoplastic manifestation? recurrence. Hypercalcemia is frequently found in (TOP 9 - FEB 2014 2014
A. Squamous cell carcinoma - hypercalcemia patients with MED BOARDS;
B. Glioblastoma multiforme - hyperglycemia squamous cell carcinoma. It may arise from bone TOPNOTCH MD)
C. Adrenal cortical carcinoma - hyperuricemia metastasis but can also be induced in a paraneoplastic
D. Hepatocellular carcinoma - polycythemia manner by secretion of parathyroid hormone-related
E. Gastric carcinoma - achlorydia protein (PTHrP).

TOPNOTCH MEDICAL BOARD PREP PATHOLOGY SUPEREXAM Page 60 of 99


For inquiries visit www.topnotchboardprep.com.ph or email us at topnotchmedicalboardprep@gmail.com
TOPNOTCH MEDICAL BOARD PREP PATHOLOGY SUPEREXAM
For inquiries visit www.topnotchboardprep.com.ph or email us at topnotchmedicalboardprep@gmail.com
Item QUESTION EXPLANATION AUTHOR TOPNOTCH
# EXAM
444 The sequence of cellular events in inflammation The sequence of cellular events in inflammation is: JULIET KRISTINE MIDTERM 2
is: Margination-diapedesis-chemotaxis-phagocytosis EVANGELISTA, MD EXAM - AUG
A. Diapedesis-margination-chemotaxis- (TOP 9 - FEB 2014 2014
phagocytosis MED BOARDS;
B. Margination-diapedesis-chemotaxis- TOPNOTCH MD)
phagocytosis
C. Diapedesis-chemotaxis-margination-
phagocytosis
D. Margination-chemotaxis-diapedesis-
phagocytosis
E. Chemotaxis-margination-diapedesis-
phagocytosis
445 A male infant was born at term. No congenital X-linked hypogammaglobulinemia or Bruton's JULIET KRISTINE MIDTERM 2
anomalies were noted at birth. About 6 months agammaglobulinemia occurs due to low levels of all EVANGELISTA, MD EXAM - AUG
later, he was noted of failure to thrive and has immunoglobulins, a virtual absence of B cells due to (TOP 9 - FEB 2014 2014
been getting bacterial pneumonia with tyrosine kinase mutation. Male infants at about 6 MED BOARDS;
Hemophilus influenzae and Streptococcus months of age present with recurrent pyogenic TOPNOTCH MD)
pneumoniae cultured from his sputum. You bacterial infections caused by Strep pneumoniae and
strongly suspect that he has: Haemophilus influenzae.
A. Selective IgA deficiency
B. Severe Combined Immunodeficiency
C. EBV Infection
D. X-linked Hypommaglobulinemia
E. Di-George Syndrome
446 A pigeon raiser suddenly developed stiff neck Cryptococcus neoformans is a type of fungus that is JULIET KRISTINE MIDTERM 2
and deteriorating sensorium was rushed to the found in the soil worldwide, usually in association EVANGELISTA, MD EXAM - AUG
ER and was admitted. Work-up was done for with bird droppings. Cryptococcal meningitis is (TOP 9 - FEB 2014 2014
probable diagnosis. Lumbar puncture was done believed to result from dissemination of the fungus MED BOARDS;
and CSF was obtained and should be stained from either an observed or unappreciated pulmonary TOPNOTCH MD)
with: infection. India ink of the CSF is a traditional
A. Acid fast stain microscopic method of diagnosis.
B. Methylene blue
C. India ink stain
D. Gram Stain
E. H&E Stain
447 A 5-year old was brought in at the out-patient Kwashiorkor is caused by protein deprivation greater JULIET KRISTINE MIDTERM 2
department because of generalized edema than reduction in caloric intake. It is the most common EVANGELISTA, MD EXAM - AUG
accompanied by easily pluckable hair, form of protein-energy malnutrition which is (TOP 9 - FEB 2014 2014
listlessness and loss of appetite. The child is associated with severe loss of the visceral protein MED BOARDS;
most likely suffering from: compartment. Clinical findings include generalized or TOPNOTCH MD)
A. Anorexia dependent edema, skin lesions, hair changes and
B. Marasmus enlarged, fatty liver.
C. Kwashiorkor
D. Bulimia
E. Vitamin Deficiency
448 A 22 year-old man has lifelong hemorrhagic Hemophilia is a group of hereditary genetic disorders JULIET KRISTINE MIDTERM 2
diathesis. The PT and bleeding time are normal, that impair the body's ability to control blood clotting EVANGELISTA, MD EXAM - AUG
but the aPTT is prolonged. The most likely or coagulation. In Hemophilia, platelet count, bleeding (TOP 9 - FEB 2014 2014
cause of the bleeding disorder is: time and prothrombin time are all normal. Only the MED BOARDS;
A. Von Willebrand's Disease aPTT is prolonged such as in Hemophilia A and B. TOPNOTCH MD)
B. Hemophilia
C. Vitamin K Deficiency
D. Factor VII Deficiency
E. Bernard-Soulier Syndrome
449 The histologic hallmark of chronic bronchitis: Chronic bronchitis is defined clinically as persistent JULIET KRISTINE MIDTERM 2
A. Inflammation and fibrosis cough and sputum production for at least 3 months in EVANGELISTA, MD EXAM - AUG
B. Hypertophy of goblet cells at least 2 consecutive years. Although the number of (TOP 9 - FEB 2014 2014
C. Hyperplasia of mucosal glands goblet cells increase slightly, the major histologic MED BOARDS;
D. Marked narrowing of bronchial lumen change is in the size of the mucous gland TOPNOTCH MD)
E. Reversible bronchoconstriction (hyperplasia). This increase is assessed by the ratio
of the thickness of mucous gland layer to the thickness
of the wall between the epithelium and the cartilage,
the Reid Index.
450 A 55 year-old man was suffering from recurrent Charcot-Leyden crystals are collections of crystalloid JULIET KRISTINE MIDTERM 2
attacks of persistent cough. He was rushed to made up of eosinophil lysophospholipase binding EVANGELISTA, MD EXAM - AUG
the Emergency Room due to severe defficulty of protein called galectin-10. Curshmann spirals are (TOP 9 - FEB 2014 2014
breathing. PE revealed wheezes over lung mucus plugs containing whorls of shed epithelium. MED BOARDS;
fields. Mucous plugs were collected and These result from mucus plugging in subepithelial TOPNOTCH MD)
histologically examined revealing collections of mucus gland ducts or bronchioles which later become
cystalloid made up of eosinophil membrane extruded.
protein. These are:
A. Charcot-Leyden Crystals
B. Reinke Crystals
C. Curschmann spirals
D. Councilman Bodies
E. Psammoma Bodies
451 The most important prognostic feature in Prognostic information is important in counseling JULIET KRISTINE MIDTERM 2
invasive breast cancer is: patients about the likely outcome of their disease and EVANGELISTA, MD EXAM - AUG
A. Grade of tumor choosing appropriate treatment. Axillary lymph (TOP 9 - FEB 2014 2014
B. Histologic type of tumor node status is the most important prognostic MED BOARDS;
C. Size of tumor factor for invasive carcinoma in the absence of TOPNOTCH MD)
D. Status of sentinel lymph nodes distant metastasis. The size of an invasive
E. Metastasis carcinoma is the second most important
prognostic factor.

TOPNOTCH MEDICAL BOARD PREP PATHOLOGY SUPEREXAM Page 61 of 99


For inquiries visit www.topnotchboardprep.com.ph or email us at topnotchmedicalboardprep@gmail.com
TOPNOTCH MEDICAL BOARD PREP PATHOLOGY SUPEREXAM
For inquiries visit www.topnotchboardprep.com.ph or email us at topnotchmedicalboardprep@gmail.com
Item QUESTION EXPLANATION AUTHOR TOPNOTCH
# EXAM
452 An 8 year-old female can bend her thumb back Inherited defects in the synthesis or structure of JULIET KRISTINE MIDTERM 2
to touch her forearm. She can pull her skin out fibrillar collagen results to Ehler's Danlos syndrome. EVANGELISTA, MD EXAM - AUG
from her abdomen about 10cm and a cut to her Skin of patients with this defect is extraordinarily (TOP 9 - FEB 2014 2014
skin gapes open with difficulty in repairing. stretchable, extremely fragile and vulnerable to MED BOARDS;
Inherited defect is suspected causing the above trauma. Affected individual also has joint TOPNOTCH MD)
findings which is defect in: hypermobility, joint laxity, dislocations or easy
A. LDL receptor bruising.
B. factor VIII
C. dystrophin
D. alpha-1 antitrypsin
E. collagen
453 Honesto is a 6 year-old boy who had fever Reye's syndrome is a rare but serious condition that JULIET KRISTINE MIDTERM 2
associated with vesicular lesions in different causes swelling in the liver and brain. Reye's EVANGELISTA, MD EXAM - AUG
ages all over his body. He was given aspirin and syndrome most often affects children and teenagers (TOP 9 - FEB 2014 2014
subsequently developed fatty change of the recovering from a viral infection (Varicella and Flu) MED BOARDS;
liver. The most likely diagnosis is: who has been given Aspirin. TOPNOTCH MD)
A. Viral Hepatitis
B. Varicella-Zoster Infection
C. Reye's Syndrome
D. Subactue sclerosing panecephalitis
E. Impetigo
454 A 45 year-old male office worker has Sjogren's syndrome is is a chronic autoimmune JULIET KRISTINE MIDTERM 2
keratoconjunctivitis. He has oral mucosal disease which destroys the exocrine glands, EVANGELISTA, MD EXAM - AUG
atrophy with buccal mucosal ulceration. A specifically the salivary and lacrimal glands which (TOP 9 - FEB 2014 2014
biopsy of minor salivary glands revealed leads to the development of xerostomia and MED BOARDS;
plasma cell infiltrates and biopsy of his lips keratoconjunctivitis sicca, which takes place in TOPNOTCH MD)
revealed marked lymphocytic infiltrates. The association with lymphocytic infiltration of the glands.
antibody that is most likely found to this The antibody that is most likely found in this patient is
patient is: anti-SS-A.
A. anti-dsDNA
B. anti-centromere
C. anti-SS-A
D. anti-Scl-70
E. anti-RNP
455 Compromise of heart function because the Constrictive pericarditis is diagnosed when heart is JULIET KRISTINE MIDTERM 2
pericardium is stiff is: completely encased by a dense fibrosis that it cannot EVANGELISTA, MD EXAM - AUG
A. Hypertrophic cardiomyopathy expand normally during diastole. Fibrinous (TOP 9 - FEB 2014 2014
B. Restrictive cardiomyopathy pericarditis is seen in patients with uremia and or MED BOARDS;
C. Cardiac tamponade viral infection. The exudate imparts an irregular TOPNOTCH MD)
D. Constrictive pericarditis appearance to the pericardial surface (bread and
E. Fibrinous pericarditis butter pericarditis).

456 It refers to a special form of necrosis usually Fibrinous pericarditis is a special form of necrosis JULIET KRISTINE MIDTERM 2
seen in immune reactions involving blood usually seen in immune reactions involving blood EVANGELISTA, MD EXAM - AUG
vessels due to deposits of fibrin complexes, vessels due to deposits of fibrin complexes, together (TOP 9 - FEB 2014 2014
together with fibrin that has leaked out of with fibrin that has leaked out of vessels resulting in a MED BOARDS;
vessels resulting in a bright pink amorphous bright pink amorphous appearance in H&E stains, TOPNOTCH MD)
appearance in H&E stains: called "fibrinoid" (fibrin-like) lesions.
A. Fat necrosis
B. Fibrinous necrosis
C. Coagulative necrosis
D. Liquefactive necrosis
E. Gangrenous necrosis
457 A 42 year-old female was diagnosed with Typical thyroid adenoma is a solitary, spherical, well- JULIET KRISTINE MIDTERM 2
follicular adenoma. The characteristics of the encapsulated lesion that is well-demarcated from the EVANGELISTA, MD EXAM - AUG
diagnosis are enumerated below, except: surrounding thyroid parenchyma. Follicular (TOP 9 - FEB 2014 2014
A. Usually solitary and spherical adenomas contain multiple nodules on their cut MED BOARDS;
B. Papillary structures within the lesion surface. Papillary formation is not a typical feature of TOPNOTCH MD)
C. Fibrous encapsulation follicular adenoma.
D. Contain multiple nodules on cut surface
E. Morphology within and outside the capsule
are different
458 A 2-month old infant, first born male was Congenital hypertrophic pyloric stenosis is a condition JULIET KRISTINE MIDTERM 2
brought to the Emergency Room for persistent which causes projectile non-bilious vomiting. It most EVANGELISTA, MD EXAM - AUG
nonbilious projectile vomiting. Physical often occurs in the first 2 to 6 weeks of life. The (TOP 9 - FEB 2014 2014
examination revealed a palpable olive-shaped pyloric hypertrophy is felt classically as an olive- MED BOARDS;
mass on the abdomen. The most probable shaped mass in the middle upper part or right upper TOPNOTCH MD)
diagnosis is: quadrant of the infant's abdomen.
A. Duodenal atresia
B. Intussusception
C. Volvulus
D. Hypertrophic pyloric stenosis
E. Hirschprung's disease
459 In viral hepatitis infection, the folllowing is Bridging fibrosis is the deposition of fibrous tissue JULIET KRISTINE MIDTERM 2
considered the histologic marker of irreversible with linking of fibrous septa which is the hallmark of EVANGELISTA, MD EXAM - AUG
liver injury: chronic hepatitis. (TOP 9 - FEB 2014 2014
A. Piecemeal necrosis MED BOARDS;
B. Bridging necrosis TOPNOTCH MD)
C. Bridging fibrosis
D. Interface hepatitis
E. Ductular reaction

TOPNOTCH MEDICAL BOARD PREP PATHOLOGY SUPEREXAM Page 62 of 99


For inquiries visit www.topnotchboardprep.com.ph or email us at topnotchmedicalboardprep@gmail.com
TOPNOTCH MEDICAL BOARD PREP PATHOLOGY SUPEREXAM
For inquiries visit www.topnotchboardprep.com.ph or email us at topnotchmedicalboardprep@gmail.com
Item QUESTION EXPLANATION AUTHOR TOPNOTCH
# EXAM
460 A bone marrow aspirate was obtained in a 48 CML is a disease primary of adults between ages 25 JULIET KRISTINE MIDTERM 2
year-old male, nosmoker who complained of and 60 years. Patient presents with mild to moderate EVANGELISTA, MD EXAM - AUG
easy fatigability, weakness, weight loss and anemia, easy fatigability, weakness, weight loss, (TOP 9 - FEB 2014 2014
anorexia associated with LUQ abdominal pain. anorexia, abdominal pain due to splenomegaly which MED BOARDS;
Laboratory revealed anemia. The bone marrow may lead to splenic infarction. The bone marrow TOPNOTCH MD)
aspirate revealed numerous scattered aspirate of CMP shows numerous scattered
macrophage with abundant wrinkled green- macrophage with abundant wrinkled green-blue
blue cytoplasm called "sea-blue histiocytes". cytoplasm called "sea-blue histiocytes".
The most probable diagnosis is:
A. Chronic myelogenous leukemia
B. Multiple myeloma
C. Hodgkin's lymphoma
D. Burkitt's lymphoma
E. Adult T-cell lymphoma
461 A 7 year old boy presents with pallor and this is a case of acute lymphoblastic leukemia, the LUISA SARANILLO, BACK-UP
recurrent fever. CBC showed increased WBC most common cancer of children. It has hypercellular MD (TOP 6 - FEB MIDTERM
and decreased RBC. Bone marrow analysis marrow packed with lymphoblasts - immature T or B 2014 MED BOARDS; EXAM AUG
showed hypercellularity with predominance of cells. It is MPO-negative, PAS positive, TDT and CALLA TOPNOTCH MD) 2014 - FOR
lymphoblasts, MPO negative, PAS positive, TDT positive. It may spread to the CNS and testes. It is the INCLUSION IN
and CALLA positive. What is not true of this most responsive to chemotherapy, and the primary THE SAMPLEX
disease? drug used is asparaginase..
A. it may spread to CNS and testes
B. least responsive to chemotherapy
C. the primary drud used for this is
asparaginase
D. it is composed of immature B or T cells
E. all of the choices are correct
462 A 35 year old male complained of bone pain The primary consideration is multiple myeloma LUISA SARANILLO, BACK-UP
and recurrent fever. His Hemoglobin was low presenting with bone pain, recurrent infection, MD (TOP 6 - FEB MIDTERM
and his creatinine is moderately high. On anemia, and renal insufficiency. Lytic bone lesions and 2014 MED BOARDS; EXAM AUG
imaging, you see lytic bone lesions with punch punched-out defects are characteristics of multiple TOPNOTCH MD) 2014 - FOR
out defects. What is your primary myeloma, which are not found in waldenstroms INCLUSION IN
consideration? macroglobulinemia. MGUS is asymptomatic. CKD will THE SAMPLEX
A. chronic kidney disease not present with bone pain and lytic bone lesions.
B. multiple myeloma
C. waldenstrom's macroglobulinemia
D. monoclonal gammopathy of undetermined
significance
E. none of the choices
463 In chronic myeloproliferative disorders, one of CML has no JAK2 mutations. LUISA SARANILLO, BACK-UP
the following has no JAK2 mutation. MD (TOP 6 - FEB MIDTERM
A. Polycythemia vera 2014 MED BOARDS; EXAM AUG
B. Essential thrombocytosis TOPNOTCH MD) 2014 - FOR
C. Primary myelofibrosis INCLUSION IN
D. Chronic myelogenous leukemia THE SAMPLEX
E. No exception

464 What is the major red cell hemoglobin present In Cooley's anemia or Beta-thalassemia major, the LUISA SARANILLO, BACK-UP
in Cooley's anemia? major red cell hemoglobin is Hemoglobin F. MD (TOP 6 - FEB MIDTERM
A. Hemoglobin F Hemoglobin A is absent. Hemoglobin H is present in 2014 MED BOARDS; EXAM AUG
B. Hemoglobin A hemoglobin H disease, while hemoglobin Barts is TOPNOTCH MD) 2014 - FOR
C. Hemoglobin H present in hydrops fetalis. INCLUSION IN
D. Hemoglobin Barts THE SAMPLEX
E. A and B

465 What is the most common inherited bleeding LUISA SARANILLO, BACK-UP
disorder? MD (TOP 6 - FEB MIDTERM
A. Hemophilia A 2014 MED BOARDS; EXAM AUG
B. Hemophilia B TOPNOTCH MD) 2014 - FOR
C. Von Willebrand disease INCLUSION IN
D. Bernard-Soulier syndrome THE SAMPLEX
E. Glanzmann's thrombasthenia

466 A 65 year old male smoker had a mass in the Warthin tumor or papillary cystadenoma LUISA SARANILLO, BACK-UP
mandibular area. Biopsy of the mass revealed lymphomatosum arises almost exclusively in the MD (TOP 6 - FEB MIDTERM
an epithelial and lymphoid elements, with parotid gland, and is associated with smoking. It is 2014 MED BOARDS; EXAM AUG
cystic spaces and follicular germinal centers. composed of epithelial and lymphoid elements with TOPNOTCH MD) 2014 - FOR
What is the diagnosis? cystic spaces and follicular germinal centers. INCLUSION IN
A. Pleomorphic adenoma THE SAMPLEX
B. paraganglioma
C. mucocoele
D. warthin tumor
E. mucoepidermoid carcinoma
467 A 5 year old girl with atopic dermatitis lichenification - thick rough skin with prominent LUISA SARANILLO, BACK-UP
constantly scratching her elbow and arm markings; excoriation- linear, trauma, epidermal MD (TOP 6 - FEB MIDTERM
producing a thickened rough skin with breakage; plaque - elevated, flat lesion, larger than 2014 MED BOARDS; EXAM AUG
prominent markings. What is the skin lesion? 5mm; scale- dry, platelike, from conification; wheal - TOPNOTCH MD) 2014 - FOR
A. plaque pruritic, erythematous with dermal edema. INCLUSION IN
B. lichenification THE SAMPLEX
C. excoriation
D. scale
E. Wheal

TOPNOTCH MEDICAL BOARD PREP PATHOLOGY SUPEREXAM Page 63 of 99


For inquiries visit www.topnotchboardprep.com.ph or email us at topnotchmedicalboardprep@gmail.com
TOPNOTCH MEDICAL BOARD PREP PATHOLOGY SUPEREXAM
For inquiries visit www.topnotchboardprep.com.ph or email us at topnotchmedicalboardprep@gmail.com
Item QUESTION EXPLANATION AUTHOR TOPNOTCH
# EXAM
468 What is the most common form of cancer in prostate adenocarcinoma is the most common form of LUISA SARANILLO, BACK-UP
men? cancer in men. Lung cancer is the leading cause of MD (TOP 6 - FEB MIDTERM
A. Testicular cancer cancer mortality overall. 2014 MED BOARDS; EXAM AUG
B. Lung cancer TOPNOTCH MD) 2014 - FOR
C. Colon cancer INCLUSION IN
D. Prostate adenocarcinoma THE SAMPLEX
E. Gastric cancer

469 A 50 year old female complained of vaginal this is a case of lichen sclerosus or chronic atrophic LUISA SARANILLO, BACK-UP
dryness, stiffening, and constriction. On vulvitis. All choices describe lichen sclerosus MD (TOP 6 - FEB MIDTERM
examination, there is a white parchment like 2014 MED BOARDS; EXAM AUG
patches of vulvar skin and atrophy of labia. TOPNOTCH MD) 2014 - FOR
Which one of the following is/are true? INCLUSION IN
A. It can lead to cancer THE SAMPLEX
B. there is thinning of the epidermis
C. there is hydropic degeneration of basal cells
D. there is lymphocytic infiltrate
E. all of the choices are correct
470 Osteopetrosis is the first genetic disease treated osteopetrosis aka marble bone disease; osteogenesis LUISA SARANILLO, BACK-UP
with bone marrow transplantation. It is also imperfecta aka brittle bone disease; paget disease aka MD (TOP 6 - FEB MIDTERM
known as: osteitis deformans; osteitis fibrosa cystica is seen in 2014 MED BOARDS; EXAM AUG
A. Marble bone disease hyperparathyroidism. TOPNOTCH MD) 2014 - FOR
B. Brittle bone disease INCLUSION IN
C. Osteitis deformans THE SAMPLEX
D. Osteitis fibrosa cystica
E. None of the choices

471 A 30 year old male was riding on his this is epidural hematoma with lucid interval and LUISA SARANILLO, BACK-UP
motorcycle, suddenly he had a head on collision lenticular shaped lesion on CT scan. The source of MD (TOP 6 - FEB MIDTERM
with another vehicle. He was thrown 1meter bleed is middle meningeal artery; Subdural hematoma 2014 MED BOARDS; EXAM AUG
away from his motorcycle hitting his head on has delayed presentation with fluctuating levels of TOPNOTCH MD) 2014 - FOR
the ground. He was brought to the ER consciousness, and on CT scan it has crescent shaped INCLUSION IN
unconscious, but later regain his consciousness. lesions. The source of bleed is the tearing of bridging THE SAMPLEX
On CT scan, there was a lenticular shaped lesion veins.
most probably a bleed. What is the source of
bleed?
A. middle cerebral artery
B. brain parenchyma
C. middle meningeal artery
D. bridging veins
E. ventricles
472 A 21 year old male noticed a painless testicular The route of lymphatic spread of testicular tumor is as LUISA SARANILLO, BACK-UP
mass. If it is a testicular tumor, what lymph follows: para-aortic nodes > mediastinal nodes > MD (TOP 6 - FEB MIDTERM
node is directly involved in its lymphatic supraclavicular nodes 2014 MED BOARDS; EXAM AUG
spread? TOPNOTCH MD) 2014 - FOR
A. Para-aortic nodes INCLUSION IN
B. Inguinal nodes THE SAMPLEX
C. Obturator nodes
D. Iliac nodes
E. All of the choices
473 What is the most common malignant primary LUISA SARANILLO, BACK-UP
brain tumor in adults? MD (TOP 6 - FEB MIDTERM
A. meningioma 2014 MED BOARDS; EXAM AUG
B. Cystic cerebellar astrocytoma TOPNOTCH MD) 2014 - FOR
C. ependymoma INCLUSION IN
D. Glioblastoma multiforme THE SAMPLEX
E. Medulloblastoma

474 A 34 year old female complained of polyuria diabetes insipidus will present with polyuria and LUISA SARANILLO, BACK-UP
and thirst. You are considering a posterior thirst. There are 2 types: nephrogenic and central DI. MD (TOP 6 - FEB MIDTERM
pituitary syndrome. If this is cause by a In central DI, there is lack of ADH due to transection of 2014 MED BOARDS; EXAM AUG
transection of the pituitary stalk. What is the pituitary stalk, hypothalamic disease or posterior TOPNOTCH MD) 2014 - FOR
treatment? pituitary disease. The treatment for central DI is INCLUSION IN
A. thiazides desmopressin. Indomethacin and thiazides are used THE SAMPLEX
B. indomethacin for nephrogenic DI. Water restriction is indicated in
C. desmopressin SIADH. demeclocycline can cause nephrogenic DI.
D. water restriction
E. demeclocycline
475 A 45 year old female sought consultation to a The following are the morphologic feature of each LUISA SARANILLO, BACK-UP
surgeon due to a mass on the upper outer cancer type: invasive lobular CA - signet ring cells MD (TOP 6 - FEB MIDTERM
quadrant of her Right breast. The surgeon arranged in Indian file pattern, or dyscohesive 2014 MED BOARDS; EXAM AUG
decided to have biopsy of the mass which infiltrating tumor cells arranged in single file or in TOPNOTCH MD) 2014 - FOR
revealed a dyscohesive infiltrating tumor cells loose clusters or sheets; invasive ductal CA- has INCLUSION IN
arranged in single file or in sheets. characteristic grating sound due to small, central THE SAMPLEX
A. invasive lobular carcinoma pinpoint foci or streaks of chalky-white elastotic
B. invasive ductal carcinoma stroma; phyllodes tumor - is a lobulated tumor with
C. inflammatory breast CA cystic spaces; inflammatory breast CA - tumor emboli
D. phyllodes tumor in dermal lymphatics.
E. fibroadenoma

TOPNOTCH MEDICAL BOARD PREP PATHOLOGY SUPEREXAM Page 64 of 99


For inquiries visit www.topnotchboardprep.com.ph or email us at topnotchmedicalboardprep@gmail.com
TOPNOTCH MEDICAL BOARD PREP PATHOLOGY SUPEREXAM
For inquiries visit www.topnotchboardprep.com.ph or email us at topnotchmedicalboardprep@gmail.com
Item QUESTION EXPLANATION AUTHOR TOPNOTCH
# EXAM
476 One of the following embryologic remnant is Right umbilical vein degenerates and has no remnant. LUISA SARANILLO, BACK-UP
not correctly paired. Left umbilical vein becomes the ligamentum teres MD (TOP 6 - FEB MIDTERM
A. Umbilical arteries : medial umbilical hepatis 2014 MED BOARDS; EXAM AUG
ligaments TOPNOTCH MD) 2014 - FOR
B. Foramen ovale : fossa ovalis INCLUSION IN
C. Ductus venosus : ligamentum venosum THE SAMPLEX
D. Right umbilical vein : ligamentum teres
hepatis
E. ductus arteriosus : ligamentum arteriosum
477 A 40 year old female with anterior neck mass diffuse nontoxic goiter is characterized by diffusely LUISA SARANILLO, BACK-UP
came in for consultation. You palpated a enlarged thyroid gland without nodularity. MD (TOP 6 - FEB MIDTERM
diffusely enlarged thyroid gland without Multinodular goiter is characterized by 2014 MED BOARDS; EXAM AUG
nodularity. The rest of the physical examination multilobulated, assymetrically enlarged thyroid gland. TOPNOTCH MD) 2014 - FOR
is unremarkable. What is your primary In graves disease, there is also diffuse enlargement of INCLUSION IN
consideration? thyroid gland but ophthalmopathy is prominent. in THE SAMPLEX
A. Multinodular goiter thyroid adenoma, there is a discrete solitary mass
B. Diffuse nontoxic goiter
C. thyroid adenoma
D. Graves disease
E. none of the choices
478 Hemorrhagic or red infarct occurs in tissues LUISA SARANILLO, BACK-UP
with dual circulation such as: MD (TOP 6 - FEB MIDTERM
A. liver 2014 MED BOARDS; EXAM AUG
B. lungs TOPNOTCH MD) 2014 - FOR
C. intestines INCLUSION IN
D. All of the above THE SAMPLEX
E. A and B only

479 A 50 year old male came in for an eye LUISA SARANILLO, BACK-UP
examination due to blurring of vision. After a MD (TOP 6 - FEB MIDTERM
thorough eye examination, he was told by the 2014 MED BOARDS; EXAM AUG
ophthalmologist that he has cataract. What are TOPNOTCH MD) 2014 - FOR
the possible systemic diseases causing cataract? INCLUSION IN
A. diabetes mellitus THE SAMPLEX
B. wilson disease
C. atopic dermatitis
D. all of the above
E. A and C only
480 A 37 year old female has a salmon-colored The lesion contains suprapapillary plates with dilated, LUISA SARANILLO, BACK-UP
plaques on her elbows, knees, and scalp with tortuous blood vessels within papillae which leads to MD (TOP 6 - FEB MIDTERM
adherent silvery-white scales. Upon scratching pinpoint bleeding when scrathed called the auspitz 2014 MED BOARDS; EXAM AUG
the lesion, pinpoint bleeding was noted. Which sign. When formation of the lesion is induced by local TOPNOTCH MD) 2014 - FOR
of the following is true: trauma, it is called koebner phenomenon. INCLUSION IN
A. there is dilated, tortuous blood vessels THE SAMPLEX
within the papillae
B. this is called koebner phenomenon
C. this is called spongiform pustules
D. this is called munro microabscesses
E. bleeding is caused by acanthosis
481 A 50 year-old man presents with recurrent SIMILAR TO PREVIOUS BOARD EXAM ANGELIS ANDREA FINAL EXAM -
fever, infections, bleeding tendencies and CONCEPT/PRINCIPLE. Sea-blue histiocytes and COCOS, MD (TOP 1 - AUG 2014
pallor. On bone marrow biopsy, sea-blue massive splenomegaly are the buzzwords for CML. FEB 2014 MED
histiocytes were seen. The most likely diagnosis BOARDS; TOPNOTCH
is: MD)
A. AML
B. CML
C. CLL
D. ALL
482 Miliary tuberculosis spreads via which route? SIMILAR TO PREVIOUS BOARD EXAM ANGELIS ANDREA FINAL EXAM -
A. hematogenous CONCEPT/PRINCIPLE. COCOS, MD (TOP 1 - AUG 2014
B. lymphogenous FEB 2014 MED
C. lymphohematogenous BOARDS; TOPNOTCH
D. Contiguous MD)

483 A patient who suffered a massive stroke years Coagulative necrosis happens in the heart, spleen, ANGELIS ANDREA FINAL EXAM -
ago would be expected to have: kidney, caseous necrosis for TB in the lungs, and COCOS, MD (TOP 1 - AUG 2014
A. Liquefactive necrosis fibrinoid necrosis for blood vessels. FEB 2014 MED
B. Coagulative necrosis BOARDS; TOPNOTCH
C. Fibrinoid necrosis MD)
D. Caseous necrosis
484 The movement of a leukocyte towards the site SIMILAR TO PREVIOUS BOARD EXAM ANGELIS ANDREA FINAL EXAM -
of injury is called: CONCEPT/PRINCIPLE. There was a big confusion here COCOS, MD (TOP 1 - AUG 2014
A. chemotaxis but to be strict on the definition: Diapedesis is the FEB 2014 MED
B. diapedesis travel of the leukocyte between endothelial cells as it BOARDS; TOPNOTCH
C. migration exits the blood vessel, migration is the condition MD)
D. rolling where in the leukocyte travels through the
interstitium to the site of injury or infection while
chemotaxis is the orientation of a cell towards a
chemical stimulus.
485 The diagnosis of endometriosis can be SIMILAR TO PREVIOUS BOARD EXAM ANGELIS ANDREA FINAL EXAM -
confirmed by the presence of: CONCEPT/PRINCIPLE. COCOS, MD (TOP 1 - AUG 2014
A. Endometrial stroma FEB 2014 MED
B. Endometrial glands BOARDS; TOPNOTCH
C. Both A and B MD)
D. Either A or B

TOPNOTCH MEDICAL BOARD PREP PATHOLOGY SUPEREXAM Page 65 of 99


For inquiries visit www.topnotchboardprep.com.ph or email us at topnotchmedicalboardprep@gmail.com
TOPNOTCH MEDICAL BOARD PREP PATHOLOGY SUPEREXAM
For inquiries visit www.topnotchboardprep.com.ph or email us at topnotchmedicalboardprep@gmail.com
Item QUESTION EXPLANATION AUTHOR TOPNOTCH
# EXAM
486 A patient palpated a mass on her left breast. Malignant breast masses are usually hard, fixed and ANGELIS ANDREA FINAL EXAM -
Which characteristic would point to a possible nontender on palpation. COCOS, MD (TOP 1 - AUG 2014
malignancy? FEB 2014 MED
A. The mass is firm and doughy. BOARDS; TOPNOTCH
B. It is movable. MD)
C. It is tender on palpation.
D. There is a palpable node on the axilla.

487 A 40 pack-year smoker presents with dyspnea Centroacinar emphysema (mostly affects respiratory ANGELIS ANDREA FINAL EXAM -
on exertion. On PE he has a barrel chest and bronchioles) is found in smokers, while panacinar COCOS, MD (TOP 1 - AUG 2014
hyperresonant lungs. Based on spirometry and emphysema (respiratory bronchioes, alveolar ducts, FEB 2014 MED
chest xray, you diagnosed him to have alveoli) is more common for those with anti- BOARDS; TOPNOTCH
emphysema. Which part of the respiratory trypsin/anti-elastase deficiency. SIMILAR TO MD)
system is damaged the most? PREVIOUS BOARD EXAM CONCEPT/PRINCIPLE.
A. respiratory bronchiole
B. alveoli
C. alveolar duct
D. major bronchi
488 A term neonate is delivered via cesarean This is a classic case describing TTN. The condition is ANGELIS ANDREA FINAL EXAM -
section because of cephalopelvic disproportion. due to retained lung fluid, commonly in term infants COCOS, MD (TOP 1 - AUG 2014
The amniotic fluid is clear and the infant cried delivered by cesarean section. FEB 2014 MED
almost immediately after birth. Minutes after BOARDS; TOPNOTCH
birth however, there was noted increase in MD)
respiratory rate and grunting. A chest xray
showed fluid in the fissures and prominent
pulmonary vascular markings. Which is the
most likely diagnosis?
A. meconium aspiration
B. idiopathic hyaline membrane disease
C. pneumonia
D. transient tachypnea of the newborn
489 A 6-day-old premature infant born at 29 weeks Necrotizing enterocolitis is a life-threatening ANGELIS ANDREA FINAL EXAM -
AOG presents with gross bloody stools, condition seen mostly in premature infants. The COCOS, MD (TOP 1 - AUG 2014
abdominal distention and autonomic instability. characteristic finding on plain radiograph is FEB 2014 MED
What is the initial diagnostic step? pneumatosis intestinalis. BOARDS; TOPNOTCH
A. Stool culture MD)
B. Plain abdominal xray
C. Meckel scan
D. Barium enema
490 A patient presents with bilateral acoustic NF1 is the same with Von Recklinghausen which ANGELIS ANDREA FINAL EXAM -
schwannoma. Which is the most likely presents with cutaneous neurofibromas among COCOS, MD (TOP 1 - AUG 2014
diagnosis? others. Von Hippel Lindau also predisposes FEB 2014 MED
A. Neurofibromatosis 1 individuals to tumors and cysts (eg. BOARDS; TOPNOTCH
B. Neurofibromatosis 2 Pheochromocytoma, and other CNS tumors). SIMILAR MD)
C. Von Hippel Lindau TO PREVIOUS BOARD EXAM CONCEPT/PRINCIPLE.
D. Von Recklinghausen
491 This type of cancer predisposes to a The syndrome described is known as Lambert-Eaton ANGELIS ANDREA FINAL EXAM -
paraneoplastic syndrome wherein antibodies syndrome which presents with muscle weakness. COCOS, MD (TOP 1 - AUG 2014
against presynaptic calcium channels are Small cell lung carcinoma is notorious for causing this FEB 2014 MED
produced: syndrome. BOARDS; TOPNOTCH
A. Squamous cell lung carcinoma MD)
B. Small cell lung carcinoma
C. Lung Adenocarcinoma
D. Lymphoma of the lung
492 In aortic dissection, the blood accumulates SIMILAR TO PREVIOUS BOARD EXAM ANGELIS ANDREA FINAL EXAM -
between: CONCEPT/PRINCIPLE. COCOS, MD (TOP 1 - AUG 2014
A. Tunica intima and tunica media FEB 2014 MED
B. The layers of tunica media BOARDS; TOPNOTCH
C. Tunica media and tunica adventitia MD)
D. Any of the above
493 The basic pathology of atherosclerosis is: Endothelial dysfunction or injury is the key event in ANGELIS ANDREA FINAL EXAM -
A. Fat deposition on the tunica intima the development of atherosclerosis. SIMILAR TO COCOS, MD (TOP 1 - AUG 2014
B. Thickening and loss of elasticity of arterial PREVIOUS BOARD EXAM CONCEPT/PRINCIPLE. FEB 2014 MED
walls BOARDS; TOPNOTCH
C. Endothelial injury MD)
D. Deposition of atheromatous plaque causing
clogged arteries
494 A patient complains of chest heaviness Myoglobin rises for the first 2-3 hours (has high ANGELIS ANDREA FINAL EXAM -
unrelieved by nitrates and rest. Which among sensitivity but poor specificity), CKMB rises 4-6 hours COCOS, MD (TOP 1 - AUG 2014
the following enzymes is expected to rise first? after, Troponins rise 6-12 hours after an MI. FEB 2014 MED
A. troponins BOARDS; TOPNOTCH
B. CKMB MD)
C. myoglobin
D. LDH

495 A patient presents with compressive symptoms The keyphrase fibrous tissue pertains to Reidel's ANGELIS ANDREA FINAL EXAM -
due to a fixed, hard and painless goiter. thyroiditis. A differential would be thyroid carcinoma, COCOS, MD (TOP 1 - AUG 2014
Thyroidectomy showed massive amounts of probably anaplastic type. FEB 2014 MED
fibrous tissue. Which is the likely diagnosis? BOARDS; TOPNOTCH
A. Papillary carcinoma MD)
B. Follicular carcinoma
C. Riedel's thyroiditis
D. de Quervain's thyroiditis

TOPNOTCH MEDICAL BOARD PREP PATHOLOGY SUPEREXAM Page 66 of 99


For inquiries visit www.topnotchboardprep.com.ph or email us at topnotchmedicalboardprep@gmail.com
TOPNOTCH MEDICAL BOARD PREP PATHOLOGY SUPEREXAM
For inquiries visit www.topnotchboardprep.com.ph or email us at topnotchmedicalboardprep@gmail.com
Item QUESTION EXPLANATION AUTHOR TOPNOTCH
# EXAM
496 A very active 17-year-old boy is noted by the A classic description for osteosarcoma is the key ANGELIS ANDREA FINAL EXAM -
family to have recurrent deep pains in his thigh phrase "sunburst appearance" (in contrast to Ewing COCOS, MD (TOP 1 - AUG 2014
that awaken him from sleep. The family brings sarcoma which is "onion skin appearance"). Periosteal FEB 2014 MED
him to your office with a complaint of swelling lifting is also a clue to the diagnosis. Osteosarcoma BOARDS; TOPNOTCH
over his distal thigh which he said is probably usually occurs at the metaphysis of the distal femur or MD)
due to playing basketball a week ago. A proximal tibia.
radiograph of the leg showed sunburst
appearance. Which would likely explain the
condition of the patient?
A. osteosarcoma
B. ewing's sarcoma
C. osteomyelitis
D. bone fracture
497 A young boy presents with gait instability and SIMILAR TO PREVIOUS BOARD EXAM ANGELIS ANDREA FINAL EXAM -
diplopia. On CT scan, there was a cerebellar CONCEPT/PRINCIPLE Craniopharyngoma is a tumor COCOS, MD (TOP 1 - AUG 2014
tumor which is hyperdense, and noncalcified. from the Rathke's pouch and ependymoma arises FEB 2014 MED
Which of the following is the most likely from the lining of the ventricles. BOARDS; TOPNOTCH
diagnosis? MD)
A. cerebellar glioma
B. craniopharyngoma
C. ependymoma
D. medulloblastoma
498 An ICU patient has "muddy brown" casts on AGN presents with RBC casts, nephrotic syndrome ANGELIS ANDREA FINAL EXAM -
urinalysis. This finding is pathognomonic for: with Fatty casts and Acute pyelonephritis with WBC COCOS, MD (TOP 1 - AUG 2014
A. Acute tubular necrosis casts. FEB 2014 MED
B. Nephrotic syndrome BOARDS; TOPNOTCH
C. Acute pyelonephritis MD)
D. Acute glomerulonephritis
499 Which is FALSE regarding autosomal dominant Congenital hepatic fibrosis is associated with ANGELIS ANDREA FINAL EXAM -
polycystic kidney disease (ADPKD)? autosomal recessive polycystic kidney disease, COCOS, MD (TOP 1 - AUG 2014
A. It is due to mutations in PKD1 or PKD2. previously infantile polycystic kidney disease. FEB 2014 MED
B. It is associated with congenital hepatic BOARDS; TOPNOTCH
fibrosis. MD)
C. It has been noted in patients with mitral
valve prolapse and benign hepatic cysts.
D. It presents with multiple, large bilateral
cysts.
500 A patient presents with ptosis and weakness Myasthenia gravis is under type 2 ANGELIS ANDREA FINAL EXAM -
more notable during the end of the day. (cytotoxic/antibody-mediated) hypersensitivity. The COCOS, MD (TOP 1 - AUG 2014
Tensilon test is positive. Which type of body produces antibodies against acetylcholine FEB 2014 MED
hypersensitivity does this patient exhibit? receptors on NMJ. BOARDS; TOPNOTCH
A. Type 1 MD)
B. Type 2
C. Type 3
D. Type 4
501 Which of the following is made up of Type 2 JAN CHARMAINE BACK-UP
collagen?? PALOMAR, MD (TOP MIDTERM
A. Cornea and lens 9 - FEB 2014 MED EXAM AUG
B. Vitreous humor and nucleus pulposus BOARDS; TOPNOTCH 2014
C. Basal lamina and bone MD)
D. Late wound repair
E. None of the above

502 Burton's lines is found in which poisoning? Burton's lines are lines on the gingiva and on the JAN CHARMAINE BACK-UP
A. Mercury epiphysis of long bones in lead poisoning. PALOMAR, MD (TOP MIDTERM
B. Copper 9 - FEB 2014 MED EXAM AUG
C. Lead BOARDS; TOPNOTCH 2014
D. Arsenic MD)
E. None of the above

503 The following are part of the Duke's Major Robbins 7th ed., 598 JAN CHARMAINE BACK-UP
Criteria for Infective endocarditis, except: PALOMAR, MD (TOP MIDTERM
A. Positive blood culture indicating 9 - FEB 2014 MED EXAM AUG
charcteristic organism BOARDS; TOPNOTCH 2014
B. New valvular regurgitation MD)
C. Echocardiographic finding of valve-related
mass or abscess
D. All of the above
E. None of the above
504 These are small erythematous or hemorrhagic, SIMILAR TO PREVIOUS BOARD EXAM JAN CHARMAINE BACK-UP
macular, nontender lesions on the palms and CONCEPT/PRINCIPLE:Osler nodes- are small, tender PALOMAR, MD (TOP MIDTERM
soles and are a consequence of septic embolic subcutaneous nodules that develop in the pulp of the 9 - FEB 2014 MED EXAM AUG
events: digitd or occassionally more proximally in the fingers BOARDS; TOPNOTCH 2014
A. Janeway lesions and persist for hours to several days; Roth spots-are MD)
B. Osler nodes oval retinal hemorrhages with pale centers Robbins
C. Roth spots 7th ed., 598
D. All of the above
E. None of the above

TOPNOTCH MEDICAL BOARD PREP PATHOLOGY SUPEREXAM Page 67 of 99


For inquiries visit www.topnotchboardprep.com.ph or email us at topnotchmedicalboardprep@gmail.com
TOPNOTCH MEDICAL BOARD PREP PATHOLOGY SUPEREXAM
For inquiries visit www.topnotchboardprep.com.ph or email us at topnotchmedicalboardprep@gmail.com
Item QUESTION EXPLANATION AUTHOR TOPNOTCH
# EXAM
505 The following features can be seen in rice bodies are aggregation of organizing fibrin JAN CHARMAINE BACK-UP
rheumatoid arthritis except: covering portions of the synovium and floating in the PALOMAR, MD (TOP MIDTERM
A. Rice bodies joint space; pannus is a mass of synovium and 9 - FEB 2014 MED EXAM AUG
B. Pannus formation synovial stromaconsisting of inflammatory cells, BOARDS; TOPNOTCH 2014
C. Heberden nodes granulation tissue, and fibroblasts, which grows over MD)
D. All of the above the articular cartilage and causes its erosion. Both rice
E. None of the above bodies and pannus is found in RA. Heberden nodes in
the fingers of patients with osteoarthritis represent
prominent osteophytes at the distal interphalangeal
joints. 1305
506 Which of the following is true regarding Sickle SIMILAR TO PREVIOUS BOARD EXAM JAN CHARMAINE BACK-UP
cell disease? CONCEPT/PRINCIPLE:Robbins 7th ed., 628 PALOMAR, MD (TOP MIDTERM
A. It is caused by a point mutation at the 6th 9 - FEB 2014 MED EXAM AUG
position of the Beta-globin chain leading to the BOARDS; TOPNOTCH 2014
substitution of a valine residue for a glutamic MD)
acid residue.
B. It is caused by a point mutation at the 9th
position of the Beta-globin chain leading to the
substitution of a valine residue for a glutamic
acid residue.
C. It is caused by a point mutation at the 6th
position of the Beta-globin chain leading to the
substitution of a glutamic acid residue for a
valine residue.
D. It is caused by a point mutation at the 9th
position of the Beta-globin chain leading to the
substitution of a glutamic acid residue for a
valine residue.
E. None of the above
507 This is the most common leukemia of adults SIMILAR TO PREVIOUS BOARD EXAM JAN CHARMAINE BACK-UP
and the elderly. Peripheral blood smear shows CONCEPT/PRINCIPLE PALOMAR, MD (TOP MIDTERM
smudge cells and nucleated red blood cells. 9 - FEB 2014 MED EXAM AUG
What is the condition? BOARDS; TOPNOTCH 2014
A. ALL MD)
B. CLL
C. AML
D. CML
E. None of the above
508 Which type of Hodkin's lymphoma has the best JAN CHARMAINE BACK-UP
prognosis? PALOMAR, MD (TOP MIDTERM
A. Nodular sclerosis 9 - FEB 2014 MED EXAM AUG
B. Mixed cellularity BOARDS; TOPNOTCH 2014
C. Lyphocyte-predominant MD)
D. Lymphocyte-rich
E. Lymphocyte-depleted

509 The following characteristics refer to Crohn Noncaseating granulomas-Crohn; No granuloma in JAN CHARMAINE BACK-UP
Disease but not to Ulcerative colitis: Ulcerative colitis PALOMAR, MD (TOP MIDTERM
A. Skip lesions or sharp demarcation of 9 - FEB 2014 MED EXAM AUG
diseased bowel segment from adjacent BOARDS; TOPNOTCH 2014
uninvolved bowel MD)
B. Fistula or sinus tract formation
C. Absence of granulomas
D. Transmural inflammation
E. All of the above statements refer to Crohn
Disease.
510 The following statements are true regarding Robbins 7th ed., 865-66 JAN CHARMAINE BACK-UP
colorectal cancer, except: PALOMAR, MD (TOP MIDTERM
A. The most common site of colorectal cancer 9 - FEB 2014 MED EXAM AUG
is in the rectosigmoid area. BOARDS; TOPNOTCH 2014
B. Carcinomas in the distal colon tend to be MD)
annular, encircling lesions that produce so-
called napkin-ring constrictions of the bowel.
C. Left-sided colonic cancers usually presnt
with occult bleeding, changes in bowel habit, or
crampy left lower quadrant discomfort.
D. Cecal or right-sided colonic cancers usually
present with fatigue, weakness and iron-
deficiency from bleeding.
E. None of the above
511 Which liver disease is characterized JAN CHARMAINE BACK-UP
histologically by hepatocyte swelling and PALOMAR, MD (TOP MIDTERM
necrosis, mallory bodies, neutrophillic reaction 9 - FEB 2014 MED EXAM AUG
and fibrosis? BOARDS; TOPNOTCH 2014
A. Fatty liver MD)
B. Alcoholic hepatitis
C. Alcoholic cirrhosis
D. All of the above
E. None of the above

TOPNOTCH MEDICAL BOARD PREP PATHOLOGY SUPEREXAM Page 68 of 99


For inquiries visit www.topnotchboardprep.com.ph or email us at topnotchmedicalboardprep@gmail.com
TOPNOTCH MEDICAL BOARD PREP PATHOLOGY SUPEREXAM
For inquiries visit www.topnotchboardprep.com.ph or email us at topnotchmedicalboardprep@gmail.com
Item QUESTION EXPLANATION AUTHOR TOPNOTCH
# EXAM
512 Hemosiderin would most likely deposit in in decreasing order of severity:liver, pancreas, JAN CHARMAINE BACK-UP
which organ in patients with hereditary myocardium, pituitary gland, adrenal gland, thyroid PALOMAR, MD (TOP MIDTERM
hemochromatosis? and parathyroid glands, joints and skin (detected by 9 - FEB 2014 MED EXAM AUG
A. Pituitary gland Prussian blue histologic reaction) p909 BOARDS; TOPNOTCH 2014
B. Myocardium MD)
C. Liver
D. Myocardium
E. Thyroid gland

513 Kayser-Fleischer ring of Wilson's disease is the JAN CHARMAINE BACK-UP


green to brown deposits of copper in which PALOMAR, MD (TOP MIDTERM
layer of the corneal limbus? 9 - FEB 2014 MED EXAM AUG
A. Corneal epithelium BOARDS; TOPNOTCH 2014
B. Bowman's layer MD)
C. Corneal stroma
D. Descemet's membrane
E. Corneal endothelium

514 Which of the following conditions refer to A. Obstruction of a single main hepatic vein by JAN CHARMAINE BACK-UP
Budd-chiari syndrome? thrombosis is clinically silent. C. Obliteration of PALOMAR, MD (TOP MIDTERM
A. Obstruction of a single main hepatic vein by hepatic vein radicles by varying amounts of 9 - FEB 2014 MED EXAM AUG
thrombosis. subendothelial swelling and fine reticulated collagen - BOARDS; TOPNOTCH 2014
B. Obstruction of 2 or more major hepatic veno-occlusive disease.D. Sinusoidal dilation with MD)
veins produces liver enargement, pain and impediment of hepatic blood efflux - Peliosis hepatica.
ascites.
C. Obliteration of hepatic vein radicles by
varying amounts of subendothelial swelling and
fine reticulated collagen.
D. Sinusoidal dilation with impediment of
hepatic blood efflux.
E. None of the above
515 These are membrane proteins that recognize a SIMILAR TO PREVIOUS BOARD EXAM JAN CHARMAINE BACK-UP
variety of microbe-derived molecules and CONCEPT/PRINCIPLE:Robbins 7th ed., 195 PALOMAR, MD (TOP MIDTERM
stimulate innate immune responses againsts 9 - FEB 2014 MED EXAM AUG
the microbes: BOARDS; TOPNOTCH 2014
A. Toll-like receptors MD)
B. Cancer antigen
C. Hemagglutinin
D. Lipopolysaccharide
E. Endotoxin
516 These mediators effect increase in vascular SIMILAR TO PREVIOUS BOARD EXAM JAN CHARMAINE BACK-UP
permeability and neutrophil recruitment to site CONCEPT/PRINCIPLE: p.208 PALOMAR, MD (TOP MIDTERM
of injury: 9 - FEB 2014 MED EXAM AUG
A. Opsonins BOARDS; TOPNOTCH 2014
B. Coagulation factors MD)
C. Complement molecules
D. Leukotrienes
E. None of the above

517 Which of the following karyotypes is associated SIMILAR TO PREVIOUS BOARD EXAM JAN CHARMAINE BACK-UP
with the classic pattern of Klinefelter CONCEPT/PRINCIPLE: Klinefelter syndrome is best PALOMAR, MD (TOP MIDTERM
syndrome? defined as male hypogonadism that occurs when there 9 - FEB 2014 MED EXAM AUG
A. 45, X are 2 or more X chromosomes and 1 or more Y BOARDS; TOPNOTCH 2014
B. 46, XY chromosomes. P.179 MD)
C. 47, XXY
D. 47, XYY
E. None of the above

518 This disease is characterized by a distinctive SIMILAR TO PREVIOUS BOARD EXAM JAN CHARMAINE BACK-UP
heliotrope discoloration of the upper eyelids CONCEPT/PRINCIPLE PALOMAR, MD (TOP MIDTERM
with periorbital edema that may accompany or 9 - FEB 2014 MED EXAM AUG
precede the onset of muscle disease? BOARDS; TOPNOTCH 2014
A. Inclusion body myositis MD)
B. Dermatomyositis
C. Myotonic Dystrophy
D. Myasthenia gravis
E. None of the above
519 These are benign tumors of adults, usually SIMILAR TO PREVIOUS BOARD EXAM JAN CHARMAINE BACK-UP
attached to the dura, that arise from the CONCEPT/PRINCIPLE PALOMAR, MD (TOP MIDTERM
meningothelial cell of the arachnoid. 9 - FEB 2014 MED EXAM AUG
A. Primary CNS lymphoma BOARDS; TOPNOTCH 2014
B. Meningioma MD)
C. Medulloblastoma
D. Ganglion cell tumor
E. None of the above

520 In Tetralogy of Fallot, the heart is often SIMILAR TO PREVIOUS BOARD EXAM JAN CHARMAINE BACK-UP
enlarged and may be boot-shaped owing to CONCEPT/PRINCIPLE: p. 569 PALOMAR, MD (TOP MIDTERM
what? 9 - FEB 2014 MED EXAM AUG
A. Marked right ventricular hypertrophy, BOARDS; TOPNOTCH 2014
particularly of the apical region MD)
B. Marked right ventricular hypertrophy,
particularly of the base of the heart
C. Marked left ventricular hypertrophy,
particularly of the apical region
D. Marked left ventricular hypertrophy,

TOPNOTCH MEDICAL BOARD PREP PATHOLOGY SUPEREXAM Page 69 of 99


For inquiries visit www.topnotchboardprep.com.ph or email us at topnotchmedicalboardprep@gmail.com
TOPNOTCH MEDICAL BOARD PREP PATHOLOGY SUPEREXAM
For inquiries visit www.topnotchboardprep.com.ph or email us at topnotchmedicalboardprep@gmail.com
Item QUESTION EXPLANATION AUTHOR TOPNOTCH
# EXAM
particularly of the base of the heart
E. None of the above

521 An otherwise healthy 16 year old girl comes to MIGUEL RAFAEL MIDTERM 1
the physician because of a 4 year history of RAMOS, MD (TOP 3 - EXAM - FEB
heavy bleeding with menses. She has a history FEB 2012 MED 2013
of excessive bleeding after a dental extraction BOARDS; TOPNOTCH
but has never had spontaneous bleeding. Her MD)
father has a history of frequent nosebleeds and
post-operative bleeding. Her mother, sisters,
and brother have no history of bleeding
disorders. Examination of the patient shows no
abnormalities except for pallor. Laboratory
studies show Hb 8, Hct 25%, Reticulocyte count
2%, platelet count 200000, bleeding time 12
min, INR 1, APTT 60 seconds. Pelvic
ultrasonography shows no abnormalities.
Which of the following is the most likely
mechanism of this patients excessive bleeding?
A) Abnormal structure of von Willebrand factor
B) Capillary fragility
C) Inadequate production of factor VIII
D) Autoimmune platelet destruction
522 A 67 year old man comes to the physician for a MIGUEL RAFAEL MIDTERM 1
follow-up examination. Three years ago, he RAMOS, MD (TOP 3 - EXAM - FEB
underwent radical dissection of a T3 N0 M0 FEB 2012 MED 2013
epidermoid carcinoma of the floow of his BOARDS; TOPNOTCH
mouth and supraomohyoid dissection of his MD)
neck. He currently takes no medications. He is a
120 pack year smoker but stopped 3 years ago.
Vital signs are within normal limits.
Examination shows well-healed surgical scars.
There are no signs of local recurrence. An X-ray
of the chest shows a 3 cm mass in the medial
upper lobe of the right lung. Which of the
following is the most likely cause of these
findings?
A) Bronchioalveolar carcinoma
B) Metastatic carcinoma
C) Primary squamos cell carcinoma
D) Mesothelioma
523 A previously healthy 57 year old man comes to MIGUEL RAFAEL MIDTERM 1
the physician because of impotence for 1 year. RAMOS, MD (TOP 3 - EXAM - FEB
Examination shows bronze-colored skin. His FEB 2012 MED 2013
serum ferritin concentration is 4050 ng/mL. BOARDS; TOPNOTCH
This patient is at increased risk for which of the MD)
following complications?
A) Hepatocellular carcinoma
B) Interstitial lung disease
C) Progressive pancytopenia
D) Renal failure
524 A 42-year-old computer science professor is MIGUEL RAFAEL MIDTERM 1
brought to the physician by her husband, who RAMOS, MD (TOP 3 - EXAM - FEB
reports insidious changes in his wife's FEB 2012 MED 2013
personality and behavior. He reports that she BOARDS; TOPNOTCH
believes that aliens have been speaking to her MD)
and tampering with their heating and air-
conditioning systems. He says that she was
upset when she turned 40 years old, and her
symptoms have developed since that time. She
was adopted, and her family history is
unknown. Physical examination shows
vermicular movements of the tongue and
bilateral writhing motions of the upper
extremities. Mental status examination shows
indifference to her condition and mild to
moderate difficulty with memory and
calculations. What is the mechanism of the
most likely diagnosis?
A) Substantia nigra degeneration
B) Build up of tau proteins in neurons
C) CAG repeats
D) Copper accumulation in tissue

TOPNOTCH MEDICAL BOARD PREP PATHOLOGY SUPEREXAM Page 70 of 99


For inquiries visit www.topnotchboardprep.com.ph or email us at topnotchmedicalboardprep@gmail.com
TOPNOTCH MEDICAL BOARD PREP PATHOLOGY SUPEREXAM
For inquiries visit www.topnotchboardprep.com.ph or email us at topnotchmedicalboardprep@gmail.com
Item QUESTION EXPLANATION AUTHOR TOPNOTCH
# EXAM
525 A 14-year-old boy is brought to the physician MIGUEL RAFAEL MIDTERM 1
by his parents because of a 2-year history of RAMOS, MD (TOP 3 - EXAM - FEB
increasing academic problems. His parents say FEB 2012 MED 2013
that he has always been hyperactive and BOARDS; TOPNOTCH
distractible, but now his academic performance MD)
has deteriorated to the point that he is failing
ninth grade. His teachers say that his
hyperactivity is disrupting the classroom. He
weighs 54 kg (120 lb) and is 152 cm (60 in) tall.
Sexual development is Tanner stage 5;
examination shows macro-orchidism, which
was not shown on previous examinations. He
has a high forehead and long, protruding ears.
He exhibits poor eye contact during the
examination. Psychoeducational testing shows
an IQ of 70. Which of the following is the most
likely diagnosis?
A) Lesch-Nyhan syndromE
B) Prader-Willi syndrome
C) Fragile X syndrome
D) Klinefelter's syndrome
526 A 19-year-old man comes to the physician MIGUEL RAFAEL MIDTERM 1
because of frequent nosebleeds over the past 3 RAMOS, MD (TOP 3 - EXAM - FEB
weeks. He has bipolar disorder currently well FEB 2012 MED 2013
controlled with lithium carbonate, bupropion, BOARDS; TOPNOTCH
and valproic acid. Physical examination shows MD)
no abnormalities except for dried blood in the
nares. Mental status examination shows an
anxious mood and slight motor restlessness.
Serum studies show a lithium carbonate level of
1.3 mEq/L (therapeutic range=0.61.2), and
valproic acid level of 77 g/mL (therapeutic
range=40100). Which of the following is the
most appropriate next step in management?
A) Measurement of serum aspartate
aminotransferase (AST, GOT) activity
B) Measurement of serum bupropion level
C) Platelet count
D) Discontinuation of lithium carbonate
therapy
527 An asymptomatic 32-year-old man comes for a buzz phrase >> anaphylactic reaction to blood MIGUEL RAFAEL MIDTERM 1
routine health maintenance examination. He transfusion RAMOS, MD (TOP 3 - EXAM - FEB
has a 10-year history of frequent sinus and FEB 2012 MED 2013
pulmonary infections. He had an anaphylactic BOARDS; TOPNOTCH
reaction to a blood transfusion following a MD)
motor vehicle collision 3 years ago. His
temperature is 37 C (98.6 F). Examination
shows mild erythema in the posterior pharynx.
The lungs are clear to auscultation. A complete
blood count and serum protein electrophoresis
are within normal limits. Which of the following
is the most likely cause of the frequent
infections?
A) Colonization with Streptococcus
pneumoniae
B) Common variable immunodeficiency
C) HIV infection
D) Selective IgA deficiency
528 A 67-year-old man with long-standing signs MIGUEL RAFAEL MIDTERM 1
and symptoms of congestive heart failure is RAMOS, MD (TOP 3 - EXAM - FEB
admitted to the hospital because of progressive FEB 2012 MED 2013
shortness of breath. Examination shows no BOARDS; TOPNOTCH
other abnormalities. An x-ray film of the chest MD)
shows cardiomegaly, cephalization of blood
vessels, and a right-sided pleural effusion.
Which of the following sets of pleural fluid
findings is most likely in this patient?
Leukocyte Segmented
Protein Glucose count neutrophils
Monocytes
(g/dL) (mg/dL) (/mm3) (%) (%)

A) 2.5 10 10,000 50 50
B) 2.5 90 2000 60 40
C) 3.8 40 30,000 80 20
D) 4.5 60 10,000 20 80

TOPNOTCH MEDICAL BOARD PREP PATHOLOGY SUPEREXAM Page 71 of 99


For inquiries visit www.topnotchboardprep.com.ph or email us at topnotchmedicalboardprep@gmail.com
TOPNOTCH MEDICAL BOARD PREP PATHOLOGY SUPEREXAM
For inquiries visit www.topnotchboardprep.com.ph or email us at topnotchmedicalboardprep@gmail.com
Item QUESTION EXPLANATION AUTHOR TOPNOTCH
# EXAM
529 A 6-year-old boy is brought to the physician by adrenoleukodystrophy MIGUEL RAFAEL MIDTERM 1
his mother because of progressive visual loss RAMOS, MD (TOP 3 - EXAM - FEB
over the past year. Over the past 2 years, he has FEB 2012 MED 2013
had deterioration of his hearing, speech, BOARDS; TOPNOTCH
writing, and intellectual performance. His MD)
maternal uncle had similar symptoms. Visual
acuity is 20/200 bilaterally. Funduscopic
examination shows optic atrophy. His hearing
is markedly impaired. There is weakness and
spasticity of all extremities. Deep tendon
reflexes are extremely hyperactive. Babinski's
sign is present bilaterally. On mental status
examination, he is not oriented to place, year,
month, or the names of his siblings. An MRI of
the brain shows marked symmetric white
matter disease involving all lobes. Diagnostic
studies are most likely to show which of the
following?
A) Abnormally decreased serum cholesterol
level
B) Acanthocytes on blood smear
C) An excess of very long chain fatty acids
D) Normal nerve conduction studies
530 A 5-year-old boy is brought to the emergency hyaline casts can be seen in setting of dehydration MIGUEL RAFAEL MIDTERM 1
department 30 minutes after he fainted at RAMOS, MD (TOP 3 - EXAM - FEB
home after standing up from a sitting position. FEB 2012 MED 2013
His symptoms began 3 days ago with diarrhea BOARDS; TOPNOTCH
and vomiting. He has had no urine output for 18 MD)
hours. He is alert but quiet. His temperature is
37.5 C (99.5 F), blood pressure is 75/45 mm Hg,
pulse is 120/min, and respirations are 28/min.
Examination shows dry lips and tenting of the
skin. There is no abdominal tenderness. Bowel
sounds are hyperactive. The remainder of the
examination shows no abnormalities. His
capillary refill time is 5 seconds. Intravenous
bolus doses of 0.9% saline are administered.
Bladder catheterization yields 5 mL of urine.
Urinalysis is most likely to show which of the
following?
A) Oxalate crystals
B) Erythrocyte casts
C) Hyaline casts
D) Leukocyte casts
531 A county health officer investigates an outbreak MIGUEL RAFAEL MIDTERM 1
of illness among persons attending a church RAMOS, MD (TOP 3 - EXAM - FEB
picnic. The illness is characterized by the onset FEB 2012 MED 2013
of nausea and vomiting 3 to 4 hours after BOARDS; TOPNOTCH
attending the picnic. All affected persons MD)
recover without specific therapy. The
investigation implicates egg salad as the vehicle
of transmission. This episode is consistent with
a foodborne outbreak caused by which of the
following?

A) Clostridium perfringens
B) Giardia lamblia
C) Salmonella species
D) Staphylococcus aureus
532 A 4-year-old girl is brought to the physician hereditary spherocytosis MIGUEL RAFAEL MIDTERM 1
because of pallor and jaundice for 2 days. She RAMOS, MD (TOP 3 - EXAM - FEB
had previously been well, although she was FEB 2012 MED 2013
treated for jaundice with phototherapy for 2 BOARDS; TOPNOTCH
weeks while a newborn. Her mother and two MD)
additional maternal relatives underwent
splenectomy during childhood for unknown
reasons. Examination of the patient shows
jaundice. The spleen tip is palpated 4 cm below
the left costal margin. Which of the following
blood smear findings is most likely to explain
this family's condition ?
A) Elliptocytes
B) Howell-Jolly bodies
C) Schistocytes
D) Spherocytes

TOPNOTCH MEDICAL BOARD PREP PATHOLOGY SUPEREXAM Page 72 of 99


For inquiries visit www.topnotchboardprep.com.ph or email us at topnotchmedicalboardprep@gmail.com
TOPNOTCH MEDICAL BOARD PREP PATHOLOGY SUPEREXAM
For inquiries visit www.topnotchboardprep.com.ph or email us at topnotchmedicalboardprep@gmail.com
Item QUESTION EXPLANATION AUTHOR TOPNOTCH
# EXAM
533 A 57-year-old man comes for a routine follow- MIGUEL RAFAEL MIDTERM 1
up examination. He has a 10-year history of an RAMOS, MD (TOP 3 - EXAM - FEB
intermittent facial rash. He has been taking FEB 2012 MED 2013
propranolol for 2 months for hypertension. BOARDS; TOPNOTCH
Examination shows several erythematous MD)
pustules and papules involving the nose and
central face. There are telangiectasias at the
base of the papules. Which of the following is
the most likely explanation for these findings?
A) Acne rosacea
B) Acne vulgaris
C) Basal cell carcinoma
D) Discoid lupus erythematosus
534 A 27-year-old woman comes to the physician MIGUEL RAFAEL MIDTERM 1
because of a 3-week history of fever, night RAMOS, MD (TOP 3 - EXAM - FEB
sweats, rash on both legs, nonproductive cough, FEB 2012 MED 2013
and pain and swelling in her wrists and knees. BOARDS; TOPNOTCH
She has not had weight loss. Her temperature is MD)
37.7 C (99.8 F), blood pressure is 110/70 mm
Hg, pulse is 96/min, and respirations are
14/min. The lungs are clear to auscultation.
Cardiac examination shows no abnormalities.
There is swelling and warmth over the wrists
and knees bilaterally and tender red nodules on
the anterior surface of both lower extremities.
An x-ray film of the chest shows bilateral hilar
fullness. Which of the following is the most
likely diagnosis?
A) Carcinoma of the lung
B) Histoplasmosis
C) Tuberculosis
D) Sarcoidosis
535 A 49-year-old woman is admitted to the MIGUEL RAFAEL MIDTERM 1
hospital because of renal failure. She has had RAMOS, MD (TOP 3 - EXAM - FEB
episodes of flank pain over the past 20 years. FEB 2012 MED 2013
She has also had nocturia 2 to 3 times nightly BOARDS; TOPNOTCH
for 10 years. Her blood pressure is 160/100 MD)
mm Hg. Examination shows pale mucous
membranes. A mass is palpated in the right
flank. Which of the following is the
most likely diagnosis?
A) Horseshoe kidney
B) Nephrolithiasis
C) Papillary necrosis
D) Polycystic kidney disease
536 A 5-year-old female presents with a new onset PSGN >> deposits between basement membrane and MIGUEL RAFAEL MIDTERM 1
hematuria and oliguria after 1 week of epithelial cells of the glomeruli RAMOS, MD (TOP 3 - EXAM - FEB
experiencing sore throat. Other pertinent FEB 2012 MED 2013
findings revealed hypertension, periorbital BOARDS; TOPNOTCH
edema and impaired renal function. A renal MD)
biopsy most likely would reveal electron-dense
deposits in which of the following sites?
A) Between basement membrane and
endothelial cells of the glomeruli
B) Between the basement membrane and
epithelial cells of the glomeruli
C) Between the basement membrane and
epithelial cells of the proximal tubules
D) Within the mesangium of the glomeruli
537 A 32-year-old female was noted to have a Sinus histiocytosis (also called reticular hyperplasia MIGUEL RAFAEL MIDTERM 1
breast mass with axillary lymph node refers to the distention and prominence of the RAMOS, MD (TOP 3 - EXAM - FEB
enlargement. Further work ups, revealed that lymphatic sinusoids. This particular form of FEB 2012 MED 2013
the mass was cancerous. A diagnosis of invasive hyperplasia is prominent in lymph nodes draining BOARDS; TOPNOTCH
ductal carcinoma was made. Modified radical cancers such as carcinoma of the breast. (SIMILAR TO MD)
mastectomy was done with axillary lymph node PREVIOUS BOARD EXAM CONCEPT/PRINCIPLE)
dissection. Pathological examination of the
axillary lymph nodes will show?
A) Follicular Hyperplasia
B) Marginal zone B-cell hyperplasia
C) Sinus histiocytosis
D) Paracortical lymphoid hyperplasia
538 A 5-year-old male developed a demyelinating Guillan Barre Syndrome >> type IV reaction MIGUEL RAFAEL MIDTERM 1
neuropathy associated with Campylobacter RAMOS, MD (TOP 3 - EXAM - FEB
jejuni. He presents clinically with ascending FEB 2012 MED 2013
neuromuscular paralysis and areflexia. He was BOARDS; TOPNOTCH
then treated with plasmapharesis, IV MD)
immunoglobulins and corticosteroids. The
diagnosis to this case belongs to what type of
hypersensitivity?
A) Type I Hypersensitivity
B) Type II Cytotoxic Hypersensitivity
C) Type III Immune Complex Hypersensitivity
D) Type IV Cell mediated Hypersensitivity

TOPNOTCH MEDICAL BOARD PREP PATHOLOGY SUPEREXAM Page 73 of 99


For inquiries visit www.topnotchboardprep.com.ph or email us at topnotchmedicalboardprep@gmail.com
TOPNOTCH MEDICAL BOARD PREP PATHOLOGY SUPEREXAM
For inquiries visit www.topnotchboardprep.com.ph or email us at topnotchmedicalboardprep@gmail.com
Item QUESTION EXPLANATION AUTHOR TOPNOTCH
# EXAM
539 A presence of necrosis with nuclear changes Karyolysis>> basophilia of the chromatin fades or MIGUEL RAFAEL MIDTERM 1
involving basophilia of the chromatin which dissolves RAMOS, MD (TOP 3 - EXAM - FEB
later on fades or dissolves is called? FEB 2012 MED 2013
A) Karyolysis BOARDS; TOPNOTCH
B) Pyknosis MD)
C) Karyorrhexis
D) Histiocytosis

540 A 32 year old male was admitted due to fatigue, red-staining peroxidase-positive structure >> auer MIGUEL RAFAEL MIDTERM 1
unexplained fever and spontaneous mucosal rods RAMOS, MD (TOP 3 - EXAM - FEB
and cutaneous bleeding lasting for about 2 FEB 2012 MED 2013
weeks. Laboratory work ups revelaed anemia, BOARDS; TOPNOTCH
neutropenia and thrombocytopenia. Peripheral MD)
blood smear examination revealed a red
staining peroxidase-positive structures with
abnormal azurophilic granules. What is the
possible diagnosis to this case?
A) Acute Lymphoblastic Leukemia
B) Acute Myelogenous Leukemia
C) Chronic Lymphoblastic Leukemia
D) Chronic Myelogenous Leukemia
541 Which of the following describes a malignant A (choristoma), B (hamartoma), and D (metaplasia) ABDELSIMAR OMAR FINAL EXAM -
condition? describe nonmalignant conditions. C refers to II, MD (TOP 2 - AUG FEB 2014
A. A small nodule of well-developed and pseudomyxoma peritonei; which occurs due to 2013 MED BOARDS;
organized pancreatic substance is found in the seeding of metastatic cells into the peritoneum from TOPNOTCH MD - 200
submucosa of the small intestine. appendiceal carcinomas. QUESTIONS) AND
B. Biopsy of a pulmonary "mass" reveals MARC DENVER
disorganized but histologically normal TIONGSON, MD (40
cartilage, bronchi and vessels QUESTIONS)
C. A gelatinous mass is seen to fill the peritoneal
cavity in a person known to have an "enlarged
appendix."
D. In chronic gastroesophageal reflux,
squamous epithelium in the lower esophagus is
replaced by glandular epithelium.
E. None of the above
542 A 67/M, a known case of lung cancer, came in Ferritin is a soluble iron binding storage protein. It is ABDELSIMAR OMAR FINAL EXAM -
due to easy fatigability and generalized decreased in IDA; but increased in sideroblastic II, MD (TOP 2 - AUG FEB 2014
weakness. Patient was noticeably pale. Your anemia (iron overload disease) and in anemia of 2013 MED BOARDS;
primary impression is anemia secondary to chronic disease / ACD (IL-1 and TNF-a induces its TOPNOTCH MD - 200
chronic disease. In this condition, one expects release.) Serum iron levels may differentiate IDA/ACD QUESTIONS) AND
low levels of the following parameters except: (decreased iron) from iron overload diseases MARC DENVER
A. Fe saturation (increased iron.) Total iron binding capacity TIONGSON, MD (40
B. Total iron binding capacity correlates with transferrin levels. Whether transferrin QUESTIONS)
C. Transferrin increases or decreases may be predicted by what
D. Ferritin happens to ferritin since decreased ferritin stores
E. Serum iron increase synthesis of transferrin in the liver. Thus high
ferritin -> low transferrin / TIBC; and vice versa. Fe
saturation represents the percentage of binding sites
on transferrin occupied by Fe. It is increased in Fe
overload states; and decreased in IDA and ACD.
543 On physical examination, a cystic 3 x 3 cm History of breastfeeding and recent pregnancy suggest ABDELSIMAR OMAR FINAL EXAM -
breast mass was palpated on a 27 year old that the cystic mass is a galactocele. Without such a II, MD (TOP 2 - AUG FEB 2014
woman who just gave birth 6 months ago, and history, primary impression would be a macrocyst. 2013 MED BOARDS;
who had exclusively breastfed her child. What TOPNOTCH MD - 200
is your primary impression? QUESTIONS) AND
A. Macrocyst MARC DENVER
B. Galactocele TIONGSON, MD (40
C. Mastitis QUESTIONS)
D. Fibrocystic changes
E. Breast abscess
544 A 66-year old diabetic, nulliparous woman History of postmenopausal bleeding in patient with ABDELSIMAR OMAR FINAL EXAM -
complains of post-menopausal vaginal bleeding. known risk factors (obesity, nulliparity) raises II, MD (TOP 2 - AUG FEB 2014
Prior to menopause, which occurred at age 55, suspicion for endometrial cancer. Increased estrogen 2013 MED BOARDS;
she had irregular menses. She denies the use of exposure increases risk. Coffee, smoking, OCP use, TOPNOTCH MD - 200
estrogen replacement therapy. Her examination physical activity, and use of raloxifene are protective. QUESTIONS) AND
is significant for obestity and hypertension. The MARC DENVER
following are risk factors for her condition TIONGSON, MD (40
except: QUESTIONS)
A. Early age at menarche
B. Late menopause
C. Diabetes
D. Smoking
E. Obesity
545 A pre-term infant born to an 18-year old Onset of bloody stools, circulatory collapse and ABDELSIMAR OMAR FINAL EXAM -
primigravid developed abdominal distension abdominal distension is typical of necrotizing II, MD (TOP 2 - AUG FEB 2014
and hypotension on the fourth day of life. You enterocolitis. Prematurity is an important risk factor. 2013 MED BOARDS;
noted that the patient passed bloody stools. Microscopically, mucosal or transmural coagulative TOPNOTCH MD - 200
Abdominal radiographs demonstrate gas within necrosis is seen. QUESTIONS) AND
the intestinal wall. Microscopic examination of MARC DENVER
the diseased bowel segment in this condition TIONGSON, MD (40
would reveal: QUESTIONS)
A. Fat necrosis
B. Coagulative necrosis
C. Liquefactive necrosis
D. Caseous necrosis

TOPNOTCH MEDICAL BOARD PREP PATHOLOGY SUPEREXAM Page 74 of 99


For inquiries visit www.topnotchboardprep.com.ph or email us at topnotchmedicalboardprep@gmail.com
TOPNOTCH MEDICAL BOARD PREP PATHOLOGY SUPEREXAM
For inquiries visit www.topnotchboardprep.com.ph or email us at topnotchmedicalboardprep@gmail.com
Item QUESTION EXPLANATION AUTHOR TOPNOTCH
# EXAM
E. Apoptosis

546 A 28/M presents with a month-long history of A classic case of infective endocarditis. The ABDELSIMAR OMAR FINAL EXAM -
low-grade fever. On PE, an apical systolic progression of symptoms over one month suggests II, MD (TOP 2 - AUG FEB 2014
murmur is appreciated on auscultation; the subacute IE; most commonly due to less virulent 2013 MED BOARDS;
Traube's space was obliterated; and organisms, most commonly, Streptococcus viridans. TOPNOTCH MD - 200
hemorrhagic nontender lesions on the palms QUESTIONS) AND
and soles were noted. Patient has an MARC DENVER
unremarkable past medical history. Blood TIONGSON, MD (40
culture woud most likely grow which infectious QUESTIONS)
agent:
A. Staphylococcus aureus
B. Cardiobacterium sp.
C. Streptococcus pyogenes
D. Haemophilus influenzae
E. Streptococcus viridans
547 A 50/M came in with a chief complaint of ABDELSIMAR OMAR FINAL EXAM -
dysphagia, heartburn and regurgitation of sour II, MD (TOP 2 - AUG FEB 2014
tasting fluid. He has weekly episodes of 2013 MED BOARDS;
retching and vomiting after drinking sprees. On TOPNOTCH MD - 200
endoscopy the distal 3rd of his esophagus is QUESTIONS) AND
seen to have tongues of red, velvety mucosa MARC DENVER
extending upward from the gastroesophageal TIONGSON, MD (40
junction alternating with residual smooth QUESTIONS)
squamous mucosa and interfaces with light
brown columnar mucosa distally. 6cm of the
esophagus is affected. Upon biopsy of the area,
histologic findings include abundant
metaplastic goblet cells, atypical mitoses,
nuclear hyperchromasia and failure of
epithelial cells to mature as they reach the
esophageal surface. What is his most likely
diagnosis?
A. Mallory-Weiss tear
B. Reflux esophagitis
C. Barrett esophagus
D. Adenocarcinoma of the esophagus
E. Squamous cell carcinoma of the esophagus
548 A 7-year old boy presents to the ER with a 3- Patient has Reye syndrome. ABDELSIMAR OMAR FINAL EXAM -
day history of vomiting. A week prior to II, MD (TOP 2 - AUG FEB 2014
consult, the patient complained of fever and 2013 MED BOARDS;
malaise; for which he was given Aspirin by his TOPNOTCH MD - 200
mother. On PE, the patient is tachycardic and QUESTIONS) AND
tachypneic. Serum chemistries reveal elevated MARC DENVER
transaminases. The key pathologic finding in TIONGSON, MD (40
the liver of patients with this condition is: QUESTIONS)
A. Microvesicular steatosis
B. Concentric bile duct fibrosis
C. Massive hepatocellular necrosis
D. Piecemeal hepatocellular necrosis
E. Portal bridging fibrosis
549 A 45.F presents with a 4-day history of an Remember the classic cases! Middle-aged woman with ABDELSIMAR OMAR FINAL EXAM -
acutely painful right sided goiter. She has no PAINLESS goiter and hypothyroid sx = Hashimoto's. II, MD (TOP 2 - AUG FEB 2014
previous history of thyroid disease; and Postpartal woman with PAINLESS goiter and signs of 2013 MED BOARDS;
clinically appears euthyroid. Thyroid gland thyrotoxicosis = Subacute lymphocytic thyroiditis. TOPNOTCH MD - 200
biopsy would reveal disruption of thyroid QUESTIONS) AND
follicles with extravasation of colloid leading to MARC DENVER
a polymorphonuclear infiltrate. A TIONGSON, MD (40
granulomatous reaction may be seen. What is QUESTIONS)
the most likely diagnosis?
A. Subacute lymphocytic thyroiditis
B. Hashimoto's thyroiditis
C. DeQuervain's thyroiditis
D. Grave's disease
E. Colloid adenomatous goiter
550 A 52-year old woman presents with a 4-month VIPoma (aka Verner Morrison syndrome) is a rare ABDELSIMAR OMAR FINAL EXAM -
history of profuse watery diarrhea, weakness endocrine tumor, usually originating from the non II, MD (TOP 2 - AUG FEB 2014
and dehydration. Testing reveals a serum islet cells of the pancreas that produces VIP. It is also 2013 MED BOARDS;
calcium level of 11.4 mg/dL and a potassium known as the WDHA syndrome because massive TOPNOTCH MD - 200
level of 2.1 mEq/L. Abdominal CT done amounts of VIP cause Watery Diarrhea with resultant QUESTIONS) AND
revealed a pancreatic mass. What is the most Dehydrataion, Hypokalemia and Achlorydia. Patients MARC DENVER
likely diagnosis? also present with hypercalcemia hyperglycemia and TIONGSON, MD (40
A. Glucagonoma metabolic acidosis. QUESTIONS)
B. Somatostatinoma
C. Insulinoma
D. Carcinoid tumor
E. VIPoma
TOPNOTCH MEDICAL BOARD PREP PATHOLOGY SUPEREXAM Page 75 of 99
For inquiries visit www.topnotchboardprep.com.ph or email us at topnotchmedicalboardprep@gmail.com
TOPNOTCH MEDICAL BOARD PREP PATHOLOGY SUPEREXAM
For inquiries visit www.topnotchboardprep.com.ph or email us at topnotchmedicalboardprep@gmail.com
Item QUESTION EXPLANATION AUTHOR TOPNOTCH
# EXAM
551 A 70/M, retired teacher, complains of multiple The histologic features of the lesion does NOT suggest ABDELSIMAR OMAR FINAL EXAM -
raised pigmented lesions over his back. Some of malignancy, ruling out C, D and E. Seborrheic keratosis II, MD (TOP 2 - AUG FEB 2014
the lesions are pruritic. On PE, you note is a common, multiple, benign skin tumor; and appear 2013 MED BOARDS;
multiple roughened brown waxy lesions that as well-circumscribed brown plaques with a stuck on TOPNOTCH MD - 200
appear "stuck on" over the patient's back. appearance. Actinic keratosis lesions on the other QUESTIONS) AND
Biopsy of the lesion reveals sheets of small cells hand are skin colored, yellowish or erythematous ill- MARC DENVER
that resemble basal cells of normal epidermis; defined irregular shaped scaly macules or plaques TIONGSON, MD (40
associated with the presence of small keratin- localized in sun-exposed areas of the body. Usually, a QUESTIONS)
filled cysts. What is your primary impression? case of actinic keratoses would feature a 'farmer' or
A. Actinic keratosis any other character with a history of chronic sun
B. Seborrheic keratosis exposure.
C. Malignant melanoma
D. Basal cell carcinoma
E. Squamous cell carcinoma
552 A 35/M with a 10-pack year history of smoking Thromboangiitis obliterans or Buerger's disease is a ABDELSIMAR OMAR FINAL EXAM -
comes in due to a cold painful fingertip with a non-atherosclerotic vascultiis resulting in segmental II, MD (TOP 2 - AUG FEB 2014
beginning ulcer. You elicit a history of Raynaud occlusion of medium-sized vessels. Patients are usuall 2013 MED BOARDS;
phenomenon in the patient. On PE, you note young, male, with a strong history of smoking. It can TOPNOTCH MD - 200
decreased brachial, ulnar and radial pulses. affect the radial artery, presenting as recurrent QUESTIONS) AND
What is your primary impression? Raynaud's phenomenon; or the tibial artery, MARC DENVER
A. Microscopic polyangitis presenting as instep claudication that persists even TIONGSON, MD (40
B. Polyarteritis nodosa after exercise. QUESTIONS)
C. Churgg-Strauss syndrome
D. Takayasu arteritis
E. Thromboangiitis obliterans
553 Urinalysis reveals increased urobilinogen, but The point of the question is to differentiate between ABDELSIMAR OMAR FINAL EXAM -
absent urine bilirubin. This is consistent with urobilinogen and urine bilirubin. Urobilinogen is II, MD (TOP 2 - AUG FEB 2014
which of the following conditions: formed by intestinal bacteria from conjugated 2013 MED BOARDS;
A. Hereditary spherocytosis bilirubin after it is secreted into the lumen; it is then TOPNOTCH MD - 200
B. Hepatitis reabsorbed by the enterohepatic circulation. Urine QUESTIONS) AND
C. Pancreatic head malignancy bilirubin, on the other hand, comes from conjugated MARC DENVER
D. Common bile duct obstruction bilirubin in the serum that is filtered into the urine. TIONGSON, MD (40
E. Rotor syndrome Recall that unconjugated bilirubin is NOT water- QUESTIONS)
soluble and bound to albumin; and is not filtered.
In obstructive jaundice (C and D), bile is not secreted
into the intestinal lumen where bacteria converts
conjugated bilirubin into urobilinogen. Thus, no
urobinogen is found in the urine.
In hepatitis, mixed bilirubinemia occurs (both
unconjugated and conjugated bilirubin increase).
Thus, we expect an increase in urine bilirubin due to
increased amounts of conjugated bilirubin filtered by
the kidney.
In hemolytic anemia, unconjugated bilirubinemia
predominates. Some of this excess bilirubin is
secreted into the lumen where it is converted into
urobilinogen. Urine bilirubin on the other hand is
decreased, since most of the serum bilirubin in
hemolytic anemia is UNCONJUGATED and not filtered
by the kidneys.
554 A 60/M presents due to gross hematuria. You ABDELSIMAR OMAR FINAL EXAM -
elicit a 3-month history of right-sided flank II, MD (TOP 2 - AUG FEB 2014
discomfort and abdominal fullness. Vital signs 2013 MED BOARDS;
are as follows: BP: 170/100, HR: 122, RR: 22, T: TOPNOTCH MD - 200
afeb. On PE, you palpate a right-sided lower QUESTIONS) AND
abdominal mass. You ordered an abdominal CT; MARC DENVER
the findings of which are consistent with renal TIONGSON, MD (40
cell carcinoma. Renal cell carcinoma is MOST QUESTIONS)
COMMONLY derived from cells of the:
A. Glomerulus
B. Proximal tubule
C. Loop of Henle
D. Distal tubule
E. Collecting tubule
555 A 7-year old boy presents with a 5-day history Medulloblastoma occurs predominantly in children ABDELSIMAR OMAR FINAL EXAM -
of gradually worsening headaches and and exclusively in the cerebellum. Histology reveals II, MD (TOP 2 - AUG FEB 2014
intermittent vomiting without fever. Patient extremely cellular tumor with sheets of anaplastic 2013 MED BOARDS;
was also noted to be walking with poor balance small blue cells. Another commonly occuring TOPNOTCH MD - 200
and coordination. MRI done revealed a cerebellar tumor in childen in plemorphic QUESTIONS) AND
cerebellar mass. Patient subsequently astrocytoma; but this benign condition would not MARC DENVER
underwent excision of the tumor. Biopsy usually present with necrosis and mitoses on TIONGSON, MD (40
reveals that the tumor is extremely cellular, histology. QUESTIONS)
with sheets of anaplastic ("small blue") cells.
Individual tumor cell are small, with little
cytoplasm and hyperchromatic nuclei. Mitotic
figures are abundant. What is the diagnosis?
A. Pilocytic astrocytoma
B. Pleomorphic xanthoastrocytoma
C. Medulloblastoma
D. Craniopharyngioma
E. Glioblastoma multiforme

TOPNOTCH MEDICAL BOARD PREP PATHOLOGY SUPEREXAM Page 76 of 99


For inquiries visit www.topnotchboardprep.com.ph or email us at topnotchmedicalboardprep@gmail.com
TOPNOTCH MEDICAL BOARD PREP PATHOLOGY SUPEREXAM
For inquiries visit www.topnotchboardprep.com.ph or email us at topnotchmedicalboardprep@gmail.com
Item QUESTION EXPLANATION AUTHOR TOPNOTCH
# EXAM
556 Your 80-year old lola has become increasingly At the microscopic level, Alzheimer's Disease is ABDELSIMAR OMAR FINAL EXAM -
forgetful. She is no longer able to help in diagnoed by the prsence of plaques and II, MD (TOP 2 - AUG FEB 2014
household tasks. She later become more neurofibrillary tangles. Lewy bodies are seen in 2013 MED BOARDS;
irritable. Systemic physical examination and Parkinson's disease; Pick bodies are seen in Pick TOPNOTCH MD - 200
routine work-up is unremarkable. Her mini- disease, a subtype of Frontotemporal lobar QUESTIONS) AND
mental status examination (MMSE) score is degeneration. (In frontotemporal lobar degeneration, MARC DENVER
20/30; there are no other significant neurologic behavior changes precede memory disturbances TIONGSON, MD (40
examination findings. Her condition is which assist in their separation from AD on clinical QUESTIONS)
characterized by which of the following grounds.) Rosenthal fibers are features of pilocytic
histologic features: astrocytoma.
A. Neurofibrillary tangles
B. Pick bodies
C. Lewy neurites
D. Fibrillary gliosis
E. Rosenthal fibers
557 A 70/M is evaluated for a heart murmur. On PE, "Dystrophic calcification of the aortic valces is an ABDELSIMAR OMAR FINAL EXAM -
a 3/6 systolic ejection murmur radiating to the important cause of aortic stenosis in elderly persons." II, MD (TOP 2 - AUG FEB 2014
neck is heard on auscultation. Echocardiogram Basic Patho 9e p.25 2013 MED BOARDS;
shows aortic stenosis and normal systolic TOPNOTCH MD - 200
function. Which of the following cellular QUESTIONS) AND
changes is BEST demonstrated in this MARC DENVER
condition? TIONGSON, MD (40
A. Atrophy QUESTIONS)
B. Coagulative necrosis
C. Hypertrophy
D. Dystrophic calcification
E. Metastatic calcification
558 On the 28th day of the menstrual cycle, a Menses is an example of apoptosis or programmed ABDELSIMAR OMAR FINAL EXAM -
woman undergoes menstrual bleeding lasting cell death. II, MD (TOP 2 - AUG FEB 2014
2-4 days consuming 3 pads per day. Which of 2013 MED BOARDS;
the following cellular changes is BEST TOPNOTCH MD - 200
demonstrated in this condition? QUESTIONS) AND
A. Atrophy MARC DENVER
B. Apoptosis TIONGSON, MD (40
C. Coagulative necrosis QUESTIONS)
D. Liquefactive necrosis
E. Hypertrophy
559 A 45/M has been drinking alcoholic beverages Hyperuricemia occurs in chronic alcohol use because ABDELSIMAR OMAR FINAL EXAM -
heavily for the past 10 years. The following are lactic acid and beta-hydroxybutyrate compete with II, MD (TOP 2 - AUG FEB 2014
laboratory findings expected in alcohol abuse uric acid for excretion in the proximal tubules. Alcohol 2013 MED BOARDS;
except: is a mitochondrial toxin that causes release of ALT, TOPNOTCH MD - 200
A. Hyperuricemia located in the mitochondria. It also induces QUESTIONS) AND
B. AST>ALT hyperplasia of SER causing increased synthesis of MARC DENVER
C. Increased GGT GGT. Increased levels of NADH also lead to the TIONGSON, MD (40
D. Hypertriglyceridemia conversion of DHAP into G3P which is used as a QUESTIONS)
E. Fasting hyperglycemia substrate for triglyceride synthesis in the liver. Fasting
HYPOglycemia occurs because excess NADH causes
pyruvate (the substrate for gluconeogenesis) to be
converted to lactate.
560 A 30/M, call-center agent, known case of HIV- Remember that FSGS is the predominant glomerular ABDELSIMAR OMAR FINAL EXAM -
AIDS, presents with progressive generalized lesion in patients with HIV-associated nephropathy. II, MD (TOP 2 - AUG FEB 2014
swelling. On PE, patient has non-pitting edema 2013 MED BOARDS;
over the lower extremities extending up to the TOPNOTCH MD - 200
mid-abdomen. Edema was refractory to steroid QUESTIONS) AND
therapy. Renal biopsy done revealed increased MARC DENVER
mesangial matrix, obliterated capillary lumina, TIONGSON, MD (40
and deposition of hyaline masses and lipid QUESTIONS)
droplets in affected glomeruli.
Immunofluorescence microscopy revealed
trapping of immunoglobulins and complement
in areas of hyalinosis. On electron microscopy,
the podocytes exhibit effacement of foot
processes. What is the most likely diagnosis?
A. Membranous nephropathy
B. Minimal change disease
C. Focal segmental glomerulosclerosis
D. Membranoproliferative glomerulonephritis
E. IgA nephropathy
561 Drugs can cause alteration in both acute and Hypersensitivity pneumonitis. Table 15-7 of Robbins BLAKE WARREN MIDTERM 2
chronic alterations in respiratory structure and ANG, MD (TOP 1 - EXAM - FEB
function. Among these drugs that cause AUG 2013 MED 2014
pulmonary disease is nitrofurantoin which is BOARDS; TOPNOTCH
associated with: MD)
a. interstitial fibrosis
b. hypersensitivity pneumonitis
c. bronchiolitis obliterans
d. eosinophilic pneumonia

TOPNOTCH MEDICAL BOARD PREP PATHOLOGY SUPEREXAM Page 77 of 99


For inquiries visit www.topnotchboardprep.com.ph or email us at topnotchmedicalboardprep@gmail.com
TOPNOTCH MEDICAL BOARD PREP PATHOLOGY SUPEREXAM
For inquiries visit www.topnotchboardprep.com.ph or email us at topnotchmedicalboardprep@gmail.com
Item QUESTION EXPLANATION AUTHOR TOPNOTCH
# EXAM
562 Certain pulmonary entities exhibit a histologic Loeffler syndrome or Simple pulmonary eosinophilia BLAKE WARREN MIDTERM 2
picture of eosinophilic infiltration. Which of the is characterized by transient pulmonary lesions, blood ANG, MD (TOP 1 - EXAM - FEB
following disease is characterized by a benign eosinophilia and a benign clinical course. AUG 2013 MED 2014
clinical course? BOARDS; TOPNOTCH
a. Acute eosinophilic pneumonia MD)
b. Loeffler syndrome
c. Tropical eosinophilia
d. Chronic eosinophilic pneumonia

563 Patient presented with fever, headache, muscle Atypical pneumonia include viral pneumonias and BLAKE WARREN MIDTERM 2
aches, leg pains and cough. PE did not show mycoplasma pneumonia. P714 Robbins. ANG, MD (TOP 1 - EXAM - FEB
findings of consolidation. Histologic pattern of AUG 2013 MED 2014
the disease show a predominant interstitial BOARDS; TOPNOTCH
nature of inflammatory reaction virtually MD)
localized within the walls of the alveoli. What is
the most probable causative agent?
a.) Klebsiella pneumonia
b.) Streptococcus pyogenes
c.) Mycoplasma pneumoniae
d.) Staphylococcus aureus
564 Fleur de lis pattern is characteristic of which BLAKE WARREN MIDTERM 2
infectious agent? ANG, MD (TOP 1 - EXAM - FEB
a. Pseudomonas aeruginosa AUG 2013 MED 2014
b. Mycoplasma pneumonia BOARDS; TOPNOTCH
c. Streptococcus pyogenes MD)
d. Staphylococcus aureus

565 Which of the following antibodies can cross the IgG4. IgG2 is the only subclass of IgG that may be BLAKE WARREN MIDTERM 2
placental circulation? unable cross the placenta. Source: Henrys Diagnostics ANG, MD (TOP 1 - EXAM - FEB
a. IgA AUG 2013 MED 2014
b. IgG2 BOARDS; TOPNOTCH
c. IgE MD)
d. IgG4

566 10 month old male presented with paralytic Brutons /x-linked aggamaglobulinemia can BLAKE WARREN MIDTERM 2
poliomyelitis after given an oral polio vaccine. predispose to paralytic poliomyelitis after live ANG, MD (TOP 1 - EXAM - FEB
What is the most probable underlying etiology? poliovirus immunization. Robbins p232 AUG 2013 MED 2014
a. Severe combined immune deficiency BOARDS; TOPNOTCH
syndrome MD)
b. Brutons Aggamaglobulinemia
c. DiGeorge Syndrome
d. Isolated IgA deficiency

567 10 month old male patient had recurrent bouts BLAKE WARREN MIDTERM 2
of infection with Haemophilus and ANG, MD (TOP 1 - EXAM - FEB
Staphylococcus aureus. He also had 3 episodes AUG 2013 MED 2014
of Giardiasis since 6 months of age. What is the BOARDS; TOPNOTCH
most probable defect? MD)
a. Complement
b. B cells
c. T cell
d. NK cells

568 15 year old patient presented with recurrent Answer is C as per Robbins p233 BLAKE WARREN MIDTERM 2
sinopulmonary infections of Haemophilus or ANG, MD (TOP 1 - EXAM - FEB
Streptococcal origin. He has had recurrent AUG 2013 MED 2014
bouts diarrhea despite treatment. What is the BOARDS; TOPNOTCH
expected histologic picture of the patient? MD)
a. Decrease number of B cells in lymphoid
tissues and blood.
b. Decrease number of B cells in the lymphoid
follicles of spleen, liver and gut.
c. Normal or near-normal numbers of B cells in
the blood and lymphoid tissues.
d. Increase numbers of B cells in the peripheral
circulation.
569 Helicobacter pylori is associated with blood Lewis as per Henrys diagnostics BLAKE WARREN MIDTERM 2
type O individuals. The reason for this lies in ANG, MD (TOP 1 - EXAM - FEB
the organisms capacity to bind to certain AUG 2013 MED 2014
antigens expressed on the surface of cells like H BOARDS; TOPNOTCH
antigen of the ABO blood group systems and MD)
the antigens of which other blood group
system?
a. Lutheran
b. Lewis
c. Kell
d. Rh blood group

TOPNOTCH MEDICAL BOARD PREP PATHOLOGY SUPEREXAM Page 78 of 99


For inquiries visit www.topnotchboardprep.com.ph or email us at topnotchmedicalboardprep@gmail.com
TOPNOTCH MEDICAL BOARD PREP PATHOLOGY SUPEREXAM
For inquiries visit www.topnotchboardprep.com.ph or email us at topnotchmedicalboardprep@gmail.com
Item QUESTION EXPLANATION AUTHOR TOPNOTCH
# EXAM
570 What is the average weight of the bone marrow 1200-1500 grams as per Henrys diagnostics BLAKE WARREN MIDTERM 2
of an adult individual? ANG, MD (TOP 1 - EXAM - FEB
a. 1000 grams AUG 2013 MED 2014
b. 1500 grams BOARDS; TOPNOTCH
c.2000 grams MD)
d. 1750 grams

571 Caspases are noted for their cysteine content caspase 8 in the nematode C. elegans but Caspase 10 BLAKE WARREN MIDTERM 2
and the ability to cleave aspartic acid residues. in humans p30 robbins ANG, MD (TOP 1 - EXAM - FEB
They are hence involve in programmed cell AUG 2013 MED 2014
death. Which caspase acts as the initiator BOARDS; TOPNOTCH
caspase in humans under the extrinsic MD)
pathway?
a. Caspase 8
b. Caspase 9
c. Caspase 10
d. Smac/DIABLO
572 40 year old male has had recurrent episodes of A as per robbins p778 in the setting of an acute H BLAKE WARREN MIDTERM 2
epigastric pain for the past 3 weeks, relieved by pylori gastritis. D occurs in the setting of chronic H. ANG, MD (TOP 1 - EXAM - FEB
eating. He had an endoscopy with biopsy done. pylori gastritis. Infection of the cardia occur at lower AUG 2013 MED 2014
Which of the following is true regarding the rates and H. pylori generally does not colonize BOARDS; TOPNOTCH
expected histologic picture of the gastric biopsy duodenal epithelium. MD)
specimen?
a. Intraepithelial neutrophils and subepithelial
plasma cells are characteristic of H. pylori
gastritis
b. Although there is a good concordance
between colonization of the antrum and cardia,
infection of the cardia occurs at somewhat
higher rates.
c. H. pylori shows tropism to gastric epithelia
and is generally generally found in association
with gastric intestinal metaplasia or duodenal
epithelium.
d. Atrophic gastric mucosa with lymphoid
aggregates can be seen.
573 Which of the following is not found the IgA is not found in PSGN but is more prominent in BLAKE WARREN MIDTERM 2
mesangium of a Post-Infectious Buergers disease. ANG, MD (TOP 1 - EXAM - FEB
Glomerulonephritis? AUG 2013 MED 2014
a. IgG BOARDS; TOPNOTCH
b.IgM MD)
c. IgA
d. Complement

574 Which of the Gatrointestinal Polyposis A is associated with an LKB1/STK11 mutation. BLAKE WARREN MIDTERM 2
syndromes does not have a hereditary Juvenile polyposis is associated with a mutation in the ANG, MD (TOP 1 - EXAM - FEB
component? SMAD4 gene. Cowden with PTEN. Cronkhite-Canada AUG 2013 MED 2014
a. Peutz-Jeghers syndrome syndrome makes its distinction by being BOARDS; TOPNOTCH
b. Juvenile Polyposis nonhereditary. MD)
c. Cowden Syndrome
d. Cronkhite-Canada syndrome

575 Most common site of AV malformation in the MCA as per robbins p1299 BLAKE WARREN MIDTERM 2
brain? ANG, MD (TOP 1 - EXAM - FEB
a. ACA AUG 2013 MED 2014
b. MCA BOARDS; TOPNOTCH
c. PCA MD)
d. Basilar artery

576 Flexner-wintersteiner rosettes is associated BLAKE WARREN MIDTERM 2


with ANG, MD (TOP 1 - EXAM - FEB
a. Neuroblastoma AUG 2013 MED 2014
b. Retinoblastoma BOARDS; TOPNOTCH
c. Medulloblastoma MD)
d. Glioblastoma

577 70 year old female presented with a slow Colloid CA as per robbins BLAKE WARREN MIDTERM 2
growing breast mass with a rubbery ANG, MD (TOP 1 - EXAM - FEB
consistency. Histologic picture show islands of AUG 2013 MED 2014
cells in pale gray-blue gelatin. What is the most BOARDS; TOPNOTCH
probable tumor? MD)
a. Medullary breast cancer
b. Ductal carcinoma
c. Colloid Carcinoma
d. Papillary breast CA

578 Which of the following is not associated with all are associated with aging except MVP robbins p532 BLAKE WARREN MIDTERM 2
advancing age in the human heart? ANG, MD (TOP 1 - EXAM - FEB
a. sigmoid septum AUG 2013 MED 2014
b. mitral valve prolapse BOARDS; TOPNOTCH
c. Lipofuscin deposistion MD)
d. Lambl excresences

TOPNOTCH MEDICAL BOARD PREP PATHOLOGY SUPEREXAM Page 79 of 99


For inquiries visit www.topnotchboardprep.com.ph or email us at topnotchmedicalboardprep@gmail.com
TOPNOTCH MEDICAL BOARD PREP PATHOLOGY SUPEREXAM
For inquiries visit www.topnotchboardprep.com.ph or email us at topnotchmedicalboardprep@gmail.com
Item QUESTION EXPLANATION AUTHOR TOPNOTCH
# EXAM
579 Cardiac hypertrophy is noted with ventricular 2cm; normal thickness is 1-1.5cm BLAKE WARREN MIDTERM 2
wall thickness of about: ANG, MD (TOP 1 - EXAM - FEB
a. 0.5cm AUG 2013 MED 2014
b. 1cm BOARDS; TOPNOTCH
c. 1.5cmcm MD)
d. 2cm

580 Intestinal metaplasia of the stomach raises a Goblet cells and intestinal columnar absorptive cells BLAKE WARREN MIDTERM 2
predisposition towards adenocarcinoma when present points to intestinal metaplasia ANG, MD (TOP 1 - EXAM - FEB
development. Which cell most likely found AUG 2013 MED 2014
confirms intestinal metaplasia? BOARDS; TOPNOTCH
a. Mucous cells MD)
b. Goblet cells
c. Isolated islands of Brunner glands
d. Simple columnar epithelia

581 A 7 year old male was admitted due to easy By definition, in ALL, blasts compose more than 25% TIMOTHY TANG LEE MIDTERM 1
fatigability, fever, and easy bruisability for 2 of the marrow cellularity. The nuclei of lymphoblasts SAY, MD (TOP 4 - EXAM - FEB
months. CBC done showed the following: in Wright-Giemsa-stained preparations have AUG 2013 MED 2014
Hgb=94 mg/L, Hct=0.29, Plt=54, WBC=45.50, somewhat coarse and clumped chromatin and one or BOARDS; TOPNOTCH
Segmenters=0.10, Lymphocytes=0.66, two nucleoli; myeloblasts tend to have finer chromatin MD)
Metamyelocyte=0.04, Blast=0.16, and more cytoplasm, which may contain granules. The
Myelocyte=0.04. Bone marrow biopsy showed cytoplasm of lymphoblasts often contains large
hypercellular marrow with cells which have aggregates of periodic acid-Schiff-positive material,
scant cytoplasm with nuclei containing coarse whereas myeloblasts are often peroxidase positive.
and clumped chromatin. Immunostaining
showed (-) myeloperoxidase and (+) periodic
acid-Schiff cytoplasmic material. What is the
most likely diagnosis?
A. Acute lymphoblastic leukemia
B. Acute myeloblastic leukemia
C. Chronic lymphocytic leukemia
D. Chronic myelogenous leukemia
E. Multiple myeloma
582 A 30-year-old woman presents with malaise Apoptosis involves single cells, not large groups of TIMOTHY TANG LEE MIDTERM 1
and increasing fatigue. On physical cells, and with apoptosis the cells shrink and there is SAY, MD (TOP 4 - EXAM - FEB
examination, she has anicteric sclerae and increased eosinophilia of cytoplasm. The shrunken AUG 2013 MED 2014
jaundice, and laboratory evaluation finds apoptotic cells form apoptotic bodies, which may be BOARDS; TOPNOTCH
elevated AST and ALT. Liver biopsy showed engulfed by adjacent cells or macrophages. With MD)
shrunken hepatocytes with highly eosinophilc apoptosis there is no inflammatory response, the cell
cytoplasm and fragmented nuclei. There were membranes do not rupture, and there is no release of
no noted inflammatory infiltrates. Which of the macromolecules
following terms best describes this process
occurring in the hepatocytes?
A. Apoptosis
B. Autophagy
C. Heterophagy
D. Necrosis
E. Pinocytosis
583 A 60-year-old man died secondary to coronary The pathogenesis of atherosclerosis depends in part TIMOTHY TANG LEE MIDTERM 1
artery disease. During autopsy, the lumen of the on the inflammatory function of macrophages, which SAY, MD (TOP 4 - EXAM - FEB
coronary arteries was covered with fibrous involves the release of numerous cytokines. Platelet- AUG 2013 MED 2014
plaques and an area with ruptured plaque and derived growth factor (PDGF) is mitogenic and BOARDS; TOPNOTCH
superimposed thrombosis. Sections from these chemotactic for smooth-muscle cells. This may explain MD)
abnormal areas revealed a fibrous cap and a the recruitment and proliferation of smooth-muscle
central core (largely lipid) core with an area cells in atherosclerosis.
showing disruption of the fibrous cap with
thrombosis. Which one of the following
substances promotes atherosclerosis by
stimulating smooth-muscle cells to migration
and proliferation in sites of endothelial injury?
A. -interferon (-INF)
B. -transforming growth factor (TGF)
C. Interleukin-1 (IL1)
D. Platelet derived growth factor (PDGF)
E. Vascular endothelial growth factor (VEGF)
584 A 56-year old post-gastrectomy patient In the peripheral blood the earliest change is usually TIMOTHY TANG LEE MIDTERM 1
consults due to pallor and easy fatigability. CBC the appearance of hypersegmented neutrophils, which SAY, MD (TOP 4 - EXAM - FEB
done showed anemia. Peripheral smear showed appear even before the onset of anemia. Normally, AUG 2013 MED 2014
large, egg-shaped macro-ovalocytes and neutrophils have three or four nuclear lobes, but in BOARDS; TOPNOTCH
hypersegmented neutrophils. Patient presents megaloblastic anemias neutrophils often have five or MD)
with? more. The red cells typically include large, egg-shaped
A. Sideroblastic anemia macro-ovalocytes
B. Iron deficiency anemia
C. Anemia of chronic disease
D. Aplastic anemia
E. Megaloblastic anemia

TOPNOTCH MEDICAL BOARD PREP PATHOLOGY SUPEREXAM Page 80 of 99


For inquiries visit www.topnotchboardprep.com.ph or email us at topnotchmedicalboardprep@gmail.com
TOPNOTCH MEDICAL BOARD PREP PATHOLOGY SUPEREXAM
For inquiries visit www.topnotchboardprep.com.ph or email us at topnotchmedicalboardprep@gmail.com
Item QUESTION EXPLANATION AUTHOR TOPNOTCH
# EXAM
585 A 40-year old consulted due to a slowly The most common neoplasm of the parotid gland is TIMOTHY TANG LEE MIDTERM 1
enlarging, soft, painless, pre-auricular mass. the pleomorphic adenoma (mixed tumor), which SAY, MD (TOP 4 - EXAM - FEB
Surgical excision of the mass was done. histologically reveals epithelial structures embedded AUG 2013 MED 2014
Microscopically, the mass was composed of within a mesenchyme-like stroma consisting of BOARDS; TOPNOTCH
small, dark epithelial cells intermingled with a mucoid, myxoid, or chondroid tissue. MD)
loose, myxoid connective tissue stroma with
islands and strands of myoepithelial cells.
These findings are suggestive of?
A. Warthin tumor
B. Pleomorphic adenoma
C. Mucoepidermoid carcinoma
D. Myoepithelioma
E. Adenoid cystic carcinoma
586 A 46-year old female came in due to prolonged Most commonly, the prolonged unremitting estrogen TIMOTHY TANG LEE MIDTERM 1
heavy menses. Dilatation and curettage was stimulation results in endometrial hyperplasia. World SAY, MD (TOP 4 - EXAM - FEB
done. Endometrial scrapings revealed an Health Organization (WHO) classification takes into AUG 2013 MED 2014
increase in the number and size of endometrial account both the architectural and cytologic features, BOARDS; TOPNOTCH
glands, marked gland crowding and branching in the sense of dividing the hyperplasias into simple MD)
(back-to-back appearance) with little and complex on the basis of the architecture, and
intervening stroma. The epithelial cells remain subdividing each into typical and atypical on the basis
cytologically normal. This is suggestive of? of their cytology. Endometrial hyperplasia is most
A. Endometrial polyp commonly seen during the perimenopausal period.
B. Leiomyoma
C. Endometriosis
D. Endometrial hyperplasia
E. Endometrial carcinoma
587 A 10-year old male complained of a painful, Ewing sarcoma and PNETs arise in the medullary TIMOTHY TANG LEE MIDTERM 1
enlarging mass on the right femur. X-ray of the cavity and invade the cortex and periosteum to SAY, MD (TOP 4 - EXAM - FEB
affected area showed layers of bone deposited produce a soft tissue mass. The tumor is tan-white, AUG 2013 MED 2014
in an onion-skin fashion. Surgical excision was frequently with hemorrhage and necrosis. It is BOARDS; TOPNOTCH
done. Microscopic examination of the mass composed of sheets of uniform small, round cells that MD)
showed sheets of uniform, round cells with are slightly larger than lymphocytes with few mitoses
small amounts of clear cytoplasm with few and little intervening stroma. Ewing sarcoma and
mitoses and little intervening cytoplasm. What PNETs typically present as painful enlarging masses in
is the diagnosis? the diaphyses of long tubular bones (especially the
A. Osteosarcoma femur) and the pelvic flat bones. Some patients have
B. Chondroblastoma systemic signs and symptoms, including fever,
C. Ewing sarcoma elevated erythrocyte sedimentation rate, anemia, and
D. Chondrosarcoma leukocytosis that can mimic infection. X-rays show a
E. Rhabdomyosarcoma destructive lytic tumor with infiltrative margins and
extension into surrounding soft tissues. There is a
characteristic periosteal reaction depositing bone in
an onionskin fashion.
588 A 50-year old male consulted due to recurrent The major morphologic manifestations of gout are TIMOTHY TANG LEE MIDTERM 1
left ankle pain and swelling relieved by intake acute arthritis, chronic tophaceous arthritis, tophi in SAY, MD (TOP 4 - EXAM - FEB
of NSAIDS. Arthrocentesis was done and various sites, and gouty nephropathy. Acute arthritis AUG 2013 MED 2014
microscopic examination of the synovial fluid is characterized by a dense neutrophilic infiltrate BOARDS; TOPNOTCH
showed long, slender, needle-shaped crystals permeating the synovium and synovial fluid. Long, MD)
which are negatively bifringent. This is slender, needle-shaped monosodium urate crystals
indicative of? are frequently found in the cytoplasm of the
A. Gouty arthritis neutrophils as well as in small clusters in the
B. Calcium pyrophosphate deposition disease synovium. The synovium is edematous and congested,
C. Viral arthritis and contains scattered mononuclear inflammatory
D. Psoriatic arthritis cells. When the episode of crystallization abates and
E. Pseudogouty arthritis the crystals resolubilize, the attack remits
589 An 80-year old female was noted to have At the microscopic level, Alzheimer disease is TIMOTHY TANG LEE MIDTERM 1
insidious onset of memory loss accompanied by diagnosed by the presence of plaques (a type of SAY, MD (TOP 4 - EXAM - FEB
alterations in mood and behavior, and extracellular lesion); and neurofibrillary tangles (a AUG 2013 MED 2014
impairment in higher level of intellectual type of intracellular lesion). Neuritic plaques are focal, BOARDS; TOPNOTCH
function. There were no noted symptoms of spherical collections of dilated, tortuous, silver- MD)
ataxia, language disturbances or alterations in staining neuritic processes (dystrophic neurites),
personality. This disease entity is differentiated often around a central amyloid core.
from other causes of dementia due to the
presence of?
A. Cytoplasmic round to oval filamentous
inclusion bodies that stain strongly with silver
(Pick Bodies)
B. Tau-containing neurofibrillary tangles
C. Ballooned neurons positive for
phosphorylated neurofilaments
D. Patchy perivacuolar or plaque type
patterns of misfolded protein deposition
E. Spherical collections of dilated, tortuous,
silver-staining neuritic processes (dystrophic
neurites), often around a central amyloid core
(neuritic/senile plaques)

TOPNOTCH MEDICAL BOARD PREP PATHOLOGY SUPEREXAM Page 81 of 99


For inquiries visit www.topnotchboardprep.com.ph or email us at topnotchmedicalboardprep@gmail.com
TOPNOTCH MEDICAL BOARD PREP PATHOLOGY SUPEREXAM
For inquiries visit www.topnotchboardprep.com.ph or email us at topnotchmedicalboardprep@gmail.com
Item QUESTION EXPLANATION AUTHOR TOPNOTCH
# EXAM
590 A 20-year old female complained of multiple In pemphigus vulgaris, acantholysis selectively TIMOTHY TANG LEE MIDTERM 1
bullae and vesicles on the face, axilla, and trunk. involves the layer of cells immediately above the basal SAY, MD (TOP 4 - EXAM - FEB
The lesions were noted to rupture easily that cell layer, giving rise to a suprabasal acantholytic AUG 2013 MED 2014
leaves shallow erosions. This disorder is caused blister. It is caused by a type II hypersensitivity BOARDS; TOPNOTCH
by autoantibodies that result in dissolution of reaction. Patient sera contain pathogenic IgG MD)
intercellular attachments within the epidermis antibodies to intercellular desmosomal proteins
and mucosal epithelium. The autoantibodies (desmoglein types 1 and 3) of skin and mucous
are directed against? membranes. The distribution of these proteins within
A. Desmoglein the epidermis determines the location of the lesions.
B. Hemidesmosomes
C. Reticulin
D. Laminin
E. Keratin
591 A 6-year old female was noted to have bipedal This relatively benign disorder is the most frequent TIMOTHY TANG LEE MIDTERM 1
edema. Urinalysis done showed no hematuria cause of the nephrotic syndrome in children. It is SAY, MD (TOP 4 - EXAM - FEB
and massive proteinuria. Blood pressure was characterized by glomeruli that have a normal AUG 2013 MED 2014
normal. There was a rapid response in the appearance by light microscopy but show diffuse BOARDS; TOPNOTCH
disease after administration of steroids. effacement of podocyte foot processes when viewed MD)
Electron microscopy will show? with the electron microscope. Even with the electron
A. Thickened GBM, and double contour or microscope, the GBM appears normal. The only
tram-track appearance of glomerular capillary obvious glomerular abnormality is the uniform and
wall diffuse effacement of the foot processes of the
B. Irregular thickening of the GBM, lamination podocytes. The cytoplasm of the podocytes thus
of lamina densa, and foci rarefraction appears flattened over the external aspect of the GBM,
C. Diffuse effacement of foot processes of obliterating the network of arcades between the
podocytes, normal GBM, and absence of podocytes and the GBM. There are also epithelial cell
deposits vacuolization, microvillus formation, and occasional
D. Diffuse effacement of foot processes of focal detachments. When the changes in the podocytes
podocytes and denudation of underlying GBM reverse (e.g., in response to corticosteroids), the
E. Thickening of the GBM with effacement of proteinuria remits.
foot processes and subendothelial deposits
592 A 16-year old female complains of a midline This is a congenital anomaly of the thyroid. A TIMOTHY TANG LEE MIDTERM 1
anterior neck mass. The mass was 2 cm in persistent sinus tract may remain as a vestigial SAY, MD (TOP 4 - EXAM - FEB
diameter, soft, painless and moves when tongue remnant of the tubular development of the thyroid AUG 2013 MED 2014
is protruded. Excision of the mass was done gland. Parts of this tube may be obliterated, leaving BOARDS; TOPNOTCH
and histology showed ducts and cysts lined by segments to form cysts. Segments of the duct and cysts MD)
stratified squamous epithelium with adjacent that occur in the neck are covered by stratified
lymphocytic infiltrate. This suggest? squamous epithelium identical to the tongue in the
A. Brachial pouch cyst region of foramen cecum. Anomalies in the lower neck
B. Thyroglossal duct cyst area more proximal to the thyroid gland are lined by
C. Epidermal inclusion cyst epithelium resembling the thyroid acinar epithelium.
D. Thyroid cyst Adjacent to the lining epithelium are lymphocytic
E. Sebaceous cyst infiltrates.
593 A 50-year old female, menopause for 4 years Intermediate cells are responsive to progesterone and TIMOTHY TANG LEE MIDTERM 1
will have a cytohormonal maturity index of? superficial cells are responsive to estrogen. At the SAY, MD (TOP 4 - EXAM - FEB
A. Parabasal=0, Intermediate=60, Superficial onset of menopause, the low levels of estrogen and AUG 2013 MED 2014
Cells=40 progesterone will favor proliferation of parabasal cells BOARDS; TOPNOTCH
B. Parabasal=20, Intermediate=80, Superficial making it the most predominant cell type on Pap MD)
Cells=0 smear.
C. Parabasal=20, Intermediate=40, Superficial
Cells=40
D. Parabasal=80, Intermediate=10, Superficial
Cells=10
E. Parabasal=20, Intermediate=0, Superficial
Cells=80
594 A 23-year old medical student who had Fat necrosis refers to focal areas of fat destruction, TIMOTHY TANG LEE MIDTERM 1
overnight binge drinking suddenly developed typically resultingfrom release of activated pancreatic SAY, MD (TOP 4 - EXAM - FEB
boring abdominal pain relieved by doubling up. lipases into the substance of the pancreas and AUG 2013 MED 2014
Serum amylase and lipase were elevated. What peritoneal cavity. On histologic examination, the BOARDS; TOPNOTCH
type of necrosis occurs when pancreatic lipases necrosis takes the form of foci of shadowy outlines of MD)
were released in the substance of the pancreas necrotic fat cells, with basophilic calcium deposits,
and peritoneal cavity? surrounded by an inflammatory reaction.
A. Liquefactive necrosis
B. Coagulative necrosis
C. Caseous necrosis
D. Fat necrosis
E. Gangrenous necrosis
595 A 30-year old male had recurrent diarrhea, Crohn disease is characterized by (1) sharply TIMOTHY TANG LEE MIDTERM 1
crampy abdominal pain, and fever for 3 months. delimited and typically transmural involvement of the SAY, MD (TOP 4 - EXAM - FEB
Colonoscopy done showed areas of long bowel by an inflammatory process with mucosal AUG 2013 MED 2014
serpentine linear ulcers oriented along the axis damage, (2) the presence of noncaseating granulomas BOARDS; TOPNOTCH
of the bowel with surrounding mucosal sparing. in 40% to 60% of cases, and (3) fissuring with MD)
Biopsy of the lesion will show? formation of fistulae.
A. Mucosal ulceration and mononuclear
inflammatory infiltrates in lamina propia
B. Transmural involvement of the bowel by
inflammatory process with mucosal damage,
non-casseating granuloma and fissuring with
formation of fistula
C. Submucosal fibrin and mucosal
architectural disarray
D. Inflammatory infiltrates predominantly
lymphocytic without granuloma formation
E. Submucosal polyps with underlying
plasmacytic infiltrates

TOPNOTCH MEDICAL BOARD PREP PATHOLOGY SUPEREXAM Page 82 of 99


For inquiries visit www.topnotchboardprep.com.ph or email us at topnotchmedicalboardprep@gmail.com
TOPNOTCH MEDICAL BOARD PREP PATHOLOGY SUPEREXAM
For inquiries visit www.topnotchboardprep.com.ph or email us at topnotchmedicalboardprep@gmail.com
Item QUESTION EXPLANATION AUTHOR TOPNOTCH
# EXAM
596 A 76-year old male consulted due to urinary Outer/peripheral glands TIMOTHY TANG LEE MIDTERM 1
frequency, dribbling, and sensation of Prostate cancers usually occur in the outer/peripheral SAY, MD (TOP 4 - EXAM - FEB
incomplete voiding. Ultrasound of kidney, glands while benign lesions usually occur in the inner AUG 2013 MED 2014
ureter, bladder and prostate showed a and transitional zones. BOARDS; TOPNOTCH
nodularly enlarged prostate with MD)
microcalcifications. PSA was 20 ng/mL.
Prostatectomy was done and histolopath
showed prostatic adenocarcinoma. Prostatic
cancers occurs mostly in what area/region of
the prostate?
A. Inner/central glands
B. Median lobe
C. Outer/peropheral glands
D. Transitional zone
E. Peri-urethral zone
597 A 37-year-old woman presents with the acute Classically the onset of bacterial pneumonia is sudden, TIMOTHY TANG LEE MIDTERM 1
onset of a productive cough, fever, chills, and with malaise, shaking chills, fever, peripheral SAY, MD (TOP 4 - EXAM - FEB
pleuritic chest pain. CBC showed leukocytosis leukocytosis, and a cough with sputum production. AUG 2013 MED 2014
with predominance of neutrophils. A chest x- Bacterial infections generally result in a BOARDS; TOPNOTCH
ray reveals consolidation of the entire lower polymorphonuclear (neutrophil) response. Bacterial MD)
lobe of her right lung. Which of the following is infection of the lung (pneumonia) results in
the most likely diagnosis? consolidation of the lung, which may be patchy or
A. Bronchiectasis diffuse. Patchy consolidation of the lung is seen in
B. Bronchopneumonia bronchopneumonia (lobular pneumonia), while
C. Interstitial pneumonitis diffuse involvement of an entire lobe is seen in lobar
D. Lobar pneumonia pneumonia.
E. Adult respiratory distress syndrome
598 A 54-year-old female consults due to epigastric Gastritis is a nonspecific term that describes any TIMOTHY TANG LEE MIDTERM 1
pain. History revealed she was taking inflammation of the gastric mucosa. Acute gastritis SAY, MD (TOP 4 - EXAM - FEB
mefenamic acid for joint pains. Gastroscopy refers to the clinical situation of gastric mucosal AUG 2013 MED 2014
revealed multiple, hemorrhagic ulcers <1 cm in erosions (not mucosal ulcers). Acute gastritis is also BOARDS; TOPNOTCH
diameter scattered throughout the stomach. known as hemorrhagic gastritis or acute erosive MD)
Mucosal folds were normal. Biopsy of the gastritis. Acute gastritis is associated with the use of
lesions reveal sharply demarcated ulcers with nonsteroidal anti-inflammatory drugs, such as aspirin,
normal adjacent mucosa. Mucosal erosions ibuprofen, and corticosteroids, and also with alcohol,
showed edema and hemorrhage. Which of the chemotherapy, ischemia, shock, and even severe
following is the most likely diagnosis? stress. Grossly acute gastritis appears as multiple,
A. Active chronic gastritis scattered, punctate (less than 1 cm) hemorrhagic
B. Active gastritis areas in the gastric mucosa. This is helpful in
C. Chronic gastritis differentiating acute gastritis from peptic ulcers,
D. Peptic ulcer disease which tend to be solitary and larger. Microscopically
E. H. pylori gastropathy the gastric mucosa from a patient with acute gastritis
is likely to reveal mucosal erosions, scattered
neutrophils, edema, and possibly hemorrhage.
599 20-year-old male consults due to red-brown When hematuria follows within 2 days of the onset of TIMOTHY TANG LEE MIDTERM 1
urine noted 2 weeks after an upper respiratory an upper respiratory infection without skin lesions in SAY, MD (TOP 4 - EXAM - FEB
tract infection. Urinalysis showed RBC of a young patient, IgA nephropathy (Bergers disease) AUG 2013 MED 2014
50/hpf and red cell casts. CBC, BUN, creatinine, should be considered. The characteristic BOARDS; TOPNOTCH
glucose, antinuclear antibodies (ANAs), and immunoflorescent picture is mesangila deposition of MD)
serum complement levels (C3 and C4) are IgA often with C3 and properdin with lesser amounts
within normal limits. Renal biopsy showed of IgG or IgM.
mesangial proliferation and matrix increase in
light microscopy. Immunofluorescence reveals
the presence of large, irregular deposits of
IgA/C3 in the mesangium. A linear staining
pattern is not found. Which of the following is
the most likely diagnosis?
A. Berger's disease
B. Focal segmental glomerulosclerosis
C. Goodpasture's syndrome
D. Membranoproliferative glomeulonephritis
E. Post-streptococcal glomerulonephritis
600 A 21-year-old male presents with a testicular Germ cell tumors of the testis often secrete enzymes TIMOTHY TANG LEE MIDTERM 1
mass. Surgical resection of the mass was done or polypeptide hormones, examples of which include SAY, MD (TOP 4 - EXAM - FEB
and microscopic examination of the mass -fetoprotein (AFP) and human chorionic AUG 2013 MED 2014
showed lace-like network of medium-sized gonadotropin (hCG). AFP is synthesized by the fetal BOARDS; TOPNOTCH
cuboidal cells and Schiller-Duval bodies. Which gut, liver, and yolk sac. It may be secreted by either MD)
of the following substances is most likely to be yolk sac tumors (endodermal sinus tumors) or
increased in this patients serum as a result of embryonal carcinomas. AFP may also be secreted by
being secreted from the cells of this tumor? liver cell carcinomas.
A. Acid phosphatase
B. Alpha-fetoprotein
C. Alkaline phosphatase
D. Human chorionic gonadotropin
E. Lactate dehydrogenase
601 The following functions can be attributed to FGFs have the following functions: RACHELLE FINAL EXAM -
fibroblast growth factors, EXCEPT: 1.New blood vessel formation (angiogenesis) MENDOZA, MD (TOP FEB 2013
A. Angiogenesis 2.Wound repair: FGFs participate in macrophage, 9 - AUG 2012 MED
B. Platelet aggregation fibroblast, and endothelial cell migration in damaged BOARDS; TOPNOTCH
C. Skeletal muscle development tissues and migration of epithelium to form new MD)
D. Hematopoiesis epidermis.
E. Lung maturation 3.Development: FGFs play a role in skeletal muscle
development and in lung maturation.
4.Hematopoiesis

TOPNOTCH MEDICAL BOARD PREP PATHOLOGY SUPEREXAM Page 83 of 99


For inquiries visit www.topnotchboardprep.com.ph or email us at topnotchmedicalboardprep@gmail.com
TOPNOTCH MEDICAL BOARD PREP PATHOLOGY SUPEREXAM
For inquiries visit www.topnotchboardprep.com.ph or email us at topnotchmedicalboardprep@gmail.com
Item QUESTION EXPLANATION AUTHOR TOPNOTCH
# EXAM
602 A 55-year old male, hypertensive, had his first Myocardial infarction would result to coagulative RACHELLE FINAL EXAM -
attack of myocardial infarction. The type of necrosis. Acute parenchymal inflammation will ensue MENDOZA, MD (TOP FEB 2013
tissue repair that will subequently occur in the along with tissue framework damage. Since 9 - AUG 2012 MED
affected areas of this patient's myocardium myocardial cells are permanent cells, no regeneration BOARDS; TOPNOTCH
should be: would occur. Healing will involve scar formation. MD)
A. Healing with scar formation Fibrosis occurs in chronic inflammatory diseases, such
B. Restitution of normal tissue as cirrhosis, pulmonary fibrosis and chronic
C. Fibrosis pancreatitis.
D. A and C
E. All of the above
603 Which of the following is NOT true regarding Seen in upward of 5% of the population, this RACHELLE FINAL EXAM -
heparin-induced thrombocytopenia? syndrome occurs when administration of MENDOZA, MD (TOP FEB 2013
A. This syndrome occurs after administration of unfractionated heparin (for purposes of 9 - AUG 2012 MED
unfractionated heparin (for purposes of therapeutic anticoagulation) induces formation of BOARDS; TOPNOTCH
therapeutic anticoagulation) antibodies that bind to molecular complexes of MD)
B. There will be induced formation of heparin and platelet factor 4 membrane protein. This
antibodies that bind to molecular complexes of antibody can also bind to similar
heparin and platelet factor 4 membrane complexes present on platelet and endothelial
protein. surfaces; the result is platelet activation, endothelial
C. The result is platelet activation, endothelial injury, and a prothrombotic state.
injury, and a prothrombotic state
D. A and C
E. None of the above
604 The following are true regarding x-linked All statements are correct regarding x-linked diseases RACHELLE FINAL EXAM -
genetic diseases, EXCEPT: MENDOZA, MD (TOP FEB 2013
A. An affected male does not transmit the 9 - AUG 2012 MED
disorder to his sons. BOARDS; TOPNOTCH
B. All daughters of an affected male are carriers. MD)
C. Sons of heterozygous women have 50%
chance of receiving the mutant gene.
D. Heterozygous female usually does not
express the full phenotypic change because of
the paired normal allele.
E. None of the above
605 Membrane proteins that recognize a variety of The Toll-like receptors (TLRs) are membrane proteins RACHELLE FINAL EXAM -
microbe-derived molecules and stimulate that recognize a variety of microbe-derived molecules MENDOZA, MD (TOP FEB 2013
innate immune responses against the microbes: and stimulate innate immune responses against the 9 - AUG 2012 MED
A. Major histocompatibility complex microbes. The first protein BOARDS; TOPNOTCH
B. T-cell receptor (TCR) to be identified in this family was the Drosophila Toll MD)
C. CD 44 adhesion molecule protein, which is involved in establishing the dorsal-
D. Toll-like receptors ventral axis during embryogenesis of the fly, as well as
E. LPS-binding proteins mediating antimicrobial
responses
606 A bone marrow aspirate was obtained from a MM is characterized by a proliferation of malignant RACHELLE FINAL EXAM -
70-year old who complained of weakness, plasma cells and a subsequent overabundance of MENDOZA, MD (TOP FEB 2013
weight loss, recurrent infections and bone pain. monoclonal paraprotein (M protein). The 9 - AUG 2012 MED
Laboratory findings showed anemia, proliferation of plasma cells in MM may interfere with BOARDS; TOPNOTCH
proteinuria and an abnormal component in the normal production of blood cells, resulting in MD)
serum proteins. The bone marrow aspirate leukopenia, anemia, and thrombocytopenia. The
showed numerous plasma cells. The MOST aberrant antibodies that are produced lead to
probable diagnosis is: impaired humoral immunity, and patients have a high
A. Multiple myeloma prevalence of infection, especially with encapsulated
B. Monocytic leukemia organisms. The overproduction of these antibodies
C. NonHodgkin lymphoma may lead to hyperviscosity, amyloidosis, and renal
D. Myelomonocytic leukemia failure
607 A patient with new skin lesions is found to have Gastric carcinoma is associated with the following RACHELLE FINAL EXAM -
a gastric carcinoma. The following cutaneous cutaneous lesions: leser-trelat sign, acanthosis MENDOZA, MD (TOP FEB 2013
lesions are consistent with this situation, nigricans, seborrheic keratosis and erythema 9 - AUG 2012 MED
EXCEPT: nodosum. BOARDS; TOPNOTCH
A. Seborrheic keratosis MD)
B. Leser-Trelat sign
C. Acanthosis nigricans
D. Erythema nodosum
E. Pyogenic granuloma
608 Which of the following is/are NOT true Familial hypercholesterolemia is a "receptor disease" RACHELLE FINAL EXAM -
regarding Familial Hypercholesterolemia? that is the consequence of a mutation in the gene MENDOZA, MD (TOP FEB 2013
A. This is the consequence of a mutation in the encoding the receptor for low density lipoprotein 9 - AUG 2012 MED
gene encoding for the production of low density (LDL), which is involved in the BOARDS; TOPNOTCH
lipoprotein (LDL). transport and metabolism of cholesterol. As a MD)
B. There is a loss of feedback control and consequence of receptor abnormalities, there is a loss
elevated levels of cholesterol that induce of feedback control and elevated levels of cholesterol
premature atherosclerosis, leading to a greatly that induce premature
increased risk of myocardial infarction atherosclerosis, leading to a greatly increased risk of
C. Homozygotes, having a double dose of the myocardial infarction.
mutant gene, are much more severely affected
and may have fivefold to sixfold elevations in
plasma cholesterol levels.
D. Affected individuals develop skin xanthomas
and coronary, cerebral, and peripheral vascular
atherosclerosis at an early age.
E. None of the above

TOPNOTCH MEDICAL BOARD PREP PATHOLOGY SUPEREXAM Page 84 of 99


For inquiries visit www.topnotchboardprep.com.ph or email us at topnotchmedicalboardprep@gmail.com
TOPNOTCH MEDICAL BOARD PREP PATHOLOGY SUPEREXAM
For inquiries visit www.topnotchboardprep.com.ph or email us at topnotchmedicalboardprep@gmail.com
Item QUESTION EXPLANATION AUTHOR TOPNOTCH
# EXAM
609 A 53-year old male, who is a chronic IV drug Bridging fibrosis is the hallmark of chronic hepatitis. RACHELLE FINAL EXAM -
user and alcoholic, presented with scleral Chronic hepatitis B infection will present with ground- MENDOZA, MD (TOP FEB 2013
icterus, abdominal tenderness on RUQ and glass hepatocytes (hepatocytes with finely granular 9 - AUG 2012 MED
positive fluid wave. Biopsy of the liver was cytoplasm), while chronic hepatitis C infection will BOARDS; TOPNOTCH
done showing bridging fibrosis and hepatocytes show lymphoid aggregates, bile duct reactive changes MD)
which appear to have finely granular and steatosis.
cytoplasm. The most likely diagnosis for this
patient is:
A. Acute alcoholic hepatitis
B. Chronic hepatitis C infection
C. Chronic hepatitis B infection
D. Acute viral hepatitis
E. None of the above
610 A 55 year old male was diagnosed with COPD. RACHELLE FINAL EXAM -
Pulmonary function test would reveal: MENDOZA, MD (TOP FEB 2013
A. Increased FRC 9 - AUG 2012 MED
B. Decreased FRC BOARDS; TOPNOTCH
C. Increased FEV1 MD)
D. Increased FVC
E. Decreased Surfactant

611 Histologic pattern of carcinoma of the breast Invasive lobular carcinoma, also known as infiltrating RACHELLE FINAL EXAM -
characterized by higher incidence of lobular carcinoma, is a type of breast cancer that MENDOZA, MD (TOP FEB 2013
multicentricity and bilaterality, but not starts in a lobule and spreads to surrounding breast 9 - AUG 2012 MED
necessarily biologic aggression: tissue. BOARDS; TOPNOTCH
A. Mucinous carcinoma MD)
B. Invasive lobular carcinoma
C. Invasive ductal carcinoma
D. Medullary carcinoma
E. None of the above
612 An 18-year old male noted recurrent painless IgA nephropathy is the most common RACHELLE FINAL EXAM -
hematuria after having bloody diarrhea and glomerulonephritis, presents with recurrent gross or MENDOZA, MD (TOP FEB 2013
fever 2 weeks before. No other sign or symptom microscopic hematuria. 9 - AUG 2012 MED
was noted. The most likely diagnosis is: BOARDS; TOPNOTCH
A. IgA Nephropathy MD)
B. Minimal change disease
C. Membranoproliferative glomerulonephritis
D. Post-streptococcal glomerulonephritis
E. None of the above
613 A 66-year old female, nonsmoker, presented Bronchioalveolar CA is a variant of adenocarcinoma RACHELLE FINAL EXAM -
with chronic cough, weight loss, occasional that is not associated with smoking. It usually MENDOZA, MD (TOP FEB 2013
chest pain and dyspnea. Chest x-ray revealed a presents as a single nodule or consolidation at the 9 - AUG 2012 MED
single nodular density at the periphery of R periphery. Histologically, alveolar architecture BOARDS; TOPNOTCH
middle lobe along with pleural effusion, remains intact along the tumor growth. MD)
blunting the R costodiaphragmatic recess. The
most likely diagnosis is:
A. Large cell lung CA
B. Small cell lung CA
C. Bronchioalveolar CA
D. Adenocarcinoma
E. Tuberculoma
614 In lung carcinoma, distant metastasis is most Adrenals is the most common organ of distant RACHELLE FINAL EXAM -
commonly found in: metastasis of a lung carcinoma (50% of the time). MENDOZA, MD (TOP FEB 2013
A. Liver 9 - AUG 2012 MED
B. Brain BOARDS; TOPNOTCH
C. Bone MD)
D. Adrenals
E. Kidney

615 Cardiac condition characterized by small sterile Marantic or nonbacterial thrombotic endocarditis is RACHELLE FINAL EXAM -
vegetations on valve leaflets, usually associated more commonly found in patients with mucinous MENDOZA, MD (TOP FEB 2013
with Systemic Lupus Erythematosus (SLE): adenoCA of the pancreas. It also presents with setrile, 9 - AUG 2012 MED
A. Marantic endocarditis nondestructive vegetations. Infective endocarditis is BOARDS; TOPNOTCH
B. Libman-Sacks disease usually due to bacteria, and vegetations are usually MD)
C. Infective endocarditis infected. Rheumatic heart disease is a chronic
D. Rheumatic heart disease complication of inadequately treated GABHS
E. Non-bacterial thrombotic endocarditis pharyngeal infection.

616 A 25-year old female came in for pre- ASD is the most common congenital heart defect in RACHELLE FINAL EXAM -
employment check up. Auscultation revealed a adults. It usually presents with fixed widely split S2, MENDOZA, MD (TOP FEB 2013
fixed widely split S2 and midsystolic murmur with midsystolic murmur along the upper border of 9 - AUG 2012 MED
along the upper parasternal border. She is the sternum. 90% of the time, ASD is of secundum BOARDS; TOPNOTCH
otherwise normal. The most likely diagnosis is: type. Primum type is usually associated with Down MD)
A. Atrial septal defect, sinus venosus type syndrome.
B. Atrial septal defect, primum type
C. Atrial septal defect, secundum type
D. Patent ductus arteriosus
E. Ventricular septal defect
617 The major risk factor/s for developing breast The major risk factors for the development of breast RACHELLE FINAL EXAM -
carcinoma is/are: cancer are hormonal and genetic (family history) MENDOZA, MD (TOP FEB 2013
A. Age 9 - AUG 2012 MED
B. Family history BOARDS; TOPNOTCH
C. Hormonal MD)
D. B and C
E. All of the above

TOPNOTCH MEDICAL BOARD PREP PATHOLOGY SUPEREXAM Page 85 of 99


For inquiries visit www.topnotchboardprep.com.ph or email us at topnotchmedicalboardprep@gmail.com
TOPNOTCH MEDICAL BOARD PREP PATHOLOGY SUPEREXAM
For inquiries visit www.topnotchboardprep.com.ph or email us at topnotchmedicalboardprep@gmail.com
Item QUESTION EXPLANATION AUTHOR TOPNOTCH
# EXAM
618 A 22-year old female was diagnosed with Cardiac manifestations are among the earliest and RACHELLE FINAL EXAM -
hyperthyroidism. Which of the following would most consistent features of hyperthyroidism MENDOZA, MD (TOP FEB 2013
be the earliest clinical manifestation that would 9 - AUG 2012 MED
be observed in this patient? BOARDS; TOPNOTCH
A. Tachycardia, palpitations MD)
B. Diarrhea
C. Proximal muscle weakness
D. Weight loss
E. Wide staring gaze and lid lag
619 The following statements characterize Alveolar epithelium, which contains a continuous RACHELLE FINAL EXAM -
Pneumocytes Type II, EXCEPT: layer of two principal cell types: flattened, platelike MENDOZA, MD (TOP FEB 2013
A. They are the source of pulmonary surfactant type I pneumocytes covering 95% of the alveolar 9 - AUG 2012 MED
B. They are the main cell type involved in the surface and rounded type II pneumocytes. Type II cells BOARDS; TOPNOTCH
repair of alveolar epithelium are important for at least two reasons: (1) They are MD)
C. They cover 95% of the alveolar epithelium the source of pulmonary surfactant, contained in
D. A and B osmiophilic lamellar bodies seen with electron
E. None of the above microscopy, and (2) they are the main cell type
involved in the repair of alveolar epithelium after
destruction of type I cells.
620 Patients who acquire nodular skin lesions Lepromatous Leprosy (LL) is found in those patients RACHELLE FINAL EXAM -
containing foamy macrophages that are filled with no or very little immune resistance to the MENDOZA, MD (TOP FEB 2013
with numerous acidfast bacilli could be Micobacterium leprae organism. When a biopsy of the 9 - AUG 2012 MED
diagnosed to have: affected LL skin is examined pathologically, it shows BOARDS; TOPNOTCH
A. Tuberculoid leprosy large collections of macrophages with foamy MD)
B. Borderline leprosy cytoplasm, called Foam Cells. Tuberculoid (TT)
C. Lepromatous leprosy leprosy - a Paucibacilliary type. Tuberculoid leprosy
D. Borderline tuberculoid leprosy pressure atrophy is due to a large collection of
lymphocytes and epithelioid cells, even in the
relatively early stages of the disease
621 A 32 year old female was noted to have a breast Sinus histiocytosis (also called reticular hyperplasia VON ANDRE DIAGNOSTIC
mass with axillary lymph node enlargement. refers to the distention and prominence of the MEDINA, MD (TOP 4 - EXAM - AUG
Further work ups, revealed that the mass was lymphatic sinusoids. This particular form of FEB 2012 MED 2012
cancerous. A diagnosis of invasive ductal hyperplasia is prominent in lymph nodes draining BOARDS; TOPNOTCH
carcinoma was made. Modified radical cancers such as carcinoma of the breast. (SIMILAR TO MD)
mastectomy was done with axillary lymph node PREVIOUS BOARD EXAM CONCEPT/PRINCIPLE)
dissection. Pathological examination of the
axillary lymph nodes will show?
A. Follicular Hyperplasia
B. Marginal zone B-cell hyperplasia
C. Sinus histiocytosis
D. Paracortical lymphoid hyperplasia
E. Myelodysplastic proliferation
622 A 32 year old male was admitted due to fatigue, This is a case of AML, the presence of a red-staining VON ANDRE DIAGNOSTIC
unexplained fever and spontaneous mucosal peroxidase-positive structure (AUER RODS) is specific MEDINA, MD (TOP 4 - EXAM - AUG
and cutaneous bleeding lasting for about 2 to AML (SIMILAR TO PREVIOUS BOARD EXAM FEB 2012 MED 2012
weeks. Laboratory work ups revelaed anemia, CONCEPT/PRINCIPLE) BOARDS; TOPNOTCH
neutropenia and thrombocytopenia. Peripheral MD)
blood smear examination revealed a red
staining peroxidase-positive structures with
abnormal azurophilic granules. What is the
possible diagnosis to this case?
A. Acute Lymphoblastic Leukemia
B. Acute Myelogenous Leukemia
C. Chronic Lymphoblastic Leukemia
D. Hodgkin's Lymphoma
E. Non-Hodgkin's Lymphoma
623 A presence of necrosis with nuclear changes Karyolysis- basophilia of the chromatin which fades or VON ANDRE DIAGNOSTIC
involving basophilia of the chromatin which dissolves (SIMILAR TO PREVIOUS BOARD EXAM MEDINA, MD (TOP 4 - EXAM - AUG
later on fades or dissolves is called? CONCEPT/PRINCIPLE) FEB 2012 MED 2012
A. Karyolysis BOARDS; TOPNOTCH
B. Pyknosis MD)
C. Karyorrhexis
D. Histiocytosis
E. Apotosis

624 A 58 year old female presented to the CHF cause increased hydrostatic pressure lcausing VON ANDRE DIAGNOSTIC
emergency department due to severe chest edema in this patient. MEDINA, MD (TOP 4 - EXAM - AUG
pain and diaphoresis. CK MB was elevated and FEB 2012 MED 2012
Trop I was positive. ST segment elevation was BOARDS; TOPNOTCH
noted in the ECG. On the 3rd hospital day, MD)
patient went into CHF accompanied by bilateral
Grade 2 edema. What is the most probable
etiology of the edema of this patient?
A. Reduced oncotic pressure
B. Increased hydrostatic pressure
C. Sodium retention
D. Inflammation
E. Lymphatic obstruction

TOPNOTCH MEDICAL BOARD PREP PATHOLOGY SUPEREXAM Page 86 of 99


For inquiries visit www.topnotchboardprep.com.ph or email us at topnotchmedicalboardprep@gmail.com
TOPNOTCH MEDICAL BOARD PREP PATHOLOGY SUPEREXAM
For inquiries visit www.topnotchboardprep.com.ph or email us at topnotchmedicalboardprep@gmail.com
Item QUESTION EXPLANATION AUTHOR TOPNOTCH
# EXAM
625 A 64 year old female was a known case of DM DM renal morphology can be
VON ANDRE DIAGNOSTIC
Type II. Futher work ups upon admission > Capillary basement membrane thickening MEDINA, MD (TOP 4 - EXAM - AUG
revealed elevated Creatinine and BUN. What > Diffuse mesanglial sclerosis FEB 2012 MED 2012
is/are the expected morphology, if her kidney > Nodular Glomerulosclerosis BOARDS; TOPNOTCH
will be examined pathologically? >Presence of laminated PAS-positive nodules ( MD)
A. Capillary basement membrane thickening Kimmelsteil-Wilson Nodules)
B. Diffuse mesanglial sclerosis
C. Nodular Glomerulosclerosis
D. Presence of laminated PAS-positive nodules
( Kimmelsteil-Wilson Nodules)
E. All of the above
626 A 59 year old man is hospitalized after suffering This patient is in shock due to cardiac tamponade VON ANDRE DIAGNOSTIC
a severe myocardial infarction. He is initially secondary to rupture of the ventricular wall, which MEDINA, MD (TOP 4 - EXAM - AUG
treated with nitrates, B blockers, and ASA and usually occurs 4-10 days after the initial M.i
FEB 2012 MED 2012
subsequently undergoues cardiac fibrinous pericarditis is common in 3-5 days, this BOARDS; TOPNOTCH
catheterization with placement of two stents. condition usually presents with chest pain, often a MD)
Following the procedure he is pericardial friction rub can be auscultated.
hemodynamically stable without recurrence of
chest pain. However, 6 days after admission his
heart rate is 134/min, BP is 70/30 mmHg and
RR is 29 cpm. PE reveals distant heart sounds
and an elevated jugular venous pressure. Which
of the following complications is most likely
causing this patient's symptoms?
A. aneurysm formation
B. cardiac arrhythmia
C. fibrinous pericarditis
D. rupture of the papillary muscle
E. rupture of the ventriclar free wall
627 A 5 year old female presents with a new onset PSGN- deposits between basement membrane and VON ANDRE DIAGNOSTIC
hematuria and oliguria after 1 week of epithelial cells of the glomeruli MEDINA, MD (TOP 4 - EXAM - AUG
experiencing sore throat. Other pertinent FEB 2012 MED 2012
findings revealed hypertension, periorbital BOARDS; TOPNOTCH
edema and impaired renal function. A renal MD)
biopsy most likely would reveal electron-dense
deposits in which of the following sites?
A. between basement membrane and
endothelial cells of the glomeruli
B. between the basement membrane and
epithelial cells of the glomeruli
C. between the basement membrane and
epithelial cells of the proximal tubules
D. Within the mesangium of the glomeruli
E. Within the juxtaglomerular apparatus
628 Which of the following colorectal polyps has the VON ANDRE DIAGNOSTIC
highest risk of malignant degeneration? MEDINA, MD (TOP 4 - EXAM - AUG
A. Villous adenoma FEB 2012 MED 2012
B. Tubular adenoma BOARDS; TOPNOTCH
C. Tubulovillous adenoma MD)
D. Hyperplastic polyp
E. None of the above

629 For the past year, a 48 year old woman has Leiomyosarcoma VON ANDRE DIAGNOSTIC
noted that her menstrual periods have been MEDINA, MD (TOP 4 - EXAM - AUG
exceptionally heavy and last 7 - 9 days. She has FEB 2012 MED 2012
noted occasional minor intermenstrual BOARDS; TOPNOTCH
bleeding. For the past 3 months, she has been MD)
taking supplemental dietary iron for IDA. On
pelvic examination, the uterine cervix appears
normal, but the uterus is enlarged to twice its
normal size. Transvaginal ultrasound reveals
the presence of a 9 cm solid mass in the uterus.
A hysterectomy is performed, and on gross
inspection with sectioning the uterus, a
reddish-tan mass is found with fleshy cut
surface. Microscopically, the mass is highly
cellular, with spindle cells having
hyperchromatic nuclei and 10-20 mitoses per
high power field. Which of the following is the
most likely diagnosis?
A. Endometrial polyp
B. Adenomyosis
C. Atypical hyperplasia
D. Leiomyosarcoma
E. Endometriosis

TOPNOTCH MEDICAL BOARD PREP PATHOLOGY SUPEREXAM Page 87 of 99


For inquiries visit www.topnotchboardprep.com.ph or email us at topnotchmedicalboardprep@gmail.com
TOPNOTCH MEDICAL BOARD PREP PATHOLOGY SUPEREXAM
For inquiries visit www.topnotchboardprep.com.ph or email us at topnotchmedicalboardprep@gmail.com
Item QUESTION EXPLANATION AUTHOR TOPNOTCH
# EXAM
630 A 40 year old male suddenly experienced Hyperplastic arteriolosclerosis VON ANDRE DIAGNOSTIC
severe headache. Upon consultation, his BP is MEDINA, MD (TOP 4 - EXAM - AUG
noted to be at 200/120. Past history shows FEB 2012 MED 2012
normal BP on regular checkups. Anti BOARDS; TOPNOTCH
hypertensive are immediately given. However, MD)
BP remains in the range of 200/120-180/100.
Nicardipine drip is instituted. Two days later,
the patient died becasue of intracerebral
hemorrhage. Biopsy of the kidney revealed
onion skin, concentric, laminated thickening of
the walls of arterioles with progressive
narrowing of the lumina consistent with what
kind of arteriolosclerosis?
A. Hyaline arteriolosclerosis
B. Hyperplastic arteriolosclerosis
C. Focal segmental nephrosclerosis
D. RPGN
E. Benign nephrosclerosis
631 A patient with hemoptysis and renal failure has Goodpasture's syndrome VON ANDRE DIAGNOSTIC
a renal biopsy which reveals crescentic MEDINA, MD (TOP 4 - EXAM - AUG
glomerulonephritis, with linear deposits of IgG FEB 2012 MED 2012
and C3 on immunofluorescence. What is the BOARDS; TOPNOTCH
probable diagnosis? MD)

A. Acute post-infectious glomerulonephritis
B. membranous nephropathy
C. Goodpastures syndrome
D. Minimal change disease
E. Wegener's granulomatosis
632 A 1 year old patient came in due to fever. On This is a case of trisomy 21, MC is nondisjunction in 95 VON ANDRE DIAGNOSTIC
physical examination , you noticed that the % of cases. MEDINA, MD (TOP 4 - EXAM - AUG
patient has epicanthal folds, flat facial profile, FEB 2012 MED 2012
macroglossia and presence of simian crease. BOARDS; TOPNOTCH
Further history revealed that the patient was MD)
born from a 48 year old mother. What is the
most common cause of this chromosal
disorder?
A. Nondisjunction
B. Robertsonian translocation
C. Mosaicism
D. Chromosomal deletion
E. None of the above
633 A 5 year old male developed a demyelinating This is a case of Guillan Barre Syndrome, a type IV VON ANDRE DIAGNOSTIC
neuropathy associated with Campylobacter Hypersensitivity reaction MEDINA, MD (TOP 4 - EXAM - AUG
jejuni. He presents clinically with ascending FEB 2012 MED 2012
neuromuscular paralysis and areflexia. He was BOARDS; TOPNOTCH
then treated with plasmapharesis, IV MD)
immunoglobulins and corticosteroids. The
diagnosis to this case belongs to what type of
hypersensitivity?
A. Type I Hypersensitivity
B. Type II Cytotoxic Hypersensitivity
C. Type III Immune Complex
Hypersensitivity
D. Type IV Cell mediated Hypersensitivity
E. Any of the above choices
634 What is the most common and the most severe Type IV- Diffuse proliferative VON ANDRE DIAGNOSTIC
form of LUPUS NEPHRITIS ? MEDINA, MD (TOP 4 - EXAM - AUG
A. Minimal mesanglial FEB 2012 MED 2012
B. Mesanglial proliferative BOARDS; TOPNOTCH
C. Focal proliferative MD)
D. Diffuse proliferative
E. Membranous

635 A 23 year old female seek consult due to Phagocyte disorder= Chediak Higashi Syndrome VON ANDRE DIAGNOSTIC
recurrent pyogenic infections caused mainly by MEDINA, MD (TOP 4 - EXAM - AUG
staphylococci and streptoccoci. Further work FEB 2012 MED 2012
up revealed that she is suffering from an BOARDS; TOPNOTCH
autosomal recessive disease characterized by a MD)
failure of phagolysosomal fusion. Moreover,
faulty microtubules was found to impair
neutrophil chemotaxis. What is the most likely
diagnosis?
A. Leukocyte Adhesion deficiency
B. Chronic Granulomatous Disease
C. Chediak-Higashi Syndrome
D. Wiskott-Aldrich Syndrome
E. Ataxia-Telangiectasia
636 Which of the following disease-malignancy All of the disease-malignancy association are correct VON ANDRE DIAGNOSTIC
association is INCORRECT? MEDINA, MD (TOP 4 - EXAM - AUG
A. Down syndrome: ALL, AML FEB 2012 MED 2012
B. Actinic keratosis: squamous cell cancer of BOARDS; TOPNOTCH
the skin MD)
C. Acanthosis nigricans: visceral malignancy
D. Dysplastic nevus: malignant melanoma
E. None of the above
TOPNOTCH MEDICAL BOARD PREP PATHOLOGY SUPEREXAM Page 88 of 99
For inquiries visit www.topnotchboardprep.com.ph or email us at topnotchmedicalboardprep@gmail.com
TOPNOTCH MEDICAL BOARD PREP PATHOLOGY SUPEREXAM
For inquiries visit www.topnotchboardprep.com.ph or email us at topnotchmedicalboardprep@gmail.com
Item QUESTION EXPLANATION AUTHOR TOPNOTCH
# EXAM
637 Cachexia is an irreversible catabolic reaction increased TNF alpha is associated with cachexia which VON ANDRE DIAGNOSTIC
(wasting syndrome) common to those suppresses the appetite center and promotes beta MEDINA, MD (TOP 4 - EXAM - AUG
individual suffering from malignancies and oxidation of fatty acids for fuel FEB 2012 MED 2012
tuberculosis. Which of the following substances BOARDS; TOPNOTCH
is correlated to the pathophysiology of MD)
cachexia?
A. elevated TNF alpha
B. elevated VEGF
C. elevated IL6
D. elevated Il 3
E. none of the above
638 Beta HCG is a tumor marker for? VON ANDRE DIAGNOSTIC
A. Hydatidiform mole MEDINA, MD (TOP 4 - EXAM - AUG
B. Choriocarcinoma FEB 2012 MED 2012
C. Gestational trophoblastic tumor BOARDS; TOPNOTCH
D. All of the above MD)
E. None of the above

639 A 65 year old male immigrant from Africa diagnosis: squamous cell carcinoma of the bladder, MC VON ANDRE DIAGNOSTIC
presents to the emergency department after an cause is chronic irritation of Schistosoma MEDINA, MD (TOP 4 - EXAM - AUG
episode of gross hematuria. He states that he haematobium. FEB 2012 MED 2012
has seen small amounts of blood in his urine BOARDS; TOPNOTCH
from time to time over the past several months. MD)
His physical examination is remarkable only for
mild hepatosplenomegaly. A urology consult is
called, and the urologist performs a bedside
cytoscopy. A large fungating mass is seen
adherent to the superior part of the bladder.
What is the most likely environmental exposure
associated with this disease in the patient?
A. Cigarette smoking
B. Exposure to aniline dyes
C. Schistosoma infection
D. Infection with papova virus
E. Long term indwelling catheter
640 Glanzmann thrombasthenia is a deficiency of? GpIIb-IIIa complex- Glanzmann GpIb- bernard VON ANDRE DIAGNOSTIC
A. GpIIb-IIIa complex soulier MEDINA, MD (TOP 4 - EXAM - AUG
B. GpIb FEB 2012 MED 2012
C. Factor IX BOARDS; TOPNOTCH
D. Fibrinogen MD)
E. vWF

641 What do you call the systemic reaction Arthus reaction is almost the same as serum sickness LITO JAY MACARAIG, DIAGNOSTIC
including rashes, brochoconstriction and but occurs as a local inflammatory reaction. MD (TOP 8 - FEB EXAM - AUG
periorbital edema after injecting an unknown Anaphylactic shock is re-exposure to a known 2013 MED BOARDS; 2013
antigen to different parts of your body? allergen, leading to severe vasodilatation and TOPNOTCH MD)
A. Arthus Reaction hemodynamic instability.
B. Serum Sickness
C. Delayed hypersensitivity reaction
D. Anaphylactic Shock
E. None of the above
642 Which among the following components of the CD8 or the cytotoxic cells are not included in the LITO JAY MACARAIG, DIAGNOSTIC
immune system is NOT included in the pathophysiology of HIV infection. It is the CD4 that is MD (TOP 8 - FEB EXAM - AUG
pathophysiology of HIV infection? affected by HIV. 2013 MED BOARDS; 2013
A. CD4 TOPNOTCH MD)
B. CD8
C. Dendritic cells
D. Macrophages
E. No excemption, all of the above are
included in the pathophysiology of HIV
infection
643 Which of the following imunoglobulins is/are As stated in Robbins, the immune system of the GIT LITO JAY MACARAIG, DIAGNOSTIC
being secreted by the immune system of the secretes dimeric IgA, IgG and IgM. MD (TOP 8 - FEB EXAM - AUG
GIT? 2013 MED BOARDS; 2013
A. Dimeric IgA TOPNOTCH MD)
B. IgG
C. IgM
D. B and C only
E. All of the above

644 While on duty as IM resident, a 28 year old this is Hemolytic-Uremic Syndrome. It is caused by LITO JAY MACARAIG, DIAGNOSTIC
female came to ER who presented with th triad shiga-like toxin from E. coli 0157:H7 from poorly MD (TOP 8 - FEB EXAM - AUG
of microangiopathic anemia, thrombocytopenia cooked burger patties. 2013 MED BOARDS; 2013
and renal failure. Further history revealed TOPNOTCH MD)
eating half-cooked burgers 2 days PTC. Which
among the choices is/are true regarding the
etiologic agent?
A. Shiga-like toxin
B. EHEC
C. E. coli 0157:H7
D. B and C only
E. All of the above

TOPNOTCH MEDICAL BOARD PREP PATHOLOGY SUPEREXAM Page 89 of 99


For inquiries visit www.topnotchboardprep.com.ph or email us at topnotchmedicalboardprep@gmail.com
TOPNOTCH MEDICAL BOARD PREP PATHOLOGY SUPEREXAM
For inquiries visit www.topnotchboardprep.com.ph or email us at topnotchmedicalboardprep@gmail.com
Item QUESTION EXPLANATION AUTHOR TOPNOTCH
# EXAM
645 A 47 year old male consulted your clinic due to the scattered macrophages with abundant wrinkled, LITO JAY MACARAIG, DIAGNOSTIC
unexplained fever accompanied by weakness. green--blue cytoplasm is the "sea-blue histiocytes" MD (TOP 8 - FEB EXAM - AUG
CBC revealed leukocytosis of 187,000/mm3. which is the pathognomonic finding for Chronic 2013 MED BOARDS; 2013
Suspecting leukemia, you requested bone Myelogenous Leukemia TOPNOTCH MD)
marrow biopsy which revealed scattered
macrophages with abundant wrinkled, green -
blue cytoplasm. What is the diagnosis?
A. AML
B. CML
C. ALL
D. CLL
E. Acute Promyelocytic Leukemia
646 During your patho rotation, the 3rd year the description given points out to the REED- LITO JAY MACARAIG, DIAGNOSTIC
resident showed you a slide containing large STERNBERG cells which are the pathognomonic for MD (TOP 8 - FEB EXAM - AUG
cells with multiple nuclei, some have single Hodgkin's lymphoma 2013 MED BOARDS; 2013
nucleus with multiple nuclear lobes (each half TOPNOTCH MD)
is a mirror image of the other). With this very
limited information, it is safe to say that the
patient whom this specimen belongs to is
suffering from?
A. Chronic Lymphocytic Leukemia
B. Hairy Cell Leukemia
C. Non-hodgkin's lymphoma
D. Hodgkin's lymphoma
E. Leukemoid reaction
647 Which among the following statements is/are topnotch notes LITO JAY MACARAIG, DIAGNOSTIC
true regarding thymoma? MD (TOP 8 - FEB EXAM - AUG
A. Associated with Pure Red Cell aplasia 2013 MED BOARDS; 2013
B. Associated with Myasthenia Gravis TOPNOTCH MD)
C. Histopath findings may show swirling
pattern of sphindle-shaped cells
D. B and C only
E. All of the above
648 This is the most common complication of Cholesteatomas are mainly deriveds from LITO JAY MACARAIG, DIAGNOSTIC
Chronic Otitis Media, characterized by cystic desquamated epithelium resulted from chronic otitis MD (TOP 8 - FEB EXAM - AUG
lesions filled with amorphous debris. media. 2013 MED BOARDS; 2013
A. Chloroma TOPNOTCH MD)
B. Branchial pouch cyst
C. Cholesteatoma
D. A and C only
E. None of the above

649 A 54 year old female presented with painless this is a case of Traumatic Fat necrosis. Usual clinical LITO JAY MACARAIG, DIAGNOSTIC
palpable mass on upper-outer quadrant of her picture is a patient with history of trauma or prior MD (TOP 8 - FEB EXAM - AUG
left breast. History revealed previous surgical surgery who presented with painless palpable mass. 2013 MED BOARDS; 2013
removal of a mass (2 months PTC) of which Choices B and C pertains to histopath findings of this TOPNOTCH MD)
histopath showed fibroadenoma. If you are condition.
about to request a biopsy of the new mass,
what findings do you expect?
A. localized area of inflammation which may
progress to abscess formation
B. ill-defined, firm, gray-white nodules
C. chalky-white foci with or without
hemorrhagic debris
D. A and B only
E. B and C only
650 This is the most common type of breast cancer About 70% of diagnosed breast cancers are invasive LITO JAY MACARAIG, DIAGNOSTIC
with characteristic grating sound (like cutting a ductal CA's. The grating sound is due to small, central MD (TOP 8 - FEB EXAM - AUG
water chestnut) when cut. pinpoint focior streaks of chalky-white elastotic 2013 MED BOARDS; 2013
A. DCIS stroma TOPNOTCH MD)
B. LCIS
C. Invasive ductal CA
D. Invasive lobular CA
E. Medullary CA

651 Psoriasis is a skin disease with predilection to All of the choices given pertainsto the series of LITO JAY MACARAIG, DIAGNOSTIC
elbows, knees, scalp, and lumbosacral areas. histopath findings can be found on psoriasis. MD (TOP 8 - FEB EXAM - AUG
Which among the following lesions is/are 2013 MED BOARDS; 2013
related to the said disease? TOPNOTCH MD)
A. Test tubes in rack appearance
B. Suprapapillary plates wuth dilated, tortuos
blood vessels
C. spongiform pustules of Kajog
D. A and C only
E. All of the above
652 A 12 year old male was brought to your clinic this is a typical description of Molluscum contagiosum. LITO JAY MACARAIG, DIAGNOSTIC
due to pruritic, pink to skin-colored papules Pathognomic finding is the central umbilication MD (TOP 8 - FEB EXAM - AUG
with central umbilication. What is your primary 2013 MED BOARDS; 2013
differential diagnosis? TOPNOTCH MD)
A. Acne vulgaris
B. Verrucae
C. Molluscum Contagiosum
D. Impetigo
E. Phemphigus vulgaris

TOPNOTCH MEDICAL BOARD PREP PATHOLOGY SUPEREXAM Page 90 of 99


For inquiries visit www.topnotchboardprep.com.ph or email us at topnotchmedicalboardprep@gmail.com
TOPNOTCH MEDICAL BOARD PREP PATHOLOGY SUPEREXAM
For inquiries visit www.topnotchboardprep.com.ph or email us at topnotchmedicalboardprep@gmail.com
Item QUESTION EXPLANATION AUTHOR TOPNOTCH
# EXAM
653 Retinoblastoma is the most common primary Homer Wright Pseudo rosettes - seen in LITO JAY MACARAIG, DIAGNOSTIC
intra-ocular malignancy of children. What do neuroblastoma, medulloblastoma. True rosettes are MD (TOP 8 - FEB EXAM - AUG
you call its pathognominic morphology which is the flexner wintersteiner rosettes. 2013 MED BOARDS; 2013
described as arrangements of a single layer of TOPNOTCH MD)
tumor cells around an apparent lumen?
A. Homer Wright Rosettes
B. Homer Wright Pseudorosettes
C. Flexner-Wintersteiner rosettes
D. Mutton-Fat precipitates
E. Hollenhorst plaques
654 This is a sequelae of chronic Escherichia coli the histopath findings described is called Michaelis- LITO JAY MACARAIG, DIAGNOSTIC
infection, characterized by histopathologic Guttman bodies which is the pathognomonic finding MD (TOP 8 - FEB EXAM - AUG
findings of large, foamy macrophages with for Malacoplakia 2013 MED BOARDS; 2013
laminatedmineralized concretions. TOPNOTCH MD)
A. Hemorrhagic cystitis
B. Interstitial cystitis
C. Malacoplakia
D. Papillary Urothelial CA
E. Papillary urothelial neoplasms of Low
malignant potential
655 A 16 month old boy was brought to your clinic Hypospadia is the condition where the meatus is LITO JAY MACARAIG, DIAGNOSTIC
due to history of 2 days fever. Being so busy, located ventrally, epispadia if dorsally. Balanitis is the MD (TOP 8 - FEB EXAM - AUG
you just requested urinalysis which showed inflamation of glans penis. 2013 MED BOARDS; 2013
PMN of 15-19 cells per HPF. You were told that TOPNOTCH MD)
the patient had 4 episodes of UTI for the past
year. That is the only time you decided to
examine his genitalia. You noticed that the
orifice of his penile prepuce is too small. What
is your diagnosis?
A. Hypospadia
B. Epispadia
C. Phimosis
D. Balanitis
E. Balanoposthitis
656 A 49 year old female consulted to your clinic On histopath, Yolk sac tumor will show lacelike LITO JAY MACARAIG, DIAGNOSTIC
due to progressively enlarging abdomen. Upon reticular cell network of structures resembling MD (TOP 8 - FEB EXAM - AUG
PE, you noted a non-painful palpable mass at endodermal sinuses called Schiller-Duval bodies. 2013 MED BOARDS; 2013
the left lower quadrant. Further studies showed TOPNOTCH MD)
elevated serum AFP, normal beta hCG and
inhibin levels. You referred the patient to an
obstetrician for operation. You are expecting to
find Schiller-Duval bodies on histopath because
you know this is a case of?
A. Leydig cell tumor
B. Sertoli cell tumor
C. Yolk sac tumor
D. Choriocarcinoma
E. Teratoma
657 Which of the following statements is/are true LITO JAY MACARAIG, DIAGNOSTIC
regarding Guillain-Barre Syndrome? MD (TOP 8 - FEB EXAM - AUG
A. Immune-mediated demyelinating 2013 MED BOARDS; 2013
neuropathy TOPNOTCH MD)
B. Associated with Campylobacter jejuni
infection
C. Ascending motor paralysis
D. A and C only
E. All of the above
658 This is a bone disease characterized by mosaic choices A and B were the same. OI is a collagen type I LITO JAY MACARAIG, DIAGNOSTIC
pattern or jigsaw puzzle appearance on problem. OP is the marble bone disease. MD (TOP 8 - FEB EXAM - AUG
microscopy. 2013 MED BOARDS; 2013
A. Osteitis Fibrosa Cystica TOPNOTCH MD)
B. Von Recklinghausen disease of the bone
C. Paget's disease
D. Osteogenesis imperfecta
E. Osteopetrosis

659 A 28 year-old construction worker fell from a 3- LITO JAY MACARAIG, DIAGNOSTIC
storey building and hit his head on the MD (TOP 8 - FEB EXAM - AUG
pavement. Patient came in conscious but after 2013 MED BOARDS; 2013
20 minutes, became stupurous and eventually TOPNOTCH MD)
GCS 3. You are entertaining Epidural
hematoma. Which among the following is/are
associated with Epidural hematoma?
A. fractured Pterion
B. Lucid interval
C. Lenticular lesion on CT scan
D. B and C only
E. All of the above
660 The triad of Graves disease includes? LITO JAY MACARAIG, DIAGNOSTIC
A. hyperthyroidism MD (TOP 8 - FEB EXAM - AUG
B. ophthalmopathy 2013 MED BOARDS; 2013
C. Pretibial myxedema TOPNOTCH MD)
D. A and B only
E. All of the above

TOPNOTCH MEDICAL BOARD PREP PATHOLOGY SUPEREXAM Page 91 of 99


For inquiries visit www.topnotchboardprep.com.ph or email us at topnotchmedicalboardprep@gmail.com
TOPNOTCH MEDICAL BOARD PREP PATHOLOGY SUPEREXAM
For inquiries visit www.topnotchboardprep.com.ph or email us at topnotchmedicalboardprep@gmail.com
Item QUESTION EXPLANATION AUTHOR TOPNOTCH
# EXAM
661 Type of cellular adaptation exhibited by genital Number of cells in organ increase, the process is HAZEL KAREN RAZ, MIDTERM 2 -
warts? reversible when stimulus stops MD (TOP 6 - FEB AUG 2013
A. hyperplasia 2013 MED BOARDS;
B. Hypertrophy TOPNOTCH MD)
C. metaplasia
D. dysplasia
E. None of the above

662 Identify the mechanism of injury from cellular Hypoxia causes injury by depleting ATP sources (no HAZEL KAREN RAZ, MIDTERM 2 -
hypoxia? ETC and Krebs cycle) and free radical injury MD (TOP 6 - FEB AUG 2013
A. ATP depletion 2013 MED BOARDS;
B. Free radical injury TOPNOTCH MD)
C. A & B
D. None
E. Hydrolysis

663 During the process of inflammation, laukocytes E-selection (rolling) HAZEL KAREN RAZ, MIDTERM 2 -
adhers to integrins found on endothelial PECAM (diapedesis) MD (TOP 6 - FEB AUG 2013
surfaces thru this athesion molecules? Sialyl lewis (rolling) 2013 MED BOARDS;
A. E-selection (rolling) TOPNOTCH MD)
B. PECAM (diapedesis)
C. Sialyl lewis (rolling)
D. ICAM - I
E. None of the above

664 True of the complement system? Consists of plasma proteins which participates in HAZEL KAREN RAZ, MIDTERM 2 -
A. Consists of plasma proteins which immune lysis of cells AND PLAYS a role in MD (TOP 6 - FEB AUG 2013
participates in immune lysis of cells but does inflammation 2013 MED BOARDS;
not play a role in inflammation C3A is chemotactic - (C5A) TOPNOTCH MD)
B. C3A is chemotactic (C5A) C5A is an anaphyiotoxin (C5A & C3A)
C. C5A is an anaphyiotoxin (C5A & C3A) C3B is part of the MAC (C5B-A)
D. C3B is part of the MAC (C5B-A)
E. None of the above

665 Observed in deep-sea divers who return to the Decompression sickness (DCS; also known as divers' HAZEL KAREN RAZ, MIDTERM 2 -
surface too rapidly, resulting in disease, the bends or caisson disease) describes a MD (TOP 6 - FEB AUG 2013
musculoskeleteal pain and infarcts in the CNS? condition arising from dissolved gases coming out of 2013 MED BOARDS;
A. Embolism Syndrome solution into bubbles inside the body on TOPNOTCH MD)
B. Decompression Sickness depressurisation.
C. Fluid Emboli
D. Paradoxical Emboli
E. None of the above

666 Newborn presents with a prominent acciput, Edwards syndrome (also known as Trisomy 18 (T18)) HAZEL KAREN RAZ, MIDTERM 2 -
micrognathia, low set ears and rodenbottom is a genetic disorder caused by the presence of all or MD (TOP 6 - FEB AUG 2013
feet genetic d/o caused by nondisjunction is part of an extra 18th chromosome. This genetic 2013 MED BOARDS;
called? condition almost always results from nondisjunction TOPNOTCH MD)
A. Down Syndrome during meiosis.
B. DiGeorge Syndrome
C. Edward's Syndrom
D. Patau Syndrome
E. None of the above
667 Type of hypersensitivity reaction manifested by Type I Anaphylaxis HAZEL KAREN RAZ, MIDTERM 2 -
a patient with weight loss, taxhycardia, Type II Cytotoxic - Graves Disease MD (TOP 6 - FEB AUG 2013
exophthalmos and elevated free thyroid Type III Immune complex 2013 MED BOARDS;
hormones? Type IV Delayed/Cell Mediated TOPNOTCH MD)
A. Type I Anaphylaxis
B. Type II Cytotoxic
C. Type III Immune complex
D. Type IV Delayed/Cell Mediated
E. None of the above
668 A patient whose mother was exposed to DES In 1971, DES was shown to cause a rare vaginal tumor HAZEL KAREN RAZ, MIDTERM 2 -
during pregnancy is prone to developing? in girls and women who had been exposed to this drug MD (TOP 6 - FEB AUG 2013
A. VSD in utero. 2013 MED BOARDS;
B. Limb Anomalies TOPNOTCH MD)
C. Acute Leukemia
D. Clear Cell CA of Vagina
E. None of the above

669 Which of the following is the most toxic Vitamin D is a group of fat- HAZEL KAREN RAZ, MIDTERM 2 -
vitamin? soluble secosteroids responsible for intestinal MD (TOP 6 - FEB AUG 2013
A. Vitamin A absorption of calcium and phosphate.The threshold 2013 MED BOARDS;
B. Vitamin D for vitamin D toxicity has not been established, TOPNOTCH MD)
C. Vitamin E however, the UL is 4000 IU/day for ages 971.
D. Vitamin K
E. None of the above

TOPNOTCH MEDICAL BOARD PREP PATHOLOGY SUPEREXAM Page 92 of 99


For inquiries visit www.topnotchboardprep.com.ph or email us at topnotchmedicalboardprep@gmail.com
TOPNOTCH MEDICAL BOARD PREP PATHOLOGY SUPEREXAM
For inquiries visit www.topnotchboardprep.com.ph or email us at topnotchmedicalboardprep@gmail.com
Item QUESTION EXPLANATION AUTHOR TOPNOTCH
# EXAM
670 A 5-Year old boy came to you with complaints HSP is a disease of the skin and other organs that most HAZEL KAREN RAZ, MIDTERM 2 -
of fever, headache and joint pains on physical commonly affects children. In the skin, the disease MD (TOP 6 - FEB AUG 2013
examination you note erythematons papules causes palpable purpura (small hemorrhages); often 2013 MED BOARDS;
and plaques appearing in crops lower abdomen with joint and abdominal pain. TOPNOTCH MD)
and buttocks. History revealed that he had
cough and colds two weeks prior to consult.
What is the diagnosis?
A. Polyartentis Nodosa
B. Buerqer Disease
C. Serum Sickness
D. Henoch-Schonlein Purpura
E. None of the above
671 A 40 year old male presented with crushing Arythmia - 24 hours HAZEL KAREN RAZ, MIDTERM 2 -
chest pain radiationg to his left arm, associated Myocardial Rupture - 4 - 7 days MD (TOP 6 - FEB AUG 2013
with dizziness and dyspnea. ECG was done at Ventricular Aneurysm - > 7 weeks 2013 MED BOARDS;
the ER showing chair-pattern tracing at chest Dressler Syndrome - pericarditis post - MI TOPNOTCH MD)
leods V2, V3 and aVf. What is the most common
complication 7 days after the incident?
A. Arythmia
B. Myocardial Rupture
C. Ventricular Aneurysm
D. Dressler Syndrome
E. None of the above
672 A 21-year old male who has a history of Aschoff bodies are nodules found in the hearts of HAZEL KAREN RAZ, MIDTERM 2 -
recurrent throat infection develops fever and individuals with rheumatic fever. They result from MD (TOP 6 - FEB AUG 2013
joint pains. On physical examination friction inflammation in the heart muscle and are 2013 MED BOARDS;
rub is heard at the latoral border of the characteristic of rheumatic heart disease. TOPNOTCH MD)
sternum, with rapid pulse. What is the
pathognomynic sign in this patient?
A. Verrucae
B. Anitschow Myocytes
C. Aschoff Body
D. Heberden's Node
E. None of the above
673 A 30 year old female with a history of oral Folate deficiency is a lack of folic acid in the diet and HAZEL KAREN RAZ, MIDTERM 2 -
contraceptive use presented with fatigue, the signs are often subtle. Folate deficiency anemia is MD (TOP 6 - FEB AUG 2013
headache, palpitations, sore tongue and the medical name given for the condition. PBS show 2013 MED BOARDS;
generalized pallor, she was diagnosed to have hypersegmented nuclei, pancytopenia and TOPNOTCH MD)
filate dificiency. The following is seen in the macrocytosis.
PBS of patients with FA deficiency except?
A. Hyposegmentad neutrophils
B. Pancytopenia
C. Macrocytosis
D. a and b
E. none
674 Characteristic histopathologic findings in In multiple myeloma, collections of abnormal plasma HAZEL KAREN RAZ, MIDTERM 2 -
multiple myeloma? cells accumulate in the bone marrow, where they MD (TOP 6 - FEB AUG 2013
A. Fiery red cytoplasm interfere with the production of normal blood cells. 2013 MED BOARDS;
B. Multiple nuclei with prominent nucleoli and Most cases of myeloma also feature the production of TOPNOTCH MD)
cytoplasmic droplets containing a paraproteinan abnormal antibody which can
immunoglobulin cause kidney problems. Bone lesions and
C. Pink gobluar cytoplasmic inclusions hypercalcemia (high calcium levels) are also often
D. AOTA encountered
E. None of the above
675 A 35-year old male came in due to fever, weight AML is a cancer of the myeloid line of blood cells, HAZEL KAREN RAZ, MIDTERM 2 -
loss, night sweats and easy bruising. On characterized by the rapid growth of abnormal white MD (TOP 6 - FEB AUG 2013
physical examination, splenomegaly and blood cells that accumulate in the bone marrow and 2013 MED BOARDS;
swollen gums are present. CBC shows interfere with the production of normal blood cells. TOPNOTCH MD)
leukocytosis with predominance of blasts. PBS AML is the most common acute leukemia affecting
shows distinctive needle-like azurophilic adults, and its incidence increases with age.
granules in myeloblasts. What is the diagnosis?
A. ALL
B. AML
C. CLL
D. CML
E. None of the above
676 In Von Willebrand disease, deficiency of VWF Inc. Bleeding time, Dec. Platelet count, Inc. PT, Inc. HAZEL KAREN RAZ, MIDTERM 2 -
leads to dysfunctional platelet adhesion. PTT (DIC) MD (TOP 6 - FEB AUG 2013
Laboratory findings diagnostic of VWF Inc. Bleeding time, Dec. Platelet Count, Normal PT, 2013 MED BOARDS;
includes? PTT (Thrombocytopenia) TOPNOTCH MD)
A. Inc. Bleeding time, Dec. Platelet count, Inc. Normal Bleeding Time, Normal Platelet Count,
PT, Inc. PTT Normal PT, Inc. PTT (Hemophilia)
B. Inc. Bleeding time, Dec. Platelet Count,
Normal PT, PTT
C. Inc. Bleeding Time, Normal Platelet Count,
Normal PT, Inc. PTT
D. Normal Bleeding Time, Normal Platelet
Count, Normal PT, Inc. PTT
E. None of the above

TOPNOTCH MEDICAL BOARD PREP PATHOLOGY SUPEREXAM Page 93 of 99


For inquiries visit www.topnotchboardprep.com.ph or email us at topnotchmedicalboardprep@gmail.com
TOPNOTCH MEDICAL BOARD PREP PATHOLOGY SUPEREXAM
For inquiries visit www.topnotchboardprep.com.ph or email us at topnotchmedicalboardprep@gmail.com
Item QUESTION EXPLANATION AUTHOR TOPNOTCH
# EXAM
677 Deficiency of alpha-1 antitrypsin leads to this Severe A1AT deficiency causes panacinar emphysema HAZEL KAREN RAZ, MIDTERM 2 -
type of emphysema? or COPD in adult life in many people with the MD (TOP 6 - FEB AUG 2013
A. Centrilobular condition (especially if they are exposed to cigarette 2013 MED BOARDS;
B. Panacinar smoke), as well as various liver diseases in a minority TOPNOTCH MD)
C. Paraseptal of children and adults, and occasionally more unusual
D. Irregular problems
E. None of the above

678 Most common type of cancer arising from the Adenocarcinoma arises from glandular cells that are HAZEL KAREN RAZ, MIDTERM 2 -
distal 1/3 of esophagus? present at the junction of the esophagus and stomach. MD (TOP 6 - FEB AUG 2013
A. Squamous cell CA 2013 MED BOARDS;
B. Barrett's esophagus TOPNOTCH MD)
C. Adenocareinoma
D. Netastasis
E. None of the above

679 Most common pituitary tumor? A prolactinoma is a benign tumor (adenoma) of the HAZEL KAREN RAZ, MIDTERM 2 -
A. Prolactinoma pituitary gland that produces a hormone called MD (TOP 6 - FEB AUG 2013
B. Somatotropic Adenoma prolactin. It is the most common type of pituitary 2013 MED BOARDS;
C. Acidophilic adenoma tumor. Symptoms of prolactinoma are caused by too TOPNOTCH MD)
D. Corticotrophic Adenoma much prolactin in the blood (hyperprolactinemia) or
E. None of the above by pressure of the tumor on surrounding tissues.

680 A 20-year old female presented with malar A wire-loop lesion may be present in stage III and IV. HAZEL KAREN RAZ, MIDTERM 2 -
rash, oral ulcers, protosensitivity and This is a glomerular capillary loop with subendothelial MD (TOP 6 - FEB AUG 2013
proteinuria with granules or casts, kidney immune complex deposition that is circumferential 2013 MED BOARDS;
biopsy was done showing a "wire loop around the loop. TOPNOTCH MD)
abnormality" on light microscope with marked
subendothelial immune complex deposition on
electron microscope. What is the diagnosis?
A. IqA nephropathy
B. Goodpasture syndrome
C. Focal segmental glomerulosclerosis
D. Lupus nephropathy
E. None of the above
681 What endothelial-leukocyte adhesion molecule Answer: D. ICAM-1 (Table 2-1, Robbins and Cotran MICHELLE JAY MIDTERM 1 -
has a major role in adhesion, arrest and Pathologic Basis of Disease, 8th ed.) FRANCISCO, MD AUG 2013
transmigration of neutrophils, monocytes and *SIMILAR TO PREVIOUS BOARD EXAM (TOP 9 - FEB 2013
lymphocytes? CONCEPT/PRINCIPLE MED BOARDS;
A. P-selectin TOPNOTCH MD)
B. E-selectin
C. VCAM-1
D. ICAM-1
682 Mrs. Kaka Awa, 32 yo presented to the ED Answer: D. Marchiafava-Micheli Syndrome MICHELLE JAY MIDTERM 1 -
because she passed blood instead of urine Notes: Paroxysmal NOCTURNAL Hemoglobinuria FRANCISCO, MD AUG 2013
morning PTA after aerobics the night before. (PNH), sometimes referred to as Marchiafava-Micheli (TOP 9 - FEB 2013
She also had 3-day history of recurrent attacks Syndrome, is a disease that results from acquired MED BOARDS;
of tolerable abdominal pain and vomiting. mutations in the PIGA gene, an enzyme that is TOPNOTCH MD)
Physical examination revealed mild jaundice, essential for the synthesis of certain cell surface
abdominal distention, and bilateral lower proteins. PNH triad: hemolysis, pancytopenia and
extremity edema. Laboratory evaluation distinct tendency to venous thrombosis. Screening
revealed hemoglobin level of 7.0 g/dL, test: Sugar water hemolysis test, confirmatory test:
pancytopenia, total serum bilirubun level 4.4 Hams test. The only form of treatment that currently
mg/dL, ALT = 51 U/L, AST = 17 U/L, serum can provide a definitive cure for PNH is allogeneic
creatinine normal. Initial screening with sugar bone marrow transplantation (BMT).
water hemolysis test was positive. Findings at
abdominal radiography showed a small amount
of bowel gas but disclosed no other
abnormalities. US and CT findings included
inferior vena caval, hepatic venous, and portal
venous thrombosis, with an associated Budd-
Chiari Syndrome. Further studies revealed
acquired mutations in the phosphatidylinositol
glycan complementation group A gene (PIGA).
In the absence of liver disease, this raises the
suspicion of what condition?
A. Paroxysmal Cold Hemoglobinuria
B. March Hemoglobinuria
C. Hereditary Spherocytosis
D. Marchiafava-Micheli Syndrome
683 Which of the following is most likely associated Answer: C. BCR-ABL gene MICHELLE JAY MIDTERM 1 -
with the Philadelphia chromosome? FRANCISCO, MD AUG 2013
A. Found in AML (TOP 9 - FEB 2013
B. t (9;23) MED BOARDS;
C. BCR-ABL gene TOPNOTCH MD)
D. Good prognosis

TOPNOTCH MEDICAL BOARD PREP PATHOLOGY SUPEREXAM Page 94 of 99


For inquiries visit www.topnotchboardprep.com.ph or email us at topnotchmedicalboardprep@gmail.com
TOPNOTCH MEDICAL BOARD PREP PATHOLOGY SUPEREXAM
For inquiries visit www.topnotchboardprep.com.ph or email us at topnotchmedicalboardprep@gmail.com
Item QUESTION EXPLANATION AUTHOR TOPNOTCH
# EXAM
684 After a week in the hospital for treatment of an Answer: B. Drug reaction MICHELLE JAY MIDTERM 1 -
upper respiratory infection complicated by Notes: The time course fits best with a drug reaction FRANCISCO, MD AUG 2013
pneumonia, a 43 yo female develops skin producing an acute erythematous dermatitis. Urticaria (TOP 9 - FEB 2013
lesions that are 2 to 4 mm in diameter. These from type I hypersensitivity is not as severe or as long MED BOARDS;
lesions are red, papulovesicular, oozing, and lasting. Sepsis rarely involves the skin with an TOPNOTCH MD)
crusted and are located on her trunk and erythematous dermatitis. Photosensitivity may be
extremities. The lesions begin to disappear enhanced by drugs, but UV light is the key component
after she is discharged from the hospital a week in light that produces photodermatitis.
later. What is the most likely pathogenesis for
her skin lesions?
A. Type I hypersensitivity
B. Drug reaction
C. Bacterial septicemia
D. Photosensitivity
685 A 45 yo female complained of headaches for Answer: B. Metastatic breast cancer MICHELLE JAY MIDTERM 1 -
about a month. She then suffered a generalized Notes: The most common cause for clinically FRANCISCO, MD AUG 2013
seizure and became obtunded. Her serum significant hypercalcemia in adults is a malignancy. (TOP 9 - FEB 2013
calcium concentration was found to be Metastatic disease from common primaries such as MED BOARDS;
markedly elevated at 15.4 mg/dL, with a serum breast, lung, and kidney tumors is much more TOPNOTCH MD)
phosphorus level of only 1.9 mg/dL. The serum frequent than parathyroid carcinoma, which tends to
albumin level was 4.2 g/dL. A chest radiograph be local but aggressive.
showed multiple lung masses, and there
appeared to be lytic lesions of the vertebral
column. Which of the following conditions best
accounts for these findings?
A. Parathyroid carcinoma
B. Metastatic breast cancer
C. Tuberculosis
D. Vitamin D toxicity
686 In the third trimester of pregnancy, a 28 yo Answer: D. Fibroadenoma MICHELLE JAY MIDTERM 1 -
woman discovers a lump in her right breast. Notes: Fibroadenomas are common, and they may FRANCISCO, MD AUG 2013
Her physician palpates a 2-cm, discrete, freely enlarge in pregnancy or late in menstrual cycle. Most (TOP 9 - FEB 2013
movable mass beneath the nipple. After intraductal papillomas are smaller than 1 cm, and they MED BOARDS;
delivery of a term infant, the mass appears to are not influenced by hormonal changes. Phyllodes TOPNOTCH MD)
decrease slightly in size. The infant breast-feeds tumors are uncommon, and they tend to be larger
without difficulty. This breast lesion is most than 4 cm. Lobular carcinoma in situ (LCIS) is typically
likely to be a (an) an ill-defined lesion without a mass effect.
A. Intraductal papilloma
B. Phyllodes tumor
C. Lobular carcinoma in situ
D. Fibroadenoma
687 An otherwise healthy 72 yo male has increasing Answer: A. Hyperplastic nodules of stroma and MICHELLE JAY MIDTERM 1 -
difficulty with urination. He has to get up glands lined by two layers of epithelium FRANCISCO, MD AUG 2013
several times each night because of a feeling of Notes: The clinical features are typical of nodular (TOP 9 - FEB 2013
urgency, but each time, the urine volume is not hyperplasia of prostate. Mild elevation of the PSA level MED BOARDS;
great. He has difficulty starting and stopping can occur with nodular hyperplasia. The area of the TOPNOTCH MD)
urination. This problem has gotten worse over prostate that is most often involved with nodular
the last few years. His serum PSA level is hyperplasia to produce significant obstruction is in the
slightly increased but stable over this time. A inner (transitional and periurethral) zone.
biopsy of the prostate is most likely to reveal
which of the following?
A. Hyperplastic nodules of stroma and glands
lined by two layers of epithelium
B. Poorly differentiated glands lined by a single
layer of epithelium and packed back to back
C. Foci of chronic inflammatory cells in the
stroma and in normal-appearing glands
D. Areas of liquefactive necrosis filled with
neutrophils
688 This term refers to an increase in white blood Answer: B. Pleocytosis MICHELLE JAY MIDTERM 1 -
cells in cerebrospinal fluid FRANCISCO, MD AUG 2013
A. Cylindruria (TOP 9 - FEB 2013
B. Pleocytosis MED BOARDS;
C. Leukocytosis TOPNOTCH MD)
D. Xanthochromia
689 A 60 yo male who has terminal carcinoma of Answer: D. Schistocytes MICHELLE JAY MIDTERM 1 -
the colon develops widespread ecchymoses Notes: This is an example of a DIC with associated FRANCISCO, MD AUG 2013
over his skin surface. The PT is 3 seconds, PTT microangiopathic haemolytic anemia. The DIC (TOP 9 - FEB 2013
is 55 seconds, platelet count is 15,200/uL, developed the setting of a mucin-secreting MED BOARDS;
fibrinogen level is 75mg/dL, and fibrin split adenocarcinoma. Schistocytes are fragmented RBCs. TOPNOTCH MD)
products levels are very elevated. Which of the
following morphologic findings would you most
expect to find on examination of the PBS?
A. Howell-Jolly bodies
B. Tear-drop cells
C. Macro-ovalocytes
D. Schistocytes
E. Target cells

TOPNOTCH MEDICAL BOARD PREP PATHOLOGY SUPEREXAM Page 95 of 99


For inquiries visit www.topnotchboardprep.com.ph or email us at topnotchmedicalboardprep@gmail.com
TOPNOTCH MEDICAL BOARD PREP PATHOLOGY SUPEREXAM
For inquiries visit www.topnotchboardprep.com.ph or email us at topnotchmedicalboardprep@gmail.com
Item QUESTION EXPLANATION AUTHOR TOPNOTCH
# EXAM
690 Mutya, a 3 yo girl presents with dark Answer: B. Basophilic stippling of erythrocytes MICHELLE JAY MIDTERM 1 -
precipitates along gingival margins, radiopaque Notes: Classic features of lead poisoning forming a FRANCISCO, MD AUG 2013
deposits in the epiphyses of her bones, and gingival lead line (composed of precipitated lead (TOP 9 - FEB 2013
urinary excretion of delta-aminolevulinic acid sulfide), radiopaque deposits in epiphyses, basophilic MED BOARDS;
(delta-ALA). Her father states that they live in stippling of erythrocytes, increased delta-ALA, and TOPNOTCH MD)
an old house that has chipped paint. The childs peripheral neuropathy and other CNS changes.
blood would most likely have which of the
following?
A. Schistocytes and helmet cells
B. Basophilic stippling of erythrocytes
C. Increased osmotic fragility of erythrocytes
D. Clumping of erythrocytes at temperatures
below 30OC
691 A 35 yr-old woman who has been taking oral Answer: E MICHELLE JAY MIDTERM 1 -
contraceptives for many years presents with Liver cell adenomas may occur after several years of FRANCISCO, MD AUG 2013
acute abdominal pain and fullness. Paracentesis taking oral contraceptives but the actual mechanism (TOP 9 - FEB 2013
harvests 200 ml of bloody fluid. Imaging studies of tumor formation is unknown. MED BOARDS;
show a 6-cm mass in the liver that is TOPNOTCH MD)
subsequently resected. Histologic exam of this
specimen would most likely reveal this to be
which of the following?
A. Angiosarcoma
B. Cholangiosarcoma
C. Focal nodular hyperplasia
D. Hepatocellular carcinoma
E. Liver cell adenoma
692 The alterations in the hemodynamic, metabolic Answer: B MICHELLE JAY MIDTERM 1 -
and immune responses evident in stressed IL-4, IL-10, IL-3 and TGF-B are all anti-inflammatory. FRANCISCO, MD AUG 2013
patients are orchestrated by endogenous (TOP 9 - FEB 2013
polypeptides known as cytokines. They are MED BOARDS;
produced by immune cells in direct response to TOPNOTCH MD)
injury, with levels correlating with the degree
of tissue damage. Despite considerable overlap
in bioactivity among cytokines, they are
commonly classified by their predominant
effect as proinflammatory or anti-
inflammatory. Which is not considered as the
former?
A. IL-1
B. IL-4
C. IL-6
D. IFN-y
693 Malignancy is second only to trauma as the Answer: B MICHELLE JAY MIDTERM 1 -
leading cause of death in children. In infants, it Solid tumor for < 2 y.o = neuroblastoma, > 2 y.o= FRANCISCO, MD AUG 2013
is the most frequent cause of death after Wilms tumor (TOP 9 - FEB 2013
prematurity and congenital anomalies. MED BOARDS;
Approximately 40% of childhood anomalies are TOPNOTCH MD)
A. Lymphoma
B. Leukemia
C. Neuroblastoma
D. Rhabdomyosarcoma
694 Upon investigation, a farmer was diagnosed to Answer: B MICHELLE JAY MIDTERM 1 -
be infected with actinomycetes. His lung was Serum sickness- type III systemic; Goodpasture- type FRANCISCO, MD AUG 2013
noted to have local, subacute hypersensitivity II; GVHD type IV (TOP 9 - FEB 2013
reaction characterized by edema and necrosis MED BOARDS;
along with complement activation. Your TOPNOTCH MD)
impression is :
A. Serum Sickness
B. Arthus reaction
C. Graft-versus-host disease
D. Goodpasture syndrome
695 A 70 y.o man with sepsis has a pH of 7.18. Answer: C MICHELLE JAY MIDTERM 1 -
which of the following statements is true Metabolic acidosis- initial compensation is FRANCISCO, MD AUG 2013
regarding his metabolic acidosis? respiratory. Hypoxia leads to anaerobic metabolism; (TOP 9 - FEB 2013
A. Tissue hypoxia leads to increased oxidative volume replacement & not vasopressors will correct MED BOARDS;
metabolism. underperfusion. TOPNOTCH MD)
B. Acute compensation for metabolic acidosis is
primarily renal.
C. Metabolic acidosis results from loss of
bicarbonate or gain of fixed acids.
D. Restoration of blood pressure with
vsopressors corrects the acidosis associated
with circulatory failure.
696 In differentiating obstructive from restrictive Answer: B MICHELLE JAY MIDTERM 1 -
lung diseases, this spirometry criteria Obstructive: Inc.TLC, FRC,RV but FEV1 is more FRANCISCO, MD AUG 2013
characterizes the former: dramatically reduced = dec.FEV1/FVC ratio (TOP 9 - FEB 2013
A. Decreased TLC MED BOARDS;
B. Decreased FEV1/FVC ratio TOPNOTCH MD)
C. Decreased FRC/FEV ratio
D. Decreased RV

TOPNOTCH MEDICAL BOARD PREP PATHOLOGY SUPEREXAM Page 96 of 99


For inquiries visit www.topnotchboardprep.com.ph or email us at topnotchmedicalboardprep@gmail.com
TOPNOTCH MEDICAL BOARD PREP PATHOLOGY SUPEREXAM
For inquiries visit www.topnotchboardprep.com.ph or email us at topnotchmedicalboardprep@gmail.com
Item QUESTION EXPLANATION AUTHOR TOPNOTCH
# EXAM
697 An 85 y.o male patient was recently found to Answer: D MICHELLE JAY MIDTERM 1 -
have an apical lung mass. A few months after, Pancoast tumor- cause Horners syndrome (triad of FRANCISCO, MD AUG 2013
patient was noted to have episodes of drooping ptosis, miosis, anhidrosis) (TOP 9 - FEB 2013
of eyelids, pupillary constriction and MED BOARDS;
anhidrosis. This condition is due to TOPNOTCH MD)
A. Neoplasm of neuroendocrine Kulchitsky cells
B. Lambert-Eaton syndrome manifested as
muscle weakness
C. Ectopic secretion of serotonin known as
carcinoid syndrome
D. Compression of cervical sympathetic plexus
698 The least important independent risk factor Answer: A MICHELLE JAY MIDTERM 1 -
associated with increased risk for DVT: Venostasis of lower extremities is asso.with prolonged FRANCISCO, MD AUG 2013
A. Obesity bed rest, standing or sitting, my immobilization and (TOP 9 - FEB 2013
B. Central venous catheter ,muscular paralysis asso.with trauma and gen.& spinal MED BOARDS;
C. Hospitalization with recent surgery anesthesia. TOPNOTCH MD)
D. Previous DVT
699 Most tumors tend to metastasize via the Answer: C MICHELLE JAY MIDTERM 1 -
lymphatics or blood vessels. Which among Pattern of spread for mesothelioma is distinctive, it FRANCISCO, MD AUG 2013
these deviate from the more common pattern of can even be diagnosed radiologically by a thick rind of (TOP 9 - FEB 2013
spread and metastasize over the surface of tumor tissue it characteristically produces over the MED BOARDS;
viscera or body cavities? surface of involved lung. TOPNOTCH MD)
A. Colon CA
B. Gastric CA
C. Mesothelioma
D. Hepatoblastoma
700 A 30 yr-old woman presents to her physician Answer: B MICHELLE JAY MIDTERM 1 -
complaining of a recent nodular growth on her Findings are consistent with nodular sclerosing HL, FRANCISCO, MD AUG 2013
left neck. Further history reveals a 16-lbs most common type (65-75%); cells described pertain (TOP 9 - FEB 2013
weight loss and intermittent fevers over the to Reed-Sternberg cells. MED BOARDS;
past3-6 months. Physical examination reveals TOPNOTCH MD)
the presence of a firm, mobile, non-tender, 2.5
cm lower cervical lymph node, left neck. A
biopsy is performed and pathology report
indicates the presence of large cells with
multilobate nuclei, abundant pale cytoplasm
and significant band of sclerosis.
Immunohistochemistry is positive for the
presence of CD15 and CD30 cell markers. These
findings are most consistent with which of the
following neoplasms?
A. Acute lymphoblastic leukemia
B. HodgkinLymphoma
C. Lymphoplasmacytic lymphoma
D. Mantle zone lymphoma
E. Mycosis fungoides

TOPNOTCH MEDICAL BOARD PREP PATHOLOGY SUPEREXAM Page 97 of 99


For inquiries visit www.topnotchboardprep.com.ph or email us at topnotchmedicalboardprep@gmail.com
TOPNOTCH MEDICAL BOARD PREP PATHOLOGY SUPEREXAM
For inquiries visit www.topnotchboardprep.com.ph or email us at topnotchmedicalboardprep@gmail.com

Item # ANSWER 88 B 176 A 264 A 352 D


1 D 89 C 177 E 265 E 353 A
2 E 90 A 178 A 266 A 354 C
3 C 91 A 179 D 267 B 355 B
4 A 92 B 180 E 268 D 356 C
5 C 93 A 181 D 269 D 357 B
6 C 94 C 182 D 270 B 358 B
7 B 95 A 183 A 271 D 359 A
8 C 96 C 184 D 272 A 360 C
9 E 97 A 185 A 273 E 361 E
10 B 98 B 186 C 274 D 362 B
11 D 99 D 187 B 275 B 363 D
12 A 100 C 188 D 276 C 364 D
13 B 101 C 189 A 277 B 365 B
14 C 102 C 190 A 278 C 366 C
15 D 103 D 191 C 279 B 367 A
16 B 104 A 192 C 280 A 368 E
17 B 105 A 193 C 281 D 369 B
18 B 106 B 194 D 282 C 370 A
19 B 107 E 195 B 283 C 371 D
20 A 108 D 196 A 284 B 372 A
21 C 109 B 197 A 285 A 373 E
22 C 110 C 198 E 286 A 374 B
23 A 111 A 199 C 287 E 375 A
24 D 112 E 200 C 288 D 376 C
25 C 113 B 201 C 289 B 377 A
26 E 114 E 202 D 290 C 378 C
27 B 115 D 203 B 291 A 379 E
28 C 116 A 204 B 292 B 380 D
29 C 117 B 205 C 293 E 381 C
30 C 118 C 206 A 294 C 382 B
31 C 119 C 207 D 295 A 383 B
32 D 120 E 208 C 296 E 384 A
33 B 121 A or B 209 B 297 A 385 B
34 C 122 A 210 C 298 C 386 D
35 E 123 E 211 D 299 E 387 A
36 D 124 B 212 A 300 B 388 B
37 C 125 E 213 C 301 B 389 A
38 C 126 B 214 E 302 C 390 C
39 D 127 C 215 C 303 A 391 A
40 B 128 C 216 B 304 D 392 B
41 B 129 B 217 B 305 C 393 B
42 D 130 B 218 C 306 B 394 C
43 D 131 B 219 D 307 A 395 E
44 E 132 B 220 B 308 D 396 B
45 A 133 D 221 C 309 C 397 D
46 C 134 A 222 D 310 B 398 A
47 D 135 E 223 D 311 D 399 C
48 A 136 A 224 A 312 E 400 D
49 E 137 B 225 A 313 D 401 B
50 B 138 C 226 D 314 D 402 D
51 A 139 A 227 B 315 B 403 C
52 A 140 D 228 D 316 A 404 D
53 C 141 B 229 C 317 C 405 D
54 C 142 C 230 D 318 C 406 D
55 B 143 B 231 D 319 B 407 C
56 C 144 D 232 C 320 C 408 B
57 B 145 C 233 B 321 C 409 E
58 A 146 D 234 C 322 D 410 A
59 B 147 B 235 A 323 D 411 A
60 A 148 C 236 C 324 A 412 D
61 D 149 B 237 A 325 B 413 A
62 A 150 B 238 B 326 C 414 D
63 C 151 D 239 A 327 B 415 C
64 E 152 C 240 B 328 D 416 B
65 D 153 C 241 E 329 C 417 D
66 A 154 C 242 B 330 A 418 B
67 D 155 D 243 A 331 D 419 A
68 C 156 D 244 B 332 D 420 E
69 B 157 E 245 D 333 A 421 B
70 A 158 B 246 B 334 B 422 B
71 D 159 D 247 D 335 C 423 D
72 D 160 B 248 C 336 D 424 C
73 A 161 B 249 D 337 C 425 A
74 D 162 E 250 E 338 D 426 C
75 C 163 A 251 C 339 B 427 D
76 B 164 C 252 D 340 D 428 B
77 C 165 D 253 B 341 A 429 C
78 A 166 A 254 B 342 C 430 D
79 D 167 D 255 C 343 D 431 A
80 E 168 C 256 D 344 D 432 C
81 B 169 E 257 C 345 B 433 C
82 C 170 A 258 D 346 D 434 C
83 C 171 B 259 B 347 A 435 B
84 B 172 C 260 B 348 B 436 C
85 C 173 D 261 C 349 A 437 D
86 B 174 E 262 D 350 B 438 A
87 D 175 D 263 E 351 B 439 B
TOPNOTCH MEDICAL BOARD PREP PATHOLOGY SUPEREXAM Page 98 of 99
For inquiries visit www.topnotchboardprep.com.ph or email us at topnotchmedicalboardprep@gmail.com
TOPNOTCH MEDICAL BOARD PREP PATHOLOGY SUPEREXAM
For inquiries visit www.topnotchboardprep.com.ph or email us at topnotchmedicalboardprep@gmail.com
440 A 529 C 618 A
441 D 530 C 619 C
442 C 531 D 620 C
443 A 532 D 621 C
444 B 533 A 622 B
445 D 534 D 623 A
446 C 535 D 624 B
447 C 536 B 625 E
448 B 537 C 626 E
449 C 538 D 627 B
450 A 539 A 628 A
451 D 540 B 629 D
452 E 541 C 630 B
453 C 542 D 631 C
454 C 543 B 632 A
455 D 544 D 633 D
456 B 545 B 634 D
457 B 546 E 635 C
458 D 547 C 636 E
459 C 548 A 637 A
460 A 549 C 638 D
461 B 550 E 639 C
462 B 551 B 640 A
463 D 552 E 641 B
464 A 553 A 642 B
465 C 554 B 643 E
466 D 555 C 644 E
467 B 556 A 645 B
468 D 557 D 646 D
469 E 558 B 647 E
470 A 559 E 648 C
471 C 560 C 649 E
472 A 561 B 650 C
473 D 562 B 651 E
474 C 563 C 652 C
475 A 564 A 653 C
476 D 565 D 654 C
477 B 566 B 655 C
478 D 567 B 656 C
479 D 568 C 657 E
480 A 569 B 658 C
481 B 570 B 659 E
482 C 571 C 660 E
483 A 572 D 661 A
484 A 573 C 662 C
485 C 574 D 663 D
486 D 575 B 664 C
487 A 576 B 665 B
488 D 577 C 666 C
489 B 578 B 667 B
490 B 579 D 668 D
491 B 580 B 669 D
492 B 581 A 670 D
493 C 582 A 671 B
494 C 583 D 672 C
495 C 584 E 673 A
496 A 585 B 674 D
497 D 586 D 675 B
498 A 587 C 676 C
499 B 588 A 677 B
500 B 589 E 678 C
501 B 590 A 679 A
502 B 591 C 680 D
503 E 592 B 681 D
504 A 593 D 682 D
505 C 594 D 683 C
506 A 595 B 684 B
507 B 596 C 685 B
508 D 597 D 686 D
509 C 598 B 687 A
510 E 599 A 688 B
511 B 600 B 689 D
512 C 601 B 690 B
513 D 602 A 691 E
514 B 603 E 692 B
515 A 604 E 693 B
516 D 605 D 694 B
517 C 606 A 695 C
518 B 607 E 696 B
519 B 608 A 697 D
520 A 609 C 698 A
521 A 610 A 699 C
522 C 611 B 700 B
523 A 612 A
524 C 613 C
525 C 614 D
526 C 615 B
527 D 616 C
528 B 617 D
TOPNOTCH MEDICAL BOARD PREP PATHOLOGY SUPEREXAM Page 99 of 99
For inquiries visit www.topnotchboardprep.com.ph or email us at topnotchmedicalboardprep@gmail.com

Potrebbero piacerti anche